Final Exam Nursing 3 Old Stuff only

Lakukan tugas rumah & ujian kamu dengan baik sekarang menggunakan Quizwiz!

Lice and body mites have been discovered in an elementary school population. Which instructions should the school nurse send home to parents?

"Look for small bugs in your children's hair or small objects attached to the hair shaft." "If lice or mites are found, stuffed animals should be washed or sealed in an airtight bag for 2 weeks."

What is an appropriate screening test for hearing that can be administered by the nurse to a 5-year-old child? a. The Rinne test b. The Weber test c. Conventional audiometry d. Eliciting the startle reflex

ANS: C Conventional audiometry is a behavioral test that measures auditory thresholds in response to speech and frequency-specific stimuli presented through earphones. The Rinne and Weber tests measure bone conduction of sound. Eliciting the startle reflex may be useful in infants. DIF: Cognitive Level: Understand REF: p. 97 TOP: Integrated Process: Nursing Process: Assessment MSC: Area of Client Needs: Health Promotion and Maintenance

A client has psoriasis. Prior to beginning education, the nurse assesses the client. Which statement indicates the client has a correct knowledge base about his illness?

"Treatment can help lessen the discomfort of my psoriasis."

________ is abroad spectrum aminopenicillin (penicillin)

Amoxicillin

Which type of play is most typical of the preschool period? a. Solitary b. Associative b. Parallel d. Team

B Associative play is group play in similar or identical activities but without rigid organization or rules. Solitary play is that of infants. Parallel play is that of toddlers. School-age children play in teams.

What is the first step in becoming a culturally competent nurse?

Developing cultural awareness

What is the therapeutic class for Cyclosporine?

Immunosuppressant

The drug that would most likely be used in the treatment of tuberculosis is

Isoniazid (INH).

Multiculturism

Rather than blending smoothly into the bigger pot as former immigrants have done, this modern immigrants maintain their own unique flavors and textures, much like the ingredients in a large tossed salad.

When nurses work with non-english speaking patients how will they measure a client's understanding?

The nurse needs to develop alternative ways through nonverbal responses.

Contraindications of cyclosporine

The only contraindication is prior hypersensitivity to the drug

which statements about HIV are accurate? (Select all that apply) A. may be acquired or congenital B. it is retrovirus C. it always progresses to AIDS D. it is a virus that attacks the immune system E. it is a parasite that forces cells to make copies of itself

b, d, e

Why is amoxacillin considered a broad spectrum antibiotic?

because it is effective against a wide range of microorganisms

What is diversity?

differences between cultures

Dermatitis is characterized by

pruritus.

What is the leading cause of miscommunication?

touching clients from different cultures that they may deem as inappropriate

Pharmacotherapy of Influenza

• Best approach—vaccination for prevention • Antivirals to prevent, decrease severity of acute symptoms - Amantadine (Symmetrel) - Rimantadine (Flumadine)

which methods or items are means of transmitting HIV (Select all that apply) a. sex b. household utensils c. breast milk d. toilet facilities e. mosquitoes

a, c

where in the body can cytomegalovirus present with symptoms? (Select all that apply) a. eyes, causing visual impairment b. kidneys as glomerulonephritis c. respiratory tract causing pneumonia d. GI tract, causing diarrhea e. heart as cardiomyopathy

a, c, d

which actions can the nurse delegate to the UAP who will be giving mouth care to a patient with HIV/AIDS (Select all that apply) a. offer mouth rinses with sodium bicarb and sterile water several times a day b. assess mouth for increased presence of lesions c. encourage the patient to drink plenty of fluids d. provide a soft bristled toothbrush e. administer oral analgesic gel

a, c, d

which conditions may be the first signs of HIV in women? (Select all that apply) A. vaginal candidiasis B. bladder infections C. cervical caner D. PID E. mononucleosis

a, c, d

which lab resluts will the nurse expect to decrease (Select all that apply) a. cd4+ b. cd8+ c. WBC d. lymphocytes e. HIV antibodies

a, c, d

which opportunistic infections can be observed in AIDS (Select all that apply) A. toxicoplasmosis B. gastroenteritis C. TB D. candidiasis E. cytomegalovirus

a, c, d, e

Which statement best describes a child who is abused by the parent(s)? a. Unintentionally contributes to the abusing situation b. Belongs to a low socioeconomic population c. Is healthier than the nonabused siblings d. Abuses siblings in the same way as child is abused by the parent(s)

A A child's temperament, position in the family, additional physical needs, activity level, or degree of sensitivity to parental needs unintentionally contributes to the abusing situation. Socioeconomic status is an environmental characteristic. This child is less likely to be abused than one who is premature, disabled, or very young. The abused child does not in turn abuse his or her siblings.

4. A client has a tracheostomy tube in place. When the nurse suctions the client, food particles are noted. What action by the nurse is best? a. Elevate the head of the client's bed. b. Measure and compare cuff pressures. c. Place the client on NPO status. d. Request that the client have a swallow study.

ANS: B Constant pressure from the tracheostomy tube cuff can cause tracheomalacia, leading to dilation of the tracheal passage. This can be manifested by food particles seen in secretions or by noting that larger and larger amounts of pressure are needed to keep the tracheostomy cuff inflated. The nurse should measure the pressures and compare them to previous ones to detect a trend. Elevating the head of the bed, placing the client on NPO status, and requesting a swallow study will not correct this situation. DIF: Analyzing/Analysis REF: 523 KEY: Tracheostomy| patient safety| nursing assessment MSC: IntegratedProcess:NursingProcess:Assessment NOT: Client Needs Category: Physiological Integrity: Physiological Adaptation

A young woman is not pregnant but has not had a menstrual period for 5 months. Which factors does the nurse explore as a possible cause of the amenorrhea? a. The client's mother having type 2 diabetes mellitus b. Running 10 to 15 miles/day c. Taking aspirin daily d. Having a diet high in protein

ANS: B Excessive exercise, with corresponding loss of body fat, is associated with insufficient estrogen levels for the maintenance of normal ovulatory and menstrual cycles. The other factors are noncontributory.

4. A client is receiving bolus feedings through a Dobhoff tube. What action by the nurse is most important? a. Auscultate lung sounds after each feeding. b. Check tube placement before each feeding. c. Check tube placement every 8 hours. d. Weigh the client daily on the same scale.

ANS: B For bolus feedings, the nurse checks placement of the tube per institutional policy prior to each feeding, which is more often than every 8 hours during the day. Auscultating lung sounds is also important, but this will indicate a complication that has already occurred. Weighing the client is important to determine if nutritional goals are being met.

The nurse knows that a client with prolonged prothrombin time (PT) values (not related to medication) probably has dysfunction in which organ? a. Kidneys b. Liver c. Spleen d. Stomach

ANS: B Severe acute or chronic liver damage leads to a prolonged PT secondary to impaired synthesis of clotting proteins. The other organs are not related to this issue.

The nurse is counseling a mother who wants her teenage daughter to have a Pap smear and pelvic examination. Which statement by the nurse is most accurate? a. "If your daughter is over 18, she needs a pelvic examination and Pap smear." b. "A teenager does not need this examination unless she is sexually active." c. "Teach her to have her first examination by the age of 21 at the latest." d. "It is not needed unless you are worried about sexually transmitted diseases."

ANS: C A woman needs to have her first pelvic examination with Pap smear by the age of 21, or within 3 years of becoming sexually active. The other statements are not accurate.

Rickets is caused by a deficiency in: a. vitamin A. b. vitamin C. c. vitamin D and calcium. d. folic acid and iron.

ANS: C Fat-soluble vitamin D and calcium are necessary in adequate amounts to prevent the development of rickets. No correlation exists between vitamins A, C, folic acid, or iron and rickets.

20. A client is receiving total parenteral nutrition (TPN). What action by the nurse is most important? a. Assessing blood glucose as directed b. Changing the IV dressing each day c. Checking the TPN with another nurse d. Performing appropriate hand hygiene

ANS: D Clients on TPN are at high risk for infection. The nurse performs appropriate hand hygiene as a priority intervention. Checking blood glucose is also an important measure, but preventing infection takes priority. The IV dressing is changed every 48 to 72 hours. TPN does not need to be double-checked with another nurse.

*Outpatient treatment is planned for a patient diagnosed with anorexia nervosa. Select the most important desired outcome related to the nursing diagnosis Imbalanced nutrition: less than body requirements. Within 1 week, the patient will:* a. weigh self accurately using balanced scales. b. limit exercise to less than 2 hours daily. c. select clothing that fits properly. d. gain 1 to 2 pounds.

ANS: D Only the outcome of a gain of 1 to 2 pounds can be accomplished within 1 week when the patient is an outpatient. The focus of an outcome would not be on the patient weighing self. Limiting exercise and selecting proper clothing are important, but weight gain takes priority.

*A patient was diagnosed with anorexia nervosa. The history shows the patient virtually stopped eating 5 months ago and lost 25% of body weight. The serum potassium is currently 2.7 mg/dL. Which nursing diagnosis applies?* a. Adult failure to thrive related to abuse of laxatives as evidenced by electrolyte imbalances and weight loss b. Disturbed energy field related to physical exertion in excess of energy produced through caloric intake as evidenced by weight loss and hyperkalemia c. Ineffective health maintenance related to self-induced vomiting as evidenced by swollen parotid glands and hyperkalemia d. Imbalanced nutrition: less than body requirements related to reduced oral intake as evidenced by loss of 25% of body weight and hypokalemia

ANS: D The patient's history and lab result support the nursing diagnosis Imbalanced nutrition: less than body requirements. Data are not present that the patient uses laxatives, induces vomiting, or exercises excessively. The patient has hypokalemia rather than hyperkalemia.

Which laboratory tests will be performed to determine whether a specific bacterium is resistant to a specific drug?

Culture and sensitivity test

the nurse is preparing to administer chemotherapy to an oncology patient who also has an order for ondansetron (Zofran). when should the nurse administer ondansetron? a. every time the patient complains of nausea b. 30-60 minutes before starting chemo c. only if the patient complains of nausea d. when the patient begins to experience vomiting during chemo

b. 30-60 minutes before starting chemo

Characteristics of Viruses

• Nonliving agents that infect bacteria, plants, animals • Intracellular parasite - Must be in host cell to replicate and cause infection - Many viruses infect specific host cells

When did the transcultural movement begin?

1974

When did the ANA recommend that multicultural content be included in nursing curricula?

1976

What is the primary reason why nurses attend continuing education programs? 1. Update professional knowledge 2. Network within the nursing profession 3. Fulfill requirements for an advanced degree 4. Graduate from an accredited nursing program

1. Continuing education programs are formal learning experiences designed to update and enhance professional knowledge or skills. This is necessary because of the explosion in information and technology within health care. Some states require evidence of continuing education units (CEUs) for license renewal.

which statements regarding HIV/AIDS among older adults are true? A. the risk for HIV infection after exposure is minimal for older adults B. older men are more susceptible to HIV C. it is not necessary to assess an older adult for history of drug use D. older adults who participate in high-risk behaviors are susceptible to HIV

d

Skin cells in the epidermis are replaced and supplied by the

stratum basale.

A client has been prescribed 0.1% tacrolimus (Protopic) for treatment of severe atopic dermatitis. The nurse would evaluate that medication teaching is successful when the client makes which statement?

"I am at increased risk for skin cancer because I am using this drug."

The nurse's neighbor has a severe sunburn and cannot sleep. What is the best advice by the nurse?

"Apply a local anesthetic to the area that is sunburned."

Initiation of Pharmacotherapy

• Pharmacotherapy may be initiated - In acute phase (symptomatic) - In chronic phase (asymptomatic)

Contraindications in Hep B vaccine

- Contraindicated in patients with hypersensitivity to yeast or HBV vaccine - Patients who demonstrated severe hypersensitivity to the first dose of the vaccine should not receive subsequent doses

To promote comfort after a colonoscopy, in what position does the nurse place the client? a. Left lateral b. Prone c. Right lateral d. Supine

ANS: A After colonoscopy, clients have less discomfort and quicker passage of flatus when placed in the left lateral position.

A nurse in the family clinic is teaching a client newly diagnosed with osteoarthritis (OA) about drugs used to treat the disease. For which medication does the nurse plan primary teaching? a. Acetaminophen (Tylenol) b. Cyclobenzaprine hydrochloride (Flexeril) c. Hyaluronate (Hyalgan) d. Ibuprofen (Motrin)

ANS: A All of the drugs are appropriate to treat OA. However, the first-line drug is acetaminophen. Cyclobenzaprine is a muscle relaxant given to treat muscle spasms. Hyaluronate is a synthetic joint fluid implant. Ibuprofen is a nonsteroidal anti-inflammatory drug.

28. After a stroke, a client has ataxia. What intervention is most appropriate to include on the client's plan of care? a. Ambulate only with a gait belt. b. Encourage double swallowing. c. Monitor lung sounds after eating. d. Perform post-void residuals.

ANS: A Ataxia is a gait disturbance. For the client's safety, he or she should have assistance and use a gait belt when ambulating. Ataxia is not related to swallowing, aspiration, or voiding. DIF: Applying/Application REF: 934 KEY: Neurologic disorders| patient safety MSC: Integrated Process: Nursing Process: Implementation NOT: Client Needs Category: Safe and Effective Care Environment: Safety and Infection Control

A client is started on etanercept (Enbrel). What teaching by the nurse is most appropriate? a. Giving subcutaneous injections b. Having a chest x-ray once a year c. Taking the medication with food d. Using heat on the injection site

ANS: A Etanercept is given as a subcutaneous injection twice a week. The nurse should teach the client how to self-administer the medication. The other options are not appropriate for etanercept.

A nurse assesses a client who has open lesions. Which action should the nurse take first? a. Put on gloves. b. Ask the client about his or her occupation. c. Assess the client's pain. d. Obtain vital signs.

ANS: A Nurses should wear gloves as part of Standard Precautions when examining skin that is not intact. The other options should be completed after gloves are put on.

A nurse assesses a client who has two skin lesions on his chest. Each lesion is the size of a nickel, flat, and darker in color than the client's skin. How should the nurse document these lesions? a. Two 2-cm hyperpigmented patches b. Two 1-inch erythematous plaques c. Two 2-mm pigmented papules d. Two 1-inch moles

ANS: A Patches are larger flat areas of the skin. The information provided does not indicate a mole or the presence of erythema.

To prevent plagiocephaly, the nurse should teach parents to: a. place infant prone for 30 to 60 minutes per day. b. buy a soft mattress. c. allow infant to nap in the car safety seat. d. have infant sleep with the parents.

ANS: A Prevention of positional plagiocephaly may begin shortly after birth by implementing prone positioning or "tummy time" for approximately 30 to 60 minutes per day when the infant is awake. Soft mattresses or sleeping with parents (co-sleeping) are not recommended because they put the infant at a higher risk for a sudden infant death incident. To prevent plagiocephaly, prolonged placement in car safety seats should be avoided.

6. A client admitted for pneumonia has been tachypneic for several days. When the nurse starts an IV to give fluids, the client questions this action, saying "I have been drinking tons of water. How am I dehydrated?" What response by the nurse is best? a. "Breathing so quickly can be dehydrating." b. "Everyone with pneumonia is dehydrated." c. "This is really just to administer your antibiotics." d. "Why do you think you are so dehydrated?"

ANS: A Tachypnea and mouth breathing, both seen in pneumonia, increase insensible water loss and can lead to a degree of dehydration. The other options do not give the client useful information.

A nurse is assessing a child with kwashiorkor disease. Which assessment findings should the nurse expect? a. Thin wasted extremities with a prominent abdomen b. Constipation c. Elevated hemoglobin d. High levels of protein

ANS: A The child with kwashiorkor has thin, wasted extremities and a prominent abdomen from edema (ascites). Diarrhea (persistent diarrhea malnutrition syndrome) not constipation commonly occurs from a lowered resistance to infection and further complicates the electrolyte imbalance. Anemia and protein deficiency are common findings in malnourished children with kwashiorkor.

The nurse is teaching high school girls about the female reproductive tract. Which statements by the nurse are accurate? (Select all that apply.) a. The vagina has an acidic environment. b. The cervix is where the Pap smear is taken from. c. The ovum is fertilized in the uterus. d. Ovaries produce sex steroid hormones. e. The breasts contain fat tissue.

ANS: A, B, D, E The acidic environment of the vagina helps protect against infection. The cervix is the site for Pap testing. The ovaries produce sex steroid hormones. The breasts contain fat, glandular, fibrous, and ductal tissue. Ova are fertilized in the fallopian tubes.

The nurse is teaching parents of preschool children consequences of inadequate sleep. What should the nurse include in the teaching session? (Select all that apply.) a. Behavior changes b. Increased appetite c. Difficulty concentrating d. Poor control of emotions e. Impaired learning ability

ANS: A, C, D, E Consequences of inadequate sleep include daytime tiredness, behavior changes, hyperactivity, difficulty concentrating, impaired learning ability, poor control of emotions and impulses, and strain on family relationships. Increased appetite is not a consequence of inadequate sleep.

The nurse understands that malnutrition can occur in hospitalized clients for several reasons. Which are possible reasons for this to occur? (Select all that apply.) a. Cultural food preferences b. Family bringing snacks c. Increased need for nutrition d. Need for NPO status e. Staff shortages

ANS: A, C, D, E FAM NO SNACKS!

3. A nurse is reviewing laboratory values for several clients. Which value causes the nurse to conduct nutritional assessments as a priority? a. Albumin: 3.5 g/dL b. Cholesterol: 142 mg/dL c. Hemoglobin: 9.8 mg/dL d. Prealbumin: 28 mg/dL

ANS: B A cholesterol level below 160 mg/dL is a possible indicator of malnutrition, so this client would be at highest priority for a nutritional assessment. The albumin and prealbumin levels are normal. The low hemoglobin could be from several problems, including dietary deficiencies, hemodilution, and bleeding.

22. A client has an intraventricular catheter. What action by the nurse takes priority? a. Document intracranial pressure readings. b. Perform hand hygiene before client care. c. Measure intracranial pressure per hospital policy. d. Teach the client and family about the device.

ANS: B All of the actions are appropriate for this client. However, performing hand hygiene takes priority because it prevents infection, which is a possibly devastating complication. DIF: Applying/Application REF: 961 KEY: Neurologic disorders| Standard Precautions| infection control MSC: IntegratedProcess:NursingProcess:Implementation NOT: Client Needs Category: Safe and Effective Care Environment: Safety and Infection Control

16. Several nurses have just helped a morbidly obese client get out of bed. One nurse accesses the clients record because I just have to know how much she weighs! What action by the clients nurse is most appropriate? a. Make an anonymous report to the charge nurse. b. State That is a violation of client confidentiality. c. Tell the nurse Dont look; Ill tell you her weight. d. Walk away and ignore the other nurses behavior.

ANS: B Ethical practice requires the nurse to speak up and tell the other nurse that he or she is violating client confidentiality rules. The other responses do not address this concern.

A nurse assesses a client and identifies that the client has pallor conjunctivae. Which focused assessment should the nurse complete next? a. Partial thromboplastin time b. Hemoglobin and hematocrit c. Liver enzymes d. Basic metabolic panel

ANS: B Pallor conjunctivae signifies anemia. The nurse should assess the client's hemoglobin and hematocrit to confirm anemia. The other laboratory results do not relate to this client's potential anemia.

16. A nurse assesses a client with pericarditis. Which assessment finding should the nurse expect to find? a. Heart rate that speeds up and slows down b. Friction rub at the left lower sternal border c. Presence of a regular gallop rhythm d. Coarse crackles in bilateral lung bases

ANS: B The client with pericarditis may present with a pericardial friction rub at the left lower sternal border. This sound is the result of friction from inflamed pericardial layers when they rub together. The other assessments are not related. DIF: Remembering/Knowledge REF: 699 KEY: Inflammatory response| assessment/diagnostic examination MSC: IntegratedProcess:NursingProcess:Assessment NOT: Client Needs Category: Physiological Integrity: Physiological Adaptation

A 5-year-old girl is having a checkup before starting kindergarten. The nurse asks her to do the "finger-to-nose" test. What is the nurse testing for? a. Deep tendon reflexes b. Cerebellar function c. Sensory discrimination d. Ability to follow directions

ANS: B The finger-to-nose test is an indication of cerebellar function. This test checks balance and coordination. Each deep tendon reflex is tested separately. Each sense is tested separately. Although this test enables the nurse to evaluate the child's ability to follow directions, it is used primarily for cerebellar function. DIF: Cognitive Level: Apply REF: p. 109 TOP: Integrated Process: Nursing Process: Assessment MSC: Area of Client Needs: Health Promotion and Maintenance

1. A nurse answers a call light and finds a client anxious, short of breath, reporting chest pain, and having a blood pressure of 88/52 mm Hg on the cardiac monitor. What action by the nurse takes priority? a. Assess the client's lung sounds. b. Notify the Rapid Response Team. c. Provide reassurance to the client. d. Take a full set of vital signs.

ANS: B This client has manifestations of a pulmonary embolism, and the most critical action is to notify the Rapid Response Team for speedy diagnosis and treatment. The other actions are appropriate also but are not the priority.

A client presents to the emergency department reporting severe abdominal pain. On assessment, the nurse finds a bulging, pulsating mass in the abdomen. What action by the nurse is the priority? a. Auscultate for bowel sounds. b. Notify the provider immediately. c. Order an abdominal flat-plate x-ray. d. Palpate the mass and measure its size.

ANS: B This observation could indicate an abdominal aortic aneurysm, which could be life threatening and should never be palpated. The nurse notifies the provider at once. An x-ray may be indicated. Auscultation is part of assessment, but the nurse's priority action is to notify the provider

17. A nurse is teaching a client about warfarin (Coumadin). What assessment finding by the nurse indicates a possible barrier to self-management? a. Poor visual acuity b. Strict vegetarian c. Refusal to stop smoking d. Wants weight loss surgery

ANS: B Warfarin works by inhibiting the synthesis of vitamin K-dependent clotting factors. Foods high in vitamin K thus interfere with its action and need to be eaten in moderate, consistent amounts. A vegetarian may have trouble maintaining this diet. The nurse should explore this possibility with the client. The other options are not related.

A nurse is designing a community education program to meet the Healthy People 2020 objectives for nutrition and weight status. What information about these goals does the nurse use to plan this event? (Select all that apply.) a. Decrease the amount of fruit to 1.1 cups/1000 calories. b. Increase the amount of vegetables to 1.1 cups/1000 calories. c. Increase the number of adults at a healthy weight by 25%. d. Reduce the number of adults who are obese by 10%. e. Reduce the consumption of saturated fat by nearly 10%.

ANS: B, D, E Take Healthy People to BED

The nurse observes yellow staining in the sclera of eyes, soles of feet, and palms of hands. How should the nurse document these findings? a. Normal b. Erythema c. Jaundice d. Ecchymosis

ANS: C Jaundice is defined as the yellow staining of the skin, usually by bile pigments. Yellow staining is not a normal appearance of the skin. Erythema is redness that results from increased blood flow to the area. Ecchymosis is large, diffuse areas, usually black and blue, caused by hemorrhage of blood into the skin. DIF: Cognitive Level: Understand REF: p. 89 TOP: Integrated Process: Nursing Process: Assessment MSC: Area of Client Needs: Health Promotion and Maintenance

Where is the best place to observe for the presence of petechiae in dark-skinned individuals? a. Face b. Buttocks c. Oral mucosa d. Palms and soles

ANS: C Petechiae, small distinct pinpoint hemorrhages, are difficult to see in dark skin unless they are in the mouth or conjunctiva. DIF: Cognitive Level: Remember REF: p. 89 TOP: Integrated Process: Nursing Process: Assessment MSC: Area of Client Needs: Health Promotion and Maintenance: Techniques of Physical Assessment

Which is the most appropriate vision acuity test for a child who is in preschool? a. Cover test b. Ishihara test c. HOTV chart d. Snellen letter chart

ANS: C The HOTV test consists of a wall chart of these letters. The child is asked to point to a corresponding card when the examiner selects one of the letters on the chart. The cover test determines ocular alignment. The Ishihara test is used for the detection of color blindness. The Snellen letter chart is usually used for older children. DIF: Cognitive Level: Understand REF: p. 93 TOP: Integrated Process: Nursing Process: Planning MSC: Area of Client Needs: Health Promotion and Maintenance

9. An intubated client's oxygen saturation has dropped to 88%. What action by the nurse takes priority? a. Determine if the tube is kinked. b. Ensure all connections are patent. c. Listen to the client's lung sounds. d. Suction the endotracheal tube.

ANS: C When an intubated client shows signs of hypoxia, check for DOPE: displaced tube (most common cause), obstruction (often by secretions), pneumothorax, and equipment problems. The nurse listens for equal, bilateral breath sounds first to determine if the endotracheal tube is still correctly placed. If this assessment is normal, the nurse would follow the mnemonic and assess the patency of the tube and connections and perform suction.

*Which nursing diagnosis is more appropriate for a patient diagnosed with anorexia nervosa who restricts intake and is 20% below normal weight than for a 130-pound patient diagnosed with bulimia nervosa who purges?* a. Powerlessness b. Ineffective coping c. Disturbed body image d. Imbalanced nutrition: less than body requirements

ANS: D The patient with bulimia nervosa usually maintains a close to normal weight, whereas the patient with anorexia nervosa may approach starvation. The incorrect options may be appropriate for patients with either anorexia nervosa or bulimia nervosa.

5. Fill in the blank: If the nurse displaying dictator behavior is in a ____________ position such as a charge nurse this type of behavior is called ________ violence.

Superior, Vertical

The nurse will know that a client with head lice understands principles of pediculicides when she can discuss

The nurse will know that a client with head lice understands principles of pediculicides when she can discuss

Normal flora contained in the colon aid digestion and produce which nutrients? Select all that apply. 1) Vitamin A 2) Vitamin B 3) Vitamin C 4) Vitamin K 5) Iron 6) Zinc

Vitamin B, Vitamin K

A patient with a colostomy complains to the nurse, "I am having really bad odors coming from my pouch." To help control odor, which foods should the nurse advise him to consume? 1) White rice and toast 2) Tomatoes and dried fruit 3) Asparagus and melons 4) Yogurt and parsley

Yogurt and parsley Yogurt, cranberry juice, parsley, and buttermilk may help control odor. White rice and toast (also bananas and applesauce) help control diarrhea. Asparagus, peas, melons, and fish are known to cause odor. Tomatoes, pears, and dried fruit are high-fiber foods that might cause blockage in a patient with an ostomy.

a patient with PJP usually presents with which symptom? A. dyspnea, tachypnea, persistent dry cough, fever b. cough with copious thick sputum, fever, and dyspnea c. chest pain and difficulty swallowing D. fever, persistant cough and vomiting

a

A client presents at the clinic with intractable diarrhea for 2 weeks. The nurse would expect to administer what type of drug for the treatment of this condition? a. Opioids b. Laxatives c. Cathartics d. Bulk-forming agents

a. Opioids

When treating a patient experiencing nausea and vomiting with antiemetics, which is important for the nurse to consider? a. Patient safety is a concern, as drowsiness is a common side effect. b. Over-the-counter antiemetics are just as effective in relieving nausea. c. Sports drinks replace the essential ingredients lost by dehydration. d. The most effective antiemetic is determined by trial and error.

a. Patient safety is a concern, as drowsiness is a common side effect

the nurse is assisting the older adult diagnosed with a gastric ulcer to schedule her medication administration. what would be the most appropriate time for this patient to take her lansoprazole (Prevacid)? a. about 30 minutes before her morning meal b. at night before bed c. after fasting at least 2 hours d. 30 minutes after each meal

a. about 30 minutes before her morning meal

Discharge instructions for patient on antibiotics

advise patients to take anti-infectives for the full length of therapy

What is an advantage of the salad bowl tradition?

allowing individuals in the dominant culture to gain an appreciation of others cultures for their unique contributions to society.

The family of a neutropenic client reports the client "is not acting right." What action by the nurse is the priority? a. Ask the client about pain. b. Assess the client for infection. c. Delegate taking a set of vital signs. d. Look at today's laboratory results.

b. Assess the client for infection.

When teaching a patient who is taking twice-daily dosages of an aluminum antacid along with other medications, which instruction should the nurse provide? a. The other medications can be taken with the antacid, as long as it is with meals. b. The antacid should be taken at least 2 hours before or after the other medications. c. The antacid should be taken at least 4-6 hours apart from the other medications. d. The patient will not be able to take the antacid therapy at this time.

b. The antacid should be taken at least 2 hours before or after the other medications.

In addition to the use of multiple antibiotics, the nurse should anticipate which medication to be included in the patient's treatment of PUD due to H. pylori infection? a. Antacids b. H2-receptor inhibitors c. Bismuth compounds d. Vitamin E compounds

c. Bismuth compounds

A female nurse has been sexually assaulted as a teenager. She finds it difficult to work with patients who have undergone the same trauma. What is the most helpful response? a. Discussing these feeling with the nurse supervisor b. Requesting that these patients not be a part of her patient assignment c. Discussing these feelings with a mental health professional d. Accepting her role in providing unbiased, respectful and professional care to all patients

c. Discussing these feelings with a mental health professional

A client has received a bone marrow transplant and is waiting for engraftment. What actions by the nurse are most appropriate? (Select all that apply.) a. Not allowing any visitors until engraftment b. Limiting the protein in the client's diet c. Placing the client in protective precautions d. Teaching visitors appropriate hand hygiene e. Telling visitors not to bring live flowers or plants

c. Placing the client in protective precautions d. Teaching visitors appropriate hand hygiene e. Telling visitors not to bring live flowers or plants

A client has been treated for a deep vein thrombus and today presents to the clinic with petechiae. Laboratory results show a platelet count of 42,000/mm3. The nurse reviews the client's medication list to determine if the client is taking which drug? a. Enoxaparin (Lovenox) b. Salicylates (aspirin) c. Unfractionated heparin d. Warfarin (Coumadin)

c. Unfractionated heparin

6. What communication technique will make patients less likely to perceive communication from the nurse, as a personal attack? a. Using a firm, confident tone of voice b. Using encouraging words such as "Okay," and "Tell me more." c. Using "I" rather than "you" statements d. Keeping the conversation light

c. Using "I" rather than "you" statements

a patient with severe diarrhea has an order for diphenoxylate with atropine (lomotil). when assessing for therapeutic effects, which of the following will the nurse expect to find? a. increased bowel sounds b. decreased belching and flatus c. decrease in loose, watery stool d. decreased abdominal cramping

c. decrease in loose, watery stool

A client with autoimmune idiopathic thrombocytopenic purpura (ITP) has had a splenectomy and returned to the surgical unit 2 hours ago. The nurse assesses the client and finds the abdominal dressing saturated with blood. What action is most important? a. Preparing to administer a blood transfusion b. Reinforcing the dressing and documenting findings c. Removing the dressing and assessing the surgical site d. Taking a set of vital signs and notifying the surgeon

d. Taking a set of vital signs and notifying the surgeon

5. A client is being prepared for a mechanical embolectomy. What action by the nurse takes priority? a. Assess for contraindications to fibrinolytics. b. Ensure that informed consent is on the chart. c. Perform a full neurologic assessment. d. Review the client's medication lists.

ANS: B For this invasive procedure, the client needs to give informed consent. The nurse ensures that this is on the chart prior to the procedure beginning. Fibrinolytics are not used. A neurologic assessment and medication review are important, but the consent is the priority. DIF: Applying/Application REF: 938 KEY: Neurologic disorders| stroke| informed consent MSC: Integrated Process: Communication and Documentation NOT: Client Needs Category: Safe and Effective Care Environment: Management of Care

Which common childhood communicable disease may cause severe defects in the fetus when it occurs in its congenital form? a. Erythema infectiosum b. Rubeola c. Roseola d. Rubella

D Rubella causes teratogenic effects on the fetus. There is a low risk of fetal death to those in contact with children affected with fifth disease. Roseola and rubeola are not dangerous to the fetus.

A client has been admitted after sustaining a humerus fracture that occurred when picking up the family cat. What test result would the nurse correlate to this condition? a. Bence-Jones protein in urine b. Epstein-Barr virus: positive c. Hemoglobin: 18 mg/dL d. Red blood cell count: 8.2/mm3

a. Bence-Jones protein in urine

pancrelipase (Pancreaze) granules are ordered for a patient. which of the following will the nurse complete before administering the drug? SATA a. sprinkle the granules on a nonacidic food b. give the granules with or just before a meal c. mix the granules with orange or grapefruit juice d. ask the patient about an allergy to pork or pork products e. administer the granules followed by an antacid

a. sprinkle the granules on a nonacidic food b. give the granules with or just before a meal d. ask the patient about an allergy to pork or pork products

A client presents to the emergency department with severe nausea and vomiting following a case of food poisoning. Which antiemetic drug should concern the nurse if the client is taking quinidine? a. Metoclopramide (Reglan) b. Prochlorperazine (Compazine) c. Scopolamine (Transderm-Scop) d. Dronabinol (Marinol)

b. Prochlorperazine (Compazine)

When is healthcare considered culturally competent?

When health care providers and institutions are able to provide care for clients that meet the clients cultural needs.

What is cultural synergy?

When healthcare providers make a commitment to learn about other cultures and to immerse themselves in these cultures

which malignancy is most common in patients with HIV/AIDS a. non-hodgkins B cell lymphoma b. anal cancer c. primary brain cancer d. kaposi's sarcoma

d

HAART causes what effects? A. reversal of a patients antibody status B. decrease of the viral load C. increase of the viral load D. moe delectable HIV

b

what methods or agents are used to treat kaposi's sarcoma (Select all that apply) a. radiotherapy b. chemo c. antibiotics d. cryotherapy e. surgery

a, b, d

a. Decrease in cardiac output d. Increase in blood pressure e. Decrease in urine output

A nurse cares for a client with congestive heart failure who has a regular cardiac rhythm of 128 beats/min. For which physiologic alterations should the nurse assess? (Select all that apply.) a. Decrease in cardiac output b. Increase in cardiac output c. Decrease in blood pressure d. Increase in blood pressure e. Decrease in urine output f. Increase in urine output

Which is a clinical manifestation of acetaminophen poisoning? a. Hyperpyrexia b. Hepatic involvement c. Severe burning pain in stomach d. Drooling and inability to clear secretions

ANS: B Hepatic involvement is the third stage of acetaminophen poisoning. Hyperpyrexia is a severe elevation in body temperature and is not related to acetaminophen poisoning. Acetaminophen does not cause burning pain in stomach or pose an airway threat.

6. Which drug is usually the best choice for patient-controlled analgesia (PCA) for a child in the immediate postoperative period? a. Codeine b. Morphine c. Methadone d. Meperidine

ANS: B The most commonly prescribed medications for PCA are morphine, hydromorphone, and fentanyl. Parenteral use of codeine is not recommended. Methadone is not available in parenteral form in the United States. Meperidine is not used for continuous and extended pain relief. DIF: Cognitive Level: Remember REF: p. 129 TOP: Integrated Process: Nursing Process: Implementation MSC: Area of Client Needs: Physiologic Integrity

a 24-year old patient has been taking sulfasalazine (Azulfidine) for IBS and complains to the nurse that he wants to stop taking the drug because of the nausea, headaches, and abdominal pain it causes. what would the nurses best recommendation be for this patient? a. the drug is absolutely necessary, even with the adverse effects b. talk to the HCP about dividing the doses throughout the day c. stop taking the drug and see if the symptoms of the IBS have resolved d. take an antidiarrheal drug such as loperamide (Imodium) along with the sulfasalazine

b. talk to the HCP about dividing the doses throughout the day

A client has been prescribed trimethoprim-sulfamethoxazole (Septra) for treatment of a urinary tract infection. Which comments, made by the client, would the nurse discuss with the prescriber before allowing the client to leave the clinic?

2. "I forgot to take my potassium supplement today."(Potassium supplements should not be taken during therapy unless directed by the health care provider.) 3. "Is it okay to take this with my warfarin?" (Sulfa drugs may enhance the effects of oral anticoagulants.)

A client has been diagnosed with multidrug-resistant tuberculosis, and drug therapy has been initiated. The nurse evaluates that medication education has been effective when the patient says, "I can expect to take this medication for up to _____ months."

24

What clinical finding in a postmenopausal client with urethritis does the nurse attribute to low estrogen levels? 1. The urinalysis indicates pyuria. 2. The urethral culture is positive for bacteria. 3. The urinalysis indicates presence of bacteria. 4. A pelvic examination shows tissue changes.

4. A pelvic examination shows tissue changes. A pelvic examination of a postmenopausal client shows tissue changes due to low estrogen. The client with urethritis does not have pyuria or white blood cells (WBCs) in the urine. The urethral culture and urinalysis is usually negative for bacteria. These clients may have improvement in their urethral symptoms with the use of estrogen vaginal cream.

what is the most important means of preventing HIV spread? A. engineering B. education C. isolation D. counseling

b

which treatments are intended to boost the immune system? a. protease inhibitors b. hematopoietic growth factors c. lymphocyte transfusion d. interleukin-2 infusion

d

The nurse is caring for a client receiving gentamicin IV. The nurse would observe for adverse effects of

ototoxicity.

Tetracycline side effects

- affects vaginal, oral, and intestinal flora and cause superinfections - nausea, vomiting, epigastric burning, diarrhea

A nurse is evaluating a patient's learning regarding nutrition. Which behavior reflects the highest level of learning in the cognitive domain? 1. Modifies favorite recipes by eliminating foods that have to be avoided 2. Evaluates the benefits associated with avoidance of certain foods 3. States why a mother's diet may affect breast-feeding 4. Identifies a list of foods to be avoided

2. This is an appropriate example of learning on the evaluation level and is the highest level of learning of the six levels of learning in the cognitive domain.

shingles results from VZV leaving the body by which route? a. mucous membrane b. pulmonary space c. body fluids and other tissues d. bone marrow

c

Fluvicin education

- given by mouth that is indicated for mycoses of the hair, skin, and nails that have not responded to conventional topical preparations - Medication given orally, increased fluids interferes with renal function - take full course of meds

The nurse is instructing a patient about performing home testing for fecal occult blood. The nurse can conclude that learning occurs if the patient says, "For 3 days prior to testing, I should avoid eating 1) beef. 2) milk. 3) eggs. 4) oatmeal.

1) beef. The nurse should instruct the patient to avoid red meat, chicken, fish, horseradish, and certain raw fruits and vegetables for 3 days prior to fecal occult blood testing.

A young child has been diagnosed with atopic dermatitis. The nurse would review the patient's history for which findings?

1. Family history of asthma 2. Allergy to soaps and lotions

A nurse recognizes which physiologic responses as a manifestation of pain in a neonate? (Select all that apply.) a. Decreased respirations b. Diaphoresis c. Decreased SaO2 d. Decreased blood pressure e. Increased heart rate

ANS: B, C, E The physiologic responses that indicate pain in neonates are increased heart rate, increased blood pressure, rapid, shallow respirations, decreased arterial oxygen saturation (SaO2), pallor or flushing, diaphoresis, and palmar sweating. DIF: Cognitive Level: Apply REF: p. 120 TOP: Integrated Process: Nursing Process: Assessment MSC: Area of Client Needs: Physiologic Integrity

Which screening tests should the school nurse perform for the adolescent? (Select all that apply.) a. Glucose b. Vision c. Hearing d. Cholesterol e. Scoliosis

ANS: B, C, E The school nurse should perform vision, hearing, and scoliosis screening tests according to the school district's required schedule. Glucose and cholesterol screening would be performed in the medical clinic setting. DIF: Cognitive Level: Apply REF: p. 461 TOP: Integrated Process: Nursing Process: Implementation MSC: Area of Client Needs: Health Promotion and Maintenance

A nursing instructor is evaluating a student nurse's knowledge. Which student behavior indicates that learning has occurred in the highest level of learning in the cognitive domain? 1. Identifies the expected properties of urine 2. Explains the importance of producing urine 3. Recognizes when something is contaminated 4. Interprets laboratory results of diagnostic urine testing

4. This is the highest level of learning in the cognitive domain of the choices offered. Interpretation of laboratory results of urine testing reflects learning on the analysis level, which is the fourth of six levels of learning in the cognitive domain.

A nurse cares for a client who has a deep wound that is being treated with a wet-to-damp dressing. Which intervention should the nurse include in this client's plan of care? a.Change the dressing every 6 hours. b.Assess the wound bed once a day. c.Change the dressing when it is saturated. d.Contact the provider when the dressing leaks.

ANS: A Wet-to-damp dressings are changed every 4 to 6 hours to provide maximum débridement. The wound should be assessed each time the dressing is changed. Dry gauze dressings should be changed when the outer layer becomes saturated. Synthetic dressings can be left in place for extended periods of time but need to be changed if the seal breaks and the exudate leaks.

A client has a gastrointestinal hemorrhage and is prescribed two units of packed red blood cells. What actions should the nurse perform prior to hanging the blood? (Select all that apply.) a. Ask a second nurse to double-check the blood. b. Prime the IV tubing with normal saline. c. Prime the IV tubing with dextrose in water. d. Take and record a set of vital signs. e. Teach the client about reaction manifestations.

ANS: A, B, D, E Prior to starting a blood transfusion, the nurse asks another nurse to double-check the blood (and client identity), primes the IV tubing with normal saline, takes and records a baseline set of vital signs, and teaches the client about manifestations to report. The IV tubing is not primed with dextrose in water.

A child with cyanide poisoning has been admitted to the emergency department. What antidote does the nurse anticipate being prescribed for the child? a. Atropine b. Glucagon c. Amyl nitrate d. Naloxone (Narcan)

ANS: C Amyl nitrate is the antidote for cyanide poisoning. Atropine is an antidote for organophosphate poisoning, glucagon is an antidote for a beta-blocker poisoning, and naloxone (Narcan) is an antidote for an opioid poisoning.

What is an important nursing consideration when a child is hospitalized for chelation therapy to treat lead poisoning? a. Maintain bed rest. b. Maintain isolation precautions. c. Keep an accurate record of intake and output. d. Institute measures to prevent skeletal fracture.

ANS: C The iron chelates are excreted though the kidneys. Adequate hydration is essential. Periodic measurement of renal function is done. Bed rest is not necessary. Often the chelation therapy is done on an outpatient basis. Chelation therapy is not infectious or dangerous. Isolation is not indicated. Skeletal weakness does not result from high levels of lead.

A nurse teaches a client who has very dry skin. Which statement should the nurse include in this client's education? a."Use lots of moisturizer several times a day to minimize dryness." b."Take a cold shower instead of soaking in the bathtub." c."Use antimicrobial soap to avoid infection of cracked skin." d."After you bathe, put lotion on before your skin is totally dry."

ANS: D The client should bathe in warm water for at least 20 minutes and then apply lotion immediately because this will keep the moisture in the skin. Just using moisturizer will not be as helpful because the moisturizer is not what rehydrates the skin; it is the water. Bathing in warm water will rehydrate skin more effectively than a cold shower, and antimicrobial soaps are actually more drying than other kinds of soap.

8. A nurse assesses a client after administering isosorbide mononitrate (Imdur). The client reports a headache. Which action should the nurse take? a. Initiate oxygen therapy. b. Hold the next dose of Imdur. c. Instruct the client to drink water. d. Administer PRN acetaminophen.

ANS: D The vasodilating effects of isosorbide mononitrate frequently cause clients to have headaches during the initial period of therapy. Clients should be told about this side effect and encouraged to take the medication with food. Some clients obtain relief with mild analgesics, such as acetaminophen. The client's headache is not related to hypoxia or dehydration; therefore, these interventions would not help. The client needs to take the medication as prescribed to prevent angina; the medication should not be held. DIF: Applying/Application REF: 686 KEY: Heart failure| nitroglycerin/nitrates| medication| pharmacologic pain management MSC: Integrated Process: Nursing Process: Implementation NOT: Client Needs Category: Physiological Integrity: Pharmacological and Parenteral Therapies

The nurse is reviewing the lipid panel of a male client who has atherosclerosis. Which finding is most concerning? a. Cholesterol: 126 mg/dL b. High-density lipoprotein cholesterol (HDL-C): 48 mg/dL c. Low-density lipoprotein cholesterol (LDL-C): 122 mg/dL d. Triglycerides: 198 mg/dL

ANS: D Triglycerides in men should be below 160 mg/dL. The other values are appropriate for adult males. DIF: Remembering/Knowledge REF: 708

A client is in the bariatric clinic 1 month after having gastric bypass surgery. The client is crying and says "I didn't know it would be this hard to live like this." What response by the nurse is best? a. Assess the client's coping and support systems. b. Inform the client that things will get easier. c. Re-educate the client on needed dietary changes. d. Tell the client lifestyle changes are always hard.

a. Assess the client's coping and support systems.

37. A 55-year-old woman has a body mass index of 35. She is at high risk for which one(s) of the following? (Select all that apply.) A Diabetes mellitus B Pulmonary disease C Hypertension D Neurologic disorders E Coronary artery disease F Some cancers of the breast

a,c,e,f A 55-year-old woman with a body mass index of 35 is at high risk for diabetes mellitus, hypertension, coronary artery disease, and some cancers of the breast and reproductive organs.

31. The recommended medication for the treatment of chlamydia would be A doxycycline. B podofilox. C acyclovir. D penicillin.

a Doxycycline is effective for treating chlamydia but should be avoided if the woman is pregnant. Penicillin is not recommended for chlamydia; it is the preferred medication for syphilis. Podofilox is a recommended treatment for nonpregnant women diagnosed with human papillomavirus infection. Acyclovir is used to treat genital herpes simplex virus infection.

32. When teaching adolescents about sexually transmitted diseases, it is important to emphasize prompt treatment when symptoms first appear to prevent complications. One example that may be used is that untreated gonorrhea may be associated with A infertility. B skin eruptions. C paralysis. D psychosis.

a Gonorrhea is associated with pelvic inflammatory disease, which increases the risk of tubal scarring and can result in infertility. The other choices are associated with syphilis.

4. Which one of the following is correct concerning the performance of a Pap test? A The woman should not douche, use vaginal medications, or have intercourse for at least 48 hours before the test. B It should be performed once a year, beginning with the onset of puberty. C A lubricant such as petroleum jelly should be used to ease speculum insertion. D The specimen for the Pap test should be obtained after a specimen is collected to test for cervical infection.

a Pap tests are performed annually for sexually active women or by age 21, especially if risk factors for cervical cancer or reproductive tract infections are present. Pap tests may be performed every 3 years in low-risk women between the ages of 21 and 29. No lubricant other than warm water should be used because accuracy of the test can be affected. The cytologic specimen should be obtained first.

A student nurse is helping a registered nurse with a blood transfusion. Which actions by the student are most appropriate? (Select all that apply.) a. Hanging the blood product using normal saline and a filtered tubing set b. Taking a full set of vital signs prior to starting the blood transfusion c. Telling the client someone will remain at the bedside for the first 5 minutes d. Using gloves to start the client's IV if needed and to handle the blood product e. Verifying the client's identity, and checking blood compatibility and expiration time

a. Hanging the blood product using normal saline and a filtered tubing set b. Taking a full set of vital signs prior to starting the blood transfusion d. Using gloves to start the client's IV if needed and to handle the blood product

A nurse is conducting a health history assessment of a patient and determines which findings as risk factors for development of peptic ulcer disease (PUD)? (Select all that apply.) a. Smoking cigarettes b. Having blood type A c. Drinking caffeinated beverages d. Using NSAIDs e. Having a family history of PUD

a. Smoking cigarettes c. Drinking caffeinated beverages d. Using NSAIDs e. Having a family history of PUD

7. A 49-year-old woman has come to the nurse practitioner for an examination, stating, "I haven't been for a physical examination since my last child was born 20 years ago and thought I should one since I have gone through menopause." When taking the health history from this woman, it is important to include questions concerning A problems that may have occurred with her labor and birth. B family history of heart disease. C history of childhood immunizations. D history of infertility or problems conceiving.

b A family history of heart disease is especially important when the woman is postmenopausal because estrogen, which provides some protection against coronary artery disease, decreases after menopause and obesity may increase. If there is a family history of heart disease, or other signs of heart disease, the woman needs further screening.

A 48-year-old woman has just been diagnosed with breast cancer in her right breast, and a simple mastectomy has been recommended. The nurse assesses the patient teaching on the surgery to be effective when the woman states: A "They are going to take only the tumor out and a couple of the lymph nodes." B "They will remove my entire right breast." C "They are going to take the right breast, some nodes, and even some chest muscle out." D "They will be removing only some lymph nodes."

b A simple mastectomy involves the removal of the entire breast. Axillary dissection is omitted. A lumpectomy is the removal of only the tumor. A modified radical mastectomy involves the removal of the breast tissue, axillary nodes, and some chest muscles. A sentinel lymph node biopsy is a technique to remove a few key lymph nodes to evaluate the spread of the cancer.

6. Select all the statements the nurse should say to family members about a loved one who is in denial about his cancer diagnosis that would be the most effective communication techniques. a. Use the hard sell to reinforce the diagnosis b. Let him talk about his feelings without interrupting c. If he yells at you, you need to yell back to keep him oriented d. Allow for periods of salience so that he can organize his thoughts e. Try to understand that denial is a protective mechanism f. Do not reinforce his denial by agreeing with him

b. Let him talk about his feelings without interrupting d. Allow for periods of salience so that he can organize his thoughts e. Try to understand that denial is a protective mechanism f. Do not reinforce his denial by agreeing with him

A client has lice infestation of the eyelids. What instruction should the nurse provide?

"Apply a thin coat of petroleum jelly to your eyelashes once a day for a week."

A client has been prescribed tretinoin (Avita) for treatment of acne. Which medication information should the nurse provide?

1. "It will take several weeks for you to see improvement in your skin." "Continue to take the tetracycline previously prescribed for your cystic acne."

The nurse would expect that most children would be using sentences of six to eight words by age: A. 18 months. B. 24 months. C. 3 years. D. 5 years.

D. 4.5-5 years Children ages 4 to 5 years use sentences of four or five words. An 18-month-old child has a vocabulary of approximately 10 words. A 24-month-old child uses two- or three-word phrases. A 3-year-old child uses sentences of three or four complete words.

The client has scabies. The nurse has taught the client about safety in using topical scabicide medications and evaluates that teaching has been effective when the client makes which statement?

"I must avoid putting this lotion on my face."

The mother of a client with head lice has completed the applications of topical medication. What is the best instruction by the nurse at this time?

"Remove all nits from the hair shaft with a nit comb or a fine-tooth comb."

The nurse works in infection control and teaches a class to staff nurses about the ways that resistance to antibiotics can occur. The nurse evaluates that learning has occurred when the nurses make which statement?

"Resistance to antibiotics can occur by the common use of them for nosocomial infections."

A patient has been prescribed ultraviolet (UV) light therapy for a skin condition. The patient says, "This treatment is so expensive, I think I will go to the tanning salon instead." The nurse would provide which information?

"The light therapy you require should be closely monitored."

The client receives topical glucocorticoids for the treatment of dermatitis. The nurse has completed medication education and evaluates that learning has occurred when the client makes which statement?

"Use of this lotion is really a lot safer and more effective than a pill."

The client calls the clinic and is frantic that her two children have been sent home from school with head lice. She has treated their scalps but does not know what else to do. What will be the best teaching by the nurse?

"Wash the bed linens and clothing that have come into contact with the children."

The client is recovering from a severe sunburn. What will the best teaching by the nurse include?

"Apply a sunscreen when you are going to be in the sun."

The physician orders penicillin for a female client who has a sinus infection. What is a priority question to ask the client prior to administering the medication?

"Are you taking birth control pills?"

The mother of a young child calls the clinic and tells the nurse that she has just discovered head lice in her daughter's hair. What is the best instruction by the nurse?

"Follow the package directions exactly on the permethrin lotion from the pharmacy."

What is culture?

May be seen as a group's acceptance of a set of attitudes, ideologies, values, beliefs, and behaviors that influence the way the members of the group express themselves.

The client receives gentamicin (Garamycin) intravenously (IV) in the clinical setting. What is a priority nursing action?

Monitor the client for hearing loss.

A client comes to the emergency department with a fever of 104°F. The nurse anticipates which actions to help identify the correct antibiotic?

Obtaining a sterile urine specimen.

The physician orders lindane (Kwell) for nursing home clients who have contracted head lice. The nurse will collaborate with the physician when one of the clients has which medical diagnosis?

Seizures

Side effects of Cipro

Serious adverse effects are uncommon - nausea, vomiting, diarrhea - phototoxicity - headache, dizziness

A client has a urinary catheter and continuous bladder irrigation after a transurethral resection of the prostate this morning. The amount of bladder irrigation solution that has infused over the past 12 hours is 1000 mL. The amount of fluid in the urinary drainage bag is 1725 mL. The nurse records that the client has had ____ mL urinary output in the past 12 hours. (Ignatavicius & Workman, p.1505)

725 mL (Ignatavicius & Workman, p.1505)

A nurse and a registered dietitian are assessing clients for partial parenteral nutrition (PPN). For which client would the nurse suggest another route of providing nutrition? a. Client with congestive heart failure b. Older client with dementia c. Client who has multiorgan failure d. Client who is post gastric resection

a. Client with congestive heart failure

During the preschool period, the emphasis of injury prevention should be placed on: a. Constant vigilance and protection. b. Punishment for unsafe behaviors. c. Education for safety and potential hazards. d. Limitation of physical activities.

C Education for safety and potential hazards is appropriate for preschoolers because they can begin to understand dangers. Constant vigilance and protection is not practical at this age since preschoolers are becoming more independent. Punishment may make children scared of trying new things. Limitation of physical activities is not appropriate.

1. Select all of the following which interfere with the encoding of a message from a sender to a receiver. a. Convoluted message b. Clear speech c. Monotone voice d. Use of jargon e. Understanding the information f. Preoccupation

a. Convoluted message c. Monotone voice d. Use of jargon f. Preoccupation

Discharge planning for the client prescribed tetracycline will include which instruction?

Do not take the medication with milk. Tetracycline effectiveness can be decreased by using milk products. Antacids can decrease the effectiveness of tetracycline. Iron can decrease the effectiveness of tetracycline. It is not necessary to decrease vitamins.

A nurse caring for a client with sickle cell disease (SCD) reviews the client's laboratory work. Which finding should the nurse report to the provider? a. Creatinine: 2.9 mg/dL b. Hematocrit: 30% c. Sodium: 147 mEq/L d. White blood cell count: 12,000/mm3

a. Creatinine: 2.9 mg/dL

A nurse working with clients with sickle cell disease (SCD) teaches about self-management to prevent exacerbations and sickle cell crises. What factors should clients be taught to avoid? (Select all that apply.) a. Dehydration b. Exercise c. Extreme stress d. High altitudes e. Pregnancy

a. Dehydration c. Extreme stress d. High altitudes e. Pregnancy

3. Identify the most important factor in the initial negotiation of a contract. a. Discussion of the important issues b. Posturing and showmanship c. Resolution of key conflicts d. Lack of willingness to negotiate

a. Discussion of the important issues

Which drug should the nurse prepare to administer to prevent constipation in a client who had a surgical procedure? a. Docusate sodium (Colace) b. Prochlorperazine (Compazine) c. Loperamide (Imodium) d. Promethazine (Phenergan)

a. Docusate sodium (Colace)

The client has MRSA and receives vancomycin (Vancocin) intravenously (IV). The nurse assesses an upper body rash and decreased urine output. What is the nurse's priority action?

Hold the next dose of vancomycin (Vancocin) and notify the physician.

Why is teaching & Learning important for nurses?

Florence Nightingales Role Patients are Active Participants in Healthcare Decisions Hospital stays are short Healthcare + Expensive Purpose of Teaching and Learning + Empower *Perform Self Care *Make informed Decisions about healthcare options

What are the food interactions with cyclosporine

Food decreases the absorption of the drug

The nurse is assessing a patient who underwent bowel resection 2 days ago. As she auscultates the patient's abdomen, she notes low-pitched, infrequent bowel sounds. How should she document this finding? 1) Hyperactive bowel sounds 2) Abdominal bruit sounds 3) Normal bowel sounds 4) Hypoactive bowel sounds

Hypoactive bowel sounds Hypoactive bowel sounds are low pitched, infrequent, and quiet. An abdominal bruit is a hollow, blowing sound found over an artery, such as the iliac artery. Normal bowel sounds are high pitched, with approximately 5 to 35 gurgles occurring every minute. Hyperactive bowel sounds are very high pitched and more frequent than normal bowel sounds.

The nurse is caring for a client with leukemia who has the priority problem of fatigue. What action by the client best indicates that an important goal for this problem has been met? a. Doing activities of daily living (ADLs) using rest periods b. Helping plan a daily activity schedule c. Requesting a sleeping pill at night d. Telling visitors to leave when fatigued

a. Doing activities of daily living (ADLs) using rest periods

A student nurse is learning about blood transfusion compatibilities. What information does this include? (Select all that apply.) a. Donor blood type A can donate to recipient blood type AB. b. Donor blood type B can donate to recipient blood type O. c. Donor blood type AB can donate to anyone. d. Donor blood type O can donate to anyone. e. Donor blood type A can donate to recipient blood type B.

a. Donor blood type A can donate to recipient blood type AB. d. Donor blood type O can donate to anyone.

A client has Crohn's disease. What type of anemia is this client most at risk for developing? a. Folic acid deficiency b. Fanconi's anemia c. Hemolytic anemia d. Vitamin B12 anemia

a. Folic acid deficiency

the nurse is teaching a patient about preventing infection through sex. which statement indicates effective teaching? a. latex condom with spermicide proves the best protection b. mutually monogamous sex with a non infected partner will best prevent HIV c. contraceptive methods like implants and injections are recommended to prevent HIV transmission d. if my partner and i are both HIV positive, unprotected sex is permitted

b

the patient with HIV/AIDS appears emaciated and has diarrhea, anorexia, mouth lesions, and peristent weight loss. what condition does the nurse suspect this patient is developing? a. AIDS dementia B. AIDS wasting syndrome C. AIDS GI opportunistic infection D. AIDS candidiasis opportunistic infection

b

which statements are true about immunodeficiency? (Select all that apply) A. it causes a decrease in the patients risk for infection B. it may be acquired or congential C. it occurs when a persons body cannot recognize antigens D. it is the same as autoimmunity E. it may cause varied reactions from mild, localized health problems to total immune system failure

b, c, e

which immune function abnormalities are a result of HIV infection? (Select all that apply) A. lymphocytosis B. CD4+ depletion C. increased CD8+ activity D. long macrophage life span E. lymphocytopenia

b, e

A client admitted for sickle cell crisis is distraught after learning her child also has the disease. What response by the nurse is best? a. "Both you and the father are equally responsible for passing it on." b. "I can see you are upset. I can stay here with you a while if you like." c. "It's not your fault; there is no way to know who will have this disease." d. "There are many good treatments for sickle cell disease these days."

b. "I can see you are upset. I can stay here with you a while if you like."

Lilly, a single mother of 4, comes to the crisis center 24 hours after a fire in which all the houses within a 1-block area were wiped out. All of Lilly's household goods and clothing were lost. Lilly has no other family in the area. Her efforts to mobilize assistance have been disorganized, and she is still without shelter. She is distraught and confused. You assess the situation as: a. A maturational crisis b. An adventitious crisis c. A crisis of confidence d. An existential crisis

b. An adventitious crisis

A client is awaiting bariatric surgery in the morning. What action by the nurse is most important? a. Answering questions the client has about surgery b. Beginning venous thromboembolism prophylaxis c. Informing the client that he or she will be out of bed tomorrow d. Teaching the client about needed dietary changes

b. Beginning venous thromboembolism prophylaxis

A client is receiving bolus feedings through a Dobhoff tube. What action by the nurse is most important? a. Auscultate lung sounds after each feeding. b. Check tube placement before each feeding. c. Check tube placement every 8 hours. d. Weigh the client daily on the same scale.

b. Check tube placement before each feeding.

A nurse is reviewing laboratory values for several clients. Which value causes the nurse to conduct nutritional assessments as a priority? a. Albumin: 3.5 g/dL b. Cholesterol: 142 mg/dL c. Hemoglobin: 9.8 mg/dL d. Prealbumin: 28 mg/dL

b. Cholesterol: 142 mg/dL A cholesterol level below 160 mg/dL is a possible indicator of malnutrition

A nurse in a hematology clinic is working with four clients who have polycythemia vera. Which client should the nurse see first? a. Client with a blood pressure of 180/98 mm Hg b. Client who reports shortness of breath c. Client who reports calf tenderness and swelling d. Client with a swollen and painful left great toe

b. Client who reports shortness of breath

A client with chronic anemia has had many blood transfusions. What medications does the nurse anticipate teaching the client about adding to the regimen? (Select all that apply.) a. Azacitidine (Vidaza) b. Darbepoetin alfa (Aranesp) c. Decitabine (Dacogen) d. Epoetin alfa (Epogen) e. Methylprednisolone (Solu-Medrol)

b. Darbepoetin alfa (Aranesp) d. Epoetin alfa (Epogen)

A client receiving a blood transfusion develops anxiety and low back pain. After stopping the transfusion, what action by the nurse is most important? a. Documenting the events in the client's medical record b. Double-checking the client and blood product identification c. Placing the client on strict bedrest until the pain subsides d. Reviewing the client's medical record for known allergies

b. Double-checking the client and blood product identification

A morbidly obese client is admitted to a community hospital that does not typically care for bariatric-sized clients. What action by the nurse is most appropriate? a. Assess the client's readiness to make lifestyle changes. b. Ensure adequate staff when moving the client. c. Leave siderails down to prevent pressure ulcers. d. Reinforce the need to be sensitive to the client.

b. Ensure adequate staff when moving the client.

A nurse is preparing to administer a blood transfusion. What action is most important? a. Correctly identifying client using two identifiers b. Ensuring informed consent is obtained if required c. Hanging the blood product with Ringer's lactate d. Staying with the client for the entire transfusion

b. Ensuring informed consent is obtained if required

A nurse is caring for a morbidly obese client. What comfort measure is most important for the nurse to delegate to the unlicensed assistive personnel (UAP)? a. Designating "quiet time" so the client can rest b. Ensuring siderails are not causing excess pressure c. Providing oral care before and after meals and snacks d. Relaying any reports of pain to the registered nurse

b. Ensuring siderails are not causing excess pressure

A patient on chemotherapy is receiving ondansetron (Zofran) for treatment of nausea. The nurse will instruct the patient to watch for which adverse effect from this drug? a. Hiccups b. Headache c. Dry mouth d. Blurred vision

b. Headache

A client has Hodgkin's lymphoma, Ann Arbor stage Ib. For what manifestations should the nurse assess the client? (Select all that apply.) a. Headaches b. Night sweats c. Persistent fever d. Urinary frequency e. Weight loss

b. Night sweats c. Persistent fever e. Weight loss

A client has a serum ferritin level of 8 ng/mL and microcytic red blood cells. What action by the nurse is best? a. Encourage high-protein foods. b. Perform a Hemoccult test on the client's stools. c. Offer frequent oral care. d. Prepare to administer cobalamin (vitamin B12).

b. Perform a Hemoccult test on the client's stools.

Influenza and Relenza

reported to shorten the normal 7-day duration of influenza symptoms to 5 days

What are cultural values?

shared beliefs about desirable end states or modes of conduct in a given culture

Contraindications in live vaccines

some attenuated vaccines cause a mild or subclinical case of the disease

Time frames for Arlalen

start 2 weeks before travel and continuing 4-6 weeks following retun

Treatment of tuberculosis usually involves

the use of two or more drugs at the same time. : Multidrug therapy for 6-12 months is the usual pharmacotherapy for tuberculosis. Surgery is not the treatment. It is not necessary to keep the client in the hospital. Use of a single drug is not usual.

What is Cyclosporine used for?

used to prevent rejection in organ transplants

Viral Hepatitis

• A, B, C types are primary • Cause inflammation, necrosis of liver cells

Treatment Failures

• Common with antiretroviral therapy - Patient nontolerance of adverse effects - Patient nonadherence to complex regimen - Emergence of resistant strains - Genetic variability • Therapy always changing • Stay current with latest treatments

Pharmacotherapy of Herpesvirus Infections

• Family of DNA viruses; causes repeated blister-like lesions on the skin, genitals, and other mucosal surfaces • Antiviral drugs lower frequency of acute episodes and diminish intensity of acute disease - Relieve acute symptoms, prevent recurrences - Do not provide cure

Highly Active Antiretroviral Therapy (HAART)

• Five drug classes used in various combinations - Nucleotide reverse transcriptase inhibitor (NtRTI) - Nonnucleoside reverse transcriptase inhibitor (NNRTI) - Protease inhibitor (PI) - Entry inhibitors (includes fusion inhibitors and CCR5 antagonists) - Integrase inhibitors and other miscellaneous antivirals

Phases of HIV Therapy

• Latent phase—Virus lies dormant - People often unaware they have HIV • Once diagnosis confirmed, decision made about starting or delaying treatment • Current protocols: Defer treatment in asymptomatic adults who have CD4 counts above 350 cells/mcL • Therapy is initiated when CD4 is under 200 cells/mcL or symptoms appear

The physician orders cefepime (Maxipime) for a client. What is a priority question for the nurse to ask the client prior to administration of this drug?

"Are you allergic to penicillin?"

A teenager is taking isotretinoin (Accutane) for treatment of severe acne. The nurse has completed medication education with her mother and evaluates additional learning is required when the mother makes which statement?

3. "At least I do not need to worry about her self-concept now."

During assessment of a client, what finding does the nurse associate with the presence of kidney stones? 1. Oliguria 2. Flank pain 3. Dysuria 4. Flank pain extending to the scrotum

2. Flank pain The major manifestation of kidney stones is renal colic, which begins suddenly and is described as "unbearable." Flank pain indicates that the stone is in the kidney or the upper ureter. Oliguria, or scant urine output, indicates an obstruction at the bladder neck or urethra. Dysuria and frequency occur when the stone reaches the bladder and causes irritation. Flank pain extending to the scrotum suggests that the stones are in the ureters or bladder. The pain is most intense when the stone is moving or when the ureter is obstructed.

A nurse is assessing a patient's readiness to learn about smoking cessation. Which patient factor does the nurse consider is most important when determining if a teaching program is needed by the patient? 1. Previous experience 2. Perceived need 3. Expectations 4. Flexibility

2. Readiness to learn and motivation, which are closely tied together, are the two most important factors contributing to the success of any learning program. The learner must recognize that the learning need exists and that the material to be learned is valuable.

What complication may the nurse expect in an older client with an indwelling catheter for urinary continence? 1. Skin breakdown 2. Risk for infection 3. Damage to tissues 4. Urine retention

2. Risk for infection An indwelling catheter increases the risk for infection in a client. An indwelling catheter can help protect the skin and reduce the risk of skin breakdown caused by urinary incontinence. Applied devices, such as intravaginal pessaries for women and penile clamps for men, can cause tissue damage. An indwelling catheter completely drains the bladder, so the client does not have urinary retention.

A client who has diabetes mellitus is diagnosed with tuberculosis and has been prescribed multiple-drug therapy. What instruction should the nurse provide to this client?

2. Test your blood glucose more frequently while on these medications.(These medications may cause hyperglycemia. The client should monitor blood glucose levels more closely.) 3. If your blood glucose levels elevate consistently, contact us.(Constant elevation of blood glucose levels may warrant alteration in medication therapy for diabetes.)

A client is prescribed prophylactic immunotherapy with intravesical instillation of bacille Calmette-Guérin (BCG) to prevent recurrence of bladder tumor. What does the nurse inform the client about this therapy? 1. The procedure is done in an inpatient setting. 2. The BCG virus compound is allowed to dwell in the bladder for 2 hours. 3. Flush the toilet three times after use. 4. Dispose of clothing that comes in contact with urine in 24 hours.

2. The BCG virus compound is allowed to dwell in the bladder for 2 hours. The BCG virus compound is allowed to dwell in the bladder for 2 hours. Live virus will be excreted when the client urinates. The instillation procedure is done in an outpatient setting. The client must not share the toilet with other household members for 24 hours after instillation; the toilet must be flushed and cleaned with a solution of 10% liquid bleach. The client must wash all clothing that has come into contact with urine during the 24 hours after instillation separately with 10% liquid bleach.

A nurse is teaching a preschool-age child. What teaching method is most appropriate for the nurse to use when teaching a child in this age group? 1. Demonstrations 2. Coloring books 3. Small groups 4. Videos

2. This is the best approach because it requires preschoolers to be active participants in their own learning. In addition, the child has a product to take home and be proud of, it reduces anxiety associated with learning because coloring is an activity most preschoolers are familiar with, and it is within a preschooler's cognitive level.

The nurse in the urology clinic is providing teaching for a female client with cystitis. Which instructions does the nurse include in the teaching plan? Select all that apply. 1. Cleanse the perineum from back to front after using the bathroom. 2. Try to take in 64 ounces of fluid each day. 3. Be sure to complete the full course of antibiotics. 4. If urine remains cloudy, call the clinic. 5. Expect some flank discomfort until the antibiotic has worked.

2. Try to take in 64 ounces of fluid each day. 3. Be sure to complete the full course of antibiotics. 4. If urine remains cloudy, call the clinic. Between 64 and 100 ounces (2-3 liters) of fluid should be taken daily to dilute bacteria and prevent infection. Not completing the course of antibiotics could suppress the bacteria, but would not destroy all bacteria, causing the infection to resurface. For persistent symptoms of infection, the client should contact the provider. The perineal area should be cleansed from front to back or "clean to dirty" to prevent infection. Cystitis produces suprapubic symptoms; flank pain occurs with infection or inflammation of the kidney.

The nurse is caring for a client who has pyuria. What does the urinalysis reveal in this client? 1. Kidney stones. 2. White blood cells in the urine. 3. Red blood cells in the urine. 4. Heavy bacteria in the urine.

2. White blood cells in the urine. Urinalysis showing pyuria means that the client has white blood cells in the urine without a large number of bacteria.

Which is a key treatment intervention for the patient admitted with diverticulitis? 1) Antacid 2) Antidiarrheal agent 3) Antibiotic therapy 4) NSAIDs

3) Antibiotic therapy A key treatment for diverticulitis (an infected diverticulum) is antibiotic therapy; if antibiotic therapy is ineffective, surgery may be necessary. Antacids, antidiarrheal agents, and NSAIDs are not indicated for treatment of diverticulitis.

The nurse is instructing an older adult female client about interventions to decrease the risk for cystitis. Which client comment indicates that the teaching was effective? 1. "I must avoid drinking carbonated beverages." 2. "I need to douche vaginally once a week." 3. "I should drink 2½ liters of fluid every day." 4. "I will not drink fluids after 8 pm each evening."

3. "I should drink 2½ liters of fluid every day." Drinking 2½ liters of fluid a day flushes out the urinary system and helps reduce the risk for cystitis. Avoiding carbonated beverages is not necessary to reduce the risk for cystitis. Douching is not a healthy behavior because it removes beneficial organisms as well as the harmful ones. Avoiding fluids after 8 pm would help prevent nocturia but not cystitis. It is recommended that clients with incontinence problems limit their late-night fluid intake to 120 mL.

A client is prescribed estrogen therapy for urinary incontinence. What does the nurse teach the client about this therapy? 1. Change positions slowly, especially in the mornings. 2. Report urine output that is significantly lower than fluid intake. 3. A thin application of cream is adequate. 4. Use hard candy to moisten the mouth.

3. A thin application of cream is adequate. Teach the client that a thin application of estrogen cream is all that is needed. The client taking tricyclic antidepressants must change positions slowly, especially in the mornings, because these drugs cause dizziness, orthostatic hypotension, and increase the risk for falls. The client taking antispasmodics or anticholinergics should report urine output that is significantly lower than fluid intake as these drugs cause urine retention. Dry mouth is another side effect of antispasmodics and anticholinergics; the client can use hard candy to moisten the mouth.

A teaching-learning concept basic to all teaching plans is to present content from the: 1. Cognitive to the affective domain 2. Formal to the informal 3. Simple to the complex 4. Broad to the specific

3. Complex material is best learned when easily understood aspects of the topic are presented first as a foundation for the more complex aspects. When moving from the simple to the complex, a person works at integrating and incorporating the less complex, new learning into one's body of knowledge and understanding before moving on to more complex information.

The student nurse asks the nursing instructor for help with her microbiology class. The student is studying bacteria. What does the best instruction by the nursing instructor include?

3. E. coli are gram-negative bacteria. 4. Gram-staining is one way to identify bacteria. 5. Spherical-shaped bacteria are called cocci.

A cognitively impaired client has urge incontinence. Which method for achieving continence does the nurse include in the client's care plan? 1. Bladder training 2. Credé method 3. Habit training 4. Kegel exercises

3. Habit training Habit training (scheduled toileting) will be most effective in reducing incontinence for a cognitively impaired client because the caregiver is responsible for helping the client to a toilet on a scheduled basis. Bladder training, the Credé method, and learning Kegel exercises require that the client be alert, cooperative, and able to assist with his or her own training.

Which behavior identified by the nurse indicates the highest level of learning in the psychomotor domain? 1. Demonstrating a well-balanced stance with crutches 2. Identifying the correct equipment that is needed for a colostomy irrigation 3. Performing a dry sterile dressing change without contaminating the equipment 4. Recognizing the difference between systolic and diastolic blood pressure sounds

3. This option reflects the highest level of learning of the options offered. When a person achieves the ability to perform a behavior that requires a complex movement pattern with confidence, learning has been achieved on the complex-overt response level of learning in the psychomotor domain

The nurse is teaching a client who is scheduled for a neobladder and a Kock's pouch. Which client statement indicates a correct understanding of these procedures? 1 . "If I restrict my oral intake of fluids, the adjustment will be easier." 2. "I must go to the restroom more often because my urine will be excreted through my anus." 3. "I need to wear loose-fitting pants so the urine can flow into my ostomy bag." 4. "I will have to drain my pouch with a catheter."

4. "I will have to drain my pouch with a catheter." For the client with a neobladder and a Kock's pouch, urine is collected in a pouch and is drained with the use of a catheter. Fluids should not be restricted. A neobladder does not require the use of an ostomy bag.

The nurse is teaching a client about pelvic muscle exercises. What information does the nurse include? 1. "For the best effect, perform all of your exercises while you are seated on the toilet." 2. "Limit your exercises to 5 minutes twice a day, or you will injure yourself." 3. "Results should be visible to you within 72 hours." 4. "You know that you are exercising correct muscles if you can stop urine flow in midstream."

4. "You know that you are exercising correct muscles if you can stop urine flow in midstream." When the client can start and stop the urine stream, the pelvic muscles are being used. Pelvic muscle exercises can be performed anywhere and should be performed more often than 5 minutes twice daily. Noticeable results take several weeks.

A client has been prescribed permethrin (Nix) for the treatment of body mites. What medication information should the nurse provide?

4. "You may feel some stinging or tingling while the lotion is being used." 5. "You should not use this medication if you are sensitive to chrysanthemums."

The health care provider requests phenazopyridine (Pyridium) for a client with cystitis. What does the nurse tell the client about the drug? 1. "It will act as an antibacterial drug." 2. "This drug will treat your infection, not the symptoms of it." 3. "You need to take the drug on an empty stomach." 4. "Your urine will turn red or orange while on the drug."

4. "Your urine will turn red or orange while on the drug." Phenazopyridine will turn the client's urine red or orange. Clients should be warned about this effect of the drug because it will be alarming to them if they are not informed, and care should be taken because it will stain undergarments. Phenazopyridine reduces bladder pain and burning by exerting a local analgesic/anesthetic effect on the mucosa of the urinary tract. It does treat the symptoms of bladder infection; it has no antibacterial action. Phenazopyridine should be taken with a meal or immediately after eating.

Which client does the nurse manager on the medical unit assign to an experienced LPN/LVN? 1. 42-year-old with painless hematuria who needs an admission assessment 2. 46-year-old scheduled for cystectomy who needs help in selecting a stoma site 3. 48-year-old receiving intravesical chemotherapy for bladder cancer 4. 55-year-old with incontinence who has intermittent catheterization prescribed

4. 55-year-old with incontinence who has intermittent catheterization prescribed Insertion of catheters is within the education and legal scope of practice for LPN/LVNs. Admission assessments and intravesical chemotherapy should be done by an RN. Preoperative preparation for cystectomy and stoma site selection should be done by an RN and either a Certified Wound, Ostomy, and Continence Nurse (CWOCN) or an enterostomal therapy (ET) nurse.

A nurse is preparing a patient with a colostomy for discharge. What patient outcome indicates that learning has occurred in the psychomotor domain? 1. Accepts the need to have a colostomy 2. Understands why certain foods should be avoided 3. Verbalizes the rationale for daily colostomy irrigations 4. Changes a colostomy bag without contaminating the hands

4. Changing a colostomy bag without contaminating the hands is an example of learning in the psychomotor domain. Learning in the psychomotor domain is related to mastering a skill and requires motor activity.

The nurse is caring for a client with cystitis. What does the nurse ask the client to include in the diet as part of nutritional therapy? 1. Carbonated beverages 2. Tomato products 3. Caffeine 4. Cranberry juice

4. Cranberry juice The client with cystitis should consume 50 mL of concentrated cranberry juice daily because it is known to decrease the ability of bacteria to adhere to the epithelial cells lining the urinary tract, decreasing the incidence of symptomatic urinary tract infections in some clients. Cranberry juice must be consumed for 3 to 4 weeks to be effective. Caffeine, carbonated beverages, and tomato products must be avoided to decrease bladder irritation during cystitis.

What does the nurse teach a client to do to decrease the risk for urinary tract infection (UTI)? 1. Limit fluid intake. 2. Increase caffeine consumption. 3. Limit sugar intake. 4. Drink about 3 liters of fluid daily.

4. Drink about 3 liters of fluid daily. Drinking about 3 liters of fluid daily, if another medical problem does not require fluid restriction, helps prevent dehydration and UTIs. Fluids flush the system and should not be limited. Increased caffeine intake and limiting sugar intake will not prevent UTIs.

A nurse is to provide nutritional counseling for an older adult. What should the nurse do first? 1. Plan educational sessions in the late afternoon 2. Speak louder when talking 3. Provide large-print books 4. Assess for readiness

4. If the patient does not recognize the need to learn or value the information to be learned, the patient will not be ready to learn.

A culturally competent nurse is planning to teach a patient about a new regimen of self-care. What must the nurse assess first about the patient before implementing the teaching plan? 1. Religious affiliation 2. Support system 3. National origin 4. Health beliefs

4. Individuals have their own beliefs associated with cultural health practices, faith beliefs, diet, illness, death and dying, and lifestyle, which all have a major impact on health beliefs.

In what location would the nurse expect to find infection in a client with acute pyelonephritis? 1. Urethra 2. Urinary bladder 3. Prostate gland 4. Kidneys

4. Kidneys Acute pyelonephritis is a kidney infection. Urinary tract infections are described by their location in the tract. Urethritis is an acute infection in the urethra, cystitis in the bladder, and prostatitis in the prostate gland.

A nurse is designing a teaching-learning program for a patient who is to be discharged from the hospital. What should the nurse do first? 1. Identify the patient's locus of control 2. Use a variety of teaching methods appropriate for the patient 3. Formulate an achievable, measurable, and realistic patient goal 4. Assess the patient's current understanding of the content to be taught

4. Learners bring their own lifetimes of learning to the learning situation. The nurse needs to customize each teaching plan, capitalize on the patient's previous experience and knowledge, and identify what the patient still needs to know before teaching can begin.

What procedure does the nurse expect the health care provider to prescribe for the removal of a large, impacted stone in a client's kidney? 1. Lithotripsy 2. Stenting 3. Pyelolithotomy 4. Nephrolithotomy

4. Nephrolithotomy Nephrolithotomy, an open surgical procedure, is often prescribed to remove a large, impacted stone in the kidney. This method is performed if all other procedures fail and there is a possible risk for a lasting injury to the ureter or kidney. Lithotripsy is the use of sound, laser, or dry shock waves to break stones into small fragments. Stenting is a minimally invasive procedure performed by placing a stent in the ureter by ureteroscopy. The stent dilates the ureter, enlarging the passageway for the stone or stone fragments. Pyelolithotomy is an open surgical procedure to remove stones in the kidney pelvis.

To be most effective, at what grade reading level should the nurse prepare educational medical material? 1. Fourth-grade 2. Eighth-grade 3. Tenth-grade 4. Sixth-grade

4. Randomized studies demonstrate that the average reading level of individuals who need health teaching is 6.8 grades of schooling.

A nurse is teaching an older adult how to perform a dressing change. Which nursing action is most important to address a developmental stress of aging? 1. Speak louder when talking to the patient 2. Use terminology understandable to the patient 3. Have the patient provide a return demonstration 4. Allow more time for the patient to process information

4. Reaction time will slow with aging; therefore, older adults need more time to process and respond to information or perform a skill. In addition, some older adults may have less energy, experience more fatigue, and may need shorter, frequent learning sessions.

Which predisposes the adolescent to feel an increased need for sleep? a. An inadequate diet b. Rapid physical growth c. Decreased activity that contributes to a feeling of fatigue d. The lack of ambition typical of this age group

ANS: B During growth spurts, the need for sleep increases. Rapid physical growth, the tendency toward overexertion, and the overall increased activity of this age contribute to fatigue. DIF: Cognitive Level: Understand REF: p. 463 TOP: Integrated Process: Nursing Process: Assessment MSC: Area of Client Needs: Health Promotion and Maintenance

Place in order the sequence of maturational changes for girls. Begin with the first change seen, sequencing to the last change. Provide answer in using lowercase letters, separated by commas (e.g., a, b, c, d, e). a. Growth of pubic hair b. Rapid increase in height and weight c. Breast changes d. Menstruation e. Appearance of axillary hair

ANS: c, b, a, e, d The usual sequence of maturational changes for girls is breast changes, rapid increase in height and weight, growth of public hair, appearance of axillary hair, and then menstruation, which usually begins 2 years after the first signs. DIF: Cognitive Level: Analyze REF: p. 450 TOP: Integrated Process: Nursing Process: Evaluation MSC: Area of Client Needs: Health Promotion and Maintenance

An infant is having an anaphylactic reaction, and the nurse is preparing to administer epinephrine 0.001 mg/kg. The child weighs 22 pounds. What is the epinephrine dose the nurse should administer? (Record your answer using two decimal places.)

ANS: 0.01 Convert the 22 pounds to kilograms by dividing 22 by 2.2 = 10. Multiply the 10 by 0.001 mg of epinephrine = 0.01 mg as the dose to be given.

A dose of oxycodone (OxyContin) 2 mg/kg has been ordered for a child weighing 33 lb. How many milligrams of OxyContin should the nurse administer? (Record your answer as a whole number.)

ANS: 30 The child's weight is divided by 2.2 to get the weight in kilograms. Kilograms in weight are then multiplied by the prescribed 2 mg. 33 lb/2.2 = 15 kg. 15 kg × 2 mg = 30 mg.

A patient on an intravenous opioid analgesic has become apneic. The nurse should implement which interventions? Place the interventions in order from the highest priority (first intervention) to the lowest priority (last intervention). Provide your answer using lowercase letters separated by commas (e.g., a, b, c, d). a. Place the patient on continuous pulse oximetry to assess SaO2. b. Administer the prescribed naloxone (Narcan) dose by slow IV push. c. Ensure oxygen is available. d. Prepare to calm the child as analgesia is reversed.

ANS: b, a, c, d The Narcan prescribed dose should be given, first by slow IV push every 2 minutes until effect is obtained. The second intervention should be assessment of the patient's SaO2 status. Oxygen should be made available and administered if the SaO2 status indicates hypoxemia. Last, the child should be calmed as the analgesia is reversed. DIF: Cognitive Level: Apply REF: p. 135 TOP: Integrated Process: Nursing Process: Implementation MSC: Area of Client Needs: Physiologic Integrity

The home health nurse is planning care for a 3-year-old boy who has Down syndrome and is receiving continuous oxygen. He recently began walking around furniture. He is spoon-fed by his parents and eats some finger foods. Which is the most appropriate goal to promote normal development? a. Encourage mobility. b. Encourage assistance in self-care. c. Promote oral-motor development. d. Provide opportunities for socialization.

ANS: A A major principle for developmental support in children with complex medical issues is that it should be flexible and tailored to the individual child's abilities, interests, and needs. This child is exhibiting readiness for ambulation. It is an appropriate time to provide activities that encourage mobility, for example, longer oxygen tubing. Parents should provide decreasing amounts of assistance with self-care as he is able to develop these skills. He is receiving oral foods and is eating finger foods. He has acquired oral-motor development. Mobility is a new developmental task. Opportunities for socialization should be ongoing.

According to Piaget, magical thinking is the belief of which? a. Thoughts are all powerful. b. God is an imaginary friend. c. Events have cause and effect. d. If the skin is broken, the insides will come out.

ANS: A Because of their egocentrism and transductive reasoning, preschoolers believe that thoughts are all powerful. Believing God is an imaginary friend is an example of concrete thinking in a preschoolers spiritual development. Cause-and-effect implies logical thought, not magical thinking. Believing that if the skin is broken, the insides will come out is an example of concrete thinking in development of body image.

Which describes a child who is abused by the parent(s)? a. Unintentionally contributes to the abusing situation b. Belongs to a low socioeconomic population c. Is healthier than the nonabused siblings d. Abuses siblings in the same way as child is abused by the parent(s)

ANS: A Child's temperament, position in the family, additional physical needs, activity level, or degree of sensitivity to parental needs unintentionally contribute to the abusing situation. Abuse occurs among all socioeconomic levels. Children who are ill or have additional physical needs are more likely to be abused. The abused child may not abuse siblings.

In terms of language and cognitive development, a 4-year-old child would be expected to have which traits (select all that apply)? a. Think in abstract terms. b. Follow directional commands. c. Understand conservation of matter. d. Use sentences of eight words. e. Tell exaggerated stories.

B, E Children ages 3 to 4 years can give and follow simple commands and tell exaggerated stories. Children cannot think abstractly at age 4 years. Conservation of matter is a developmental task of the school-age child. Five-year-old children use sentences with eight words with all parts of speech.

Who are the learners?

Clients Families /Others who care for the client Coworkers Nursing students Community Members

When a patient with heartburn takes antacids, for which problem is he especially at risk? 1) Diarrhea 2) Constipation 3) Stomach ulceration 4) Flatulence

Constipation Antacids slow peristalsis, placing the patient at risk for constipation. Antibiotics increase the risk for diarrhea. Stomach ulceration is an adverse effect associated with NSAIDs. Iron supplementation may cause flatulence.

What lab tests correlate with cyclosporine

Cyclosporine may increase serum triglycerides and uric acid. It may decrease hepatic enzymes and urinary function test values

The nurse is teaching the importance of a low purine diet to a client admitted with urolithiasis consisting of uric acid. Which statement by the client indicates that teaching was effective? A. "I am so relieved that I can continue eating my fried fish meals every week." B. "I will quit growing rhubarb in my garden since I'm not supposed to eat it anymore." C. "My wife will be happy to know that I can keep enjoying her liver and onions recipe." D. "I will no longer be able to have red wine with my dinner."

D. "I will no longer be able to have red wine with my dinner." Nutrition therapy depends on the type of stone formed. When stones consist of uric acid (urate), the client should decrease intake of purine sources such as organ meats, poultry, fish, gravies, red wines, and sardines. Reduction of urinary purine content may help prevent these stones from forming. Avoiding oxalate sources such as spinach, black tea, and rhubarb is appropriate when the stone consists of calcium oxalate.

2. True or False: Anyone can learn to use an assertive communication style and develop assertiveness.

True

9. True or False: By venting and then emotionally and/or physically withdrawing, chronic complainers are seeking to gain the sympathy of others and develop some type of connection, even if it is dysfunctional.

True

A client has heparin-induced thrombocytopenia (HIT). The student nurse asks how this is treated. About what drugs does the nurse instructor teach? (Select all that apply.) a. Argatroban (Argatroban) b. Bivalirudin (Angiomax) c. Clopidogrel (Plavix) d. Lepirudin (Refludan) e. Methylprednisolone (Solu-Medrol)

a. Argatroban (Argatroban) b. Bivalirudin (Angiomax) d. Lepirudin (Refludan)

HIV-AIDS Antiretrovirals Classification

• Nucleotide reverse transcriptase inhibitors (NtRTIs) - Resemble natural building blocks of DNA • Nonnucleoside reverse transcriptase inhibitors (NNRTIs) - Target the enzyme needed for reverse transcriptase • Protease inhibitors (PIs) - Block the viral enzyme protease, inhibiting final assembly of HIV virions • Entry inhibitors - Block the entry of viral nucleic acid into the T4 lymphocyte • Integrase inhibitors/Miscellaneous antivirals - Integrase enzyme inserts its viral DNA strand into human chromosome

Role of the Nurse: NtRTI, NNRTI, and PI Therapy

• Nursing care similar for NtRTIs, NNRTIs, and PIs • Establish trusting, nonjudgmental relationship with patient • Assess patient's understanding of HIV disease process • Assess for symptoms of HIV and any opportunistic infections • Monitor plasma HIV RNA (viral load) assays, CD4 counts, complete blood count, liver and renal profiles, blood glucose levels • Assess for bone marrow suppression, liver toxicity, and Stevens-Johnson syndrome • Patients should not drive or perform hazardous activities until medication reactions are known • Be aware of conditions and drugs that are problematic with antiretroviral therapy • Teach patients how to practice blood and body fluid precautions

Therapeutic Goals

• Reduce HIV RNA load in the blood - To undetectable level or less than 50 copies/mL • Increased lifespan • Higher quality of life • Decreased risk of transmission from mother to child

Fluvin (griseofluvin)

- if symptoms worsen or don't improve call the doctor - Given orally - increase fluids interferes with renal function - take full course

A 16-year-old patient is admitted to the emergency room after attempting to commit suicide by overdosing on the isoniazid (INH) prescribed for newly diagnosed tuberculosis. What information does the nurse provide to the family?

1. INH overdose is very serious. 3. Treatment will include infusion of vitamin B6. 4. Liver damage may occur.

A client's medical record reveals presence of an erythematous urticarial rash with pruritus. What assessment findings would the nurse expect?

1. The area of the rash is red. 2. The area has a raised, bumpy texture. 4. The area itches.

Every person who attended a smoking cessation educational program completed a questionnaire. What is this type of evaluation called? 1. Survey 2. Post-test 3. Case study 4. Focus group

1. The terms questionnaire and survey are used interchangeably to describe a type of evaluation tool designed to gather data about a topic

A patient asks the nurse, "What does 96 indicate when my blood pressure is 140 over 96?" What is the best response by the nurse? 1. "The 96 is the pressure within an artery when the heart is resting between beats." 2. "The 96 reflects the lowest pressure within a vein when blood moves through it." 3. "Everyone is different so it's really relative to each individual what it means." 4. "Let's talk about the concerns you may have about your blood pressure."

1. This response is simple, is direct, and uses language that is easily understood.

A nurse is planning a teaching plan for an older adult. Which common factor among older adult patients must be considered by the nurse? 1. Learning may require more energy 2. Intelligence decreases as people age 3. Older adults rely more on visual rather than auditory learning 4. Older adult patients are more resistant to change that accompanies new learning

1. Various physiological changes of aging impact on the rate of learning (e.g., declines in sensory perception and speed of mental processing and more time needed for recall), requiring the use of multisensory teaching strategies and a slower approach. In addition, older adults may have less physical and emotional stamina because of more chronic illnesses, so they may require shorter and more frequent learning sessions.

*Physical assessment of a patient diagnosed with bulimia often reveals:* a. prominent parotid glands. b. peripheral edema. c. thin, brittle hair. d. 25% underweight.

ANS: A Prominent parotid glands are associated with repeated vomiting. The other options are signs of anorexia nervosa and not usually seen in bulimia.

3. A nurse assesses a client who is recovering after a coronary catheterization. Which assessment findings in the first few hours after the procedure require immediate action by the nurse? (Select all that apply.) a. Blood pressure of 140/88 mm Hg b. Serum potassium of 2.9 mEq/L c. Warmth and redness at the site d. Expanding groin hematoma e. Rhythm changes on the cardiac monitor

ANS: B, D, E In the first few hours postprocedure, the nurse monitors for complications such as bleeding from the insertion site, hypotension, acute closure of the vessel, dye reaction, hypokalemia, and dysrhythmias. The client's blood pressure is slightly elevated but does not need immediate action. Warmth and redness at the site would indicate an infection, but this would not be present in the first few hours. DIF: Applying/Application REF: 643 KEY: Assessment/diagnostic examination MSC: IntegratedProcess:NursingProcess:Assessment NOT: Client Needs Category: Physiological Integrity: Reduction of Risk Potential

12. A client just returned to the surgical unit after a gastric bypass. What action by the nurse is the priority? a. Assess the clients pain. b. Check the surgical incision. c. Ensure an adequate airway. d. Program the morphine pump.

ANS: C All actions are appropriate care measures for this client; however, airway is always the priority. Bariatric clients tend to have short, thick necks that complicate airway management.

What is the action of bactericidal drugs?

They will kill the bacteria.

What is a disadvantage of the salad bowl tradition?

a type of cultural confusion can be created that may lead to increased tension and anxiety.

Following surgery, a client is placed on cefotaxime (Claforan). The assessment for possible adverse effects should include observing for

diarrhea.

Is culture a monolithic concept?

no

HIV-AIDS—Nucleoside and Nucleotide Reverse Transcriptase Inhibitors Prototype drug: zidovudine (Retrovir, AZT)

• Mechanism of action: Virus mistakenly uses zidovudine as nucleoside, thus creating defective DNA strand • Primary use: with other antiretrovirals for symptomatic and asymptomatic HIV- infected patients - Also for post exposure prophylaxis (preventive health care) in HIV- exposed health care workers - To reduce transmission rate from HIV-positive mother to fetus • Adverse effects - Toxicity to blood cells at high doses - Anemia and neutropenia - Anorexia, nausea, diarrhea - Fatigue, generalized weakness

Agents for HIV-AIDS—Nonnucleoside Reverse Transcriptase Inhibitors Prototype drug #2: efavirenz (Sustiva)

• Mechanism of action: to bind directly to reverse transcriptase, disrupting enzyme's active site • Primary use: in combination with other antivirals in treatment using HAART • Adverse effects: CNS effects: sleep disorders, inability to concentrate, delusions, dizziness; rash

Agents for HIV-AIDS—Protease Inhibitors: Prototype drug: lopinavir with ritonavir (Kaletra)

• Mechanism of action: to inhibit HIV protease • Primary use: In combination with other antiretrovirals for HIV-infected patients, it is the preferred drug for initial treatment • Adverse effects: nausea, vomiting, dyspepsia, diarrhea, general fatigue, headache - Hyperglycemia has been reported, lipodystrophy syndrome occurs in many patients receiving long-term therapy, pancreatitis is rare but possible serious effect

Role of the nurse in general

• Monitor patient's condition • Provide patient education • Obtain medical, surgical, drug history • Assess lifestyle and dietary habits • Obtain description of symptomology and current therapies

HIV Pharmacotherapy

• No cure yet, but many new drugs developed • Some therapeutic successes - People live symptom-free longer - Rates of transmission from mother to newborn reduced - 70% decline in death rate in U.S. § Incidence of infections still very high in African nations

Role of the Nurse: Antiviral Therapy

• Use drugs with extreme caution with pre- existing renal or hepatic disease • Judicious use is warranted during pregnancy • Emphasize compliance with antiviral therapy • Some drugs cause digestive distress and should be taken with food

Pharmacotherapy of Viral Hepatitis

• Vaccination available for A and B, not C • Prophylaxis or post exposure treatment - Hepatitis A immunoglobulins (HAIg) - Hepatitis B immunoglobulins (HBIg) • Symptomatic treatment for chronic hepatitis • Hepatitis C - Interferon - Antiviral ribavirin

Challenges of Antiviral Therapy

• Viruses mutate rapidly, and drug becomes ineffective • Difficult for drug to find virus without injuring normal cells • Each antiviral drug specific to one particular virus

What type of precautions should the nurse take for a patient suspected of having TB as a result of HIV? A. universal b. airborne c. enteric d. protective isolation

b

Diet education for Flagyl

must be taken on an empty stomach

The client is prescribed amoxicillin (Amoxil) for 10 days to treat strep throat. After 5 days, the client tells the nurse he plans to stop the medication because he feels better. What is the best response by the nurse?

"If you stop the medicine early, you have not effectively killed out the bacteria making you sick."

The client receives multiple drugs for treatment of tuberculosis. The nurse teaches the client the rationale for multiple drug treatment and evaluates learning as effective when the client makes which statement?

"Multiple drugs are necessary because the bacteria are likely to develop resistance to just one drug."

List the adverse effects of Intron-A

- A fulike syndrome of fever, chills, dizziness, and fatigue occurs in 50% of patients - Headache, nausea, vomiting, diarrhea, and anorexia are relatively common - Depression and suicidal intention - With prolonged therapy, serious toxicity such as immunosuppression, hepatotoxicity, and neurotoxicity may be observed

A patient has been advised to use an over-the-counter acne product that contains benzoyl peroxide and salicylic acid. The nurse prioritizes which information in patient teaching?

1. "Test this product on a small area of skin for 3 days before applying to your face." 3. "Some serious allergic reactions have occurred when using this medication."

A patient diagnosed with rosacea has been prescribed topical metronidazole (MetroGel). The nurse would teach which other management strategies?

1. "Avoid drinking alcohol." 3. "Avoid eating spicy foods." 4. "Drink your coffee at room temperature."

A patient has been prescribed ciprofloxacin (Cipro) for a severe sinus infection. The nurse evaluates that medication education has been effective when the patient makes which statements?

1. "I should avoid milk while taking this medication."(Dairy products can decrease the absorption of ciprofloxacin.) 2. "I should avoid coffee while taking this medication."(Ciprofloxacin can increase serum levels of caffeine. ) 4. "I may have some diarrhea while taking this medication." 5. "If my stomach gets upset, I should take this medication with food."

A client wants to lose 1.5 pounds a week. After reviewing a diet history, the nurse determines the client typically eats 2450 calories a day. What should the client's calorie goal be to achieve this weight loss? (Record your answer using a whole number.) __ calories/day

1700 calories/day

A client is prescribed trimethoprim/sulfamethoxazole (Septra) for urinary tract infection (UTI). What does the nurse instruct the client about this therapy? Select all that apply. 1. Disclose any allergies to sulfa drugs before beginning therapy. 2. Wear sunscreen and protective clothing when out in the sun. 3. Monitor the pulse twice daily while taking this drug. 4. Drink a full glass of water with each dose of the drug. 5. Avoid taking the drug within 2 hours of taking an antacid.

1. Disclose any allergies to sulfa drugs before beginning therapy. 2. Wear sunscreen and protective clothing when out in the sun. 4. Drink a full glass of water with each dose of the drug. The nurse should ensure that the client does not have any allergies to sulfa drugs before beginning therapy, since allergies to sulfa drugs are common and may require changing the drug therapy. The client should wear sunscreen and protective clothing when out in the sun because sulfamethoxazole increases sensitivity to the sun and can lead to severe sunburns. The client must consume a full glass of water with each dose because the drug can form crystals that precipitate in the kidney tubules; drinking at least 3 L of fluids daily prevents this complication. The client taking fluoroquinolone is asked to monitor the pulse twice daily as this class of drugs induces serious cardiac dysrhythmias. Fluoroquinolone must not be taken within 2 hours of taking an antacid; antacids containing magnesium or aluminum interfere with drug absorption.

The nurse is teaching a group of older adult women about the signs and symptoms of urinary tract infection (UTI). Which concepts does the nurse explain in the presentation? Select all that apply. 1. Dysuria 2. Enuresis 3. Frequency 4. Nocturia 5. Urgency 6. Polyuria

1. Dysuria 3. Frequency 4. Nocturia 5. Urgency Dysuria (painful urination), nocturia (frequent urinating at night), urgency (having the urge to urinate quickly), and frequency are symptoms of UTI. Enuresis (bed-wetting) and polyuria (increased amounts of urine production) are not signs of a UTI.

A nurse is providing health teaching for a patient with a comprehension deficit. Which is the best intervention by the nurse that will support this patient's learning? 1. Establishing a structured environment 2. Asking that unclear words be repeated 3. Speaking directly in front of the patient 4. Making a referral for a hearing evaluation

1. For people who have difficulty with comprehension, participating in a learning program often makes them feel overwhelmed and threatened. The teacher needs to provide a structured environment in which variables are controlled to reduce anxiety and support comprehension. The nurse should minimize ambiguity, provide a familiar environment, teach at the same time each day, limit environmental distractions, and provide simple learning materials.

A nurse uses computer-assisted instruction as a strategy when providing preoperative teaching. The nurse explains to preoperative patients that the greatest advantage of computer-assisted instruction is that: 1. Learners can progress at their own rate 2. It is the least expensive teaching strategy 3. There are opportunities for pre- and post-testing 4. Information is presented in a well-organized format

1. Learners progress through a program at their own pace viewing informational material, answering questions, and receiving immediate feedback. Some programs feature simulated situations that require critical thinking and a response. Correct responses are rationalized, praise is offered, and incorrect responses trigger an explanation of why the wrong answer is wrong and offer encouragement to try again. This is a superior teaching strategy for the learner who may find that group lessons are paced either too fast or too slow for effective learning.

A nurse is assessing a patient to determine educational needs. Which is most important for the nurse to consider? 1. Make no assumptions about the patient 2. Teaching may be informal or formal in nature 3. The teaching plan should be documented on appropriate records 4. A copy of the teaching/learning contract should be given to the patient

1. Many variables influence an individual's willingness and ability to learn (e.g., readiness, motivation, physical and emotional abilities, education, age, cultural and health beliefs, cognitive abilities). Because everyone is unique with individual needs, the nurse must avoid making assumptions and generalizations.

What nonsurgical methods does the nurse teach the client to manage stress incontinence? Select all that apply. 1. Reduce excess body weight. 2. Walk to strengthen pelvic muscles. 3. Perform Kegel exercises. 4. Use artificial sweeteners instead of sugar. 5. Practice vaginal cone therapy.

1. Reduce excess body weight. 3. Perform Kegel exercises. 5. Practice vaginal cone therapy. The client with stress incontinence should reduce excess weight because increased abdominal pressure aggravates stress incontinence. Kegel exercise therapy strengthens the muscles of the pelvic floor, and weighted vaginal cones are used to strengthen pelvic muscles and decrease stress incontinence. Walking is a good exercise; however, it does not help to increase pelvic strength. The client must avoid foods that irritate the bladder such as artificial sweeteners, alcohol, nicotine, citrus, and caffeine.

What clinical findings does the nurse attribute to the presence of kidney stones? Select all that apply. 1. Smoky urine 2. Odorless urine 3. Increased serum calcium 4. Increased serum phosphate 5. Urine pH of 6

1. Smoky urine 3. Increased serum calcium 4. Increased serum phosphate Smoky or rusty urine is common in clients with kidney stones, indicating hematuria. Increases in serum calcium and phosphate levels indicate that excess minerals are present and may contribute to stone formation. The urine generally has an odor, indicating infection. Normal urine is alkaline, with a pH between 5 and 6. If urine contains uric acid or cystine stones, it is highly acidic. If urine contains calcium phosphate and struvite stones, it is alkaline.

What method of emptying the bladder is helpful for a client with a large cystocele? 1. Splinting 2. Credé method 3. Double-voiding 4. Valsalva maneuver

1. Splinting The client with a large cystocele or prolapse of the bladder into the vagina may use splinting to reduce renal urinary incontinence. This is achieved by inserting fingers into the vagina and lifting the cystocele to urinate. The client using the Credé method presses over the bladder area to increase pressure. The client may also trigger nerve stimulation by tugging at the pubic hair or massaging the genital area. In the double-voiding technique, the client empties the bladder once and then attempts a second voiding within a few minutes. The Valsalva maneuver is a breathing technique that increases chest and abdominal pressure. Increased pressure is directed toward the bladder during exhalation.

The nurse is providing community education regarding ways to reduce development of antibiotic resistance. Which information should be included?

1. The best way to prevent antibiotic resistance is to prevent infections from occurring. 2. Do not expect to receive an antibiotic prescription for colds and influenza. 3. Take the full amount of any prescribed medication. 5. Use good infection control measures.

A nurse must implement a teaching plan for a patient recently diagnosed with heart failure. What should the nurse do first? 1. Identify the patient's level of recognition of the need for learning 2. Frame the goal within the patient's value system 3. Determine the patient's preferred learning style 4. Assess the patient's personal support system

1. The learner must recognize that the need exists and that the material to be learned is valuable. Motivation is the most important factor influencing learning.

The nursing instructor teaches the student nurses about the structure and function of the skin. What will the best teaching plan of the instructor include?

1. The outermost layer of the epidermis serves as the major waterproof barrier to the environment. 3. The amount of subcutaneous tissue varies and is determined by nutritional status and heredity. 3. The amount of subcutaneous tissue varies and is determined by nutritional status and heredity.

Which best describes a patient with an external locus of control? Select all that apply. 1. _____ Behaving appropriately to obtain the right to watch a television program 2. _____ Is self-motivated when implementing health promotion behaviors 3. _____ Wants to please family members with efforts to get well 4. _____ Understands the expected outcome of therapy 5. _____ Is a self-actualized adult

1. The person with an external locus of control is motivated by rewards that center on privileges, incentives, or praise received from pleasing significant others or members of the health-care team. Watching television is a privilege in this situation. 3. Pleasing others precipitates feedback that is often viewed as positive by the recipient. Positive verbal or nonverbal communication from another is an external reward.

The nurse must irrigate the colostomy of a patient who is unable to move independently. How should the nurse position the patient for this procedure? 1) Semi-Fowler's position 2) Left side-lying position 3) Supine, with the head of the bed lowered flat 4) Supine, with the head of bed raised to 30 degrees

2) Left side-lying position The nurse should position an immobile patient in a left side-lying position to irrigate his colostomy. Semi-Fowler's, supine with the bed lowered flat, and the supine position with the head of bed elevated to 30 degrees are not appropriate positions for colostomy irrigation.

The nurse is teaching a client with a neurogenic bladder to use intermittent self-catheterization for bladder emptying. Which client statement indicates a need for further clarification? 1. "A small-lumen catheter will help prevent injury to my urethra." 2. "I will use a new, sterile catheter each time I do the procedure." 3. "My family members can be taught to help me if I need it." 4. "Proper handwashing before I start the procedure is very important."

2. "I will use a new, sterile catheter each time I do the procedure." Catheters are cleaned and reused. With proper handwashing and cleaning of the catheter, no increase in bacterial complications has been shown. Catheters are replaced when they show signs of deteriorating. The smallest lumen possible and the use of a lubricant help reduce urethral trauma to this sensitive mucous tissue. Research shows that family members in the home can be taught to perform straight catheterizations using a clean (rather than a sterile) catheter with good outcomes. Proper handwashing is extremely important in reducing the risk for infection in clients who use intermittent self-catheterization and is a principle that should be stressed.

The nurse receives the change-of-shift report on four clients. Which client does the nurse decide to assess first? 1. 26-year-old admitted 2 days ago with urosepsis with an oral temperature of 99.4° F (37.4° C) 2. 28-year-old with urolithiasis who has been receiving morphine sulfate and has not voided for 8 hours 3. 32-year-old admitted with hematuria and possible bladder cancer who is scheduled for cystoscopy 4. 40-year-old with noninfectious urethritis who is reporting "burning" and has estrogen cream prescribed

2. 28-year-old with urolithiasis who has been receiving morphine sulfate and has not voided for 8 hours Anuria may indicate urinary obstruction at the bladder neck or urethra and is an emergency because obstruction can cause acute kidney failure. The client who has been receiving morphine sulfate may be oversedated and may not be aware of any discomfort caused by bladder distention. The 26-year-old admitted with urosepsis and slight fever, the 32-year-old scheduled for cystoscopy, and the 40-year-old with noninfectious urethritis are not at immediate risk for complications or deterioration.

A nurse is teaching a postoperative patient deep breathing and coughing exercises. Which method of instruction is most appropriate in this situation? 1. Explanation 2. Demonstration 3. Video presentation 4. Brochure with pictures

2. A demonstration is the best strategy for teaching a psychomotor skill. A demonstration is an actual performance of the skill by the teacher who is acting as a role model. A demonstration usually is followed by a return demonstration. The learner can imitate the teacher during a return demonstration, ask questions, and receive feedback from the instructor.

A patient is readmitted to the hospital because of complications resulting from nonadherence to the prescribed health-care regimen. What should the nurse do first? 1. Encourage healthy behaviors 2. Develop a trusting relationship 3. Use educational aids to reinforce teaching 4. Establish why the client is not following the regimen

2. A trusting relationship between the patient and the nurse is essential. Patients have to be confident that the nurse will maintain confidentiality, has credibility, and is genuinely interested in their success

A patient has recurrent skin infections. The nurse anticipates administering an aminopenicillin such as which drug?

2. Ampicillin 4. Amoxicillin

The client is to receive an injection of penicillin G benzathine (Bicillin LA) in the outpatient clinic. What are the priority nursing actions prior to administering this injection?

2. Ask the client if she has ever had an allergy to penicillin before. 3. Inform the client that she will need to wait 30 minutes before leaving the clinic.

A nurse is attending a class about a new intravenous pump presented by the hospital staff education department. What is this type of educational program? 1. Continuing education program 2. Inservice education program 3. Certification program 4. Orientation program

2. Inservice programs generally are provided by health-care agencies to reinforce current knowledge and skills or provide new information about such issues as policies, theory, skills, practice or equipment

A nurse is planning to engage a patient in a program to learn about a newly diagnosed illness. Which psychosocial response to the illness will have the greatest impact on the patient's future success with learning? 1. Fear 2. Denial 3. Fatigue 4. Anxiety

2. Of all the options presented, the patient in denial is the person least ready and motivated to learn. The patient in denial is unable to recognize the need for the learning.

A nurse is assessing the results of dietary teaching for a patient with diabetes mellitus. What patient behavior indicates that learning occurred in the affective domain? 1. Discusses which food on the ordered diet must be avoided 2. Eats the food on the special diet ordered by the physician 3. Compiles a list of foods that are permitted on the diet 4. Asks about which foods can be eaten

2. This is an example of learning on the valuing level in the affective domain. Valuing is demonstrated when learning is incorporated into the learner's behavior because it is perceived as important. Affective learning involves the expression of feelings and the changing of beliefs, attitudes, or values.

A nurse is teaching a patient colostomy care in relation to the affective domain. Which teaching method is most effective for this situation? 1. Discussing a pamphlet about colostomy care from the American Cancer Society 2. Exploring how the patient feels about having a colostomy 3. Providing a demonstration on how to do colostomy care 4. Showing a videotape demonstrating colostomy care

2. This option reflects learning in the affective domain. Affective learning is concerned with feelings, emotions, values, beliefs, and attitudes about the colostomy

A client receiving chemotherapy has a very low white blood cell count. Antibiotic therapy is initiated. What rationales should the nurse provide for the addition of this drug?

2. We would like to prevent you from developing any infections.

A client is receiving continuous tube feeding at 70 mL/hr. When the bag is empty, how much formula does the nurse add? (Record your answer using a whole number.) _____ mL

280 mL

A 53-year-old postmenopausal woman reports "leaking urine" when she laughs, and is diagnosed with stress incontinence. What does the nurse tell the client about how certain drugs may be able to help with her stress incontinence? 1. "They can relieve your anxiety associated with incontinence." 2. "They help your bladder to empty." 3. "They may be used to improve urethral resistance." 4. "They decrease your bladder's tone."

3. "They may be used to improve urethral resistance." Bladder pressure is greater than urethral resistance; drugs may be used to improve urethral resistance. Relieving anxiety has not been shown to improve stress incontinence. No drugs have been shown to promote bladder emptying, and this is not usually the problem with stress incontinence. Emptying the bladder is accomplished by the individual, or if that is not possible, by using a catheter. Decreasing bladder tone would not be a desired outcome for a woman with incontinence.

A nurse is planning teaching about weight reduction strategies to an obese patient. Before implementing the teaching plan the nurse first should assess the patient's: 1. Intelligence 2. Experience 3. Motivation 4. Strengths

3. If the patient does not recognize the need to learn or value the information to be learned, the patient will not be ready to learn.

A school nurse is teaching a class of adolescents about avoiding smoking and includes role-playing as a creative learning activity. What is the primary reason for using role-playing? 1. Provides more fun than other methods 2. Eliminates the need for media equipment 3. Requires active participation by the learner 4. Gives the learner the opportunity to be another person

3. Learning activities that actively engage the learner have been shown to be more effective as well as more fun than methods that do not actively engage the learner. When learners are actively involved, they assume more responsibility for their own learning and develop more self-interest in learning the content.

The certified Wound, Ostomy, and Continence Nurse (CWOCN) or enterostomal therapist (ET) teaches a client who has had a cystectomy about which care principles for the client's postdischarge activities? 1. Nutritional and dietary care 2. Respiratory care 3. Stoma and pouch care 4. Wiping from front to back (asepsis)

3. Stoma and pouch care The enterostomal therapist demonstrates external pouch application, local skin care, pouch care, methods of adhesion, and drainage mechanisms. The registered dietitian (RD) teaches the cystectomy client about nutritional care. The respiratory therapist teaches the cystectomy client about respiratory care. The client with a cystectomy does not require instruction about front-to-back wiping.

A client is ordered phenazopyridine (Prodium) to reduce bladder pain and burning on urination. What does the nurse teach the client about this drug regimen? 1. Report if the urine turns red. 2. Report blurred vision. 3. Take the drug with a meal. 4. Wear dark glasses in sunlight.

3. Take the drug with a meal. The client should take the drug with a meal to prevent gastrointestinal disturbances. The client need not report if the urine turns red or orange because this is an expected response to the drug. The client taking antispasmodics for relieving bladder spasms is asked to report blurred vision,which is a manifestation of toxicity. The client taking antispasmodics, not analgesics, is asked to wear dark glasses in sunlight as the drug dilates the pupil and increases eye sensitivity to light.

Which word best describes the nurse's role when functioning as a teacher? 1. Provide 2. Comfort 3. Empower 4. Collaborate

3. The purpose of teaching patients is to ensure that they have the knowledge and authority to respond most effectively to their own situation.

A nurse educator designed various educational programs that employ role-playing as a teaching strategy. Which group of people should the nurse anticipate will benefit the most from role-playing? 1. Older adults preparing to retire from the workforce 2. Men unwilling to admit that they have a drinking problem 3. Adolescents learning to abstain from recreational drug use 4. Middle-aged adults preparing for total-knee replacement surgery

3. This group should benefit most from role-playing. Role-playing provides a safe environment in which to practice interpersonal skills. It enables the adolescen to rehearse what should be said, learn to respond to the emotional environment, and experience the pressures of the person playing the peer using drugs.

A nurse is teaching a patient with a hearing impairment. What should the nurse do? 1. Limit educational sessions to 10 minutes 2. Provide information in written format 3. Use at least 2 teaching methods 4. Teach in group settings

3. Varieties of teaching methods facilitate learning because multiple senses are stimulated. When we see, hear, and touch, learning is more effective than when we see or hear alone. In addition, research demonstrates that we remember only 10% of what we read, 20% of what we hear, 30% of what we see, 50% of what we see and hear, and 80% of what we say and do.

A nurse is teaching a patient recently diagnosed with diabetes mellitus the step-by step procedure of administering an insulin injection. However, after two sessions the patient is still reluctant to self-administer the insulin. What should the nurse do? 1. Have the patient administer the injection to an orange 2. Keep reinforcing the principles that have been presented 3. Give the patient an opportunity to explore concerns about the injection 4. Determine if a member of the family is willing to administer the insulin

3. When a teaching plan is ineffective the nurse must gather more data and revise the teaching plan to achieve the desired goal.

A client reports experiencing involuntary loss and constant dribbling of urine due to an enlarged prostate. How does the nurse document this incontinence? 1. Stress incontinence 2. Urge incontinence 3. Reflex incontinence 4. Overflow incontinence

4. Overflow incontinence This client's condition is known as overflow incontinence. The urethra in the client is obstructed due to the enlarged prostate; the urethra fails to relax sufficiently to allow urine to flow, resulting in incomplete bladder emptying or complete urinary retention. Loss of urine following physical exertion, cough, or sneeze is documented as stress incontinence. Stress incontinence occurs due to intrinsic sphincter deficiency or acquired anatomic damage to the urethral sphincter. The client with urge incontinence experiences an involuntary loss of urine with a strong desire to urinate. The client with reflex incontinence has a post-void residual less than 50 mL.

A client comes to the emergency department complaining of a sore throat. He has white patches on his tonsils, and he has swollen cervical lymph nodes. What will the best plan by the nurse include?

4. Plan to obtain a throat culture.

A nurse is planning a weight reduction program with an obese patient. What should the nurse anticipate will be the most important component that will determine the success or failure of this plan? 1. Rewarding compliant behavior with favorite foods 2. Encouraging at least 1 hour of exercise daily 3. Using an 800-calorie daily dietary regimen 4. Setting realistic goals

4. Setting realistic goals is important to the success of a weight-loss plan. Because achieving success is dependent largely on motivation, the teacher and patient should design goals that demonstrate immediate progress or growth. One strategy is to design numerous realistic short-term intermediary goals that are achieved more easily than one long-term goal.

A client weighs 228 pounds (103.6 kg) and is 5'3" (160 cm) tall. What is this client's body mass index (BMI)? (Record your answer using a decimal rounded up to the nearest tenth.) _____

40.4

a. Mid-sternal chest pain

A nurse assesses a client with tachycardia. Which clinical manifestation requires immediate intervention by the nurse? a. Mid-sternal chest pain b. Increased urine output c. Mild orthostatic hypotension d. P wave touching the T wave

d. Sinus rhythm with premature ventricular contractions (PVCs)

A nurse assesses a client's electrocardiogram (ECG) and observes the reading shown below: How should the nurse document this client's ECG strip? a. Ventricular tachycardia b. V entricular fibrillation c. Sinus rhythm with premature atrial contractions (PACs) d. Sinus rhythm with premature ventricular contractions (PVCs)

a. Smoking cessation b. Stress reduction and management d. Adverse effects of medications

A nurse is teaching a client with premature ectopic beats. Which education should the nurse include in this client's teaching? (Select all that apply.) a. Smoking cessation b. Stress reduction and management c. Avoiding vagal stimulation d. Adverse effects of medications e. Foods high in potassium

d. Ventricular and atrial depolarizations are initiated from different sites.

A nurse assesses a client's electrocardiograph tracing and observes that not all QRS complexes are preceded by a P wave. How should the nurse interpret this observation? a. The client has hyperkalemia causing irregular QRS complexes. b. Ventricular tachycardia is overriding the normal atrial rhythm. c. The client's chest leads are not making sufficient contact with the skin. d. Ventricular and atrial depolarizations are initiated from different sites.

b. Turn off oxygen therapy.

A nurse assists with the cardioversion of a client experiencing acute atrial fibrillation. Which action should the nurse take prior to the initiation of cardioversion? a. Administer intravenous adenosine. b. Turn off oxygen therapy. c. Ensure a tongue blade is available. d. Position the client on the left side.

b. "Avoid straining while having a bowel movement."

A nurse cares for a client who has a heart rate averaging 56 beats/min with no adverse symptoms. Which activity modification should the nurse suggest to avoid further slowing of the heart rate? a. "Make certain that your bath water is warm." b. "Avoid straining while having a bowel movement." c. "Limit your intake of caffeinated drinks to one a day." d. "Avoid strenuous exercise such as running."

a. Assess airway, breathing, and level of consciousness. V tach

A nurse cares for a client who is on a cardiac monitor. The monitor displayed the rhythm shown below: Which action should the nurse take first? a. Assess airway, breathing, and level of consciousness. b. Administer an amiodarone bolus followed by a drip. c. Cardiovert the client with a biphasic defibrillator. d. Begin cardiopulmonary resuscitation (CPR).

b. Assess vital signs and level of consciousness.

A nurse cares for a client with an intravenous temporary pacemaker for bradycardia. The nurse observes the presence of a pacing spike but no QRS complex on the client's electrocardiogram. Which action should the nurse take next? a. Administer intravenous diltiazem (Cardizem). b. Assess vital signs and level of consciousness. c. Administer sublingual nitroglycerin. d. Assess capillary refill and temperature.

Preschoolers' fears can best be dealt with by which intervention? a. Actively involving them in finding practical methods to deal with the frightening experience b. Forcing them to confront the frightening object or experience in the presence of their parents c. Using logical persuasion to explain away their fears and help them recognize how unrealistic the fears are d. Ridiculing their fears so they understand that there is no need to be afraid

A Actively involving the child in finding practical methods to deal with the frightening experience is the best way to deal with fears. Forcing a child to confront fears may make the child more afraid. Preconceptual thought prevents logical understanding. Ridiculing fears does not make them go away.

In terms of cognitive development, the 5-year-old child would be expected to: a. Use magical thinking. b. Think abstractly. c. Understand conservation of matter. d. Be able to comprehend another person's perspective.

A Magical thinking is believing that thoughts can cause events. Abstract thought does not develop until school-age years. The concept of conservation is the cognitive task of school-age children ages 5 to 7 years. Five-year-olds cannot understand another's perspective.

A common characteristic of those who sexually abuse children is that they: a. Pressure the victim into secrecy. b. Are usually unemployed and unmarried. c. Are unknown to victims and victims' families. d. Have many victims that are each abused only once.

A Sex offenders may pressure the victim into secrecy, regarding the activity as a "secret between us" that other people may take away if they find out. Abusers are often employed upstanding members of the community. Most sexual abuse is committed by men and persons who are well known to the child. Abuse is often repeated with the same child over time. The relationship may start insidiously without the child realizing that sexual activity is part of the offer.

A useful skill that the nurse should expect a 5-year-old child to be able to master is to: a. Tie shoelaces. b. Hammer a nail. c. Use a knife to cut meat. d. Make change from a quarter.

A Tying shoelaces is a fine motor task typical of 5-year-olds. Using a knife to cut meat is a fine motor task of a 7-year-old. Hammering a nail and making change from a quarter are fine motor tasks of an 8- to 9-year-old.

c. Schedule periods of exercise and rest during the day.

A nurse cares for a client with atrial fibrillation who reports fatigue when completing activities of daily living. What interventions should the nurse implement to address this client's concerns? a. Administer oxygen therapy at 2 liters per nasal cannula. b. Provide the client with a sleeping pill to stimulate rest. c. Schedule periods of exercise and rest during the day. d. Ask unlicensed assistive personnel to help bathe the client.

b. Warfarin (Coumadin)

A nurse evaluates prescriptions for a client with chronic atrial fibrillation. Which medication should the nurse expect to find on this client's medication administration record to prevent a common complication of this condition? a. Sotalol (Betapace) b. Warfarin (Coumadin) c. Atropine (Sal-Tropine) d. Lidocaine (Xylocaine)

b. A 50-year-old who is post coronary artery bypass graft surgery

A nurse is assessing clients on a medical-surgical unit. Which client should the nurse identify as being at greatest risk for atrial fibrillation? a. A 45-year-old who takes an aspirin daily b. A 50-year-old who is post coronary artery bypass graft surgery c. A 78-year-old who had a carotid endarterectomy d. An 80-year-old with chronic obstructive pulmonary disease

c. Short period of asystole

A nurse administers prescribed adenosine (Adenocard) to a client. Which response should the nurse assess for as the expected therapeutic response? a. Decreased intraocular pressure b. Increased heart rate c. Short period of asystole d. Hypertensive crisis

b. Speech alterations

A nurse assesses a client with atrial fibrillation. Which manifestation should alert the nurse to the possibility of a serious complication from this condition? a. Sinus tachycardia b. Speech alterations c. Fatigue d. Dyspnea with activity

c. Ask the client what medications he or she takes. sinus brady

A nurse performs an admission assessment on a 75-year-old client with multiple chronic diseases. The client's blood pressure is 135/75 mm Hg and oxygen saturation is 94% on 2 liters per nasal cannula. The nurse assesses the client's rhythm on the cardiac monitor and observes the reading shown below: Which action should the nurse take first? a. Begin external temporary pacing. b. Assess peripheral pulse strength. c. Ask the client what medications he or she takes. d. Administer 1 mg of atropine.

d. Ensure that everyone is clear of contact with the client and the bed.

A nurse prepares to defibrillate a client who is in ventricular fibrillation. Which priority intervention should the nurse perform prior to defibrillating this client? a. Make sure the defibrillator is set to the synchronous mode. b. Administer 1 mg of intravenous epinephrine. c. Test the equipment by delivering a smaller shock at 100 joules. d. Ensure that everyone is clear of contact with the client and the bed.

a. Medication reconciliation

A nurse prepares to discharge a client with cardiac dysrhythmia who is prescribed home health care services. Which priority information should be communicated to the home health nurse upon discharge? a. Medication reconciliation b. Immunization history c. Religious beliefs d. Nutrition preferences

a. "Clean the skin and clip hairs if needed."

A nurse supervises an unlicensed assistive personnel (UAP) applying electrocardiographic monitoring. Which statement should the nurse provide to the UAP related to this procedure? a. "Clean the skin and clip hairs if needed." b. "Add gel to the electrodes prior to applying them." c. "Place the electrodes on the posterior chest." d. "Turn off oxygen prior to monitoring the client."

a. "Minimize or abstain from caffeine."

A nurse teaches a client who experiences occasional premature atrial contractions (PACs) accompanied by palpitations that resolve spontaneously without treatment. Which statement should the nurse include in this client's teaching? a. "Minimize or abstain from caffeine." b. "Lie on your side until the attack subsides." c. "Use your oxygen when you experience PACs." d. "Take amiodarone (Cordarone) daily to prevent PACs."

a. "Until your incision is healed, do not submerge your pacemaker. Only take showers." b. "Report any pulse rates lower than your pacemaker settings." e. "Do not lift your left arm above the level of your shoulder for 8 weeks."

A nurse teaches a client with a new permanent pacemaker. Which instructions should the nurse include in this client's teaching? (Select all that apply.) a. "Until your incision is healed, do not submerge your pacemaker. Only take showers." b. "Report any pulse rates lower than your pacemaker settings." c. "If you feel weak, apply pressure over your generator." d. "Have your pacemaker turned off before having magnetic resonance imaging (MRI)." e. "Do not lift your left arm above the level of your shoulder for 8 weeks."

c. Level of consciousness

A telemetry nurse assesses a client with third-degree heart block who has wide QRS complexes and a heart rate of 35 beats/min on the cardiac monitor. Which assessment should the nurse complete next? a. Pulmonary auscultation b. Pulse strength and amplitude c. Level of consciousness d. Mobility and gait stability

Strict isolation is required for a child who is hospitalized with (select all that apply): a. Mumps. b. Chickenpox. c. Exanthema subitum (roseola). d. Erythema infectiosum (fifth disease). e. Parvovirus B19.

A, B, C, D Childhood communicable diseases requiring strict transmission-based precautions (Contact, Airborne, and Droplet Precautions) include diphtheria, chickenpox, measles, mumps, tuberculosis, adenovirus, Haemophilus influenzae type B, mumps, pertussis, plague, streptococcal pharyngitis, and scarlet fever. Strict isolation is not required for parvovirus B19.

Which play patterns does a 3-year-old child typically display (select all that apply)? a. Imaginary play b. Parallel play c. Cooperative play d. Structured play e. Associative play

A, B, C, E Children between ages 3 and 5 years enjoy parallel and associative play. Children learn to share and cooperate as they play in small groups. Play is often imitative, dramatic, and creative. Imaginary friends are common around age 3 years. Structured play is typical of school-age children.

Which toys should a nurse provide to promote imaginative play for a 3-year-old hospitalized child (select all that apply)? a. Plastic telephone b. Hand puppets c. Jigsaw puzzle (100 pieces) d. Farm animals and equipment e. Jump rope

A, B, D To promote imaginative play for a 3-year-old child, the nurse should provide: dress-up clothes, dolls and dollhouses, housekeeping toys, play-store toys, telephones, farm animals and equipment, village sets, trains, trucks, cars, planes, hand puppets, and medical kits. A 100-piece jigsaw puzzle and a jump rope would be appropriate for a young, school-age child but not a 3-year-old child.

What information will the nurse provide to a client who is scheduled for extracorporeal shock wave lithotripsy? Select all that apply. A. "Your urine will be strained after the procedure." B. "Be sure to finish all of your antibiotics." C. "Immediately call the health care provider if you notice bruising." D. "Remember to drink at least 3 liters of fluid a day to promote urine flow." E. "You will need to change the incisional dressing once a day."

A. "Your urine will be strained after the procedure." B. "Be sure to finish all of your antibiotics." D. "Remember to drink at least 3 liters of fluid a day to promote urine flow." After lithotripsy, urine is strained to monitor the passage of stone fragments. Clients must finish the entire antibiotic prescription to decrease the risk of developing a urinary tract infection. Drinking at least 3 L of fluid a day dilutes potential stone-forming crystals, prevents dehydration, and promotes urine flow. Bruising on the flank of the affected side is expected after lithotripsy as a result of the shock waves that break the stone into small fragments. The client must notify the health care provider if he or she develops pain, fever, chills, or difficulty with urination because these signs and symptoms may signal the beginning of an infection or the formation of another stone. There is no incision with extracorporeal shock wave lithotripsy. There may be a small incision when intracorporeal lithotripsy is performed.

What is the correct sequence used when performing an abdominal assessment? Begin with the first technique and end with the last. Provide answer using lowercase letters separated by commas (e.g., a, b, c, d). a. Auscultation b. Palpation c. Inspection d. Percussion

ANS: c, a, d, b The correct order of abdominal examination is inspection, auscultation, percussion, and palpation. Palpation is always performed last because it may distort the normal abdominal sounds. DIF: Cognitive Level: Apply REF: p. 104 TOP: Integrated Process: Nursing Process: Assessment MSC: Area of Client Needs: Health Promotion and Maintenance

During the rehabilitative phase of care, pressure dressings are primarily applied to burned areas to A. relieve pain. B. decrease blood supply to scar. C. limit motion during the healing process. D. encourage healing through scar formation.

ANS. B Uniform pressure to the scar decreases the blood supply. The use of pressure garments serves to decrease the blood supply to the hypertrophic tissue. This is done to prevent scarring and contractures. The goal of the pressure dressing is to improve the appearance of scars by decreasing the blood supply to the area. Motion is encouraged because it prevents contractures. Movement should take place to the point of pain, but no further. The goal of the pressure dressing is to minimize the development of scar tissue.

SHORT ANSWER 1. A client in the emergency department is having a stroke and the provider has prescribed the tissue plasminogen activator (t-PA) alteplase (Activase). The client weighs 146 pounds. How much medication will this client receive? (Record your answer using a whole number.) _____ mg

ANS: 60 mg The dose of t-PA is 0.9 mg/kg with a maximum dose of 90 mg. The client weighs 66.4 kg. 0.9 mg × 66.4 = 59.76 mg, which rounds to 60 mg. DIF: Applying/Application REF: 939 KEY: Neurologic disorders| thrombolytic agents| drug calculation MSC: Integrated Process: Nursing Process: Analysis NOT: Client Needs Category: Physiological Integrity: Pharmacological and Parenteral Therapies

1. A 242-pound client is being mechanically ventilated. To prevent lung injury, what setting should the nurse anticipate for tidal volume? (Record your answer using a whole number.) ___ mL

ANS: 660 mL A low tidal volume of 6 mL/kg is used to prevent lung injury. 242 pounds = 110 kg. 110 kg × 6 mL/kg = 660 mL.

The nurse preparing a nutritional teaching plan for the parents of a preschool child should include which information? A. The quality of the food consumed is more important than the quantity. B. Nutrition requirements for preschoolers are very different from requirements for toddlers. C. Requirement for calories per unit of body weight increases slightly during the preschool period. D. Average daily intake of preschoolers should be about 3000 calories.

A. The quality of the food consumed is more important than the quantity. It is essential that the child eat a balanced diet with essential nutrients; the amount of food is less important than the quality of the food. Requirements are similar for both toddlers and preschoolers. The caloric requirement decreases slightly for preschoolers. The average intake is about 1800 calories each day for preschoolers.

Which statement is correct about young children who report sexual abuse? A. They may exhibit various behavioral manifestations. B. In most cases the child has fabricated the story. C. Their stories are not believed unless other evidence is apparent. D. They should be able to retell the story the same way to another person.

A. They may exhibit various behavioral manifestations.

What is the most important nursing consideration in the management of cellulitis? A. Application of Burow solution compresses B. Administration of oral or parenteral antibiotics C. Topical application of an antibiotic D. Incision and drainage of severe lesions

ANS. B Oral or parenteral antibiotics are indicated depending on the extent of the cellulitis. Warm water compresses may be indicated for limited cellulitis. Antibiotics need to be administered systemically (orally or parenterally), not topically. If incision and drainage are implemented, there is a risk of spreading infection or making the lesion worse.

The nurse is assessing a preschool age child who is stuttering when answering the nurse's questions. The nurse should offer alternate methods of responding to the stuttering when observing the parent: A. completing the child's sentences. B. listening attentively. C. encouraging the child to speak slowly. D. helping the child relax.

A. completing the child's sentences.

The nurse is caring for a 12-year-old who sustained major burns when putting charcoal lighter on a campfire. The nurse observes that the child is "very brave" and appears to accept pain with little or no response. What is the most appropriate nursing action related to this? A. Request a psychological consultation. B. Ask the child why the child does not have pain. C. Praise the child for the ability to withstand pain. D. Encourage continued bravery as a coping strategy.

ANS. A A psychological consultation will assist the child in verbalizing fears. This age group is very concerned with physical appearance. The psychologist can help integrate the issues the child is facing. It is likely that the child is having pain but not acknowledging the pain. Speaking with a psychologist might assist the child in relaying his or her fears and pain. If the child is feeling pain, the nurse should not praise the child for hiding the pain. The nurse should encourage the child to speak up during painful episodes so that the pain can be managed appropriately. Bravery may not be an effective coping strategy if the child is in severe pain.

A child has been stung by a bee and the parents call the walk in clinic asking for instructions on what to do as they make their way to the clinic. The nurse responds by stating? A. Tell the parents to remove all of the child's clothing and apply warm water to the affected area. B. Remove the stinger from the site. C. Encourage the child to take slow deep breaths to minimize associuated anxiety that has occured due to the event. D. Have the parents offer the child water.

ANS. B First action is to remove the stinger, then cleanse the area with soap and water and apply a cool compress. There is no need to remove the child's clothing or provide fluid hydration. There is no indication tha the child is experiencing any evidence of anxiety provided by the parent's communication.

A child is brought to the emergency department after falling down the basement stairs. On assessment, what findings may cause the nurse to suspect child abuse? Select all that apply. A. The child's bruises are located only on the right arm and leg. B. The child is brought to the emergency department by an unrelated adult. C. The child has a history of a broken arm last year from falling off a swing. D. The child's caregiver is anxious that the child get immediate medical attention. E. The child has red, green, and yellow bruises on more than one plane of the body.

ANS. B, E A child brought to a health care provider for a trauma or suspicious injury by an unrelated adult or if the primary care provider is totally unavailable is a warning sign of abuse. Varying degrees of healing of bruises in more than one plane of the body is a warning of abuse. Falling down stairs can be an unintentional injury. A child with an isolated documented injury is not a warning sign of abuse. Multiple fractures of differing ages are a warning sign of abuse. An anxious caregiver is a normal response for an injured child. A delay in seeking care is a warning sign of abuse.

Which statement by a student nurse indicates that additional instruction is needed regarding topical agents being used to treat burns? A. They eliminate bacterial growth but do not remove the bacteria from the skin B. They are not considered to be toxic substances C. They are associated with electrolyte derangement of surronding tissues D. They are able to penetrate through eschar levels to reach the wound

ANS. C Topical agents used in the treatment of burns should provide minimal electrolyte derangement. The other options stated are all consistent with the expceted actions of topical agents uised in the treatment of burns.

The school nurse is seeing a child who brought poison ivy to school in a leaf collection. The child says that only hands touched it. The most appropriate nursing action is to A. apply Burow solution compresses immediately. B. soak hands in warm water. C. rinse hands in cold, running water. D. scrub hands thoroughly with antibacterial soap.

ANS. C Washing the child's hands in cold running water is the recommended first action. Once contact has been made, it is desirable to flush the skin with cold running water within 15 minutes of exposure to neutralize the effect. Applying Burrow solution is effective for soothing the skin lesions once the dermatitis has begun. Antibacterial soap is not recommended as it removes protective skin oils, and may allow spread of contact.

2. A client in the emergency department is having a stroke. The client weighs 225 pounds. After the initial bolus of t-Pa, at what rate should the nurse set the IV pump? (Record your answer using a decimal rounded to the nearest tenth.) ____ mL/hr

ANS: 1.4 mL/hr The client weighs 102 kg. The dose of t-PA is 0.9 mg/kg with a maximum of 90 mg, so the client's dose is 90 mg. 10% of the dose is given as a bolus IV over the first minute (9 mg). That leaves 81 mg to run in over 59 minutes. , which rounds to 1.4 mL/hr. DIF: Applying/Application REF: 939 KEY: Neurologic disorders| drug calculation| thrombolytic agents MSC: Integrated Process: Nursing Process: Implementation NOT: Client Needs Category: Physiological Integrity: Pharmacological and Parenteral Therapies

The estimated average requirement of calcium for an adolescent is _____ milligrams. (Record your answer in a whole number.)

ANS: 1100 The EAR (estimated average requirement) for calcium in adolescents 14 to 18 years of age is 1100 mg. DIF: Cognitive Level: Understand REF: p. 459 TOP: Integrated Process: Nursing Process: Planning MSC: Area of Client Needs: Health Promotion and Maintenance

COMPLETION 1. A nurse prepares a client with acute renal insufficiency for a cardiac catheterization. The provider prescribes 0.9% normal saline to infuse at 125 mL/hr for renal protection. The nurse obtains gravity tubing with a drip rate of 15 drops/mL. At what rate (drops/min) should the nurse infuse the fluids? (Record your answer using a whole number, and rounding to the nearest drop.) _____ drops/min

ANS: 31 drops/min DIF: Applying/Application REF: 641 KEY: Medication administration MSC: Integrated Process: Nursing Process: Implementation NOT: Client Needs Category: Physiological Integrity: Pharmacological and Parenteral Therapies

1. An emergency department nurse cares for a client who is severely dehydrated and is prescribed 3 L of intravenous fluid over 6 hours. At what rate (mL/hr) should the nurse set the intravenous pump to infuse the fluids? (Record your answer using a whole number.) ____ mL/hr

ANS: 500 mL/hr Because IV pumps deliver in units of milliliters per hour, the pump would have to be set at 500 mL/hr to deliver 3 L (3000 mL) over 6 hours. 6x = 3000 x = 500

Place in order the correct sequence for emergency treatment of poisoning in a child. Provide answer using lowercase letters separated by commas (e.g., a, b, c, d). a. Locate the poison. b. Assess the child. c. Prevent absorption of poison. d. Terminate exposure to the toxic substance.

ANS: b, d, a, c The initial step in treating poisonings is to assess the child, treat immediate life-threatening conditions, and initiate cardiopulmonary resuscitation (CPR) if indicated. Terminating the exposure to the toxic substance is the second step. Locating the poison for identification is the third step. Preventing absorption of poison is the fourth step.

The nurse is caring for a child with suspected ingestion of some type of poison. What action should the nurse take next after initiating cardiopulmonary resuscitation (CPR)? a. Empty the mouth of pills, plants, or other material. b. Question the victim and witness. c. Place the child in a side-lying position. d. Call poison control.

ANS: A Emptying the mouth of any leftover pills, plants, or other ingested material is the next step after assessment and initiation of CPR if needed. Questioning the victim and witnesses, calling poison control, and placing the child in a side-lying position are follow-up steps.

The nurse is teaching parents of a preschool child strategies to implement when the child delays going to bed. What strategy should the nurse recommend? a. Use consistent bedtime rituals. b. Give in to attention-seeking behavior. c. Take the child into the parents bed for an hour. d. Allow the child to stay up past the decided bedtime.

ANS: A For children who delay going to bed, a recommended approach involves a consistent bedtime ritual and emphasizing the normalcy of this type of behavior in young children. Parents should ignore attention-seeking behavior, and the child should not be taken into the parents bed or allowed to stay up past a reasonable hour.

A nurse assesses a client who has a chronic wound. The client states, "I do not clean the wound and change the dressing every day because it costs too much for supplies." How should the nurse respond? a."You can use tap water instead of sterile saline to clean your wound." b."If you don't clean the wound properly, you could end up in the hospital." c."Sterile procedure is necessary to keep this wound from getting infected." d."Good hand hygiene is the only thing that really matters with wound care."

ANS: A For chronic wounds in the home, clean tap water and nonsterile supplies are acceptable and serve as cheaper alternatives to sterile supplies. Of course, if the wound becomes grossly infected, the client may end up in the hospital, but this response does not provide any helpful information. Good handwashing is important, but it is not the only consideration.

A child is admitted with a suspected diagnosis of Munchausen syndrome by proxy (MSBP). What is an important consideration in the care of this child? a. Monitoring the parents whenever they are with the child b. Reassuring the parents that the cause of the disorder will be found c. Teaching the parents how to obtain necessary specimens d. Supporting the parents as they cope with diagnosis of a chronic illness

ANS: A MSBP refers to an illness that one person fabricates or induces in another. The child must be continuously observed for development of symptoms to determine the cause. MSBP is caused by an individual harming the child for the purpose of gaining attention. Nursing staff should obtain all specimens for analyzing. This minimizes the possibility of the abuser contaminating the sample. The child must be supported through the diagnosis of MSBP. The abuser must be identified and the child protected from that individual.

The nurse is using calipers to measure skinfold thickness over the triceps muscle in a school-age child. What is the purpose of doing this? a. To measure body fat b. To measure muscle mass c. To determine arm circumference d. To determine accuracy of weight measurement

ANS: A Measurement of skinfold thickness is an indicator of body fat. Arm circumference is an indirect measure of muscle mass. The accuracy of weight measurement should be verified with a properly balanced scale. Body fat is just one indicator of weight. DIF: Cognitive Level: Remember REF: p. 80 TOP: Integrated Process: Nursing Process: Assessment MSC: Area of Client Needs: Health Promotion and Maintenance

11. A client is receiving oxygen at 4 liters per nasal cannula. What comfort measure may the nurse delegate to unlicensed assistive personnel (UAP)? a. Apply water-soluble ointment to nares and lips. b. Periodically turn the oxygen down or off. c. Remove the tubing from the client's nose. d. Turn the client every 2 hours or as needed.

ANS: A Oxygen can be drying, so the UAP can apply water-soluble lubricant to the client's lips and nares. The UAP should not adjust the oxygen flow rate or remove the tubing. Turning the client is not related to comfort measures for oxygen. DIF: Applying/Application REF: 515 KEY: Oxygen| comfort measures| oral care| skin care| delegation MSC: Integrated Process: Communication and Documentation NOT: Client Needs Category: Physiological Integrity: Basic Care and Comfort

Which is a common characteristic of those who sexually abuse children? a. Pressure victim into secrecy b. Are usually unemployed and unmarried c. Are unknown to victims and victims' families d. Have many victims that are each abused once only

ANS: A Sex offenders may pressure the victim into secrecy regarding the activity as a "secret between us" that other people may take away if they find out. The offender may be anyone, including family members and persons from any level of society. Sex offenders are usually trusted acquaintances of the victims and victims' families. Many victims are abused many times over a long period.

15. A nurse cares for a client with a 40-year smoking history who is experiencing distended neck veins and dependent edema. Which physiologic process should the nurse correlate with this client's history and clinical manifestations? a. Increased pulmonary pressure creating a higher workload on the right side of the heart b. Exposure to irritants resulting in increased inflammation of the bronchi and bronchioles c. Increased number and size of mucus glands producing large amounts of thick mucus d. Left ventricular hypertrophy creating a decrease in cardiac output

ANS: A Smoking increases pulmonary hypertension, resulting in cor pulmonale, or right-sided heart failure. Increased pressures in the lungs make it more difficult for blood to flow through the lungs. Blood backs up into the right side of the heart and then into the peripheral venous system, creating distended neck veins and dependent edema. Inflammation in bronchi and bronchioles creates an airway obstruction which manifests as wheezes. Thick mucus in the lungs has no impact on distended neck veins and edema. Left ventricular hypertrophy is associated with left heart failure and is not caused by a 40-year smoking history. DIF: Remembering/Knowledge REF: 58 KEY: Heart failure| cor pulmonale MSC: Integrated Process: Nursing Process: Implementation NOT: Client Needs Category: Physiological Integrity: Physiological Adaptation

A nurse who manages client placements prepares to place four clients on a medical-surgical unit. Which client should be placed in isolation awaiting possible diagnosis of infection with methicillin-resistant Staphylococcus aureus(MRSA)? a.Client admitted from a nursing home with furuncles and folliculitis b.Client with a leg cut and other trauma from a motorcycle crash c.Client with a rash noticed after participating in sporting events d.Client transferred from intensive care with an elevated white blood cell count

ANS: A The client in long-term care and other communal environments is at high risk for MRSA. The presence of furuncles and folliculitis is also an indication that MRSA may be present. A client with an open wound from a motorcycle crash would have the potential to develop MRSA, but no signs are visible at present. The rash following participation in a sporting event could be caused by several different things. A client with an elevated white blood cell count has the potential for infection but should be at lower risk for MRSA than the client admitted from the communal environment.

A nurse assesses an older client who is scratching and rubbing white ridges on the skin between the fingers and on the wrists. Which action should the nurse take? a.Place the client in a single room. b.Administer an antihistamine. c.Assess the client's airway. d.Apply gloves to minimize friction.

ANS: A The client's presentation is most likely to be scabies, a contagious mite infestation. The client needs to be admitted to a single room and treated for the infestation. Secondary interventions may include medication to decrease the itching. This is not an allergic manifestation; therefore, antihistamine and airway assessments are not indicated. Gloves may decrease skin breakdown but would not address the client's infectious disorder.

A nurse prepares to discharge a client who has a wound and is prescribed home health care. Which information should the nurse include in the hand-off report to the home health nurse? a.Recent wound assessment, including size and appearance b.Insurance information for billing and coding purposes c.Complete health history and physical assessment findings d.Resources available to the client for wound care supplies

ANS: A The hospital nurse should provide details about the wound, including size and appearance and any special wound needs, in a hand-off report to the home health nurse. Insurance information is important to the home health agency and manager, but this is not appropriate during this hand-off report. The nurse should report focused assessment findings instead of a complete health history and physical assessment. The home health nurse should work with the client to identify community resources.

What statement is correct about young children who report sexual abuse? a. They may exhibit various behavioral manifestations. b. In more than half the cases, the child has fabricated the story. c. Their stories should not be believed unless other evidence is apparent. d. They should be able to retell the story the same way to another person.

ANS: A Victims of sexual abuse have no typical profile. The child may exhibit various behavioral manifestations, none of which is diagnostic for sexual abuse. When children report potentially sexually abusive experiences, their reports need to be taken seriously. Other children in the household also need to be evaluated. In children who are sexually abused, it is often difficult to identify other evidence. In one study, approximately 96% of children who were sexually abused had normal genital and anal findings. The ability to retell the story is partly dependent on the childs cognitive level. Children who repeatedly tell identical stories may have been coached.

A nurse cares for a client with ulcerative colitis. The client states, "I feel like I am tied to the toilet. This disease is controlling my life." How should the nurse respond? a. "Let's discuss potential factors that increase your symptoms." b. "If you take the prescribed medications, you will no longer have diarrhea." c. "To decrease distress, do not eat anything before you go out." d. "You must retake control of your life. I will consult a therapist to help."

ANS: A Clients with ulcerative colitis often express that the disorder is disruptive to their lives. Stress factors can increase symptoms. These factors should be identified so that the client will have more control over his or her condition. Prescription medications and anorexia will not eliminate exacerbations. Although a therapist may assist the client, this is not an appropriate response. DIF: Applying/Application REF: 1180 KEY: Ulcerative colitis| coping MSC: Integrated Process: Caring NOT: Client Needs Category: Psychosocial Integrity

A nurse reviews the chart of a client who has Crohn's disease and a draining fistula. Which documentation should alert the nurse to urgently contact the provider for additional prescriptions? a. Serum potassium of 2.6 mEq/L b. Client ate 20% of breakfast meal c. White blood cell count of 8200/mm3 d. Client's weight decreased by 3 pounds

ANS: A Fistulas place the client with Crohn's disease at risk for hypokalemia which can lead to serious dysrhythmias. This potassium level is low and should cause the nurse to intervene. The white blood cell count is normal. The other two findings are abnormal and also warrant intervention, but the potassium level takes priority. DIF: Applying/Application REF: 1184 KEY: Crohn's disease| electrolyte imbalance MSC: Integrated Process: Nursing Process: Analysis NOT: Client Needs Category: Physiological Integrity: Reduction of Risk Potential

A nurse cares for a client who has a Giardia infection. Which medication should the nurse anticipate being prescribed for this client? a. Metronidazole (Flagyl) b. Ciprofloxacin (Cipro) c. Sulfasalazine (Azulfidine) d. Ceftriaxone (Rocephin)

ANS: A Metronidazole is the drug of choice for a Giardia infection. Ciprofloxacin and ceftriaxone are antibiotics used for bacterial infections. Sulfasalazine is used for ulcerative colitis and Crohn's disease. DIF: Remembering/Knowledge REF: 1190 KEY: Parasitic infection| medication MSC: Integrated Process: Nursing Process: Planning NOT: Client Needs Category: Physiological Integrity: Pharmacological and Parenteral Therapies

3. A client has a tracheostomy that is 3 days old. Upon assessment, the nurse notes the client's face is puffy and the eyelids are swollen. What action by the nurse takes priority? a. Assess the client's oxygen saturation. b. Notify the Rapid Response Team. c. Oxygenate the client with a bag-valve-mask. d. Palpate the skin of the upper chest.

ANS: A REF: 514 This client may have subcutaneous emphysema, which is air that leaks into the tissues surrounding the tracheostomy. The nurse should first assess the client's oxygen saturation and other indicators of oxygenation. If the client is stable, the nurse can palpate the skin of the upper chest to feel for the air. If the client is unstable, the nurse calls the Rapid Response Team. Using a bag-valve-mask device may or may not be appropriate for the unstable client. DIF: Applying/Application REF: 523 KEY: Oxygenation| tracheostomy| nursing assessment MSC: IntegratedProcess:NursingProcess:Assessment NOT: Client Needs Category: Safe and Effective Care Environment: Management of Care

A nurse assesses a client who is recovering from an ileostomy placement. Which clinical manifestation should alert the nurse to urgently contact the health care provider? a. Pale and bluish stoma b. Liquid stool c. Ostomy pouch intact d. Blood-smeared output

ANS: A The nurse should assess the stoma for color and contact the health care provider if the stoma is pale, bluish, or dark. The nurse should expect the client to have an intact ostomy pouch with dark green liquid stool that may contain some blood. DIF: Applying/Application REF: 1177 KEY: Ostomy care| postoperative nursing MSC: Integrated Process: Nursing Process: Analysis NOT: Client Needs Category: Safe and Effective Care Environment: Management of Care

A nurse assesses a client with Crohn's disease and colonic strictures. Which clinical manifestation should alert the nurse to urgently contact the health care provider? a. Distended abdomen b. Temperature of 100.0° F (37.8° C) c. Loose and bloody stool d. Lower abdominal cramps

ANS: A The presence of strictures predisposes the client to intestinal obstruction. Abdominal distention may indicate that the client has developed an obstruction of the large bowel, and the client's provider should be notified right away. Low-grade fever, bloody diarrhea, and abdominal cramps are common symptoms of Crohn's disease. DIF: Applying/Application REF: 1179 KEY: Crohn's disease| assessment/diagnostic examination MSC: Integrated Process: Nursing Process: Analysis NOT: Client Needs Category: Physiological Integrity: Reduction of Risk Potential

12. A nurse is caring for four clients in the neurologic intensive care unit. After receiving the hand-off report, which client should the nurse see first? a. Client with a Glasgow Coma Scale score that was 10 and is now is 8 b. Client with a Glasgow Coma Scale score that was 9 and is now is 12 c. Client with a moderate brain injury who is amnesic for the event d. Client who is requesting pain medication for a headache

ANS: A A 2-point decrease in the Glasgow Coma Scale score is clinically significant and the nurse needs to see this client first. An improvement in the score is a good sign. Amnesia is an expected finding with brain injuries, so this client is lower priority. The client requesting pain medication should be seen after the one with the declining Glasgow Coma Scale score. DIF: Applying/Application REF: 952 KEY: Neurologic disorders| neurologic assessment| critical rescue MSC: Integrated Process: Nursing Process: Analysis NOT: Client Needs Category: Safe and Effective Care Environment: Management of Care

During a funduscopic examination of a school-age child, the nurse notes a brilliant, uniform red reflex in both eyes. How should the nurse interpret this finding? a. Normal finding b. Abnormal finding, so child needs referral to ophthalmologist c. Sign of possible visual defect, so child needs vision screening d. Sign of small hemorrhages, which will usually resolve spontaneously

ANS: A A brilliant, uniform red reflex is an important normal finding. It rules out many serious defects of the cornea, aqueous chamber, lens, and vitreous chamber. DIF: Cognitive Level: Understand REF: p. 91 TOP: Integrated Process: Nursing Process: Assessment MSC: Area of Client Needs: Health Promotion and Maintenance

A nurse prepares to discharge a client with chronic pancreatitis. Which question should the nurse ask to ensure safety upon discharge? a. "Do you have a one- or two-story home?" b. "Can you check your own pulse rate?" c. "Do you have any alcohol in your home?" d. "Can you prepare your own meals?"

ANS: A A client recovering from chronic pancreatitis should be limited to one floor until strength and activity increase. The client will need a bathroom on the same floor for frequent defecation. Assessing pulse rate and preparation of meals is not specific to chronic pancreatitis. Although the client should be encouraged to stop drinking alcoholic beverages, asking about alcohol availability is not adequate to assess this client's safety.

24. A nurse auscultates a client's lung fields. Which pathophysiologic process should the nurse associate with this breath sound? (Click the media button to hear the audio clip.) a. Inflammation of the pleura b. Constriction of the bronchioles c. Upper airway obstruction d. Pulmonary vascular edema

ANS: A A pleural friction rub can be heard when the pleura is inflamed and rubbing against the lung wall. The other pathophysiologic processes would not cause a pleural friction rub. Constriction of the bronchioles may be heard as a wheeze, upper airway obstruction may be heard as stridor, and pulmonary vascular edema may be heard as crackles. DIF: Applying/Application REF: 575 KEY: Assessment/diagnostic examination MSC: Integrated Process: Nursing Process: Implementation NOT: Client Needs Category: Physiological Integrity: Physiological Adaptation

20. A client has been diagnosed with a very large pulmonary embolism (PE) and has a dropping blood pressure. What medication should the nurse anticipate the client will need as the priority? a. Alteplase (Activase) b. Enoxaparin (Lovenox) c. Unfractionated heparin d. Warfarin sodium (Coumadin)

ANS: A Activase is a "clot-busting" agent indicated in large PEs in the setting of hemodynamic instability. The nurse knows this drug is the priority, although heparin may be started initially. Enoxaparin and warfarin are not indicated in this setting.

Which aspect of cognition develops during adolescence? a. Capability to use a future time perspective b. Ability to place things in a sensible and logical order c. Ability to see things from the point of view of another d. Progress from making judgments based on what they see to making judgments based on what they reason

ANS: A Adolescents are no longer restricted to the real and actual. They also are concerned with the possible; they think beyond the present. During concrete operations (between ages 7 and 11 years), children exhibit these characteristic thought processes. DIF: Cognitive Level: Remember REF: p. 449 TOP: Integrated Process: Nursing Process: Assessment MSC: Area of Client Needs: Health Promotion and Maintenance

21. A nurse admits a client from the emergency department. Client data are listed below: History Physical Assessment Laboratory Values: 70 years of age History of diabetes On insulin twice a day Reports new-onset dyspnea and productive cough Crackles and rhonchi heard throughout the lungs Dullness to percussion LLL Afebrile Oriented to person only WBC: 5,200/mm3 PaO2 on room air 65mm Hg What action by the nurse is the priority? a. Administer oxygen at 4 liters per nasal cannula. b. Begin broad-spectrum antibiotics. c. Collect a sputum sample for culture. d. Start an IV of normal saline at 50 mL/hr.

ANS: A All actions are appropriate for this client who has manifestations of pneumonia. However, airway and breathing come first, so begin oxygen administration and titrate it to maintain saturations greater than 95%. Start the IV and collect a sputum culture, and then begin antibiotics.

The nurse is working with a client who has rheumatoid arthritis (RA). The nurse has identified the priority problem of poor body image for the client. What finding by the nurse indicates goals for this client problem are being met? a. Attends meetings of a book club b. Has a positive outlook on life c. Takes medication as directed d. Uses assistive devices to protect joints

ANS: A All of the activities are appropriate for a client with RA. Clients who have a poor body image are often reluctant to appear in public, so attending public book club meetings indicates that goals for this client problem are being met.

19. A client in the emergency department has several broken ribs. What care measure will best promote comfort? a. Allowing the client to choose the position in bed b. Humidifying the supplemental oxygen c. Offering frequent, small drinks of water d. Providing warmed blankets

ANS: A Allow the client with respiratory problems to assume a position of comfort if it does not interfere with care. Often the client will choose a more upright position, which also improves oxygenation. The other options are less effective comfort measures.

1. A nurse assesses clients on a cardiac unit. Which client should the nurse identify as being at greatest risk for the development of left-sided heart failure? a. A 36-year-old woman with aortic stenosis b. A 42-year-old man with pulmonary hypertension c. A 59-year-old woman who smokes cigarettes daily d. A 70-year-old man who had a cerebral vascular accident

ANS: A Although most people with heart failure will have failure that progresses from left to right, it is possible to have left-sided failure alone for a short period. It is also possible to have heart failure that progresses from right to left. Causes of left ventricular failure include mitral or aortic valve disease, coronary artery disease, and hypertension. Pulmonary hypertension and chronic cigarette smoking are risk factors for right ventricular failure. A cerebral vascular accident does not increase the risk of heart failure. DIF: Applying/Application REF: 679 KEY: Heart failure| health screening MSC: IntegratedProcess:NursingProcess:Assessment NOT: Client Needs Category: Safe and Effective Care Environment: Management of Care

2. A client had an embolic stroke and is having an echocardiogram. When the client asks why the provider ordered "a test on my heart," how should the nurse respond? a. "Most of these types of blood clots come from the heart." b. "Some of the blood clots may have gone to your heart too." c. "We need to see if your heart is strong enough for therapy." d. "Your heart may have been damaged in the stroke too."

ANS: A An embolic stroke is caused when blood clots travel from one area of the body to the brain. The most common source of the clots is the heart. The other statements are inaccurate. DIF: Understanding/Comprehension REF: 931 KEY: Neurologic disorders| stroke| patient education MSC: IntegratedProcess:Teaching/Learning NOT: Client Needs Category: Physiological Integrity: Physiological Adaptation

6. A nurse is teaching a client with heart failure who has been prescribed enalapril (Vasotec). Which statement should the nurse include in this client's teaching? a. "Avoid using salt substitutes." b. "Take your medication with food." c. "Avoid using aspirin-containing products." d. "Check your pulse daily."

ANS: A Angiotensin-converting enzyme (ACE) inhibitors such as enalapril inhibit the excretion of potassium. Hyperkalemia can be a life-threatening side effect, and clients should be taught to limit potassium intake. Salt substitutes are composed of potassium chloride. ACE inhibitors do not need to be taken with food and have no impact on the client's pulse rate. Aspirin is often prescribed in conjunction with ACE inhibitors and is not contraindicated. DIF: Applying/Application REF: 685 KEY: Heart failure| angiotensin-converting enzyme (ACE) inhibitor| medication| patient education MSC: IntegratedProcess:Teaching/Learning NOT: Client Needs Category: Physiological Integrity: Pharmacological and Parenteral Therapies

11. A nurse admits a client who is experiencing an exacerbation of heart failure. Which action should the nurse take first? a. Assess the client's respiratory status. b. Draw blood to assess the client's serum electrolytes. c. Administer intravenous furosemide (Lasix). d. Ask the client about current medications.

ANS: A Assessment of respiratory and oxygenation status is the priority nursing intervention for the prevention of complications. Monitoring electrolytes, administering diuretics, and asking about current medications are important but do not take priority over assessing respiratory status. DIF: Applying/Application REF: 687 KEY: Heart failure| respiratory distress/failure| assessment/diagnostic examination MSC: Integrated Process: Nursing Process: Implementation NOT: Client Needs Category: Safe and Effective Care Environment: Management of Care

A nurse is preparing to assess a 3-year-old child. What communication technique should the nurse use for this child? a. Focus communication on child. b. Explain experiences of others to child. c. Use easy analogies when possible. d. Assure child that communication is private.

ANS: A Because children of this age are able to see things only in terms of themselves, the best approach is to focus communication directly on them. Children should be provided with information about what they can do and how they will feel. With children who are egocentric, experiences of others, analogies, and assurances that the communication is private will not be effective because the child is not capable of understanding. DIF: Cognitive Level: Apply REF: p. 61 TOP: Integrated Process: Communication and Documentation MSC: Area of Client Needs: Psychosocial Integrity

21. A client has a brain abscess and is receiving phenytoin (Dilantin). The spouse questions the use of the drug, saying the client does not have a seizure disorder. What response by the nurse is best? a. "Increased pressure from the abscess can cause seizures." b. "Preventing febrile seizures with an abscess is important." c. "Seizures always occur in clients with brain abscesses." d. "This drug is used to sedate the client with an abscess."

ANS: A Brain abscesses can lead to seizures as a complication. The nurse should explain this to the spouse. Phenytoin is not used to prevent febrile seizures. Seizures are possible but do not always occur in clients with brain abscesses. This drug is not used for sedation. DIF: Understanding/Comprehension REF: 955 KEY: Neurologic disorders| antiseizure medications MSC: IntegratedProcess:Teaching/Learning NOT: Client Needs Category: Physiological Integrity: Pharmacological and Parenteral Therapies

A client who has had systemic lupus erythematosus (SLE) for many years is in the clinic reporting hip pain with ambulation. Which action by the nurse is best? a. Assess medication records for steroid use. b. Facilitate a consultation with physical therapy. c. Measure the range of motion in both hips. d. Notify the health care provider immediately.

ANS: A Chronic steroid use is seen in clients with SLE and can lead to osteonecrosis (bone necrosis). The nurse should determine if the client has been taking a steroid. Physical therapy may be beneficial, but there is not enough information about the client yet. Measuring range of motion is best done by the physical therapist. Notifying the provider immediately is not warranted.

8. A nurse and a registered dietitian are assessing clients for partial parenteral nutrition (PPN). For which client would the nurse suggest another route of providing nutrition? a. Client with congestive heart failure b. Older client with dementia c. Client who has multiorgan failure d. Client who is post gastric resection

ANS: A Clients receiving PPN typically get large amounts of fluid volume, making the client with heart failure a poor candidate. The other candidates are appropriate for this type of nutritional support.

A nurse is planning a teaching session for a group of adolescents. The nurse understands that by adolescence the individual is in which stage of cognitive development? a. Formal operations b. Concrete operations c. Conventional thought d. Post-conventional thought

ANS: A Cognitive thinking culminates with capacity for abstract thinking. This stage, the period of formal operations, is Piaget's fourth and last stage. Concrete operations usually occur between ages 7 and 11 years. Conventional and postconventional thought refers to Kohlberg's stages of moral development. DIF: Cognitive Level: Understand REF: p. 453 TOP: Integrated Process: Nursing Process: Planning MSC: Area of Client Needs: Health Promotion and Maintenance

A parent of an infant with colic tells the nurse, "All this baby does is scream at me; it is a constant worry." What is the nurse's best action? a. Encourage parent to verbalize feelings. b. Encourage parent not to worry so much. c. Assess parent for other signs of inadequate parenting. d. Reassure parent that colic rarely lasts past age 9 months.

ANS: A Colic is multifactorial, and no single treatment is effective for all infants. The parent is verbalizing concern and worry. The nurse should allow the parent to put these feelings into words. An empathic, gentle, and reassuring attitude, in addition to suggestions about remedies, will help alleviate the parent's anxieties. The nurse should reassure the parent that he or she is not doing anything wrong. Colic is multifactorial. The infant with colic is experiencing spasmodic pain that is manifested by loud crying, in some cases up to 3 hours each day. Telling the parent that it will eventually go away does not help him or her through the current situation.

A nurse assesses a client who is recovering from a Whipple procedure. Which assessment finding alerts the nurse to urgently contact the health care provider? a. Drainage from a fistula b. Absent bowel sounds c. Pain at the incision site d. Nasogastric (NG) tube drainage

ANS: A Complications of a Whipple procedure include secretions that drain from a fistula and peritonitis. Absent bowel sounds, pain at the incision site, and NG tube drainage are normal postoperative findings.

5. A nurse cares for a client with right-sided heart failure. The client asks, "Why do I need to weigh myself every day?" How should the nurse respond? a. "Weight is the best indication that you are gaining or losing fluid." b. "Daily weights will help us make sure that you're eating properly." c. "The hospital requires that all inpatients be weighed daily." d. "You need to lose weight to decrease the incidence of heart failure."

ANS: A Daily weights are needed to document fluid retention or fluid loss. One liter of fluid equals 2.2 pounds. The other responses do not address the importance of monitoring fluid retention or loss. DIF: Remembering/Knowledge REF: 683 KEY: Heart failure| patient education MSC: Integrated Process: Teaching/Learning NOT: Client Needs Category: Physiological Integrity: Physiological Adaptation

9. A client with a stroke has damage to Broca's area. What intervention to promote communication is best for this client? a. Assess whether or not the client can write. b. Communicate using "yes-or-no" questions. c. Reinforce speech therapy exercises. d. Remind the client not to use neologisms.

ANS: A Damage to Broca's area often leads to expressive aphasia, wherein the client can understand what is said but cannot express thoughts verbally. In some instances the client can write. The nurse should assess to see if that ability is intact. "Yes-or-no" questions are not good for this type of client because he or she will often answer automatically but incorrectly. Reinforcing speech therapy exercises is good for all clients with communication difficulties. Neologisms are made-up "words" often used by clients with sensory aphasia. DIF: Applying/Application REF: 943 KEY: Neurologic disorders| stroke| communication MSC: Integrated Process: Nursing Process: Implementation NOT: Client Needs Category: Psychosocial Integrity

21. A nurse cares for an older adult client with heart failure. The client states, "I don't know what to do. I don't want to be a burden to my daughter, but I can't do it alone. Maybe I should die." How should the nurse respond? a. "Would you like to talk more about this?" b. "You are lucky to have such a devoted daughter." c. "It is normal to feel as though you are a burden." d. "Would you like to meet with the chaplain?"

ANS: A Depression can occur in clients with heart failure, especially older adults. Having the client talk about his or her feelings will help the nurse focus on the actual problem. Open-ended statements allow the client to respond safely and honestly. The other options minimize the client's concerns and do not allow the nurse to obtain more information to provide client-centered care. DIF: Applying/Application REF: 683 KEY: Heart failure| support| psychosocial response MSC: IntegratedProcess:Caring NOT: Client Needs Category: Psychosocial Integrity

A client takes celecoxib (Celebrex) for chronic osteoarthritis in multiple joints. After a knee replacement, the health care provider has prescribed morphine sulfate for postoperative pain relief. The client also requests the celecoxib in addition to the morphine. What action by the nurse is best? a. Consult with the health care provider about administering both drugs to the client. b. Inform the client that the celecoxib will be started when he or she goes home. c. Teach the client that, since morphine is stronger, celecoxib is not needed. d. Tell the client he or she should not take both drugs at the same time.

ANS: A Despite getting an opioid analgesic for postoperative pain, the nurse should be aware that the client may be on other medications for arthritis in other joints. The nonsteroidal anti-inflammatory drug celecoxib will also help with the postoperative pain. The nurse should consult the provider about continuing the celecoxib while the client is in the hospital. The other responses are not warranted, as the client should be restarted on this medication postoperatively.

5. An emergency room nurse obtains the health history of a client. Which statement by the client should alert the nurse to the occurrence of heart failure? a. "I get short of breath when I climb stairs." b. "I see halos floating around my head." c. "I have trouble remembering things." d. "I have lost weight over the past month."

ANS: A Dyspnea on exertion is an early manifestation of heart failure and is associated with an activity such as stair climbing. The other findings are not specific to early occurrence of heart failure. DIF: Applying/Application REF: 635 KEY: Health screening| heart failure MSC: IntegratedProcess:NursingProcess:Assessment NOT: Client Needs Category: Health Promotion and Maintenance

5. A nurse evaluates a client with acute glomerulonephritis (GN). Which manifestation should the nurse recognize as a positive response to the prescribed treatment? a. The client has lost 11 pounds in the past 10 days. b. The client's urine specific gravity is 1.048. c. No blood is observed in the client's urine. d. The client's blood pressure is 152/88 mm Hg.

ANS: A Fluid retention is a major feature of acute GN. This weight loss represents fluid loss, indicating that the glomeruli are performing the function of filtration. A urine specific gravity of 1.048 is high. Blood is not usually seen in GN, so this finding would be expected. A blood pressure of 152/88 mm Hg is too high; this may indicate kidney damage or fluid overload.

Which nursing intervention has the highest priority for a patient diagnosed with bulimia nervosa? a. Assist the patient to identify triggers to binge eating. b. Provide corrective consequences for weight loss. c. Assess for signs of impulsive eating. d. Explore needs for health teaching.

ANS: A For most patients with bulimia nervosa, certain situations trigger the urge to binge; purging then follows. Often the triggers are anxiety-producing situations. Identification of triggers makes it possible to break the binge-purge cycle. Because binge eating and purging directly affect physical status, the need to promote physical safety assumes highest priority.

11. The emergency department (ED) manager is reviewing client charts to determine how well the staff performs when treating clients with community-acquired pneumonia. What outcome demonstrates that goals for this client type have been met? a. Antibiotics started before admission b. Blood cultures obtained within 20 minutes c. Chest x-ray obtained within 30 minutes d. Pulse oximetry obtained on all clients

ANS: A Goals for treatment of community-acquired pneumonia include initiating antibiotics prior to inpatient admission or within 6 hours of presentation to the ED. Timely collection of blood cultures, chest x-ray, and pulse oximetry are important as well but do not coincide with established goals.

A client recently diagnosed with systemic lupus erythematosus (SLE) is in the clinic for a follow-up visit. The nurse evaluates that the client practices good self-care when the client makes which statement? a. "I always wear long sleeves, pants, and a hat when outdoors." b. "I try not to use cosmetics that contain any type of sunblock." c. "Since I tend to sweat a lot, I use a lot of baby powder." d. "Since I can't be exposed to the sun, I have been using a tanning bed."

ANS: A Good self-management of the skin in SLE includes protecting the skin from sun exposure, using sunblock, avoiding drying agents such as powder, and avoiding tanning beds.

A young adolescent boy tells the nurse he "feels gawky." How should the nurse explain why this occurs in adolescents? a. Growth of the extremities and neck precedes growth in other areas b. Growth is in the trunk and chest c. The hip and chest breadth increases d. The growth spurt occurs earlier in boys than it does in girls

ANS: A Growth in length of the extremities and neck precedes growth in other areas, and, because these parts are the first to reach adult length, the hands and feet appear larger than normal during adolescence. Increases in hip and chest breadth take place in a few months followed several months later by an increase in shoulder width. These changes are followed by increases in length of the trunk and depth of the chest. This sequence of changes is responsible for the characteristic long-legged, gawky appearance of early adolescent children. The growth spurt occurs earlier in girls than in boys. DIF: Cognitive Level: Apply REF: p. 450 TOP: Integrated Process: Teaching/Learning MSC: Area of Client Needs: Health Promotion and Maintenance

24. A nurse is providing community screening for risk factors associated with stroke. Which client would the nurse identify as being at highest risk for a stroke? a. A 27-year-old heavy cocaine user b. A 30-year-old who drinks a beer a day c. A 40-year-old who uses seasonal antihistamines d. A 65-year-old who is active and on no medications

ANS: A Heavy drug use, particularly cocaine, is a risk factor for stroke. Heavy alcohol use is also a risk factor, but one beer a day is not considered heavy drinking. Antihistamines may contain phenylpropanolamine, which also increases the risk for stroke, but this client uses them seasonally and there is no information that they are abused or used heavily. The 65-year-old has only age as a risk factor. DIF: Remembering/Knowledge REF: 933 KEY: Neurologic disorders| stroke| health screening MSC: IntegratedProcess:NursingProcess:Assessment NOT: Client Needs Category: Health Promotion and Maintenance

A client comes to the family medicine clinic and reports joint pain and stiffness. The nurse is asked to assess the client for Heberden's nodules. What assessment technique is correct? a. Inspect the client's distal finger joints. b. Palpate the client's abdomen for tenderness. c. Palpate the client's upper body lymph nodes. d. Perform range of motion on the client's wrists.

ANS: A Herberden's nodules are seen in osteoarthritis and are bony nodules at the distal interphalangeal joints. To assess for this finding, the nurse inspects the client's distal fingertips. These nodules are not found in the abdomen, lymph nodes, or wrists.

The nurse has just started assessing a young child who is febrile and appears very ill. There is hyperextension of the child's head (opisthotonos) with pain on flexion. Which is the most appropriate action? a. Refer for immediate medical evaluation. b. Continue assessment to determine cause of neck pain. c. Ask parent when neck was injured. d. Record "head lag" on assessment record, and continue assessment of child.

ANS: A Hyperextension of the child's head with pain on flexion is indicative of meningeal irritation and needs immediate evaluation; it is not descriptive of head lag. The pain is indicative of meningeal irritation. No indication of injury is present. DIF: Cognitive Level: Apply REF: p. 90 TOP: Integrated Process: Nursing Process: Assessment MSC: Area of Client Needs: Health Promotion and Maintenance

A 14-year-old boy seems to be always eating, although his weight is appropriate for his height. What is the best explanation for this? a. This is normal because of increase in body mass. b. This is abnormal and suggestive of future obesity. c. His caloric intake would have to be excessive. d. He is substituting food for unfilled needs.

ANS: A In adolescence, nutritional needs are closely related to the increase in body mass. The peak requirements occur in the years of maximal growth. The caloric and protein requirements are higher than at almost any other time of life. Seemingly always eating describes the expected eating pattern for young adolescents; as long as weight and height are appropriate, obesity is not a concern. DIF: Cognitive Level: Understand REF: p. 459 TOP: Integrated Process: Nursing Process: Assessment MSC: Area of Client Needs: Health Promotion and Maintenance

A nurse provides care for an adolescent patient diagnosed with an eating disorder. Which behavior by this nurse indicates that additional clinical supervision is needed? a. The nurse interacts with the patient in a protective fashion. b. The nurse's comments to the patient are compassionate and nonjudgmental. c. The nurse teaches the patient to recognize signs of increasing anxiety and ways to intervene. d. The nurse refers the patient to a self-help group for individuals with eating disorders.

ANS: A In the effort to motivate the patient and take advantage of the decision to seek help and be healthier, the nurse must take care not to cross the line toward authoritarianism and assumption of a parental role. Protective behaviors are part of the parent's role. The helpful nurse uses a problem-solving approach and focuses on the patient's feelings of shame and low self-esteem. Referring a patient to a self-help group is an appropriate intervention.

What are symptoms of abusive head trauma (AHT) in the more severe form that may be present? (Select all that apply.) a. Seizures b. Posturing c. Tachypnea d. Tachycardia e. Altered level of consciousness

ANS: A, B, E In more severe forms, presenting symptoms of abusive head trauma may include seizures, posturing, alterations in level of consciousness, apnea, bradycardia, or death.

Which approach would be best to use to ensure a positive response from a toddler? a. Assume an eye-level position and talk quietly. b. Call the toddler's name while picking him or her up. c. Call the toddler's name and say, "I'm your nurse." d. Stand by the toddler, addressing him or her by name.

ANS: A It is important that the nurse assume a position at the child's level when communicating with the child. By speaking quietly and focusing on the child, the nurse should be able to obtain a positive response. The nurse should engage the child and inform the toddler what is going to occur. If the nurse picks up the child without explanation, the child is most likely going to become upset. The toddler may not understand the meaning of the phrase, "I'm your nurse." If a positive response is desired, the nurse should assume the child's level when speaking if possible. DIF: Cognitive Level: Apply REF: p. 60 TOP: Integrated Process: Communication and Documentation MSC: Area of Client Needs: Psychosocial Integrity

The nurse is teaching a client with gout dietary strategies to prevent exacerbations or other problems. Which statement by the nurse is most appropriate? a. "Drink 1 to 2 liters of water each day." b. "Have 10 to 12 ounces of juice a day." c. "Liver is a good source of iron." d. "Never eat hard cheeses or sardines."

ANS: A Kidney stones are common in clients with gout, so drinking plenty of water will help prevent this from occurring. Citrus juice is high in ash, which can help prevent the formation of stones, but the value of this recommendation is not clear. Clients with gout should not eat organ meats or fish with bones, such as sardines.

Three months ago a patient diagnosed with binge eating disorder weighed 198 pounds. Lorcaserin (Belviq) was prescribed. Which current assessment finding indicates the need for reevaluation of this treatment approach? The patient: a. now weighs 196 pounds. b. says, "I am using contraceptives." c. says, "I feel full after eating a small meal." d. reports problems with dry mouth and constipation.

ANS: A Lorcaserin is designed to make people feel full after eating smaller meals by activating a serotonin 2c receptor in the brain and blocking appetite signals. According to the FDA, this drug should be stopped if a patient does not have 5% weight loss after 12 weeks of use. If the patient now weighs 196 pounds, the medication has not been effective. The distracters indicate patient learning was effective and expected side effects of this medication.

The nurse on an inpatient rheumatology unit receives a hand-off report on a client with an acute exacerbation of systemic lupus erythematosus (SLE). Which reported laboratory value requires the nurse to assess the client further? a. Creatinine: 3.9 mg/dL b. Platelet count: 210,000/mm3 c. Red blood cell count: 5.2/mm3 d. White blood cell count: 4400/mm3

ANS: A Lupus nephritis is the leading cause of death in clients with SLE. The creatinine level is very high and the nurse needs to perform further assessments related to this finding. The other laboratory values are normal.

A nurse is preparing to administer an oral iron supplement to a hospitalized infant. Which should not be given simultaneously with the iron supplement? a. Milk b. Multivitamin c. Fruit juice d. Meat, fish, poultry

ANS: A Many foods interfere with iron absorption and should be avoided when the iron is consumed. These foods include phosphates found in milk, phytates found in cereals, and oxalates found in many vegetables. Multivitamins may contain iron; no contraindication exists to taking the two together. Vitamin C-containing juices enhance the absorption of iron. Meat, fish, and poultry do not have an effect on absorption.

A nurse is preparing to accompany a medical mission's team to a third world country. Marasmus is seen frequently in children 6 months to 2 years in this country. Which symptoms should the nurse expect for this condition? a. Loose, wrinkled skin b. Edematous skin c. Depigmentation of the skin d. Dermatoses

ANS: A Marasmus is characterized by gradual wasting and atrophy of body tissues, especially of subcutaneous fat. The child appears to be very old, with loose and wrinkled skin, unlike the child with kwashiorkor, who appears more rounded from the edema. Fat metabolism is less impaired than in kwashiorkor; thus, deficiency of fat-soluble vitamins is usually minimal or absent. In general, the clinical manifestations of marasmus are similar to those seen in kwashiorkor with the following exceptions: With marasmus, there is no edema from hypoalbuminemia or sodium retention, which contributes to a severely emaciated appearance; no dermatoses caused by vitamin deficiencies; little or no depigmentation of hair or skin; moderately normal fat metabolism and lipid absorption; and a smaller head size and slower recovery after treatment.

5. Nonpharmacologic strategies for pain management: a. may reduce pain perception. b. make pharmacologic strategies unnecessary. c. usually take too long to implement. d. trick children into believing they do not have pain.

ANS: A Nonpharmacologic techniques provide coping strategies that may help reduce pain perception, make the pain more tolerable, decrease anxiety, and enhance the effectiveness of analgesics. Nonpharmacologic techniques should be learned before the pain occurs. With severe pain, it is best to use both pharmacologic and nonpharmacologic measures for pain control. The nonpharmacologic strategy should be matched with the child's pain severity and taught to the child before the onset of the painful experience. Some of the techniques may facilitate the child's experience with mild pain, but the child will still know the discomfort was present. DIF: Cognitive Level: Understand REF: p. 124 TOP: Integrated Process: Nursing Process: Planning MSC: Area of Client Needs: Physiologic Integrity

12. A nurse assesses a client who is recovering from a myocardial infarction. The client's pulmonary artery pressure reading is 25/12 mm Hg. Which action should the nurse take first? a. Compare the results with previous pulmonary artery pressure readings. b. Increase the intravenous fluid rate because these readings are low. c. Immediately notify the health care provider of the elevated pressures. d. Document the finding in the client's chart as the only action.

ANS: A Normal pulmonary artery pressures range from 15 to 26 mm Hg for systolic and from 5 to 15 mm Hg for diastolic. Although this client's readings are within normal limits, the nurse needs to assess any trends that may indicate a need for medical treatment to prevent complications. There is no need to increase intravenous fluids or notify the provider. DIF: Applying/Application REF: 638 KEY: Coronary perfusion| assessment/diagnostic examination| vascular perfusion MSC: Integrated Process: Nursing Process: Implementation NOT: Client Needs Category: Safe and Effective Care Environment: Management of Care

Clinical manifestations of failure to thrive caused by behavioral problems resulting in inadequate intake of calories include: a. avoidance of eye contact. b. an associated malabsorption defect. c. weight that falls below the 15th percentile. d. normal achievement of developmental landmarks.

ANS: A One of the clinical manifestations of nonorganic failure to thrive is the child's avoidance of eye contact with the health professional. A malabsorption defect would result in a physiologic problem, not behavioral. Weight (but not height) below the 5th percentile is indicative of failure to thrive. Developmental delays, including social, motor, adaptive, and language, exist.

A client in the orthopedic clinic has a self-reported history of osteoarthritis. The client reports a low-grade fever that started when the weather changed and several joints started "acting up," especially both hips and knees. What action by the nurse is best? a. Assess the client for the presence of subcutaneous nodules or Baker's cysts. b. Inspect the client's feet and hands for podagra and tophi on fingers and toes. c. Prepare to teach the client about an acetaminophen (Tylenol) regimen. d. Reassure the client that the problems will fade as the weather changes again.

ANS: A Osteoarthritis is not a systemic disease, nor does it present bilaterally. These are manifestations of rheumatoid arthritis. The nurse should assess for other manifestations of this disorder, including subcutaneous nodules and Baker's cysts. Podagra and tophi are seen in gout. Acetaminophen is not used for rheumatoid arthritis. Telling the client that the symptoms will fade with weather changes is not accurate.

The nurse must assess a 10-month-old infant. The infant is sitting on the father's lap and appears to be afraid of the nurse and of what might happen next. Which initial action by the nurse would be most appropriate? a. Initiate a game of peek-a-boo. b. Ask father to place the infant on the examination table. c. Undress the infant while he is still sitting on his father's lap. d. Talk softly to the infant while taking him from his father.

ANS: A Peek-a-boo is an excellent means of initiating communication with infants while maintaining a safe, nonthreatening distance. The child will most likely become upset if separated from his father. As much of the assessment as possible should be done on the father's lap. The nurse should have the father undress the child as needed for the examination. DIF: Cognitive Level: Apply REF: p. 62 TOP: Integrated Process: Communication and Documentation MSC: Area of Client Needs: Psychosocial Integrity

13. A client is in the clinic for a follow-up visit after a moderate traumatic brain injury. The client's spouse is very frustrated, stating that the client's personality has changed and the situation is intolerable. What action by the nurse is best? a. Explain that personality changes are common following brain injuries. b. Ask the client why he or she is acting out and behaving differently. c. Refer the client and spouse to a head injury support group. d. Tell the spouse this is expected and he or she will have to learn to cope.

ANS: A Personality and behavior often change permanently after head injury. The nurse should explain this to the spouse. Asking the client about his or her behavior isn't useful because the client probably cannot help it. A referral might be a good idea, but the nurse needs to do something in addition to just referring the couple. Telling the spouse to learn to cope belittles the spouse's concerns and feelings. DIF: Applying/Application REF: 953 KEY: Neurologic disorders| therapeutic communication| psychosocial response| coping MSC: Integrated Process: Communication and Documentation NOT: Client Needs Category: Psychosocial Integrity

*One bed is available on the inpatient eating disorders unit. Which patient should be admitted to this bed? The patient whose weight decreased from:* a. 150 to 100 pounds over a 4-month period. Vital signs are temperature, 35.9° C; pulse, 38 beats/min; blood pressure 60/40 mm Hg b. 120 to 90 pounds over a 3-month period. Vital signs are temperature, 36° C; pulse, 50 beats/min; blood pressure 70/50 mm Hg c. 110 to 70 pounds over a 4-month period. Vital signs are temperature 36.5° C; pulse, 60 beats/min; blood pressure 80/66 mm Hg d. 90 to 78 pounds over a 5-month period. Vital signs are temperature, 36.7° C; pulse, 62 beats/min; blood pressure 74/48 mm Hg

ANS: A Physical criteria for hospitalization include weight loss of more than 30% of body weight within 6 months, temperature below 36° C (hypothermia), heart rate less than 40 beats/min, and systolic blood pressure less than 70 mm Hg.

6. A nurse is caring for a client receiving enteral feedings through a Dobhoff tube. What action by the nurse is best to prevent hyperosmolarity? a. Administer free-water boluses. b. Change the clients formula. c. Dilute the clients formula. d. Slow the rate of infusion.

ANS: A Proteins and sugar molecules in the enteral feeding product contribute to dehydration due to increased osmolarity. The nurse can administer free-water boluses after consulting with the provider on the appropriate amount and timing of the boluses, or per protocol. The client may not be able to switch formulas. Diluting the formula is not appropriate. Slowing the rate of the infusion will not address the problem.

7. A client appears dyspneic, but the oxygen saturation is 97%. What action by the nurse is best? a. Assess for other manifestations of hypoxia. b. Change the sensor on the pulse oximeter. c. Obtain a new oximeter from central supply. d. Tell the client to take slow, deep breaths.

ANS: A Pulse oximetry is not always the most accurate assessment tool for hypoxia as many factors can interfere, producing normal or near-normal readings in the setting of hypoxia. The nurse should conduct a more thorough assessment. The other actions are not appropriate for a hypoxic client.

The nurse has determined the rate of both the child's radial pulse and heart. What is the normal finding when comparing the two rates? a. Are the same b. Differ, with heart rate faster c. Differ, with radial pulse faster d. Differ, depending on quality and intensity

ANS: A Pulses are the fluid wave through the blood vessel as a result of each heartbeat. Therefore, they should be the same. DIF: Cognitive Level: Understand REF: p. 103 TOP: Integrated Process: Nursing Process: Assessment MSC: Area of Client Needs: Health Promotion and Maintenance

25. A client has a shoulder injury and is scheduled for a magnetic resonance imaging (MRI). The nurse notes the presence of an aneurysm clip in the client's record. What action by the nurse is best? a. Ask the client how long ago the clip was placed. b. Have the client sign an informed consent form. c. Inform the provider about the aneurysm clip. d. Reschedule the client for computed tomography.

ANS: A Some older clips are metal, which would preclude the use of MRI. The nurse should determine how old the clip is and relay that information to the MRI staff. They can determine if the client is a suitable candidate for this examination. The client does not need to sign informed consent. The provider will most likely not know if the client can have an MRI with this clip. The nurse does not independently change the type of diagnostic testing the client receives. DIF: Applying/Application REF: 940 KEY: Neurologic disorders| patient safety| communication| nursing assessment MSC: IntegratedProcess:NursingProcess:Assessment NOT: Client Needs Category: Safe and Effective Care Environment: Safety and Infection Control

25. A nurse auscultates a client's lung fields. Which action should the nurse take based on the lung sounds? (Click the media button to hear the audio clip.) a. Assess for airway obstruction. b. Initiate oxygen therapy. c. Assess vital signs. d. Elevate the client's head.

ANS: A Stridor is the sound heard, and it indicates severe airway constriction. The nurse must administer a bronchodilator to get air into the lungs. Administering oxygen, assessing vital signs, and elevating the client's head will not help until the client's airways are open. DIF: Applying/Application REF: 575 KEY: Assessment/diagnostic examination MSC: Integrated Process: Nursing Process: Implementation NOT: Client Needs Category: Safe and Effective Care Environment: Management of Care

7. A student is practicing suctioning a tracheostomy in the skills laboratory. What action by the student demonstrates that more teaching is needed? a. Applying suction while inserting the catheter b. Preoxygenating the client prior to suctioning c. Suctioning for a total of three times if needed d. Suctioning for only 10 to 15 seconds each time

ANS: A Suction should only be applied while withdrawing the catheter. The other actions are appropriate. DIF: Remembering/Knowledge REF: 525 KEY: Tracheostomy| tracheostomy care| suctioning| supervision MSC: IntegratedProcess:NursingProcess:Assessment NOT: Client Needs Category: Physiological Integrity: Reduction of Risk Potential

Which would be best for the nurse to use when determining the temperature of a preterm infant under a radiant heater? a. Axillary sensor b. Tympanic membrane sensor c. Rectal mercury glass thermometer d. Rectal electronic thermometer

ANS: A The axillary sensor measures the infrared heat energy radiating from the axilla. It can be used on wet skin, in incubators, or under radiant warmers. Ear thermometry does not show sufficient correlation with established methods of measurement. It should not be used when body temperature must be assessed with precision. Mercury thermometers should never be used. The release of mercury, should the thermometer be broken, can cause harmful vapors. Rectal temperatures should be avoided unless no other suitable way exists for the temperature to be measured. DIF: Cognitive Level: Apply REF: p. 85 TOP: Integrated Process: Nursing Process: Assessment MSC: Area of Client Needs: Health Promotion and Maintenance

A nurse cares for a client with end-stage pancreatic cancer. The client asks, "Why is this happening to me?" How should the nurse respond? a. "I don't know. I wish I had an answer for you, but I don't." b. "It's important to keep a positive attitude for your family right now." c. "Scientists have not determined why cancer develops in certain people." d. "I think that this is a trial so you can become a better person because of it."

ANS: A The client is not asking the nurse to actually explain why the cancer has occurred. The client may be expressing his or her feelings of confusion, frustration, distress, and grief related to this diagnosis. Reminding the client to keep a positive attitude for his or her family does not address the client's emotions or current concerns. The nurse should validate that there is no easy or straightforward answer as to why the client has cancer. Telling a client that cancer is a trial is untrue and may diminish the client-nurse relationship.

A client is getting out of bed into the chair for the first time after an uncemented hip replacement. What action by the nurse is most important? a. Have adequate help to transfer the client. b. Provide socks so the client can slide easier. c. Tell the client full weight bearing is allowed. d. Use a footstool to elevate the client's leg.

ANS: A The client with an uncemented hip will be on toe-touch only right after surgery. The nurse should ensure there is adequate help to transfer the client while preventing falls. Slippery socks will encourage a fall. Elevating the leg greater than 90 degrees is not allowed.

A client has dumping syndrome after a partial gastrectomy. Which action by the nurse would be most helpful? a. Arrange a dietary consult. b. Increase fluid intake. c. Limit the client's foods. d. Make the client NPO.

ANS: A The client with dumping syndrome after a gastrectomy has multiple dietary needs. A referral to the registered dietitian will be extremely helpful. Food and fluid intake is complicated and needs planning. The client should not be NPO.

15. A nurse cares for a client with infective endocarditis. Which infection control precautions should the nurse use? a. Standard Precautions b. Bleeding precautions c. Reverse isolation d. Contact isolation

ANS: A The client with infective endocarditis does not pose any specific threat of transmitting the causative organism. Standard Precautions should be used. Bleeding precautions or reverse or contact isolation is not necessary. DIF: Applying/Application REF: 697 KEY: Infection| Standard Precautions MSC: Integrated Process: Nursing Process: Implementation NOT: Client Needs Category: Safe and Effective Care Environment: Safety and Infection Control

11. A nurse assesses a client who is prescribed fluticasone (Flovent) and notes oral lesions. Which action should the nurse take? a. Encourage oral rinsing after fluticasone administration. b. Obtain an oral specimen for culture and sensitivity. c. Start the client on a broad-spectrum antibiotic. d. Document the finding as a known side effect.

ANS: A The drug reduces local immunity and increases the risk for local infection, especially Candida albicans. Rinsing the mouth after using the inhaler will decrease the risk for developing this infection. Use of mouthwash and broad-spectrum antibiotics is not warranted in this situation. The nurse should document the finding, but the best action to take is to have the client start rinsing his or her mouth after using fluticasone. An oral specimen for culture and sensitivity will not provide information necessary to care for this client. DIF: Applying/Application REF: 554 KEY: Medication| fungal infection MSC: Integrated Process: Nursing Process: Implementation NOT: Client Needs Category: Physiological Integrity: Pharmacological and Parenteral Therapies 12. A nurse cares for a client who is infected with Burkholderia cepacia. Which action should the nurse take first when admitting this client to a pulmonary care unit? a. Instruct the client to wash his or her hands after contact with other people. b. Implement Droplet Precautions and don a surgical mask. c. Keep the client isolated from other clients with cystic fibrosis. d. Obtain blood, sputum, and urine culture specimens. ANS: C Burkholderia cepacia infection is spread through casual contact between cystic fibrosis clients, thus the need for these clients to be separated from one another. Strict isolation measures will not be necessary. Although the client should wash his or her hands frequently, the most important measure that can be implemented on the unit is isolation of the client from other clients with cystic fibrosis. There is no need to implement Droplet Precautions or don a surgical mask when caring for this client. Obtaining blood, sputum, and urine culture specimens will not provide information necessary to care for a client with Burkholderia cepacia infection. DIF: Applying/Application REF: 568 KEY: Pulmonary infection| infection control MSC: Integrated Process: Nursing Process: Implementation NOT: Client Needs Category: Safe and Effective Care Environment: Safety and Infection Control

The nurse is seeing an adolescent boy and his parents in the clinic for the first time. What should the nurse do first? a. Introduce self. b. Make family comfortable. c. Explain purpose of interview. d. Give assurance of privacy.

ANS: A The first thing that nurses should do is to introduce themselves to the patient and family. Parents and other adults should be addressed with appropriate titles unless they specify a preferred name. During the initial part of the interview, the nurse should include general conversation to help make the family feel at ease. Clarification of the purpose of the interview and the nurse's role is the next thing that should be done. The interview should take place in an environment as free of distraction as possible. In addition, the nurse should clarify which information will be shared with other members of the health care team and any limits to the confidentiality. DIF: Cognitive Level: Apply REF: p. 57 TOP: Integrated Process: Communication and Documentation MSC: Area of Client Needs: Psychosocial Integrity

Which is an important nursing consideration when caring for an infant with failure to thrive? a. Establish a structured routine and follow it consistently. b. Maintain a nondistracting environment by not speaking to child during feeding. c. Place child in an infant seat during feedings to prevent overstimulation. d. Limit sensory stimulation and play activities to alleviate fatigue.

ANS: A The infant with failure to thrive should have a structured routine that is followed consistently. Disruptions in other activities of daily living can have a great impact on feeding behaviors. Bathing, sleeping, dressing, playing, and feeding are structured. The nurse should talk to the child by giving directions about eating. This will help the child maintain focus. Young children should be held while being fed, and older children can sit at a feeding table. The child should be fed in the same manner at each meal. The child can engage in sensory and play activities at times other than mealtime.

8. The nurse is caring for a child receiving intravenous (IV) morphine for severe postoperative pain. The nurse observes a slower respiratory rate, and the child cannot be aroused. What is the priority nursing action? a. Administer naloxone (Narcan) b. Discontinue IV infusion c. Discontinue morphine until child is fully awake d. Stimulate child by calling name, shaking gently, and asking to breathe deeply

ANS: A The management of opioid-induced respiratory depression includes lowering the rate of infusion and stimulating the child. If the respiratory rate is depressed and the child cannot be aroused, then IV naloxone should be administered. The child will be in pain because of the reversal of the morphine. The morphine should be discontinued, but naloxone is indicated if the child is unresponsive. The child is unresponsive, therefore naloxone is indicated. DIF: Cognitive Level: Apply REF: p. 143 TOP: Integrated Process: Nursing Process: Implementation MSC: Area of Client Needs: Physiologic Integrity

A client has been diagnosed with rheumatoid arthritis. The client has experienced increased fatigue and worsening physical status and is finding it difficult to maintain the role of elder in his cultural community. The elder is expected to attend social events and make community decisions. Stress seems to exacerbate the condition. What action by the nurse is best? a. Assess the client's culture more thoroughly. b. Discuss options for performing duties. c. See if the client will call a community meeting. d. Suggest the client give up the role of elder.

ANS: A The nurse needs a more thorough understanding of the client's culture, including the meaning of illness and the ramifications of the elder not being able to perform traditional duties. This must be done prior to offering any possible solutions. If the nurse does not understand the consequences of what is suggested, the client may simply be unwilling to listen or participate in problem solving. The other options may be reasonable depending on the outcome of a better cultural understanding.

12. A nurse is caring for a client on mechanical ventilation and finds the client agitated and thrashing about. What action by the nurse is most appropriate? a. Assess the cause of the agitation. b. Reassure the client that he or she is safe. c. Restrain the client's hands. d. Sedate the client immediately.

ANS: A The nurse needs to determine the cause of the agitation. The inability to communicate often makes clients anxious, even to the point of panic. Pain and confusion can also cause agitation. Once the nurse determines the cause of the agitation, he or she can implement measures to relieve the underlying cause. Reassurance is also important but may not address the etiology of the agitation. Restraints and more sedation may be necessary, but not as a first step.

15. A client has been hospitalized with tuberculosis (TB). The client's spouse is fearful of entering the room where the client is in isolation and refuses to visit. What action by the nurse is best? a. Ask the spouse to explain the fear of visiting in further detail. b. Inform the spouse the precautions are meant to keep other clients safe. c. Show the spouse how to follow the isolation precautions to avoid illness. d. Tell the spouse that he or she has already been exposed, so it's safe to visit.

ANS: A The nurse needs to obtain further information about the spouse's specific fears so they can be addressed. This will decrease stress and permit visitation, which will be beneficial for both client and spouse. Precautions for TB prevent transmission to all who come into contact with the client. Explaining isolation precautions and what to do when entering the room will be helpful, but this is too narrow in scope to be the best answer. Telling the spouse it's safe to visit is demeaning of the spouse's feelings.

14. A client is in the bariatric clinic 1 month after having gastric bypass surgery. The client is crying and says I didnt know it would be this hard to live like this. What response by the nurse is best? a. Assess the clients coping and support systems. b. Inform the client that things will get easier. c. Re-educate the client on needed dietary changes. d. Tell the client lifestyle changes are always hard.

ANS: A The nurse should assess this clients coping styles and support systems in order to provide holistic care. The other options do not address the clients distress.

29. A client in the emergency department is having a stroke and needs a carotid artery angioplasty with stenting. The client's mental status is deteriorating. What action by the nurse is most appropriate? a. Attempt to find the family to sign a consent. b. Inform the provider that the procedure cannot occur. c. Nothing; no consent is needed in an emergency. d. Sign the consent form for the client.

ANS: A The nurse should attempt to find the family to give consent. If no family is present or can be found, under the principle of emergency consent, a life-saving procedure can be performed without formal consent. The nurse should not just sign the consent form. DIF: Applying/Application REF: 938 KEY: Neurologic disorders| informed consent| ethics MSC: Integrated Process: Communication and Documentation NOT: Client Needs Category: Safe and Effective Care Environment: Management of Care

A nurse cares for a client who has a new colostomy. Which action should the nurse take? a. Empty the pouch frequently to remove excess gas collection. b. Change the ostomy pouch and wafer every morning. c. Allow the pouch to completely fill with stool prior to emptying it. d. Use surgical tape to secure the pouch and prevent leakage.

ANS: A The nurse should empty the new ostomy pouch frequently because of excess gas collection, and empty the pouch when it is one-third to one-half full of stool. The ostomy pouch does not need to be changed every morning. Ostomy wafers with paste should be used to secure and seal the ostomy appliance; surgical tape should not be used.

The nurse is preparing to perform a physical assessment on a 10-year-old girl. The nurse gives her the option of her mother either staying in the room or leaving. How should this action be interpreted? a. Appropriate because of child's age b. Appropriate because mother would be uncomfortable making decisions for child c. Inappropriate because of child's age d. Inappropriate because child is same sex as mother

ANS: A The older school-age child should be given the option of having the parent present or not. During the examination, the nurse should respect the child's need for privacy. Although the question was appropriate for the child's age, the mother is responsible for making decisions for the child. It is appropriate because of the child's age. During the examination, the nurse must respect the child's privacy. The child should help determine who is present during the examination. DIF: Cognitive Level: Apply REF: p. 77 TOP: Integrated Process: Nursing Process: Assessment MSC: Area of Client Needs: Health Promotion and Maintenance

The nurse is completing a pain assessment on a 4-year-old child. Which of the depicted pain scale tools should the nurse use with a child this age? a. FACES b. Numeric pain scales c. Word graphic scales d. Visual analogue scales

ANS: A The pain scale appropriate for a 4-year-old child is the FACES pain scale. Numeric pain scales can be used on children as young as age 5 as long as they can count and have some concept of numbers and their values in relation to other numbers. Word graphic scales and visual analogue scales are used preferably for school-age children. DIF: Cognitive Level: Analyze REF: p. 115 TOP: Integrated Process: Nursing Process: Implementation MSC: Area of Client Needs: Physiologic Integrity

The nurse is interviewing the mother of an infant. She reports, "I had a difficult delivery, and my baby was born preterm." This information should be recorded under which of the following headings? a. Past history b. Present illness c. Chief complaint d. Review of systems

ANS: A The past history refers to information that relates to previous aspects of the child's health, not to the current problem. The mother's difficult delivery and prematurity are important parts of the past history of an infant. The history of the present illness is a narrative of the chief complaint from its earliest onset through its progression to the present. Unless the chief complaint is directly related to the prematurity, this information is not included in the history of present illness. The chief complaint is the specific reason for the child's visit to the clinic, office, or hospital. It would not include the birth information. The review of systems is a specific review of each body system. It does not include the preterm birth. Sequelae such as pulmonary dysfunction would be included. DIF: Cognitive Level: Understand REF: p. 65 TOP: Integrated Process: Communication and Documentation MSC: Area of Client Needs: Health Promotion and Maintenance

4. While assessing a client on a cardiac unit, a nurse identifies the presence of an S3 gallop. Which action should the nurse take next? a. Assess for symptoms of left-sided heart failure. b. Document this as a normal finding. c. Call the health care provider immediately. d. Transfer the client to the intensive care unit.

ANS: A The presence of an S3 gallop is an early diastolic filling sound indicative of increasing left ventricular pressure and left ventricular failure. The other actions are not warranted. DIF: Remembering/Knowledge REF: 683 KEY: Heart failure| assessment/diagnostic examination MSC: IntegratedProcess:NursingProcess:Assessment NOT: Client Needs Category: Physiological Integrity: Reduction of Risk Potential

5. An unlicensed assistive personnel (UAP) was feeding a client with a tracheostomy. Later that evening, the UAP reports that the client had a coughing spell during the meal. What action by the nurse takes priority? a. Assess the client's lung sounds. b. Assign a different UAP to the client. c. Report the UAP to the manager. d. Request thicker liquids for meals.

ANS: A The priority is to check the client's oxygenation because he or she may have aspirated. Once the client has been assessed, the nurse can consult with the registered dietitian about appropriately thickened liquids. The UAP should have reported the incident immediately, but addressing that issue is not the immediate priority. DIF: Applying/Application REF: 524 KEY: Delegation| aspiration| tracheostomy| nursing assessment| unlicensed assistive personnel(UAP) MSC: IntegratedProcess:NursingProcess:Assessment NOT: Client Needs Category: Physiological Integrity: Reduction of Risk Potential

9. A client is receiving total parenteral nutrition (TPN). On assessment, the nurse notes the clients pulse is 128 beats/min, blood pressure is 98/56 mm Hg, and skin turgor is dry. What action should the nurse perform next? a. Assess the 24-hour fluid balance. b. Assess the clients oral cavity. c. Prepare to hang a normal saline bolus. d. Turn up the infusion rate of the TPN.

ANS: A This client has clinical indicators of dehydration, so the nurse calculates the clients 24-hour intake, output, and fluid balance. This information is then reported to the provider. The clients oral cavity assessment may or may not be consistent with dehydration. The nurse may need to give the client a fluid bolus, but not as an independent action. The clients dehydration is most likely due to fluid shifts from the TPN, so turning up the infusion rate would make the problem worse, and is not done as an independent action.

26. The nurse instructs a client on the steps needed to obtain a peak expiratory flow rate. In which order should these steps occur? 1. "Take as deep a breath as possible." 2. "Stand up (unless you have a physical disability)." 3. "Place the meter in your mouth, and close your lips around the mouthpiece." 4. "Make sure the device reads zero or is at base level." 5. "Blow out as hard and as fast as possible for 1 to 2 seconds." 6. "Write down the value obtained." 7. "Repeat the process two additional times, and record the highest number in your chart." a. 4, 2, 1, 3, 5, 6, 7 b. 3, 4, 1, 2, 5, 7, 6 c. 2, 1, 3, 4, 5, 6, 7 d. 1, 3, 2, 5, 6, 7, 4

ANS: A The proper order for obtaining a peak expiratory flow rate is as follows. Make sure the device reads zero or is at base level. The client should stand up (unless he or she has a physical disability). The client should take as deep a breath as possible, place the meter in the mouth, and close the lips around the mouthpiece. The client should blow out as hard and as fast as possible for 1 to 2 seconds. The value obtained should be written down. The process should be repeated two more times, and the highest of the three numbers should be recorded in the client's chart. DIF: Applying/Application REF: 552 KEY: Assessment/diagnostic examination MSC: IntegratedProcess:NursingProcess:Assessment NOT: Client Needs Category: Physiological Integrity: Reduction of Risk Potential

A nurse cares for a client who had a colostomy placed in the ascending colon 2 weeks ago. The client states, "The stool in my pouch is still liquid." How should the nurse respond? a. "The stool will always be liquid with this type of colostomy." b. "Eating additional fiber will bulk up your stool and decrease diarrhea." c. "Your stool will become firmer over the next couple of weeks." d. "This is abnormal. I will contact your health care provider."

ANS: A The stool from an ascending colostomy can be expected to remain liquid because little large bowel is available to reabsorb the liquid from the stool. This finding is not abnormal. Liquid stool from an ascending colostomy will not become firmer with the addition of fiber to the client's diet or with the passage of time.

The nurse has a 2-year-old boy sit in "tailor" position during palpation for the testes. What is the rationale for this position? a. It prevents cremasteric reflex. b. Undescended testes can be palpated. c. This tests the child for an inguinal hernia. d. The child does not yet have a need for privacy.

ANS: A The tailor position stretches the muscle responsible for the cremasteric reflex. This prevents its contraction, which pulls the testes into the pelvic cavity. Undescended testes cannot be predictably palpated. Inguinal hernias are not detected by this method. This position is used for inhibiting the cremasteric reflex. Privacy should always be provided for children. DIF: Cognitive Level: Understand REF: p. 107 TOP: Integrated Process: Nursing Process: Assessment MSC: Area of Client Needs: Health Promotion and Maintenance: Techniques of Physical Assessment

8. A client has been diagnosed with tuberculosis (TB). What action by the nurse takes highest priority? a. Educating the client on adherence to the treatment regimen b. Encouraging the client to eat a well-balanced diet c. Informing the client about follow-up sputum cultures d. Teaching the client ways to balance rest with activity

ANS: A The treatment regimen for TB ranges from 6 to 12 months, making adherence problematic for many people. The nurse should stress the absolute importance of following the treatment plan for the entire duration of prescribed therapy. The other options are appropriate topics to educate this client on but do not take priority.

When scheduling an annual pelvic examination and Pap test, the client asks if she should abstain from intercourse before the test. Which is the nurse's best response? a. "Yes. Avoid having intercourse for 24 hours before the test." b. "Yes. Avoid having intercourse for 2 hours before the test." c. "No. Intercourse does not interfere with this test." d. "No. Intercourse can actually enhance the test results."

ANS: A The woman should not douche, use vaginal medications or deodorants, or have sexual intercourse for at least 24 hours before the test. Such activities may prevent the accurate evaluation of smears, cultures, and cytologic data.

11. A client has a traumatic brain injury. The nurse assesses the following: pulse change from 82 to 60 beats/min, pulse pressure increase from 26 to 40 mm Hg, and respiratory irregularities. What action by the nurse takes priority? a. Call the provider or Rapid Response Team. b. Increase the rate of the IV fluid administration. c. Notify respiratory therapy for a breathing treatment. d. Prepare to give IV pain medication.

ANS: A These manifestations indicate Cushing's syndrome, a potentially life-threatening increase in intracranial pressure (ICP), which is an emergency. Immediate medical attention is necessary, so the nurse notifies the provider or the Rapid Response Team. Increasing fluids would increase the ICP. The client does not need a breathing treatment or pain medication. DIF: Applying/Application REF: 952 KEY: Neurologic disorders| Rapid Response Team| critical rescue MSC: Integrated Process: Communication and Documentation NOT: Client Needs Category: Safe and Effective Care Environment: Management of Care

17. A client who had a severe traumatic brain injury is being discharged home, where the spouse will be a full-time caregiver. What statement by the spouse would lead the nurse to provide further education on home care? a. "I know I can take care of all these needs by myself." b. "I need to seek counseling because I am very angry." c. "Hopefully things will improve gradually over time." d. "With respite care and support, I think I can do this."

ANS: A This caregiver has unrealistic expectations about being able to do everything without help. Acknowledging anger and seeking counseling show a realistic outlook and plans for accomplishing goals. Hoping for improvement over time is also realistic, especially with the inclusion of the word "hopefully." Realizing the importance of respite care and support also is a realistic outlook. DIF: Evaluating/Synthesis REF: 957 KEY: Neurologic disorders| discharge teaching| psychosocial response| coping MSC: IntegratedProcess:Caring NOT: Client Needs Category: Psychosocial Integrity

A 3-month-old infant dies shortly after arrival to the emergency department. The infant has subdural and retinal hemorrhages but no external signs of trauma. What should the nurse suspect? a. Unintentional injury b. Shaken-baby syndrome c. Sudden infant death syndrome (SIDS) d. Congenital neurologic problem

ANS: B Shaken-baby syndrome causes internal bleeding but may have no external signs. Unintentional injury would not cause these injuries. SIDS and congenital neurologic problems would not appear this way.

1. A client is in the family practice clinic. Today the client weighs 186.4 pounds (84.7 kg). Six months ago the client weighed 211.8 pounds (96.2 kg). What action by the nurse is best? a. Ask the client if the weight loss was intentional. b. Determine if there are food allergies or intolerances. c. Perform a comprehensive nutritional assessment. d. Perform a rapid bedside blood glucose test.

ANS: A This client has had a 12% weight loss. The nurse first determines if the weight loss was intentional. If not, then the nurse proceeds to a comprehensive nutritional assessment. Food intolerances are part of this assessment. Depending on risk factors and other findings, a blood glucose test may be warranted.

The nurse on the postoperative inpatient unit assesses a client after a total hip replacement. The client's surgical leg is visibly shorter than the other one and the client reports extreme pain. While a co-worker calls the surgeon, what action by the nurse is best? a. Assess neurovascular status in both legs. b. Elevate the affected leg and apply ice. c. Prepare to administer pain medication. d. Try to place the affected leg in abduction.

ANS: A This client has manifestations of hip dislocation, a critical complication of this surgery. Hip dislocation can cause neurovascular compromise. The nurse should assess neurovascular status, comparing both legs. The nurse should not try to move the extremity to elevate or abduct it. Pain medication may be administered if possible, but first the nurse should thoroughly assess the client.

20. A nurse assesses a client with the National Institutes of Health (NIH) Stroke Scale and determines the client's score to be 36. How should the nurse plan care for this client? a. The client will need near-total care. b. The client will need cuing only. c. The client will need safety precautions. d. The client will be discharged home.

ANS: A This client has severe neurologic deficits and will need near-total care. Safety precautions are important but do not give a full picture of the client's dependence. The client will need more than cuing to complete tasks. A home discharge may be possible, but this does not help the nurse plan care for a very dependent client. DIF: Analyzing/Analysis REF: 935 KEY: Neurologic disorders| neurologic assessment MSC: Integrated Process: Nursing Process: Analysis NOT: Client Needs Category: Physiological Integrity: Reduction of Risk Potential

23. A client has a subarachnoid bolt. What action by the nurse is most important? a. Balancing and recalibrating the device b. Documenting intracranial pressure readings c. Handling the fiberoptic cable with care to avoid breakage d. Monitoring the client's phlebostatic axis

ANS: A This device needs frequent balancing and recalibration in order to read correctly. Documenting readings is important, but it is more important to ensure the device's accuracy. The fiberoptic transducer-tipped catheter has a cable that must be handled carefully to avoid breaking it, but ensuring the device's accuracy is most important. The phlebostatic axis is not related to neurologic monitoring. DIF: Applying/Application REF: 956 KEY: Neurologic disorders| neurologic assessment| equipment safety MSC: Integrated Process: Nursing Process: Implementation NOT: Client Needs Category: Safe and Effective Care Environment: Safety and Infection Control

15. A client has been prescribed lorcaserin (Belviq). What teaching is most appropriate? a. Increase the fiber and water in your diet. b. Reduce fat to less than 30% each day. c. Report dry mouth and decreased sweating. d. Lorcaserin may cause loose stools for a few days.

ANS: A This drug can cause constipation, so the client should increase fiber and water in the diet to prevent this from occurring. Reducing fat in the diet is important with orlistat. Lorcaserin can cause dry mouth but not decreased sweating. Loose stools are common with orlistat.

A nurse is teaching a client with psoriatic arthritis about the medication golimumab (Simponi). What information is most important to include? a. "Avoid large crowds or people who are ill." b. "Stay upright for 1 hour after taking this drug." c. "This drug may cause your hair to fall out." d. "You may double the dose if pain is severe."

ANS: A This drug has a Food and Drug Administration black box warning about opportunistic or other serious infections. Teach the client to avoid large crowds and people who are ill. The other instructions do not pertain to golimumab.

What action by the perioperative nursing staff is most important to prevent surgical wound infection in a client having a total joint replacement? a. Administer preoperative antibiotic as ordered. b. Assess the client's white blood cell count. c. Instruct the client to shower the night before. d. Monitor the client's temperature postoperatively.

ANS: A To prevent surgical wound infection, antibiotics are given preoperatively within an hour of surgery. Simply taking a shower will not help prevent infection unless the client is told to use special antimicrobial soap. The other options are processes to monitor for infection, not prevent it.

Which figure depicts a nurse performing a test for the triceps reflex? a. b. c. d.

ANS: A To test the triceps reflex, the child is placed supine, with the forearm resting over the chest and the triceps tendon is struck with the reflex hammer. The other figures depict tests for biceps reflex (slightly above the antecubital space) patellar (knee), and Achilles (behind the foot). DIF: Cognitive Level: Analyze REF: p. 110 TOP: Integrated Process: Nursing Process: Assessment MSC: Area of Client Needs: Health Promotion and Maintenance

Cow's milk allergy (CMA) is diagnosed in a 6-month-old infant. Which should the nurse recommend as a substitute formula? a. Nutramigen b. Goat's milk c. Similac d. Enfamil

ANS: A Treatment of CMA is elimination of cow's milk-based formula and all other dairy products. For infants fed cow's milk formula, this primarily involves changing the formula to a casein hydrolysate milk formula (Pregestimil, Nutramigen, or Alimentum). Goat's milk (raw) is not an acceptable substitute because it cross-reacts with cow's milk protein, is deficient in folic acid, has a high sodium and protein content, and is unsuitable as the only source of calories. Cow's milk protein is contained in both Enfamil and Similac.

A patient diagnosed with anorexia nervosa virtually stopped eating 5 months ago and lost 25% of body weight. A nurse asks, "Describe what you think about your present weight and how you look." Which response by the patient is most consistent with the diagnosis? a. "I am fat and ugly." b. "What I think about myself is my business." c. "I'm grossly underweight, but that's what I want." d. "I'm a few pounds overweight, but I can live with it."

ANS: A Untreated patients with anorexia nervosa do not recognize their thinness. They perceive themselves to be overweight and unattractive. The patient with anorexia will usually tell people perceptions of self. The patient with anorexia does not recognize his or her thinness and will persist in trying to lose more weight.

A nurse assesses a client who is recovering from an open Whipple procedure. Which action should the nurse perform first? a. Assess the client's endotracheal tube with 40% FiO2. b. Insert an indwelling Foley catheter to gravity drainage. c. Place the client's nasogastric tube to low intermittent suction. d. Start lactated Ringer's solution through an intravenous catheter.

ANS: A Using the ABCs, airway and oxygenation status should always be assessed first, so checking the endotracheal tube is the first action. Next, the nurse should start the IV line (circulation). After that, the Foley catheter can be inserted and the nasogastric tube can be set.

What type of breath sound is normally heard over the entire surface of the lungs except for the upper intrascapular area and the area beneath the manubrium? a. Vesicular b. Bronchial c. Adventitious d. Bronchovesicular

ANS: A Vesicular breath sounds are heard over the entire surface of lungs, with the exception of the upper intrascapular area and the area beneath the manubrium. Bronchial breath sounds are heard only over the trachea near the suprasternal notch. Adventitious breath sounds are not usually heard over the chest. These sounds occur in addition to normal or abnormal breath sounds. Bronchovesicular breath sounds are heard over the manubrium and in the upper intrascapular regions where trachea and bronchi bifurcate. DIF: Cognitive Level: Remember REF: p. 101 TOP: Integrated Process: Nursing Process: Assessment MSC: Area of Client Needs: Health Promotion and Maintenance

Recent studies indicate that a deficiency of which vitamin correlates with increased morbidity and mortality in children with measles? a.A b.C c.Niacin d.Folic acid

ANS: A Vitamin A deficiency is correlated with increased morbidity and mortality in children with measles. This vitamin deficiency also is associated with complications from diarrhea, and infections are often increased in infants and children with vitamin A deficiency. No correlation exists between vitamin C, niacin, or folic acid and measles.

*An outpatient diagnosed with anorexia nervosa has begun refeeding. Between the first and second appointments, the patient gained 8 pounds. The nurse should:* a. assess lung sounds and extremities. b. suggest use of an aerobic exercise program. c. positively reinforce the patient for the weight gain. d. establish a higher goal for weight gain the next week.

ANS: A Weight gain of more than 2 to 5 pounds weekly may overwhelm the heart's capacity to pump, leading to cardiac failure. The nurse must assess for signs of pulmonary edema and congestive heart failure. The incorrect options are undesirable because they increase the risk for cardiac complications.

1. A nurse assesses a client who had a myocardial infarction and is hypotensive. Which additional assessment finding should the nurse expect? a. Heart rate of 120 beats/min b. Cool, clammy skin c. Oxygen saturation of 90% d. Respiratory rate of 8 breaths/min

ANS: A When a client experiences hypotension, baroreceptors in the aortic arch sense a pressure decrease in the vessels. The parasympathetic system responds by lessening the inhibitory effect on the sinoatrial node. This results in an increase in heart rate and respiratory rate. This tachycardia is an early response and is seen even when blood pressure is not critically low. An increased heart rate and respiratory rate will compensate for the low blood pressure and maintain oxygen saturations and perfusion. The client may not be able to compensate for long, and decreased oxygenation and cool, clammy skin will occur later. DIF: Applying/Application REF: 638 KEY: Coronary perfusion| hemodynamics MSC: IntegratedProcess:NursingProcess:Assessment NOT: Client Needs Category: Physiological Integrity: Physiological Adaptation

A nurse is caring for a client with systemic sclerosis. The client's facial skin is very taut, limiting the client's ability to open the mouth. After consulting with a registered dietitian for appropriate nutrition, what other consultation should the nurse facilitate? a. Dentist b. Massage therapist c. Occupational therapy d. Physical therapy

ANS: A With limited ability to open the mouth, dental hygiene may be lacking. The nurse should encourage the client to see a dentist. The other referrals are not related to the mouth.

A nurse is performing an otoscopic exam on a school-age child. Which direction should the nurse pull the pinna for this age of child? a. Up and back b. Down and back c. Straight back d. Straight

ANS: A With older children, usually those older than 3 years of age, the canal curves downward and forward. Therefore, pull the pinna up and back during otoscopic examinations. In infants, the canal curves upward. Therefore, pull the pinna down and back to straighten the canal. Pulling the pinna straight back or straight up will not open the inner ear canal. DIF: Cognitive Level: Understand REF: p. 95 TOP: Integrated Process: Nursing Process: Assessment MSC: Area of Client Needs: Health Promotion and Maintenance

A client tells the nurse she is happy that she never had children because she has less risk of developing cancer. Which response by the nurse is best? a. "Actually, your risk of breast cancer is slightly higher." b. "You're right; your risk of all reproductive cancer is quite low." c. "In reality, smoking is the leading risk factor for all types of cancer." d. "Your risk of uterine cancer is higher because you had no children."

ANS: A Women who have never had children have a slightly higher risk of breast cancer than the general population. Smoking is a major risk factor for many, but not all, cancers. Uterine cancer is not influenced by pregnancy.

A client is in the clinic for an annual examination and questions the need for a pelvic examination and Pap smear because she had a hysterectomy many years ago. Which response by the nurse is most appropriate? a. "Do you still have your cervix?" b. "Are you sexually active?" c. "We can skip it if you like." d. "Let's see what the doctor says."

ANS: A Women who still have their cervix after hysterectomy still need a Pap smear according to the guidelines established for other women. Sexual activity is not relevant. Simply stating that it can be skipped does not help the woman protect her health. Asking the provider does not help the nurse further assess the client.

A nurse is teaching a larger female client about alcohol intake and how it affects hypertension. The client asks if drinking two beers a night is an acceptable intake. What answer by the nurse is best? a. "No, women should only have one beer a day as a general rule." b. "No, you should not drink any alcohol with hypertension." c. "Yes, since you are larger, you can have more alcohol." d. "Yes, two beers per day is an acceptable amount of alcohol."

ANS: A Alcohol intake should be limited to two drinks a day for men and one drink a day for women. A "drink" is classified as one beer, 1.5 ounces of hard liquor, or 5 ounces of wine. Limited alcohol intake is acceptable with hypertension. The woman's size does not matter. DIF: Understanding/Comprehension REF: 712

A nurse cares for a client with an increased blood urea nitrogen (BUN)/creatinine ratio. Which action should the nurse take first? a. Assess the client's dietary habits. b. Inquire about the use of nonsteroidal anti-inflammatory drugs (NSAIDs). c. Hold the client's metformin (Glucophage). d. Contact the health care provider immediately.

ANS: A An elevated BUN/creatinine ratio is often indicative of dehydration, urinary obstruction, catabolism, or a high-protein diet. The nurse should inquire about the client's dietary habits. Kidney damage related to NSAID use most likely would manifest with elevations in both BUN and creatinine, but no change in the ratio. The nurse should obtain more assessment data before holding any medications or contacting the provider. DIF: Applying/Application REF: 1355

A nurse reviews the allergy list of a client who is scheduled for an intravenous urography. Which client allergy should alert the nurse to urgently contact the health care provider? a. Seafood b. Penicillin c. Bee stings d. Red food dye

ANS: A Clients with seafood allergies often have severe allergic reactions to the standard dyes used during intravenous urography. The other allergies have no impact on the client's safety during an intravenous urography. DIF: Applying/Application REF: 1361

A nurse cares for a client with diabetes mellitus who is prescribed metformin (Glucophage) and is scheduled for an intravenous urography. Which action should the nurse take first? a. Contact the provider and recommend discontinuing the metformin. b. Keep the client NPO for at least 6 hours prior to the examination. c. Check the client's capillary artery blood glucose and administer prescribed insulin. d. Administer intravenous fluids to dilute and increase the excretion of dye.

ANS: A Metformin can cause lactic acidosis and renal impairment as the result of an interaction with the dye. This drug must be discontinued for 48 hours before the procedure and not started again after the procedure until urine output is well established. The client's health care provider needs to provide alternative therapy for the client until the metformin can be resumed. Keeping the client NPO, checking the client's blood glucose, and administering intravenous fluids should be part of the client's plan of care, but are not the priority, as the examination should not occur while the client is still taking metformin. DIF: Applying/Application REF: 1361 KEY

A nurse contacts the health care provider after reviewing a client's laboratory results and noting a blood urea nitrogen (BUN) of 35 mg/dL and a creatinine of 1.0 mg/dL. For which action should the nurse recommend a prescription? a. Intravenous fluids b. Hemodialysis c. Fluid restriction d. Urine culture and sensitivity

ANS: A Normal BUN is 10 to 20 mg/dL. Normal creatinine is 0.6 to 1.2 mg/dL (males) or 0.5 to 1.1 mg/dL (females). Creatinine is more specific for kidney function than BUN, because BUN can be affected by several factors (dehydration, high-protein diet, and catabolism). This client's creatinine is normal, which suggests a non-renal cause for the elevated BUN. A common cause of increased BUN is dehydration, so the nurse should anticipate giving the client more fluids, not placing the client on fluid restrictions. Hemodialysis is not an appropriate treatment for dehydration. The lab results do not indicate an infection; therefore, a urine culture and sensitivity is not appropriate. DIF: Applying/Application REF: 1355

A nurse reviews laboratory results for a client who was admitted for a myocardial infarction and cardiogenic shock 2 days ago. Which laboratory test result should the nurse expect to find? a. Blood urea nitrogen (BUN) of 52 mg/dL b. Creatinine of 2.3 mg/dL c. BUN of 10 mg/dL d. BUN/creatinine ratio of 8:1

ANS: A Shock leads to decreased renal perfusion. An elevated BUN accompanies this condition. The creatinine should be normal because no kidney damage occurred. A low BUN signifies overhydration, malnutrition, or liver damage. A low BUN/creatinine ratio indicates fluid volume excess or acute renal tubular acidosis. DIF: Applying/Application REF: 1359

After delegating to an unlicensed assistive personnel (UAP) the task of completing a bladder scan examination for a client, the nurse evaluates the UAP's performance. Which action by the UAP indicates the nurse must provide additional instructions when delegating this task? a. Selecting the female icon for all female clients and male icon for all male clients b. Telling the client, "This test measures the amount of urine in your bladder." c. Applying ultrasound gel to the scanning head and removing it when finished d. Taking at least two readings using the aiming icon to place the scanning head

ANS: A The UAP should use the female icon for women who have not had a hysterectomy. This allows the scanner to subtract the volume of the uterus from readings. If a woman has had a hysterectomy, the UAP should choose the male icon. The UAP should explain the procedure to the client, apply gel to the scanning head and clean it after use, and take at least two readings. DIF: Applying/Application REF: 1360

A nurse is caring for a client with a nonhealing arterial lower leg ulcer. What action by the nurse is best? a. Consult with the Wound Ostomy Care Nurse. b. Give pain medication prior to dressing changes. c. Maintain sterile technique for dressing changes. d. Prepare the client for eventual amputation.

ANS: A A nonhealing wound needs the expertise of the Wound Ostomy Care Nurse (or Wound Ostomy Continence Nurse). Premedicating prior to painful procedures and maintaining sterile technique are helpful, but if the wound is not healing, more needs to be done. The client may need an amputation, but other options need to be tried first. DIF: Applying/Application REF: 734

A nurse is interested in providing community education and screening on hypertension. In order to reach a priority population, to what target audience should the nurse provide this service? a. African-American churches b. Asian-American groceries c. High school sports camps d. Women's health clinics

ANS: A African Americans in the United States have one of the highest rates of hypertension in the world. The nurse has the potential to reach this priority population by providing services at African-American churches. Although hypertension education and screening are important for all groups, African Americans are the priority population for this intervention. DIF: Remembering/Knowledge REF: 711

A nurse is assessing a client with peripheral artery disease (PAD). The client states walking five blocks is possible without pain. What question asked next by the nurse will give the best information? a. "Could you walk further than that a few months ago?" b. "Do you walk mostly uphill, downhill, or on flat surfaces?" c. "Have you ever considered swimming instead of walking?" d. "How much pain medication do you take each day?"

ANS: A As PAD progresses, it takes less oxygen demand to cause pain. Needing to cut down on activity to be pain free indicates the client's disease is worsening. The other questions are useful, but not as important. DIF: Applying/Application REF: 719

A client has peripheral arterial disease (PAD). What statement by the client indicates misunderstanding about self-management activities? a. "I can use a heating pad on my legs if it's set on low." b. "I should not cross my legs when sitting or lying down." c. "I will go out and buy some warm, heavy socks to wear." d. "It's going to be really hard but I will stop smoking."

ANS: A Clients with PAD should never use heating pads as skin sensitivity is diminished and burns can result. The other statements show good understanding of self-management. DIF: Evaluating/Synthesis REF: 722

A client had a femoropopliteal bypass graft with a synthetic graft. What action by the nurse is most important to prevent wound infection? a. Appropriate hand hygiene before giving care b. Assessing the client's temperature every 4 hours c. Clean technique when changing dressings d. Monitoring the client's daily white blood cell count

ANS: A Hand hygiene is the best way to prevent infections in hospitalized clients. Dressing changes should be done with sterile technique. Assessing vital signs and white blood cell count will not prevent infection. DIF: Applying/Application REF: 724

The nurse is caring for four hypertensive clients. Which drug-laboratory value combination should the nurse report immediately to the health care provider? a. Furosemide (Lasix)/potassium: 2.1 mEq/L b. Hydrochlorothiazide (Hydrodiuril)/potassium: 4.2 mEq/L c. Spironolactone (Aldactone)/potassium: 5.1 mEq/L d. Torsemide (Demadex)/sodium: 142 mEq/L

ANS: A Lasix is a loop diuretic and can cause hypokalemia. A potassium level of 2.1 mEq/L is quite low and should be reported immediately. Spironolactone is a potassium-sparing diuretic that can cause hyperkalemia. A potassium level of 5.1 mEq/L is on the high side, but it is not as critical as the low potassium with furosemide. The other two laboratory values are normal. DIF: Applying/Application REF: 714

A client is taking warfarin (Coumadin) and asks the nurse if taking St. John's wort is acceptable. What response by the nurse is best? a. "No, it may interfere with the warfarin." b. "There isn't any information about that." c. "Why would you want to take that?" d. "Yes, it is a good supplement for you."

ANS: A Many foods and drugs interfere with warfarin, St. John's wort being one of them. The nurse should advise the client against taking it. The other answers are not accurate. DIF: Understanding/Comprehension REF: 733

A client has been diagnosed with a deep vein thrombosis and is to be discharged on warfarin (Coumadin). The client is adamant about refusing the drug because "it's dangerous." What action by the nurse is best? a. Assess the reason behind the client's fear. b. Remind the client about laboratory monitoring. c. Tell the client drugs are safer today than before. d. Warn the client about consequences of noncompliance.

ANS: A The first step is to assess the reason behind the client's fear, which may be related to the experience of someone the client knows who took warfarin. If the nurse cannot address the specific rationale, teaching will likely be unsuccessful. Laboratory monitoring once every few weeks may not make the client perceive the drug to be safe. General statements like "drugs are safer today" do not address the root cause of the problem. Warning the client about possible consequences of not taking the drug is not therapeutic and is likely to lead to an adversarial relationship. DIF: Applying/Application REF: 709

A nurse is working with a client who takes atorvastatin (Lipitor). The client's recent laboratory results include a blood urea nitrogen (BUN) of 33 mg/dL and creatinine of 2.8 mg/dL. What action by the nurse is best? a. Ask if the client eats grapefruit. b. Assess the client for dehydration. c. Facilitate admission to the hospital. d. Obtain a random urinalysis.

ANS: A There is a drug-food interaction between statins and grapefruit that can lead to acute kidney failure. This client has elevated renal laboratory results, indicating some degree of kidney involvement. The nurse should assess if the client eats grapefruit or drinks grapefruit juice. Dehydration can cause the BUN to be elevated, but the elevation in creatinine is more specific for a kidney injury. The client does not necessarily need to be admitted. A urinalysis may or may not be ordered. DIF: Applying/Application REF: 709

A client with a history of heart failure and hypertension is in the clinic for a follow-up visit. The client is on lisinopril (Prinivil) and warfarin (Coumadin). The client reports new-onset cough. What action by the nurse is most appropriate? a. Assess the client's lung sounds and oxygenation. b. Instruct the client on another antihypertensive. c. Obtain a set of vital signs and document them. d. Remind the client that cough is a side effect of Prinivil.

ANS: A This client could be having an exacerbation of heart failure or be experiencing a side effect of lisinopril (and other angiotensin-converting enzyme inhibitors). The nurse should assess the client's lung sounds and other signs of oxygenation first. The client may or may not need to switch antihypertensive medications. Vital signs and documentation are important, but the nurse should assess the respiratory system first. If the cough turns out to be a side effect, reminding the client is appropriate, but then more action needs to be taken. DIF: Applying/Application REF: 715

The nurse is assessing a client on admission to the hospital. The client's leg appears as shown below: What action by the nurse is best? a. Assess the client's ankle-brachial index. b. Elevate the client's leg above the heart. c. Obtain an ice pack to provide comfort. d. Prepare to teach about heparin sodium. *Black big toe*

ANS: A This client has dependent rubor, a classic finding in peripheral arterial disease. The nurse should measure the client's ankle-brachial index. Elevating the leg above the heart will further impede arterial blood flow. Ice will cause vasoconstriction, also impeding circulation and perhaps causing tissue injury. Heparin sodium is not the drug of choice for this condition. DIF: Applying/Application REF: 720

A nurse delegates care for a client who has open skin lesions. Which statements should the nurse include when delegating this client's hygiene care to an unlicensed assistive personnel (UAP)? (Select all that apply.) a."Wash your hands before touching the client." b."Wear gloves when bathing the client." c."Assess skin for breakdown during the bath." d."Apply lotion to lesions while the skin is wet." e."Use a damp cloth to scrub the lesions."

ANS: A, B All health care providers should follow Standard Precautions when caring for clients who have any open skin areas. This includes hand hygiene and wearing gloves when in contact with the lesions. The UAP is not qualified to assess the client's skin. The other statements are not appropriate for the care of open skin lesions.

10. A nurse is seeing many clients in the neurosurgical clinic. With which clients should the nurse plan to do more teaching? (Select all that apply.) a. Client with an aneurysm coil placed 2 months ago who is taking ibuprofen (Motrin) for sinus headaches b. Client with an aneurysm clip who states that his family is happy there is no chance of recurrence c. Client who had a coil procedure who says that there will be no problem following up for 1 year d. Client who underwent a flow diversion procedure 3 months ago who is taking docusate sodium (Colace) for constipation e. Client who underwent surgical aneurysm ligation 3 months ago who is planning to take a Caribbean cruise

ANS: A, B After a coil procedure, up to 20% of clients experience re-bleeding in the first year. The client with this coil should not be taking drugs that interfere with clotting. An aneurysm clip can move up to 5 years after placement, so this client and family need to be watchful for changing neurologic status. The other statements show good understanding. DIF: Evaluating/Synthesis REF: 940 KEY: Neurologic disorders MSC: Integrated Process: Nursing Process: Evaluation NOT: Client Needs Category: Physiological Integrity: Physiological Adaptation

A nurse teaches a client to perform total skin self-examinations on a monthly basis. Which statements should the nurse include in this client's teaching? (Select all that apply.) a. "Look for asymmetry of shape and irregular borders." b. "Assess for color variation within each lesion." c. "Examine the distribution of lesions over a section of the body." d. "Monitor for edema or swelling of tissues." e. "Focus your assessment on skin areas that itch."

ANS: A, B Clients should be taught to examine each lesion following the ABCDE features associated with skin cancer: asymmetry of shape, border irregularity, color variation within one lesion, diameter greater than 6 mm, and evolving or changing in any feature.

A nurse cares for a client who presents with tachycardia and prostration related to biliary colic. Which actions should the nurse take? (Select all that apply.) a. Contact the provider immediately. b. Lower the head of the bed. c. Decrease intravenous fluids. d. Ask the client to bear down. e. Administer prescribed opioids.

ANS: A, B Clients who are experiencing biliary colic may present with tachycardia, pallor, diaphoresis, prostration, or other signs of shock. The nurse should stay with the client, lower the client's head, and contact the provider or Rapid Response Team for immediate assistance. Treatment for shock usually includes intravenous fluids; therefore, decreasing fluids would be an incorrect intervention. The client's tachycardia is a result of shock, not pain. Performing the vagal maneuver or administering opioids could knock out the client's compensation mechanism.

A parent asks the nurse about the characteristics of a nightmare. What response should the nurse give to the parent? (Select all that apply.) a. Nightmares are scary dreams. b. The child can describe the nightmare. c. The child is reassured by your presence. d. Nightmares occur usually 1 to 4 hours after falling asleep. e. Nightmares take place during nonrapid eye movement sleep

ANS: A, B, C Nightmares are scary dreams, the child can describe the nightmare, and the child is reassured by a parents presence. Sleep terrors occur usually 1 to 4 hours after falling asleep, but nightmares occur in the second half of sleep. Sleep terrors occur during nonrapid eye movement sleep, but nightmares occur during rapid eye movement sleep.

2. A hospital nurse is participating in a drill during which many clients with inhalation anthrax are being admitted. What drugs should the nurse anticipate administering? (Select all that apply.) a. Amoxicillin (Amoxil) b. Ciprofloxacin (Cipro) c. Doxycycline (Vibramycin) d. Ethambutol (Myambutol) e. Sulfamethoxazole-trimethoprim (SMX-TMP) (Septra)

ANS: A, B, C Amoxicillin, ciprofloxacin, and doxycycline are all possible treatments for inhalation anthrax. Ethambutol is used for tuberculosis. SMX-TMP is commonly used for urinary tract infections and other common infections.

8. A nurse assesses a client who is recovering from a heart transplant. Which assessment findings should alert the nurse to the possibility of heart transplant rejection? (Select all that apply.) a. Shortness of breath b. Abdominal bloating c. New-onset bradycardia d. Increased ejection fraction e. Hypertension

ANS: A, B, C Clinical manifestations of heart transplant rejection include shortness of breath, fatigue, fluid gain, abdominal bloating, new-onset bradycardia, hypotension, atrial fibrillation or flutter, decreased activity tolerance, and decreased ejection fraction. DIF: Remembering/Knowledge REF: 703 KEY: Transplant| heart failure MSC: IntegratedProcess:NursingProcess:Assessment NOT: Client Needs Category: Physiological Integrity: Reduction of Risk Potential

3. A nurse teaches a client who has chronic obstructive pulmonary disease. Which statements related to nutrition should the nurse include in this client's teaching? (Select all that apply.) a. "Avoid drinking fluids just before and during meals." b. "Rest before meals if you have dyspnea." c. "Have about six small meals a day." d. "Eat high-fiber foods to promote gastric emptying." e. "Increase carbohydrate intake for energy."

ANS: A, B, C Fluids can make a client feel bloated and should be avoided with meals. Resting before the meal will help a client with dyspnea. Six small meals a day also will help to decrease bloating. Fibrous foods can produce gas, which can cause abdominal bloating and can increase shortness of breath. The client should increase calorie and protein intake to prevent malnourishment. The client should not increase carbohydrate intake as this will increase carbon dioxide production and increase the client's risk of for acidosis. DIF: Applying/Application REF: 565 KEY: Nutrition| patient education MSC: Integrated Process: Teaching/Learning NOT: Client Needs Category: Health Promotion and Maintenance

20. A nurse assesses a client who has aortic regurgitation. In which location in the illustration shown below should the nurse auscultate to best hear a cardiac murmur related to aortic regurgitation? a. Location A b. Location B c. Location C d. Location D ANS: A The aortic valve is auscultated in the second intercostal space just to the right of the sternum. DIF: Applying/Application REF: 639 KEY: Assessment/diagnostic examination MSC: IntegratedProcess:NursingProcess:Assessment NOT: Client Needs Category: Health Promotion and Maintenance MULTIPLE RESPONSE 1. A nurse is caring for a client with a history of renal insufficiency who is scheduled for a cardiac catheterization. Which actions should the nurse take prior to the catheterization? (Select all that apply.) a. Assess for allergies to iodine. b. Administer intravenous fluids. c. Assess blood urea nitrogen (BUN) and creatinine results. d. Insert a Foley catheter. e. Administer a prophylactic antibiotic. f. Insert a central venous catheter.

ANS: A, B, C If the client has kidney disease (as indicated by BUN and creatinine results), fluids and Mucomyst may be given 12 to 24 hours before the procedure for renal protection. The client should be assessed for allergies to iodine, including shellfish; the contrast medium used during the catheterization contains iodine. A Foley catheter and central venous catheter are not required for the procedure and would only increase the client's risk for infection. Prophylactic antibiotics are not administered prior to a cardiac catheterization. DIF: Applying/Application REF: 643 KEY: Assessment/diagnostic examination MSC: Integrated Process: Nursing Process: Implementation NOT: Client Needs Category: Physiological Integrity: Reduction of Risk Potential

1. A home health nurse is visiting a new client who uses oxygen in the home. For which factors does the nurse assess when determining if the client is using the oxygen safely? (Select all that apply.) a. The client does not allow smoking in the house. b. Electrical cords are in good working order. c. Flammable liquids are stored in the garage. d. Household light bulbs are the fluorescent type. e. The client does not have pets inside the home.

ANS: A, B, C Oxygen is an accelerant, which means it enhances combustion, so precautions are needed whenever using it. The nurse should assess if the client allows smoking near the oxygen, whether electrical cords are in good shape or are frayed, and if flammable liquids are stored (and used) in the garage away from the oxygen. Light bulbs and pets are not related to oxygen safety. DIF: Understanding/Comprehension REF: 515 KEY: Patient safety| fire| oxygen| home safety| nursing assessment MSC: IntegratedProcess:NursingProcess:Assessment NOT: Client Needs Category: Safe and Effective Care Environment: Safety and Infection Control

7. A client has meningitis following brain surgery. What comfort measures may the nurse delegate to the unlicensed assistive personnel (UAP)? (Select all that apply.) a. Applying a cool washcloth to the head b. Assisting the client to a position of comfort c. Keeping voices soft and soothing d. Maintaining low lighting in the room e. Providing antipyretics for fever

ANS: A, B, C, D The client with meningitis often has high fever, pain, and some degree of confusion. Cool washcloths to the forehead are comforting and help with pain. Allowing the client to assume a position of comfort also helps manage pain. Keeping voices low and lights dimmed also helps convey caring in a nonthreatening manner. The nurse provides antipyretics for fever. DIF: Applying/Application REF: 962 KEY: neurologic disorders| delegation| comfort measures| communication| unlicensed assistive personnel (UAP) MSC: Integrated Process: Communication and Documentation NOT: Client Needs Category: Physiological Integrity: Basic Care and Comfort

9. A client has a small-bore feeding tube (Dobhoff tube) inserted for continuous enteral feedings while recovering from a traumatic brain injury. What actions should the nurse include in the client's care? (Select all that apply.) a. Assess tube placement per agency policy. b. Keep the head of the bed elevated at least 30 degrees. c. Listen to lung sounds at least every 4 hours. d. Run continuous feedings on a feeding pump. e. Use blue dye to determine proper placement.

ANS: A, B, C, D All of these options are important for client safety when continuous enteral feedings are in use. Blue dye is not used because it can cause lung injury if aspirated. DIF: Applying/Application REF: 955 KEY: Neurologic disorders| enteral feedings MSC: Integrated Process: Nursing Process: Implementation NOT: Client Needs Category: Safe and Effective Care Environment: Safety and Infection Control

A client with fibromyalgia is in the hospital for an unrelated issue. The client reports that sleep, which is always difficult, is even harder now. What actions by the nurse are most appropriate? (Select all that apply.) a. Allow the client uninterrupted rest time. b. Assess the client's usual bedtime routine. c. Limit environmental noise as much as possible. d. Offer a massage or warm shower at night. e. Request an order for a strong sleeping pill.

ANS: A, B, C, D Clients with fibromyalgia often have sleep disturbances, which can be exacerbated by the stress, noise, and unfamiliar environment of the hospital. Allowing uninterrupted rest time, adhering to the client's usual bedtime routine as much as possible, limiting noise and light, and offering massages or warm showers can help. The client does not need a strong sleeping pill unless all other options fail and the client requests something for sleep. At that point a mild sleeping agent can be tried.

The nurse working in the gastrointestinal clinic sees clients who are anemic. What are common causes for which the nurse assesses in these clients? (Select all that apply.) a. Colon cancer b. Diverticulitis c. Inflammatory bowel disease d. Peptic ulcer disease e. Pernicious anemia

ANS: A, B, C, D In adults, the most common cause of anemia is GI bleeding. This is commonly associated with colon cancer, diverticulitis, inflammatory bowel disease, and peptic ulcer disease. Pernicious anemia is not associated with GI bleeding.

. The nurse caring for mechanically ventilated clients uses best practices to prevent ventilator-associated pneumonia. What actions are included in this practice? (Select all that apply.) a. Adherence to proper hand hygiene b. Administering anti-ulcer medication c. Elevating the head of the bed d. Providing oral care per protocol e. Suctioning the client on a regular schedule

ANS: A, B, C, D The "ventilator bundle" is a group of care measures to prevent ventilator-associated pneumonia. Actions in the bundle include using proper hand hygiene, giving anti-ulcer medications, elevating the head of the bed, providing frequent oral care per policy, preventing aspiration, and providing pulmonary hygiene measures. Suctioning is done as needed.

4. A client has been diagnosed with an empyema. What interventions should the nurse anticipate providing to this client? (Select all that apply.) a. Assisting with chest tube insertion b. Facilitating pleural fluid sampling c. Performing frequent respiratory assessment d. Providing antipyretics as needed e. Suctioning deeply every 4 hours

ANS: A, B, C, D The client with an empyema is often treated with chest tube insertion, which facilitates obtaining samples of the pleural fluid for analysis and re-expands the lungs. The nurse should perform frequent respiratory system assessments. Antipyretic medications are also used. Suction is only used when needed and is not done deeply to prevent tissue injury.

When working with older adults to promote good nutrition, what actions by the nurse are most appropriate? (Select all that apply.) a. Allow uninterrupted time for eating. b. Assess dentures for appropriate fit. c. Ensure the client has glasses on when eating. d. Provide salty foods that the client can taste. e. Serve high-calorie, high-protein snacks.

ANS: A, B, C, E NO SALTY FOOD!

The student nurse learns about risk factors for gastric cancer. Which factors does this include? (Select all that apply.) a. Achlorhydria b. Chronic atrophic gastritis c. Helicobacter pylori infection d. Iron deficiency anemia e. Pernicious anemia

ANS: A, B, C, E Achlorhydria, chronic atrophic gastritis, H. pylori infection, and pernicious anemia are all risk factors for developing gastric cancer. Iron deficiency anemia is not a risk factor.

3. A client with a new pulmonary embolism (PE) is anxious. What nursing actions are most appropriate? (Select all that apply.) a. Acknowledge the frightening nature of the illness. b. Delegate a back rub to the unlicensed assistive personnel (UAP). c. Give simple explanations of what is happening. d. Request a prescription for antianxiety medication. e. Stay with the client and speak in a quiet, calm voice.

ANS: A, B, C, E Clients with PEs are often anxious. The nurse can acknowledge the client's fears, delegate comfort measures, give simple explanations the client will understand, and stay with the client. Using a calm, quiet voice is also reassuring. Sedatives and antianxiety medications are not used routinely because they can contribute to hypoxia. If the client's anxiety is interfering with diagnostic testing or treatment, they can be used, but there is no evidence that this is the case.

5. A nurse is teaching a client about possible complications and hazards of home oxygen therapy. About which complications does the nurse plan to teach the client? that apply.) a. Absorptive atelectasis b. Combustion c. Dried mucous membranes d. Oxygen-induced hyperventilation e. Toxicity (Select all

ANS: A, B, C, E Complications of oxygen therapy include absorptive atelectasis, combustion, dried mucous membranes, and oxygen toxicity. Oxygen-induced hypoventilation is also a complication. DIF: Understanding/Comprehension REF: 515 KEY: Respiratory system| oxygen therapy| home safety| patient education MSC: IntegratedProcess:Teaching/Learning NOT: Client Needs Category: Safe and Effective Care Environment: Safety and Infection Control

5. When working with older adults to promote good nutrition, what actions by the nurse are most appropriate? (Select all that apply.) a. Allow uninterrupted time for eating. b. Assess dentures for appropriate fit. c. Ensure the client has glasses on when eating. d. Provide salty foods that the client can taste. e. Serve high-calorie, high-protein snacks.

ANS: A, B, C, E Older adults need unhurried and uninterrupted time for eating. Dentures should fit appropriately and glasses, if used, should be on. High-calorie, high-protein snacks are a good choice. Salty snacks are not recommended because all adults should limit sodium in their diets.

The student nurse studying stomach disorders learns that the risk factors for acute gastritis include which of the following? (Select all that apply.) a. Alcohol b. Caffeine c. Corticosteroids d. Fruit juice e. Nonsteroidal anti-inflammatory drugs (NSAIDs)

ANS: A, B, C, E Risk factors for acute gastritis include alcohol, caffeine, corticosteroids, and chronic NSAID use. Fruit juice is not a risk factor, although in some people it does cause distress.

The nurse working with older clients understands age-related changes in the gastrointestinal system. Which changes does this include? (Select all that apply.) a. Decreased hydrochloric acid production b. Diminished sensation that can lead to constipation c. Fat not digested as well in older adults d. Increased peristalsis in the large intestine e. Pancreatic vessels become calcified

ANS: A, B, C, E Several age-related changes occur in the gastrointestinal system. These include decreased hydrochloric acid production, diminished nerve function that leads to decreased sensation of the need to pass stool, decreased fat digestion, decreased peristalsis in the large intestine, and calcification of pancreatic vessels.

A nurse assesses a client with ulcerative colitis. Which complications are paired correctly with their physiologic processes? (Select all that apply.) a. Lower gastrointestinal bleeding - Erosion of the bowel wall b. Abscess formation - Localized pockets of infection develop in the ulcerated bowel lining c. Toxic megacolon - Transmural inflammation resulting in pyuria and fecaluria d. Nonmechanical bowel obstruction - Paralysis of colon resulting from colorectal cancer e. Fistula - Dilation and colonic ileus caused by paralysis of the colon

ANS: A, B, D Lower GI bleeding can lead to erosion of the bowel wall. Abscesses are localized pockets of infection that develop in the ulcerated bowel lining. Nonmechanical bowel obstruction is paralysis of the colon that results from colorectal cancer. When the inflammation is transmural, fistulas can occur between the bowel and bladder resulting in pyuria and fecaluria. Paralysis of the colon causing dilation and subsequent colonic ileus is known as a toxic megacolon. DIF: Understanding/Comprehension REF: 1181 KEY: Ulcerative colitis MSC: Integrated Process: Nursing Process: Analysis NOT: Client Needs Category: Physiological Integrity: Physiological Adaptation

5. A nurse teaches a client with polycystic kidney disease (PKD). Which statements should the nurse include in this client's discharge teaching? (Select all that apply.) a. "Take your blood pressure every morning." b. "Weigh yourself at the same time each day." c. "Adjust your diet to prevent diarrhea." d. "Contact your provider if you have visual disturbances." e. "Assess your urine for renal stones."

ANS: A, B, D A client who has PKD should measure and record his or her blood pressure and weight daily, limit salt intake, and adjust dietary selections to prevent constipation. The client should notify the provider if urine smells foul or has blood in it, as these are signs of a urinary tract infection or glomerular injury. The client should also notify the provider if visual disturbances are experienced, as this is a sign of a possible berry aneurysm, which is a complication of PKD. Diarrhea and renal stones are not manifestations or complications of PKD; therefore, teaching related to these concepts would be inappropriate.

3. A nurse assesses clients on a cardiac unit. Which clients should the nurse identify as at greatest risk for the development of acute pericarditis? (Select all that apply.) a. A 36-year-old woman with systemic lupus erythematosus (SLE) b. A 42-year-old man recovering from coronary artery bypass graft surgery c. A 59-year-old woman recovering from a hysterectomy d. An 80-year-old man with a bacterial infection of the respiratory tract e. An 88-year-old woman with a stage III sacral ulcer

ANS: A, B, D Acute pericarditis is most commonly associated acute exacerbations of systemic connective tissue disease, including SLE; with Dressler's syndrome, or inflammation of the cardiac sac after cardiac surgery or a myocardial infarction; and with infective organisms, including bacterial, viral, and fungal infections. Abdominal and reproductive surgeries and pressure ulcers do not increase clients' risk for acute pericarditis. DIF: Applying/Application REF: 699 KEY: Inflammatory response| health screening MSC: IntegratedProcess:NursingProcess:Assessment NOT: Client Needs Category: Safe and Effective Care Environment: Management of Care

6. The nurse caring for mechanically ventilated clients knows that older adults are at higher risk for weaning failure. What age-related changes contribute to this? (Select all that apply.) a. Chest wall stiffness b. Decreased muscle strength c. Inability to cooperate d. Less lung elasticity e. Poor vision and hearing

ANS: A, B, D Age-related changes that increase the difficulty of weaning older adults from mechanical ventilation include increased stiffness of the chest wall, decreased muscle strength, and less elasticity of lung tissue. Not all older adults have an inability to cooperate or poor sensory acuity.

A client has rheumatoid arthritis (RA) and the visiting nurse is conducting a home assessment. What options can the nurse suggest for the client to maintain independence in activities of daily living (ADLs)? (Select all that apply.) a. Grab bars to reach high items b. Long-handled bath scrub brush c. Soft rocker-recliner chair d. Toothbrush with built-up handle e. Wheelchair cushion for comfort

ANS: A, B, D Grab bars, long-handled bath brushes, and toothbrushes with built-up handles all provide modifications for daily activities, making it easier for the client with RA to complete ADLs independently. The rocker-recliner and wheelchair cushion are comfort measures but do not help increase independence.

6. A nurse plans care for a client who has chronic obstructive pulmonary disease and thick, tenacious secretions. Which interventions should the nurse include in this client's plan of care? (Select all that apply.) a. Ask the client to drink 2 liters of fluids daily. b. Add humidity to the prescribed oxygen. c. Suction the client every 2 to 3 hours. d. Use a vibrating positive expiratory pressure device. e. Encourage diaphragmatic breathing.

ANS: A, B, D Interventions to decrease thick tenacious secretions include maintaining adequate hydration and providing humidified oxygen. These actions will help to thin secretions, making them easier to remove by coughing. The use of a vibrating positive expiratory pressure device can also help clients remove thick secretions. Although suctioning may assist with the removal of secretions, frequent suctioning can cause airway trauma and does not support the client's ability to successfully remove secretions through normal coughing. Diaphragmatic breathing is not used to improve the removal of thick secretions. DIF: Applying/Application REF: 564 KEY: Respiratory distress/failure MSC: IntegratedProcess:NursingProcess:Implementation NOT: Client Needs Category: Physiological Integrity: Reduction of Risk Potential

2. A nurse assesses a client with nephrotic syndrome. For which clinical manifestations should the nurse assess? (Select all that apply.) a. Proteinuria b. Hypoalbuminemia c. Dehydration d. Lipiduria e. Dysuria f. Costovertebral angle (CVA) tenderness

ANS: A, B, D Nephrotic syndrome is caused by glomerular damage and is characterized by proteinuria (protein level higher than 3.5 g/24 hr), hypoalbuminemia, edema, and lipiduria. Fluid overload leading to edema and hypertension is common with nephrotic syndrome; dehydration does not occur. Dysuria is present with cystitis. CVA tenderness is present with inflammatory changes in the kidney.

5. A nurse is caring for a client who is on mechanical ventilation. What actions will promote comfort in this client? (Select all that apply.) a. Allow visitors at the client's bedside. b. Ensure the client can communicate if awake. c. Keep the television tuned to a favorite channel. d. Provide back and hand massages when turning. e. Turn the client every 2 hours or more.

ANS: A, B, D, E There are many basic care measures that can be employed for the client who is on a ventilator. Allowing visitation, providing a means of communication, massaging the client's skin, and routinely turning and repositioning the client are some of them. Keeping the TV on will interfere with sleep and rest.

6. A nurse prepares to discharge a client who has heart failure. Based on the Heart Failure Core Measure Set, which actions should the nurse complete prior to discharging this client? (Select all that apply.) a. Teach the client about dietary restrictions. b. Ensure the client is prescribed an angiotensin-converting enzyme (ACE) inhibitor. c. Encourage the client to take a baby aspirin each day. d. Confirm that an echocardiogram has been completed. e. Consult a social worker for additional resources.

ANS: A, B, D The Heart Failure Core Measure Set includes discharge instructions on diet, activity, medications, weight monitoring and plan for worsening symptoms, evaluation of left ventricular systolic function (usually with an echocardiogram), and prescribing an ACE inhibitor or angiotensin receptor blocker. Aspirin is not part of the Heart Failure Core Measure Set and is usually prescribed for clients who experience a myocardial infarction. Although the nurse may consult the social worker or case manager for additional resources, this is not part of the Core Measures. DIF: Understanding/Comprehension REF: 689 KEY: Heart failure| discharge| Core Measures| The Joint Commission MSC: Integrated Process: Nursing Process: Analysis NOT: Client Needs Category: Safe and Effective Care Environment: Management of Care

A client has been diagnosed with fibromyalgia syndrome but does not want to take the prescribed medications. What nonpharmacologic measures can the nurse suggest to help manage this condition? (Select all that apply.) a. Acupuncture b. Stretching c. Supplements d. Tai chi e. Vigorous aerobics

ANS: A, B, D There are many nonpharmacologic means for controlling the symptoms of fibromyalgia, including acupuncture, stretching, tai chi, low-impact aerobics, swimming, biking, strengthening, massage, stress management, and hypnosis. Dietary supplements and vigorous aerobics are not recommended.

7. A nurse prepares to discharge a client who has heart failure. Which questions should the nurse ask to ensure this client's safety prior to discharging home? (Select all that apply.) a. "Are your bedroom and bathroom on the first floor?" b. "What social support do you have at home?" c. "Will you be able to afford your oxygen therapy?" d. "What spiritual beliefs may impact your recovery?" e. "Are you able to accurately weigh yourself at home?"

ANS: A, B, D To ensure safety upon discharge, the nurse should assess for structural barriers to functional ability, such as stairs. The nurse should also assess the client's available social support, which may include family, friends, and home health services. The client's ability to adhere to medication and treatments, including daily weights, should also be reviewed. The other questions do not address the client's safety upon discharge. DIF: Applying/Application REF: 689 KEY: Heart failure| discharge| safety MSC: Integrated Process: Nursing Process: Analysis NOT: Client Needs Category: Safe and Effective Care Environment: Safety and Infection Control

A home health care nurse is visiting a client discharged home after a hip replacement. The client is still on partial weight bearing and using a walker. What safety precautions can the nurse recommend to the client? (Select all that apply.) a. Buy and install an elevated toilet seat. b. Install grab bars in the shower and by the toilet. c. Step into the bathtub with the affected leg first. d. Remove all throw rugs throughout the house. e. Use a shower chair while taking a shower.

ANS: A, B, D, E Buying and installing an elevated toilet seat, installing grab bars, removing throw rugs, and using a shower chair will all promote safety for this client. The client is still on partial weight bearing, so he or she cannot step into the bathtub leading with the operative side. Stepping into a bathtub may also require the client to bend the hip more than the allowed 90 degrees.

8. A nurse is working with many stroke clients. Which clients would the nurse consider referring to a mental health provider on discharge? (Select all that apply.) a. Client who exhibits extreme emotional lability b. Client with an initial National Institutes of Health (NIH) Stroke Scale score of 38 c. Client with mild forgetfulness and a slight limp d. Client who has a past hospitalization for a suicide attempt e. Client who is unable to walk or eat 3 weeks post-stroke

ANS: A, B, D, E Clients most at risk for post-stroke depression are those with a previous history of depression, severe stroke (NIH Stroke Scale score of 38 is severe), and post-stroke physical or cognitive impairment. The client with mild forgetfulness and a slight limp would be a low priority for this referral. DIF: Applying/Application REF: 935 KEY: Neurologic disorders| stroke| psychosocial response| depression| nursing assessment MSC: IntegratedProcess:NursingProcess:Assessment NOT: Client Needs Category: Psychosocial Integrity

A nurse is teaching a female client with rheumatoid arthritis (RA) about taking methotrexate (MTX) (Rheumatrex) for disease control. What information does the nurse include? (Select all that apply.) a. "Avoid acetaminophen in over-the-counter medications." b. "It may take several weeks to become effective on pain." c. "Pregnancy and breast-feeding are not affected by MTX." d. "Stay away from large crowds and people who are ill." e. "You may find that folic acid, a B vitamin, reduces side effects."

ANS: A, B, D, E MTX is a disease-modifying antirheumatic drug and is used as a first-line drug for RA. MTX can cause liver toxicity, so the client should be advised to avoid medications that contain acetaminophen. It may take 4 to 6 weeks for effectiveness. MTX can cause immunosuppression, so avoiding sick people and crowds is important. Folic acid helps reduce side effects for some people. Pregnancy and breast-feeding are contraindicated while on this drug.

4. A nurse is planning discharge teaching on tracheostomy care for an older client. What factors does the nurse need to assess before teaching this particular client? (Select all that apply.) a. Cognition b. Dexterity c. Hydration d. Range of motion e. Vision

ANS: A, B, D, E The older adult is at risk for having impairments in cognition, dexterity, range of motion, and vision that could limit the ability to perform tracheostomy care and should be assessed. Hydration is not directly related to the ability to perform self-care. DIF: Understanding/Comprehension REF: 529 KEY: Older adult| tracheostomy| patient education MSC: IntegratedProcess:NursingProcess:Assessment NOT: Client Needs Category: Health Promotion and Maintenance

A nurse is teaching parents methods to reduce lead levels in their home. Which should the nurse include in the teaching? (Select all that apply.) a. Plant bushes around the outside of the house. b. Ensure your child eats frequent meals. c. Use hot water from the tap when boiling vegetables. d. Food can be stored in ceramic in the refrigerator. e. Ensure that your child's diet contains sufficient iron and calcium.

ANS: A, B, E Methods to reduce lead levels in homes include: planting bushes around the outside of the house if soil is contaminated with lead, so children cannot play there; ensuring that children eat regular meals because more lead is absorbed on an empty stomach; and ensuring that children's diets contain sufficient iron and calcium. Cold water should only be used for drinking, cooking, and reconstituting powder infant formula. Hot water dissolves lead more quickly than cold water and thus contains higher levels of lead. Do not use pottery or ceramic ware that was inadequately fired or is meant for decorative use for food storage or service.

What can the nurse suggest to families to reduce blood lead levels? (Select all that apply.) a. Do not store food in open cans. b. Ensure the child eats regular meals. c. Mix formula with hot water from the tap. d. Vacuum hard-surfaced floors and window wells. e. Wash and dry the childs hands and face frequently.

ANS: A, B, E To reduce blood lead levels, the family should ensure the child eats regular meals because more lead is absorbed on an empty stomach. The childs hands and face should be washed and dried frequently, especially before eating. Food should not be stored in open cans, particularly if cans are imported. Hot water dissolves lead more quickly than cold water and thus contains higher levels of lead. Hot water should not be used to mix formula. Hard-surfaced floors or window sills or wells should not be vacuumed because this spreads dust.

A nurse cares for older adult clients in a long-term acute care facility. Which interventions should the nurse implement to prevent skin breakdown in these clients? (Select all that apply.) a.Use a lift sheet when moving the client in bed. b.Avoid tape when applying dressings. c.Avoid whirlpool therapy. d.Use loose dressing on all wounds. e.Implement pressure-relieving devices.

ANS: A, B, E Using a lift sheet will prevent shearing forces from tearing skin. Tape should be avoided so that the skin won't tear. Using pressure-relieving devices for clients who are at risk for pressure ulcer formation, including older adults, is a proactive approach to prevent skin breakdown. No contraindication to using whirlpool therapy for the older client is known. Dressings should be applied as prescribed, not so loose that they do not provide required treatment, and not so tight that they decrease blood flow to tissues.

A nurse assesses a client with peritonitis. Which clinical manifestations should the nurse expect to find? (Select all that apply.) a. Distended abdomen b. Inability to pass flatus c. Bradycardia d. Hyperactive bowel sounds e. Decreased urine output

ANS: A, B, E A client with peritonitis may present with a distended abdomen, diminished bowel sounds, inability to pass flatus or feces, tachycardia, and decreased urine output secondary to dehydration. Bradycardia and hyperactive bowel sounds are not associated with peritonitis. DIF: Remembering/Knowledge REF: 1170 KEY: Inflammatory bowel disorder| assessment/diagnostic examination MSC: Integrated Process: Nursing Process: Assessment NOT: Client Needs Category: Physiological Integrity: Physiological Adaptation

The nurse is working with clients who have connective tissue diseases. Which disorders are correctly paired with their manifestations? (Select all that apply.) a. Dry, scaly skin rash - Systemic lupus erythematosus (SLE) b. Esophageal dysmotility - Systemic sclerosis c. Excess uric acid excretion - Gout d. Footdrop and paresthesias - Osteoarthritis e. Vasculitis causing organ damage - Rheumatoid arthritis

ANS: A, B, E A dry, scaly skin rash is the most frequent dermatologic manifestation of SLE. Systemic sclerosis can lead to esophageal motility problems. Vasculitis leads to organ damage in rheumatoid arthritis. Gout is caused by hyperuricemia; the production of uric acid exceeds the excretion capability of the kidneys. Footdrop and paresthesias occur in rheumatoid arthritis.

MULTIPLE RESPONSE 1. A nurse is assessing a client with left-sided heart failure. For which clinical manifestations should the nurse assess? (Select all that apply.) a. Pulmonary crackles b. Confusion, restlessness c. Pulmonary hypertension d. Dependent edema e. Cough that worsens at night

ANS: A, B, E Left-sided heart failure occurs with a decrease in contractility of the heart or an increase in afterload. Most of the signs will be noted in the respiratory system. Right-sided heart failure occurs with problems from the pulmonary vasculature onward including pulmonary hypertension. Signs will be noted before the right atrium or ventricle including dependent edema. DIF: Remembering/Knowledge REF: 682 KEY: Heart failure| assessment/diagnostic examination MSC: IntegratedProcess:NursingProcess:Assessment NOT: Client Needs Category: Physiological Integrity: Physiological Adaptation

A nurse assesses a client with irritable bowel syndrome (IBS). Which questions should the nurse include in this client's assessment? (Select all that apply.) a. "Which food types cause an exacerbation of symptoms?" b. "Where is your pain and what does it feel like?" c. "Have you lost a significant amount of weight lately?" d. "Are your stools soft, watery, and black in color?" e. "Do you experience nausea associated with defecation?"

ANS: A, B, E The nurse should ask the client about factors that may cause exacerbations of symptoms, including food, stress, and anxiety. The nurse should also assess the location, intensity, and quality of the client's pain, and nausea associated with defecation or meals. Clients who have IBS do not usually lose weight and stools are not black in color.

2. A nurse evaluates laboratory results for a client with heart failure. Which results should the nurse expect? (Select all that apply.) a. Hematocrit: 32.8% b. Serum sodium: 130 mEq/L c. Serum potassium: 4.0 mEq/L d. Serum creatinine: 1.0 mg/dL e. Proteinuria f. Microalbuminuria

ANS: A, B, E, F A hematocrit of 32.8% is low (should be 42.6%), indicating a dilutional ratio of red blood cells to fluid. A serum sodium of 130 mEq/L is low because of hemodilution. Microalbuminuria and proteinuria are present, indicating a decrease in renal filtration. These are early warning signs of decreased compliance of the heart. The potassium level is on the high side of normal and the serum creatinine level is normal. DIF: Applying/Application REF: 683 KEY: Heart failure| assessment/diagnostic examination MSC: Integrated Process: Nursing Process: Evaluation NOT: Client Needs Category: Physiological Integrity: Reduction of Risk Potential

After teaching a client with an anal fissure, a nurse assesses the client's understanding. Which client actions indicate that the client correctly understands the teaching? (Select all that apply.) a. Taking a warm sitz bath several times each day b. Utilizing a daily enema to prevent constipation c. Using bulk-producing agents to aid elimination d. Self-administering anti-inflammatory suppositories e. Taking a laxative each morning

ANS: A, C, D Taking warm sitz baths each day, using bulk-producing agents, and administering anti-inflammatory suppositories are all appropriate actions for the client with an anal fissure. The client should not use enemas or laxatives to promote elimination, but rather should rely on bulk-producing agents such as psyllium hydrophilic mucilloid (Metamucil). DIF: Applying/Application REF: 1189 KEY: Skin lesions/wounds| bowel care MSC: Integrated Process: Nursing Process: Evaluation NOT: Client Needs Category: Physiological Integrity: Basic Care and Comfort

MULTIPLE RESPONSE 1. A nursing student studying the neurologic system learns which information? (Select all that apply.) a. An aneurysm is a ballooning in a weakened part of an arterial wall. b. An arteriovenous malformation is the usual cause of strokes. c. Intracerebral hemorrhage is bleeding directly into the brain. d. Reduced perfusion from vasospasm often makes stroke worse. e. Subarachnoid hemorrhage is caused by high blood pressure.

ANS: A, C, D An aneurysm is a ballooning of the weakened part of an arterial wall. Intracerebral hemorrhage is bleeding directly into the brain. Vasospasm often makes the damage from the initial stroke worse because it causes decreased perfusion. An arteriovenous malformation (AVM) is unusual. Subarachnoid hemorrhage is usually caused by a ruptured aneurysm or A VM. DIF: Remembering/Knowledge REF: 932 KEY: Neurologic disorders MSC: Integrated Process: Teaching/Learning NOT: Client Needs Category: Physiological Integrity: Physiological Adaptation

A nurse is performing an assessment on a school-age child. Which findings suggest the child is getting an excess of vitamin A? (Select all that apply.) a. Delayed sexual development b. Edema c. Pruritus d. Jaundice e. Paresthesia

ANS: A, C, D Excess vitamin A can cause delayed sexual development, pruritus, and jaundice. Edema is seen with excess sodium. Paresthesia occurs with excess riboflavin. DIF: Cognitive Level: Apply REF: p. 73 TOP: Integrated Process: Nursing Process: Assessment MSC: Area of Client Needs: Health Promotion and Maintenance

A nurse plans care for a client who has a wound that is not healing. Which focused assessments should the nurse complete to develop the client's plan of care? (Select all that apply.) a. Height b. Allergies c. Alcohol use d. Prealbumin laboratory results e. Liver enzyme laboratory results

ANS: A, C, D Nutritional status can have a significant impact on skin health and wound healing. The care plan for a client with poor nutritional status should include a high-protein, high-calorie diet. To determine the client's nutritional status, the nurse should assess height and weight, alcohol use, and prealbumin laboratory results. These data will provide information related to vitamin and protein deficiencies, and obesity. Allergies and liver enzyme laboratory results will not provide information about nutrition status or wound healing.

6. A nurse cares for older clients who have traumatic brain injury. What should the nurse understand about this population? (Select all that apply.) a. Admission can overwhelm the coping mechanisms for older clients. b. Alcohol is typically involved in most traumatic brain injuries for this age group. c. These clients are more susceptible to systemic and wound infections. d. Other medical conditions can complicate treatment for these clients. e. Very few traumatic brain injuries occur in this age group.

ANS: A, C, D Older clients often tolerate stress poorly, which includes being admitted to a hospital that is unfamiliar and noisy. Because of decreased protective mechanisms, they are more susceptible to both local and systemic infections. Other medical conditions can complicate their treatment and recovery. Alcohol is typically not related to traumatic brain injury in this population; such injury is most often from falls and motor vehicle crashes. The 65- to 76-year-old age group has the second highest rate of brain injuries compared to other age groups. DIF: Remembering/Knowledge REF: 951 KEY: Neurologic disorders| older adults| trauma MSC: IntegratedProcess:Teaching/Learning NOT: Client Needs Category: Health Promotion and Maintenance

5. A nurse collaborates with an unlicensed assistive personnel (UAP) to provide care for a client with congestive heart failure. Which instructions should the nurse provide to the UAP when delegating care for this client? (Select all that apply.) a. "Reposition the client every 2 hours." b. "Teach the client to perform deep-breathing exercises." c. "Accurately record intake and output." d. "Use the same scale to weigh the client each morning." e. "Place the client on oxygen if the client becomes short of breath."

ANS: A, C, D The UAP should reposition the client every 2 hours to improve oxygenation and prevent atelectasis. The UAP can also accurately record intake and output, and use the same scale to weigh the client each morning before breakfast. UAPs are not qualified to teach clients or assess the need for and provide oxygen therapy. DIF: Applying/Application REF: 684 KEY: Heart failure| delegation| interdisciplinary team| unlicensed assistive personnel (UAP) MSC: Integrated Process: Communication and Documentation NOT: Client Needs Category: Safe and Effective Care Environment: Management of Care

A nurse teaches a client how to avoid becoming ill with Salmonella infection again. Which statements should the nurse include in this client's teaching? (Select all that apply.) a. "Wash leafy vegetables carefully before eating or cooking them." b. "Do not ingest water from the garden hose or the pool." c. "Wash your hands before and after using the bathroom." d. "Be sure meat is cooked to the proper temperature." e. "Avoid eating eggs that are sunny side up or undercooked."

ANS: A, C, D, E Salmonella is usually contracted via contaminated eggs, beef, poultry, and green leafy vegetables. It is not transmitted through water in garden hoses or pools. Clients should wash leafy vegetables well, wash hands before and after using the restroom, make sure meat and eggs are cooked properly, and, because it can be transmitted by flies, keep flies off of food. DIF: Applying/Application REF: 1191 KEY: Inflammatory bowel disorder| infection control MSC: Integrated Process: Teaching/Learning NOT: Client Needs Category: Safe and Effective Care Environment: Safety and Infection Control

*Which personality characteristic is a nurse most likely to assess in a patient diagnosed with anorexia nervosa?* a. Carefree flexibility b. Rigidity, perfectionism c. Open displays of emotion d. High spirits and optimism

ANS: B Rigid thinking, inability to demonstrate flexibility, and difficulty changing cognitions are characteristic of patients with eating disorders. The incorrect options are rare in a patient with an eating disorder. Inflexibility, controlled emotions, and pessimism are more the rule.

2. The nurse working in the emergency department assesses a client who has symptoms of stroke. For what modifiable risk factors should the nurse assess? (Select all that apply.) a. Alcohol intake b. Diabetes c. High-fat diet d. Obesity e. Smoking

ANS: A, C, D, E Alcohol intake, a high-fat diet, obesity, and smoking are all modifiable risk factors for stroke. Diabetes is not modifiable but is a risk factor that can be controlled with medical intervention. DIF: Remembering/Knowledge REF: 933 KEY: Neurologic disorders| stroke| nursing assessment MSC: IntegratedProcess:NursingProcess:Assessment NOT: Client Needs Category: Physiological Integrity: Physiological Adaptation

1. A nurse is providing pneumonia vaccinations in a community setting. Due to limited finances, the event organizers must limit giving the vaccination to priority groups. What clients would be considered a priority when administering the pneumonia vaccination? (Select all that apply.) a. 22-year-old client with asthma b. Client who had a cholecystectomy last year c. Client with well-controlled diabetes d. Healthy 72-year-old client e. Client who is taking medication for hypertension

ANS: A, C, D, E Clients over 65 years of age and any client (no matter what age) with a chronic health condition would be considered a priority for a pneumonia vaccination. Having a cholecystectomy a year ago does not qualify as a chronic health condition.

1. The nurse understands that malnutrition can occur in hospitalized clients for several reasons. Which are possible reasons for this to occur? (Select all that apply.) a. Cultural food preferences b. Family bringing snacks c. Increased need for nutrition d. Need for NPO status e. Staff shortages

ANS: A, C, D, E Many factors increase the hospitalized clients risk for nutritional deficits. Cultural food preferences may make hospital food unpalatable. Ill clients have increased nutritional needs but may be NPO for testing or treatment, or have a loss of appetite from their illness. Staff shortages impact clients who need to be fed or assisted with meals. The family may bring snacks that are either healthy or unhealthy, so without further information, the nurse cannot assume the snacks are leading to malnutrition.

A patient referred to the eating disorders clinic has lost 35 pounds in 3 months. For which physical manifestations of anorexia nervosa should a nurse assess? Select all that apply. a. Peripheral edema b. Parotid swelling c. Constipation d. Hypotension e. Dental caries f. Lanugo

ANS: A, C, D, F Peripheral edema is often present because of hypoalbuminemia. Constipation related to starvation is often present. Hypotension is often present because of dehydration. Lanugo is often present and is related to starvation. Parotid swelling is associated with bulimia. Dental caries are associated with bulimia. See relationship to audience response question.

The nurse is teaching parents of preschool-aged children strategies to prevent sexual abuse. What should the nurse include in the teaching session? (Select all that apply.) a. Back up a childs right to say no. b. Dont take what your child says too seriously. c. Take a second look at signals of potential danger. d. Dont be too detailed about examples of sexual assault. e. Remind children that even nice people sometimes do mean things.

ANS: A, C, E To provide protection and preparation from sexual abuse, parents should back up a childs right to say no, take a second look at signals of potential danger, and remind children that even nice people sometimes do mean things. Parents should take what children say seriously and they should give specific definitions and examples of sexual assault.

A nurse teaches a community group about food poisoning and gastroenteritis. Which statements should the nurse include in this group's teaching? (Select all that apply.) a. "Rotavirus is more common among infants and younger children." b. "Escherichia coli diarrhea is transmitted by contact with infected animals." c. "To prevent E. coli infection, don't drink water when swimming." d. "Clients who have botulism should be quarantined within their home." e. "Parasitic diseases may not show up for 1 to 2 weeks after infection."

ANS: A, C, E Rotavirus is more common among the youngest of clients. Not drinking water while swimming can help prevent E. coli infection. Parasitic diseases may take up to 2 weeks to become symptomatic. People with botulism need to be hospitalized to monitor for respiratory failure and paralysis. Escherichia coli is not transmitted by contact with infected animals. DIF: Applying/Application REF: 1172 KEY: Inflammatory bowel disorder| infection control MSC: Integrated Process: Teaching/Learning NOT: Client Needs Category: Health Promotion and Maintenance

4. A nurse reviews a client's laboratory results. Which findings should alert the nurse to the possibility of atherosclerosis? (Select all that apply.) a. Total cholesterol: 280 mg/dL b. High-density lipoprotein cholesterol: 50 mg/dL c. Triglycerides: 200 mg/dL d. Serum albumin: 4 g/dL e. Low-density lipoprotein cholesterol: 160 mg/dL

ANS: A, C, E A lipid panel is often used to screen for cardiovascular risk. Total cholesterol, triglycerides, and low-density lipoprotein cholesterol levels are all high, indicating higher risk for cardiovascular disease. High-density lipoprotein cholesterol is within the normal range for both males and females. Serum albumin is not assessed for atherosclerosis. DIF: Applying/Application REF: 636 KEY: Assessment/diagnostic examination| health screening MSC: IntegratedProcess:NursingProcess:Assessment NOT: Client Needs Category: Physiological Integrity: Reduction of Risk Potential

Which interventions should the nurse implement when caring for a family of a sudden infant death syndrome (SIDS) infant? (Select all that apply.) a. Allow parents to say goodbye to their infant. b. Once parents leave the hospital, no further follow-up is required. c. Arrange for someone to take the parents home from the hospital. d. Avoid requesting an autopsy of the deceased infant. e. Conduct a debriefing session with the parents before they leave the hospital.

ANS: A, C, E An important aspect of compassionate care for parents experiencing a SIDS incident is allowing them to say good-bye to their infant. These are the parents' last moments with their infant, and they should be as quiet, meaningful, peaceful, and undisturbed as possible. Because the parents leave the hospital without their infant, it is helpful to accompany them to the car or arrange for someone else to take them home. A debriefing session may help health care workers who dealt with the family and deceased infant to cope with emotions that are often engendered when a SIDS victim is brought into the acute care facility. An autopsy may clear up possible misconceptions regarding the death. When the parents return home, a competent, qualified professional should visit them after the death as soon as possible.

A nurse plans care for a client who has acute pancreatitis and is prescribed nothing by mouth (NPO). With which health care team members should the nurse collaborate to provide appropriate nutrition to this client? (Select all that apply.) a. Registered dietitian b. Nursing assistant c. Clinical pharmacist d. Certified herbalist e. Health care provider

ANS: A, C, E Clients who are prescribed NPO while experiencing an acute pancreatitis episode may need enteral or parenteral nutrition. The nurse should collaborate with the registered dietitian, clinical pharmacist, and health care provider to plan and implement the more appropriate nutritional interventions. The nursing assistant and certified herbalist would not assist with this clinical decision.

6. A nurse cares for a client who is recovering from a right-sided heart catheterization. For which complications of this procedure should the nurse assess? (Select all that apply.) a. Thrombophlebitis b. Stroke c. Pulmonary embolism d. Myocardial infarction e. Cardiac tamponade

ANS: A, C, E Complications from a right-sided heart catheterization include thrombophlebitis, pulmonary embolism, and vagal response. Cardiac tamponade is a risk of both right- and left-sided heart catheterizations. Stroke and myocardial infarction are complications of left-sided heart catheterizations. DIF: Remembering/Knowledge REF: 643 KEY: Assessment/diagnostic examination MSC: IntegratedProcess:NursingProcess:Assessment NOT: Client Needs Category: Physiological Integrity: Reduction of Risk Potential

7. A nurse cares for a client who is prescribed an intravenous prostacyclin agent. Which actions should the nurse take to ensure the client's safety while on this medication? (Select all that apply.) a. Keep an intravenous line dedicated strictly to the infusion. b. Teach the client that this medication increases pulmonary pressures. c. Ensure that there is always a backup drug cassette available. d. Start a large-bore peripheral intravenous line. e. Use strict aseptic technique when using the drug delivery system.

ANS: A, C, E Intravenous prostacyclin agents should be administered in a central venous catheter with a dedicated intravenous line for this medication. Death has been reported when the drug delivery system is interrupted; therefore, a backup drug cassette should also be available. The nurse should use strict aseptic technique when using the drug delivery system. The nurse should teach the client that this medication decreases pulmonary pressures and increases lung blood flow. DIF: Understanding/Comprehension REF: 571 KEY: Medication administration| safety MSC: Integrated Process: Nursing Process: Implementation NOT: Client Needs Category: Physiological Integrity: Pharmacological and Parenteral Therapies

The nurse should teach the adolescent that the long-term effects of tanning can cause which conditions? (Select all that apply.) a. Phototoxic reactions b. Increased number of moles c. Premature aging d. Striae e. Increased risk of skin cancer

ANS: A, C, E Long-term effects of tanning include premature aging of the skin, increased risk of skin cancer, and, in susceptible individuals, phototoxic reactions. There has been no correlation to an increase in moles or striae (streaks or stripes on the skin, usually on the abdomen) development. DIF: Cognitive Level: Apply REF: p. 463 TOP: Integrated Process: Teaching/Learning MSC: Area of Client Needs: Health Promotion and Maintenance

An older client returning to the postoperative nursing unit after a hip replacement is disoriented and restless. What actions does the nurse delegate to the unlicensed assistive personnel (UAP)? (Select all that apply.) a. Apply an abduction pillow to the client's legs. b. Assess the skin under the abduction pillow straps. c. Place pillows under the heels to keep them off the bed. d. Monitor cognition to determine when the client can get up. e. Take and record vital signs per unit/facility policy.

ANS: A, C, E The UAP can apply an abduction pillow, elevate the heels on a pillow, and take/record vital signs. Assessing skin is the nurse's responsibility, although if the UAP notices abnormalities, he or she should report them. Determining when the client is able to get out of bed is also a nursing responsibility.

A nurse inserts a nasogastric (NG) tube for an adult client who has a bowel obstruction. Which actions does the nurse perform correctly? (Select all that apply.) a. Performs hand hygiene and positions the client in high-Fowler's position, with pillows behind the head and shoulders b. Instructs the client to extend the neck against the pillow once the NG tube has reached the oropharynx c. Checks for correct placement by checking the pH of the fluid aspirated from the tube d. Secures the NG tube by taping it to the client's nose and pinning the end to the pillowcase e. Connects the NG tube to intermittent medium suction with an anti-reflux valve on the air vent

ANS: A, C, E The client's head should be flexed forward once the NG tube has reached the oropharynx. The NG tube should be secured to the client's gown, not to the pillowcase, because it could become dislodged easily. All the other actions are appropriate.

Which dietary recommendations should a nurse make to an adolescent patient to manage constipation related to opioid analgesic administration? (Select all that apply.) a. Bran cereal b. Decrease fluid intake c. Prune juice d. Cheese e. Vegetables

ANS: A, C, E To manage the side effect of constipation caused by opioids, fluids should be increased, and bran cereal and vegetables are recommended to increase fiber. Prune juice can act as a nonpharmacologic laxative. Fluids should be increased, not decreased, and cheese can cause constipation so it should not be recommended. DIF: Cognitive Level: Apply REF: p. 132 TOP: Integrated Process: Nursing Process: Implementation MSC: Area of Client Needs: Physiologic Integrity

Where do the lesions of atopic dermatitis (eczema) most commonly occur in the infant? (Select all that apply.) a. Cheeks b. Buttocks c. Extensor surfaces of arms and legs d. Back e. Trunk f. Scalp

ANS: A, C, E, F The lesions of atopic dermatitis are generalized in the infant. They are most commonly on the cheeks, scalp, trunk, and extensor surfaces of the extremities. The buttocks and back are not common locations for the lesions of atopic dermatitis in infants.

1. A nursing student caring for a client removes the client's oxygen as prescribed. The client is now breathing what percentage of oxygen in the room air? a. 14% b. 21% c. 28% d. 31%

ANS: B Room air is 21% oxygen. DIF: Remembering/Knowledge KEY: Oxygen| physiology MSC: IntegratedProcess:NursingProcess:Assessment NOT: Client Needs Category: Physiological Integrity: Physiological Adaptation

Which of the following data would be included in a health history? (Select all that apply.) a. Review of systems b. Physical assessment c. Sexual history d. Growth measurements e. Nutritional assessment f. Family medical history

ANS: A, C, E, F The review of systems, sexual history, nutritional assessment, and family medical history are part of the health history. Physical assessment and growth measurements are components of the physical examination. DIF: Cognitive Level: Apply REF: p. 64 TOP: Integrated Process: Nursing Process: Assessment MSC: Area of Client Needs: Health Promotion and Maintenance

2. A nurse is caring for a client who has a tracheostomy tube. What actions may the nurse delegate to unlicensed assistive personnel (UAP)? (Select all that apply.) a. Applying water-soluble lip balm to the client's lips b. Ensuring the humidification provided is adequate c. Performing oral care with alcohol-based mouthwash d. Reminding the client to cough and deep breathe often e. Suctioning excess secretions through the tracheostomy

ANS: A, D The UAP can perform hygiene measures such as applying lip balm and reinforce teaching such as reminding the client to perform coughing and deep-breathing exercises. Oral care can be accomplished with normal saline, not products that dry the mouth. Ensuring the humidity is adequate and suctioning through the tracheostomy are nursing functions. DIF: Applying/Application REF: 515 KEY: Tracheostomy| oral care| delegation| unlicensed assistive personnel (UAP) MSC: Integrated Process: Communication and Documentation NOT: Client Needs Category: Safe and Effective Care Environment: Management of Care

A nurse teaches a community group ways to prevent Escherichia coli infection. Which statements should the nurse include in this group's teaching? (Select all that apply.) a. "Wash your hands after any contact with animals." b. "It is not necessary to buy a meat thermometer." c. "Stay away from people who are ill with diarrhea." d. "Use separate cutting boards for meat and vegetables." e. "Avoid swimming in backyard pools and using hot tubs."

ANS: A, D Washing hands after contact with animals and using separate cutting boards for meat and other foods will help prevent E. coli infection. The other statements are not related to preventing E. coli infection. DIF: Applying/Application REF: 1172 KEY: Inflammatory bowel disorder| infection control MSC: Integrated Process: Teaching/Learning NOT: Client Needs Category: Safe and Effective Care Environment: Safety and Infection Control

A young adult client is in the clinic for evaluation of amenorrhea lasting 3 months. She takes birth control pills but is on no other medications. Which actions by the nurse are most appropriate? (Select all that apply.) a. Instruct the client on collecting a urinalysis for a pregnancy test. b. Assess the client's urinary and bowel habits. c. Perform a physical assessment on the client's abdomen. d. Weigh the client and calculate the body mass index. e. Reassure the client that amenorrhea can occur with oral contraception.

ANS: A, D Amenorrhea can be caused by several things, but not by urinary or bowel problems. Pregnancy should always be considered, even if the woman is on birth control of any type. Too little body fat can lead to menstrual irregularities. Simply reassuring the client is not as helpful as conducting further assessment.

A client has dumping syndrome. What menu selections indicate the client understands the correct diet to manage this condition? (Select all that apply.) a. Canned unsweetened apricots b. Coffee cake c. Milk shake d. Potato soup e. Steamed broccoli

ANS: A, D Canned apricots and potato soup are appropriate selections as they are part of a high-protein, high-fat, low- to moderate-carbohydrate diet. Coffee cake and other sweets must be avoided. Milk products and sweet drinks such as shakes must be avoided. Gas-forming foods such as broccoli must also be avoided.

A parent asks the nurse about the characteristics of a sleep terror. What response should the nurse give to the parent? (Select all that apply.) a. The child screams during the sleep terror. b. Return to sleep is delayed because of persistent fear. c. The night terror occurs during the second half of night. d. The child has no memory of the dream with a sleep terror. e. The child is not aware of anothers presence during a sleep terror.

ANS: A, D, E During sleep terrors, the child screams and has no memory of the dream. The child is not aware of anothers presence during a sleep terror. Return to sleep is usually rapid with a sleep terror, but it is delayed with a nightmare. The sleep terror occurs usually within 1 to 4 hours of sleep, but nightmares occur during the second half of night.

4. After teaching a client with congestive heart failure (CHF), the nurse assesses the client's understanding. Which client statements indicate a correct understanding of the teaching related to nutritional intake? (Select all that apply.) a. "I'll read the nutritional labels on food items for salt content." b. "I will drink at least 3 liters of water each day." c. "Using salt in moderation will reduce the workload of my heart." d. "I will eat oatmeal for breakfast instead of ham and eggs." e. "Substituting fresh vegetables for canned ones will lower my salt intake."

ANS: A, D, E Nutritional therapy for a client with CHF is focused on decreasing sodium and water retention to decrease the workload of the heart. The client should be taught to read nutritional labels on all food items, omit table salt and foods high in sodium (e.g., ham and canned foods), and limit water intake to a normal 2 L/day. DIF: Applying/Application REF: 682 KEY: Heart failure| patient education MSC: Integrated Process: Teaching/Learning NOT: Client Needs Category: Physiological Integrity: Basic Care and Comfort

Based on the nurse's knowledge of wounds and wound healing, what are factors that can delay or cause dysfunctional wound healing? Select all that apply. A. Overweight B. Hypoxemia C. Hypervolemia D. Prolonged infection E. Corticosteroid therapy

ANS: A, D, E Poor nutrition without proper protein and calorie intake affects healing more than being overweight itself. Corticosteroid therapy or other immunocompromising therapy prevents macrophages from migrating to the site of injury, thus suppressing epithelialization. Hypovolemia, not hypervolemia, inhibits wound healing due to low circulating blood volume and oxygenation of tissues. Hypoxemia makes tissues more susceptible to infection due to insufficient oxygenation. Prolonged infection affects the healing process and causes increased scarring.

Where is the appropriate placement of a tongue blade for assessment of the mouth and throat? a. Center back area of tongue b. Side of the tongue c. Against the soft palate d. On the lower jaw

ANS: B Side of the tongue is the correct position. It avoids the gag reflex yet allows visualization. Placement in the center back area of the tongue will elicit the gag reflex. Against the soft palate and on the lower jaw are not appropriate places for the tongue blade. DIF: Cognitive Level: Understand REF: p. 98 TOP: Integrated Process: Nursing Process: Assessment MSC: Area of Client Needs: Health Promotion and Maintenance: Techniques of Physical Assessment

5. A nursing student studying traumatic brain injuries (TBIs) should recognize which facts about these disorders? (Select all that apply.) a. A client with a moderate trauma may need hospitalization. b. A Glasgow Coma Scale score of 10 indicates a mild brain injury. c. Only open head injuries can cause a severe TBI. d. A client with a Glasgow Coma Scale score of 3 has severe TBI. e. The terms "mild TBI" and "concussion" have similar meanings.

ANS: A, D, E "Mild TBI" is a term used synonymously with the term "concussion." A moderate TBI has a Glasgow Coma Scale (GCS) score of 9 to 12, and these clients may need to be hospitalized. Both open and closed head injuries can cause a severe TBI, which is characterized by a GCS score of 3 to 8. DIF: Remembering/Knowledge REF: 947 KEY: Neurologic disorders| trauma MSC: Integrated Process: Teaching/Learning NOT: Client Needs Category: Physiological Integrity: Physiological Adaptation

4. A nurse has applied to work at a hospital that has National Stroke Center designation. The nurse realizes the hospital adheres to eight Core Measures for ischemic stroke care. What do these Core Measures include? (Select all that apply.) a. Discharging the client on a statin medication b. Providing the client with comprehensive therapies c. Meeting goals for nutrition within 1 week d. Providing and charting stroke education e. Preventing venous thromboembolism

ANS: A, D, E Core Measures established by The Joint Commission include discharging stroke clients on statins, providing and recording stroke education, and taking measures to prevent venous thromboembolism. The client must be assessed for therapies but may go elsewhere for them. Nutrition goals are not part of the Core Measures. DIF: Remembering/Knowledge REF: 945 KEY: Neurologic disorders| stroke| Core Measures MSC: IntegratedProcess:Teaching/Learning NOT: Client Needs Category: Safe and Effective Care Environment: Management of Care

A 13-year-old is being seen in the clinic for a routine health check. The adolescent has not been in the clinic for 3 years but was up to date on immunizations at that time. Which immunizations should the adolescent receive? (Select all that apply.) a. DTaP (tetanus, diphtheria, acellular pertussis) b. MMR (measles, mumps, rubella) c. Hepatitis B d. Influenza e. MCV4 (meningococcal)

ANS: A, D, E The DTaP (tetanus, diphtheria, acellular pertussis) vaccine is recommended for adolescents 11 to 18 years old who have not received a tetanus booster (Td) or DTaP dose and have completed the childhood DTaP/DTP series. Meningococcal vaccine (MCV4) should be given to adolescents 11 to 12 years of age with a booster dose at age 16 years. Annual influenza vaccination with either the live attenuated influenza vaccine or trivalent influenza vaccine is recommended for all children and adolescents. The adolescent, previously up to date on vaccinations, would have received the MMR and hepatitis B as a child. DIF: Cognitive Level: Apply REF: p. 462 TOP: Integrated Process: Nursing Process: Implementation MSC: Area of Client Needs: Health Promotion and Maintenance

3. A client is being discharged home after having a tracheostomy placed. What suggestions does the nurse offer to help the client maintain self-esteem? (Select all that apply.) a. Create a communication system. b. Don't go out in public alone. c. Find hobbies to enjoy at home. d. Try loose-fitting shirts with collars. e. Wear fashionable scarves.

ANS: A, D, E The client with a tracheostomy may be shy and hesitant to go out in public. The client should have a sound communication method to ease frustration. The nurse can also suggest ways of enhancing appearance so the client is willing to leave the house. These can include wearing scarves and loose-fitting shirts to hide the stoma. Keeping the client homebound is not good advice. DIF: Understanding/Comprehension REF: 528 KEY: Tracheostomy| psychosocial response| patient education MSC: IntegratedProcess:Caring NOT: Client Needs Category: Psychosocial Integrity

A nurse is planning to use an interpreter during a health history interview of a non-English speaking patient and family. Which nursing care guidelines should the nurse include when using an interpreter? (Select all that apply.) a. Elicit one answer at a time. b. Interrupt the interpreter if the response from the family is lengthy. c. Comments to the interpreter about the family should be made in English. d. Arrange for the family to speak with the same interpreter, if possible. e. Introduce the interpreter to the family.

ANS: A, D, E When using an interpreter, the nurse should pose questions to elicit only one answer at a time, such as: "Do you have pain?" rather than "Do you have any pain, tiredness, or loss of appetite?" Refrain from interrupting family members and the interpreter while they are conversing. Introduce the interpreter to family and allow some time before the interview for them to become acquainted. Refrain from interrupting family members and the interpreter while they are conversing. Avoid commenting to the interpreter about family members because they may understand some English. DIF: Cognitive Level: Apply REF: p. 60 TOP: Integrated Process: Nursing Process: Assessment MSC: Area of Client Needs: Health Promotion and Maintenance

A nurse plans care for a client who is immobile. Which interventions should the nurse include in this client's plan of care to prevent pressure sores? (Select all that apply.) a.Place a small pillow between bony surfaces. b.Elevate the head of the bed to 45 degrees. c.Limit fluids and proteins in the diet. d.Use a lift sheet to assist with re-positioning. e.Re-position the client who is in a chair every 2 hours. f.Keep the client's heels off the bed surfaces. g.Use a rubber ring to decrease sacral pressure when up in the chair.

ANS: A, D, F A small pillow decreases the risk for pressure between bony prominences, a lift sheet decreases friction and shear, and heels have poor circulation and are at high risk for pressure sores, so they should be kept off hard surfaces. Head-of-the-bed elevation greater than 30 degrees increases pressure on pelvic soft tissues. Fluids and proteins are important for maintaining tissue integrity. Clients should be repositioned every hour while sitting in a chair. A rubber ring impairs capillary blood flow, increasing the risk for a pressure sore.

A nurse assesses a client with cholelithiasis. Which assessment findings should the nurse identify as contributors to this client's condition? (Select all that apply.) a. Body mass index of 46 b. Vegetarian diet c. Drinking 4 ounces of red wine nightly d. Pregnant with twins e. History of metabolic syndrome f. Glycosylated hemoglobin level of 15%

ANS: A, D, F Obesity, pregnancy, and diabetes are all risk factors for the development of cholelithiasis. A diet low in saturated fats and moderate alcohol intake may decrease the risk. Although metabolic syndrome is a precursor to diabetes, it is not a risk factor for cholelithiasis. The client should be informed of the connection.

A nurse assesses a wife who is caring for her husband. She has a Braden Scale score of 9. Which question should the nurse include in this assessment? a."Do you have a bedpan at home?" b."How are you coping with providing this care?" c."What are you doing to prevent pediculosis?" d."Are you sharing a bed with your husband?"

ANS: B A client with a Braden Scale score of 9 is at high risk for skin breakdown and requires moderate to maximum assistance to prevent further breakdown. Family members who care for clients at home may experience a disruption in family routines and added stress. The nurse should assess the wife's feelings and provide support for coping with changes. Asking about the client's toileting practices, prevention of pediculosis, and sleeping arrangements do not provide information about the caregiver's support and coping mechanisms and ability to continue to care for her husband.

A child has been admitted to the emergency department with an acetaminophen (Tylenol) poisoning. An antidote is being prescribed by the health care provider. Which antidote should the nurse prepare to administer? a. Naloxone (Narcan) b. N-acetylcysteine (Mucomyst) c. Flumazenil (Romazicon) d. Digoxin immune Fab (Digibind)

ANS: B Antidotes available to treat toxin ingestion include N-acetylcysteine for acetaminophen poisoning, naloxone for opioid overdose, flumazenil (Romazicon) for benzodiazepine (diazepam [Valium], midazolam [Versed]) overdose, and digoxin immune Fab (Digibind) for digoxin toxicity.

A nurse is beginning chelation therapy on a child for lead poisoning. Which intervention should the nurse implement during the time the child is receiving chelation therapy? a. Calorie counts b. Strict intake and output c. Telemetry monitoring d. Contact isolation

ANS: B Because calcium disodium edetate (EDTA) and lead are toxic to the kidneys, a nurse should keep strict records of intake and output to monitor renal functioning. Adequate hydration is essential during therapy because the chelates are excreted via the kidneys. Calorie counts, telemetry, or contact isolation would not be nursing interventions appropriate for a child undergoing chelation therapy.

A child is admitted to the hospital with lesions on his abdomen that appear like cigarette burns. What should accurate documentation by the nurse include? a. Two unhealed lesions are on the childs abdomen. b. Two round 4-mm lesions are on the childs lower abdomen. c. Two round symmetrical lesions are on the childs lower abdomen. d. Two round lesions on the childs abdomen that appear to be cigarette burns.

ANS: B Burn documentation should include the location, pattern, demarcation lines, and presence of eschar or blisters. The option that includes the size of the lesions is the most accurate.

What is the most common form of child maltreatment? a. Sexual abuse b. Child neglect c. Physical abuse d. Emotional abuse

ANS: B Child neglect, which is characterized by the failure to provide for the childs basic needs, is the most common form of child maltreatment. Sexual abuse, physical abuse, and emotional abuse are individually not as common as neglect.

A nurse assesses a client who has psoriasis. Which action should the nurse take first? a.Don gloves and an isolation gown. b.Shake the client's hand and introduce self. c.Assess for signs and symptoms of infections. d.Ask the client if she might be pregnant.

ANS: B Clients with psoriatic lesions are often self-conscious of their skin. The nurse should first provide direct contact and touch without gloves to establish a good report with the client. Psoriasis is not an infectious disease, nor is it contagious. The nurse would not need to wear gloves or an isolation gown. Obtaining a health history and assessing for an infection and pregnancy should be completed after establishing a report with the client.

The parents of a 7-year-old boy tell the nurse that lately he has been cruel to their family pets and actually caused physical harm. The nurses recommendation should be based on remembering what? a. This is an expected behavior at this age. b. This is a warning sign of a serious problem. c. This is harmless venting of anger and frustration. d. This is common in children who are physically abused.

ANS: B Cruelty to family pets is not an expected behavior. Hurting animals can be one of the earliest symptoms of a conduct disorder. Abusing animals does not dissipate violent emotions; rather, the acts may fuel the abusive behaviors. Referral for evaluation is essential. This behavior may be seen in emotional abuse or neglect, not physical abuse

After educating a caregiver of a home care client, a nurse assesses the caregiver's understanding. Which statement indicates that the caregiver needs additional education? a."I can help him shift his position every hour when he sits in the chair." b."If his tailbone is red and tender in the morning, I will massage it with baby oil." c."Applying lotion to his arms and legs every evening will decrease dryness." d."Drinking a nutritional supplement between meals will help maintain his weight."

ANS: B Massage of reddened areas over bony prominences such as the coccyx, or tailbone, is contraindicated because the pressure of the massage can cause damage to the skin and subcutaneous tissue layers. The other statements are appropriate for the care of a client at home.

After teaching a client who has psoriasis, a nurse assesses the client's understanding. Which statement indicates the client needs additional teaching? a."At the next family reunion, I'm going to ask my relatives if they have psoriasis." b."I have to make sure I keep my lesions covered, so I do not spread this to others." c."I expect that these patches will get smaller when I lie out in the sun." d."I should continue to use the cortisone ointment as the patches shrink and dry out."

ANS: B Psoriasis is not a contagious disorder. The client does not have to worry about spreading the condition to others. It is a condition that has hereditary links, the patches will decrease in size with ultraviolet light exposure, and cortisone ointment should be applied directly to lesions to suppress cell division.

6. What is a significant secondary prevention nursing activity for lead poisoning? a. Chelation therapy b. Screening children for blood lead levels c. Removing lead-based paint from older homes d. Questioning parents about ethnic remedies containing lead

ANS: B Screening children for lead poisoning is an important secondary prevention activity. Screening does not prevent the initial exposure of the child to lead. It can lead to identification and treatment of children who are exposed. Chelation therapy is treatment, not prevention. Removing lead-based paints from older homes before children are affected is primary prevention. Questioning parents about ethnic remedies containing lead is part of the assessment to determine the potential source of lead.

After teaching a client who is at risk for the formation of pressure ulcers, a nurse assesses the client's understanding. Which dietary choice by the client indicates a good understanding of the teaching? a.Low-fat diet with whole grains and cereals and vitamin supplements b.High-protein diet with vitamins and mineral supplements c.Vegetarian diet with nutritional supplements and fish oil capsules d.Low-fat, low-cholesterol, high-fiber, low-carbohydrate diet

ANS: B The preferred diet is high in protein to assist in wound healing and prevention of new wounds. Fat is also needed to ensure formation of cell membranes, so any of the options with low fat would not be good choices. A vegetarian diet would not provide fat and high levels of protein

A nurse performs a skin screening for a client who has numerous skin lesions. Which lesion does the nurse evaluate first? a.Beige freckles on the backs of both hands b.Irregular blue mole with white specks on the lower leg c.Large cluster of pustules in the right axilla d.Thick, reddened papules covered by white scales

ANS: B This mole fits two of the criteria for being cancerous or precancerous: variation of color within one lesion, and an indistinct or irregular border. Melanoma is an invasive malignant disease with the potential for a fatal outcome. Freckles are a benign condition. Pustules could mean an infection, but it is more important to take care of the potentially cancerous lesion first. Psoriasis vulgaris manifests as thick reddened papules covered by white scales. This is a chronic disorder and is not the priority.

A father calls the clinic because he found his young daughter squirting Visine eyedrops into her mouth. What is the most appropriate nursing action? a. Reassure the father that Visine is harmless. b. Direct him to seek immediate medical treatment. c. Recommend inducing vomiting with ipecac. d. Advise him to dilute Visine by giving his daughter several glasses of water to drink.

ANS: B Visine is a sympathomimetic and if ingested may cause serious consequences. Medical treatment is necessary. Inducing vomiting is no longer recommended for ingestions. Dilution will not decrease risk.

3. Which teaching point is most important for the client with bacterial pharyngitis? a. Gargle with warm salt water. b. Take all antibiotics as directed. c. Use a humidifier in the bedroom. d. Wash hands frequently.

ANS: B Any client on antibiotics must be instructed to complete the entire course of antibiotics. Not completing them can lead to complications or drug-resistant strains of bacteria. The other instructions are appropriate, just not the most important.

A nurse assesses a client who is hospitalized for botulism. The client's vital signs are temperature: 99.8° F (37.6° C), heart rate: 100 beats/min, respiratory rate: 10 breaths/min, and blood pressure: 100/62 mm Hg. Which action should the nurse take? a. Decrease stimulation and allow the client to rest. b. Stay with the client while another nurse calls the provider. c. Increase the client's intravenous fluid replacement rate. d. Check the client's blood glucose and administer orange juice.

ANS: B A client with botulism is at risk for respiratory failure. This client's respiratory rate is slow, which could indicate impending respiratory distress or failure. The nurse should remain with the client while another nurse notifies the provider. The nurse should monitor and document the IV infusion per protocol, but this client does not require additional intravenous fluids. Allowing the client to rest or checking the client's blood glucose and administering orange juice are not appropriate actions. DIF: Applying/Application REF: 1191 KEY: Inflammatory bowel disorder| respiratory distress/failure MSC: Integrated Process: Nursing Process: Analysis NOT: Client Needs Category: Physiological Integrity: Reduction of Risk Potential

After teaching a client who is prescribed adalimumab (Humira) for severe ulcerative colitis, the nurse assesses the client's understanding. Which statement made by the client indicates a need for additional teaching? a. "I will avoid large crowds and people who are sick." b. "I will take this medication with my breakfast each morning." c. "Nausea and vomiting are common side effects of this drug." d. "I must wash my hands after I play with my dog."

ANS: B Adalimumab (Humira) is an immune modulator that must be given via subcutaneous injection. It does not need to be given with food or milk. Nausea and vomiting are two common side effects. Adalimumab can cause immune suppression, so clients receiving the medication should avoid large crowds and people who are sick, and should practice good handwashing. DIF: Applying/Application REF: 1183 KEY: Ulcerative colitis| medication safety MSC: Integrated Process: Nursing Process: Evaluation NOT: Client Needs Category: Physiological Integrity: Pharmacological and Parenteral Therapies

A nurse plans care for a client with Crohn's disease who has a heavily draining fistula. Which intervention should the nurse indicate as the priority action in this client's plan of care? a. Low-fiber diet b. Skin protection c. Antibiotic administration d. Intravenous glucocorticoids

ANS: B Protecting the client's skin is the priority action for a client who has a heavily draining fistula. Intestinal fluid enzymes are caustic and can cause skin breakdown or fungal infections if the skin is not protected. The plan of care for a client who has Crohn's disease includes adequate nutrition focused on high-calorie, high-protein, high-vitamin, and low-fiber meals, antibiotic administration, and glucocorticoids. DIF: Applying/Application REF: 1181 KEY: Crohn's disease| bowel care MSC: Integrated Process: Nursing Process: Planning NOT: Client Needs Category: Physiological Integrity: Reduction of Risk Potential

A nurse cares for a client with a new ileostomy. The client states, "I don't think my friends will accept me with this ostomy." How should the nurse respond? a. "Your friends will be happy that you are alive." b. "Tell me more about your concerns." c. "A therapist can help you resolve your concerns." d. "With time you will accept your new body."

ANS: B Social anxiety and apprehension are common in clients with a new ileostomy. The nurse should encourage the client to discuss concerns. The nurse should not minimize the client's concerns or provide false reassurance. DIF: Applying/Application REF: 1180 KEY: Ostomy care| coping| support MSC: Integrated Process: Caring NOT: Client Needs Category: Psychosocial Integrity

After teaching a client who has a new colostomy, the nurse provides feedback based on the client's ability to complete self-care activities. Which statement should the nurse include in this feedback? a. "I realize that you had a tough time today, but it will get easier with practice." b. "You cleaned the stoma well. Now you need to practice putting on the appliance." c. "You seem to understand what I taught you today. What else can I help you with?" d. "You seem uncomfortable. Do you want your daughter to care for your ostomy?"

ANS: B The nurse should provide both approval and room for improvement in feedback after a teaching session. Feedback should be objective and constructive, and not evaluative. Reassuring the client that things will improve does not offer anything concrete for the client to work on, nor does it let him or her know what was done well. The nurse should not make the client convey learning needs because the client may not know what else he or she needs to understand. The client needs to become the expert in self-management of the ostomy, and the nurse should not offer to teach the daughter instead of the client. DIF: Applying/Application REF: 1179 KEY: Ostomy care| psychosocial response| coping MSC: Integrated Process: Teaching/Learning NOT: Client Needs Category: Health Promotion and Maintenance

After teaching a client with perineal excoriation caused by diarrhea from acute gastroenteritis, a nurse assesses the client's understanding. Which statement by the client indicates a need for additional teaching? a. "I'll rinse my rectal area with warm water after each stool and apply zinc oxide ointment." b. "I will clean my rectal area thoroughly with toilet paper after each stool and then apply aloe vera gel." c. "I must take a sitz bath three times a day and then pat my rectal area gently but thoroughly to make sure I am dry." d. "I shall clean my rectal area with a soft cotton washcloth and then apply vitamin A and D ointment."

ANS: B Toilet paper can irritate the sensitive perineal skin, so warm water rinses or soft cotton washcloths should be used instead. Although aloe vera may facilitate healing of superficial abrasions, it is not an effective skin barrier for diarrhea. Skin barriers such as zinc oxide and vitamin A and D ointment help protect the rectal area from the excoriating effects of liquid stools. Patting the skin is recommended instead of rubbing the skin dry. DIF: Applying/Application REF: 1179 KEY: Bowel care| inflammatory bowel disorder MSC: Integrated Process: Nursing Process: Evaluation NOT: Client Needs Category: Safe and Effective Care Environment: Management of Care

22. A nurse teaches a client with heart failure about energy conservation. Which statement should the nurse include in this client's teaching? a. "Walk until you become short of breath, and then walk back home." b. "Gather everything you need for a chore before you begin." c. "Pull rather than push or carry items heavier than 5 pounds." d. "Take a walk after dinner every day to build up your strength."

ANS: B A client who has heart failure should be taught to conserve energy. Gathering all supplies needed for a chore at one time decreases the amount of energy needed. The client should not walk until becoming short of breath because he or she may not make it back home. Pushing a cart takes less energy than pulling or lifting. Although walking after dinner may help the client, the nurse should teach the client to complete activities when he or she has the most energy. This is usually in the morning. DIF: Applying/Application REF: 696 KEY: Heart failure| functional ability| patient education MSC: IntegratedProcess:Teaching/Learning NOT: Client Needs Category: Health Promotion and Maintenance

The nurse is meeting a 5-year-old child for the first time and would like the child to cooperate during a dressing change. The nurse decides to do a simple magic trick using gauze. How should this action be interpreted? a. Inappropriate, because of child's age b. A way to establish rapport c. Too distracting, when cooperation is important d. Acceptable, if there is adequate time

ANS: B A magic trick or other simple game may help alleviate anxiety for a 5-year-old. It is an excellent method to build rapport and facilitate cooperation during a procedure. Magic tricks appeal to the natural curiosity of young children. The nurse should establish rapport with the child. Failure to do so may cause the procedure to take longer and be more traumatic. DIF: Cognitive Level: Analyze REF: p. 64 TOP: Integrated Process: Communication and Documentation MSC: Area of Client Needs: Psychosocial Integrity

An African-American client has a prostate-specific antigen (PSA) of 12 ng/mL. Which action by the nurse is best? a. Remind the client to repeat the test in 1 year. b. Prepare the client for further diagnostic testing. c. Ask if the client ejaculated within 48 hours of the test. d. Assess the client for alcohol and tobacco use.

ANS: B A normal PSA level is less than 4 ng/mL. Elevated PSA levels, particularly those over 10 ng/mL, are associated with cancer. African Americans tend to have higher PSA levels as they age, but this level is so high that the nurse must suspect cancer and prepare the client for further diagnostic testing. The client should not wait a year to repeat the test. The client should not ejaculate for 24 hours before having blood drawn. Alcohol and tobacco use does not cause an elevation in PSA.

A patient diagnosed with anorexia nervosa is resistant to weight gain. What is the rationale for establishing a contract with the patient to participate in measures designed to produce a specified weekly weight gain? a. Because severe anxiety concerning eating is expected, objective and subjective data may be unreliable. b. Patient involvement in decision making increases sense of control and promotes compliance with treatment. c. Because of increased risk of physical problems with refeeding, the patient's permission is needed. d. A team approach to planning the diet ensures that physical and emotional needs will be met.

ANS: B A sense of control for the patient is vital to the success of therapy. A diet that controls weight gain can allay patient fears of too-rapid weight gain. Data collection is not the reason for contracting. A team approach is wise but is not a guarantee that needs will be met. Permission for treatment is a separate issue. The contract for weight gain is an additional aspect of treatment.

7. After administering newly prescribed captopril (Capoten) to a client with heart failure, the nurse implements interventions to decrease complications. Which priority intervention should the nurse implement for this client? a. Provide food to decrease nausea and aid in absorption. b. Instruct the client to ask for assistance when rising from bed. c. Collaborate with unlicensed assistive personnel to bathe the client. d. Monitor potassium levels and check for symptoms of hypokalemia.

ANS: B Administration of the first dose of angiotensin-converting enzyme (ACE) inhibitors is often associated with hypotension, usually termed first-dose effect. The nurse should instruct the client to seek assistance before arising from bed to prevent injury from postural hypotension. ACE inhibitors do not need to be taken with food. Collaboration with unlicensed assistive personnel to provide hygiene is not a priority. The client should be encouraged to complete activities of daily living as independently as possible. The nurse should monitor for hyperkalemia, not hypokalemia, especially if the client has renal insufficiency secondary to heart failure. DIF: Applying/Application REF: 685 KEY: Heart failure| angiotensin-converting enzyme (ACE) inhibitor| medication| patient education MSC: Integrated Process: Nursing Process: Implementation NOT: Client Needs Category: Physiological Integrity: Pharmacological and Parenteral Therapies

When the nurse interviews an adolescent, which is especially important? a. Focus the discussion on the peer group. b. Allow an opportunity to express feelings. c. Emphasize that confidentiality will always be maintained. d. Use the same type of language as the adolescent.

ANS: B Adolescents, like all children, need an opportunity to express their feelings. Often they will interject feelings into their words. The nurse must be alert to the words and feelings expressed. Although the peer group is important to this age group, the focus of the interview should be on the adolescent. The nurse should clarify which information will be shared with other members of the health care team and any limits to confidentiality. The nurse should maintain a professional relationship with adolescents. To avoid misinterpretation of words and phrases that the adolescent may use, the nurse should clarify terms frequently. DIF: Cognitive Level: Understand REF: p. 62 TOP: Integrated Process: Communication and Documentation

18. A nurse is caring for a morbidly obese client. What comfort measure is most important for the nurse to delegate to the unlicensed assistive personnel (UAP)? a. Designating quiet time so the client can rest b. Ensuring siderails are not causing excess pressure c. Providing oral care before and after meals and snacks d. Relaying any reports of pain to the registered nurse

ANS: B All actions are good for client comfort, but when dealing with an obese client, the staff should take extra precautions, such as ensuring the siderails are not putting pressure on the clients tissues. The other options are appropriate for any client, and are not specific to obese clients.

A 14-year-old adolescent never had chickenpox as a child. What should the nurse expect the health care provider to recommend? a. One dose of the varicella vaccination b. Two doses of the varicella vaccination 4 weeks apart c. One dose of the varicella immune globulin d. No vaccinations—the child is past the age to receive it

ANS: B All adolescents should also be assessed for previous history of varicella infection or vaccination. Vaccination with the varicella vaccine is recommended for those with no previous history; for those with no previous infection or history, the varicella vaccine may be given in two doses 4 or more weeks apart to adolescents 13 years or older. The varicella immune globulin is given to immunosuppressed children exposed to chickenpox to boost immunity; it is only temporary. The varicella vaccination should be given to adolescents, no matter the age, who have not had chickenpox as a child. DIF: Cognitive Level: Apply REF: p. 462 TOP: Integrated Process: Nursing Process: Implementation MSC: Area of Client Needs: Health Promotion and Maintenance

12. A nurse provides health screening for a community health center with a large population of African-American clients. Which priority assessment should the nurse include when working with this population? a. Measure height and weight. b. Assess blood pressure. c. Observe for any signs of abuse. d. Ask about medications.

ANS: B All interventions are important for the visiting nurse to accomplish. However, African Americans have a high rate of hypertension leading to end-stage renal disease. Each encounter that the nurse has with an African-American client provides a chance to detect hypertension and treat it. If the client is already on antihypertensive medication, assessing blood pressure monitors therapy.

A nurse works in the rheumatology clinic and sees clients with rheumatoid arthritis (RA). Which client should the nurse see first? a. Client who reports jaw pain when eating b. Client with a red, hot, swollen right wrist c. Client who has a puffy-looking area behind the knee d. Client with a worse joint deformity since the last visit

ANS: B All of the options are possible manifestations of RA. However, the presence of one joint that is much redder, hotter, or more swollen that the other joints may indicate infection. The nurse needs to see this client first.

The clinic nurse assesses a client with diabetes during a checkup. The client also has osteoarthritis (OA). The nurse notes the client's blood glucose readings have been elevated. What question by the nurse is most appropriate? a. "Are you compliant with following the diabetic diet?" b. "Have you been taking glucosamine supplements?" c. "How much exercise do you really get each week?" d. "You're still taking your diabetic medication, right?"

ANS: B All of the topics are appropriate for a client whose blood glucose readings have been higher than usual. However, since this client also has OA, and glucosamine can increase blood glucose levels, the nurse should ask about its use. The other questions all have an element of nontherapeutic communication in them. "Compliant" is a word associated with negative images, and the client may deny being "noncompliant." Asking how much exercise the client "really" gets is accusatory. Asking if the client takes his or her medications "right?" is patronizing.

The nurse is taking a sexual history on an adolescent girl. Which is the best way to determine whether she is sexually active? a. Ask her, "Are you sexually active?" b. Ask her, "Are you having sex with anyone?" c. Ask her, "Are you having sex with a boyfriend?" d. Ask both the girl and her parent whether she is sexually active.

ANS: B Asking the adolescent girl whether she is having sex with anyone is a direct question that is well understood. The phrase sexually active is broadly defined and may not provide specific information to the nurse to provide necessary care. The word anyone is preferred to using gender-specific terms such as boyfriend or girlfriend. Because homosexual experimentation may occur, it is preferable to use gender-neutral terms. Questioning about sexual activity should occur when the adolescent is alone. DIF: Cognitive Level: Apply REF: p. 65 TOP: Integrated Process: Nursing Process: Assessment MSC: Area of Client Needs: Health Promotion and Maintenance

2. A nurse cares for a client with arthritis who reports frequent asthma attacks. Which action should the nurse take first? a. Review the client's pulmonary function test results. b. Ask about medications the client is currently taking. c. Assess how frequently the client uses a bronchodilator. d. Consult the provider and request arterial blood gases.

ANS: B Aspirin and other nonsteroidal anti-inflammatory drugs (NSAIDs) can trigger asthma in some people. This results from increased production of leukotriene when aspirin or NSAIDs suppress other inflammatory pathways and is a high priority given the client's history. Reviewing pulmonary function test results will not address the immediate problem of frequent asthma attacks. This is a good intervention for reviewing response to bronchodilators. Questioning the client about the use of bronchodilators will address interventions for the attacks but not their cause. Reviewing arterial blood gas results would not be of use in a client between attacks because many clients are asymptomatic when not having attacks. DIF: Applying/Application REF: 553 KEY: Respiratory distress/failure| medication MSC: IntegratedProcess:NursingProcess:Assessment NOT: Client Needs Category: Physiological Integrity: Pharmacological and Parenteral Therapies

24. A nurse assesses a client who has mitral valve regurgitation. For which cardiac dysrhythmia should the nurse assess? a. Preventricular contractions b. Atrial fibrillation c. Symptomatic bradycardia d. Sinus tachycardia

ANS: B Atrial fibrillation is a clinical manifestation of mitral valve regurgitation and stenosis. Preventricular contractions and bradycardia are not associated with valvular problems. These are usually identified in clients with electrolyte imbalances, myocardial infarction, and sinus node problems. Sinus tachycardia is a manifestation of aortic regurgitation due to a decrease in cardiac output. DIF: Understanding/Comprehension REF: 692 KEY: Valve disorder| cardiac dysrhythmia MSC: IntegratedProcess:NursingProcess:Assessment NOT: Client Needs Category: Physiological Integrity: Reduction of Risk Potential

Parents of a newborn are concerned because the infant's eyes often "look crossed" when the infant is looking at an object. The nurse's response is that this is normal based on the knowledge that binocularity is normally present by what age? a. 1 month b. 3 to 4 months c. 6 to 8 months d. 12 months

ANS: B Binocularity is usually achieved by ages 3 to 4 months. 1 month is too young. If binocularity is not achieved by ages 6 to 12 months, the child must be observed for strabismus. DIF: Cognitive Level: Understand REF: p. 91 TOP: Integrated Process: Nursing Process: Assessment MSC: Area of Client Needs: Health Promotion and Maintenance

The nurse is teaching a postmenopausal woman about nutrition. Which statement by the nurse is most appropriate? a. "Be sure to eat cereal fortified with folic acid and B vitamins." b. "Make sure you take a calcium supplement every day." c. "Vitamin C is important for the postmenopausal woman." d. "You can get all the iron you need in two daily meat servings."

ANS: B Calcium is important throughout life, but for the postmenopausal woman, it is vital to help prevent osteoporosis. Folic acid and B and C vitamins are very important for the woman taking oral contraceptives. Iron might be important for this client for other reasons but is especially important for women with heavy menstrual bleeding.

23. A nurse administers medications to a client who has asthma. Which medication classification is paired correctly with its physiologic response to the medication? a. Bronchodilator - Stabilizes the membranes of mast cells and prevents the release of inflammatory mediators b. Cholinergic antagonist - Causes bronchodilation by inhibiting the parasympathetic nervous system c. Corticosteroid - Relaxes bronchiolar smooth muscles by binding to and activating pulmonary beta2 receptors d. Cromone - Disrupts the production of pathways of inflammatory mediators

ANS: B Cholinergic antagonist drugs cause bronchodilation by inhibiting the parasympathetic nervous system. This allows the sympathetic nervous system to dominate and release norepinephrine that actives beta2 receptors. Bronchodilators relax bronchiolar smooth muscles by binding to and activating pulmonary beta2 receptors. Corticosteroids disrupt the production of pathways of inflammatory mediators. Cromones stabilize the membranes of mast cells and prevent the release of inflammatory mediators. DIF: Remembering/Knowledge REF: 554 KEY: Medications MSC: Integrated Process: Nursing Process: Analysis NOT: Client Needs Category: Physiological Integrity: Pharmacological and Parenteral Therapies

14. A nurse teaches a client with diabetes mellitus and a body mass index of 42 who is at high risk for coronary artery disease. Which statement related to nutrition should the nurse include in this client's teaching? a. "The best way to lose weight is a high-protein, low-carbohydrate diet." b. "You should balance weight loss with consuming necessary nutrients." c. "A nutritionist will provide you with information about your new diet." d. "If you exercise more frequently, you won't need to change your diet."

ANS: B Clients at risk for cardiovascular diseases should follow the American Heart Association guidelines to combat obesity and improve cardiac health. The nurse should encourage the client to eat vegetables, fruits, unrefined whole-grain products, and fat-free dairy products while losing weight. High-protein food items are often high in fat and calories. Although the nutritionist can assist with client education, the nurse should include nutrition education and assist the client to make healthy decisions. Exercising and eating nutrient-rich foods are both important components in reducing cardiovascular risk. DIF: Applying/Application REF: 633 KEY: Nutrition| patient education MSC: Integrated Process: Teaching/Learning NOT: Client Needs Category: Physiological Integrity: Basic Care and Comfort

A client who has been taking antibiotics reports severe, watery diarrhea. About which test does the nurse teach the client? a. Colonoscopy b. Enzyme-linked immunosorbent assay (ELISA) toxin A+B c. Ova and parasites d. Stool culture

ANS: B Clients taking antibiotics are at risk for Clostridium difficile infection. The most common test for this disorder is a stool sample for ELISA toxin A+B. Colonoscopy, ova and parasites, and stool culture are not warranted at this time.

14. After teaching a client who is being discharged home after mitral valve replacement surgery, the nurse assesses the client's understanding. Which client statement indicates a need for additional teaching? a. "I'll be able to carry heavy loads after 6 months of rest." b. "I will have my teeth cleaned by my dentist in 2 weeks." c. "I must avoid eating foods high in vitamin K, like spinach." d. "I must use an electric razor instead of a straight razor to shave."

ANS: B Clients who have defective or repaired valves are at high risk for endocarditis. The client who has had valve surgery should avoid dental procedures for 6 months because of the risk for endocarditis. When undergoing a mitral valve replacement surgery, the client needs to be placed on anticoagulant therapy to prevent vegetation forming on the new valve. Clients on anticoagulant therapy should be instructed on bleeding precautions, including using an electric razor. If the client is prescribed warfarin, the client should avoid foods high in vitamin K. Clients recovering from open heart valve replacements should not carry anything heavy for 6 months while the chest incision and muscle heal. DIF: Applying/Application REF: 694 KEY: Valve disorder| patient education| hygiene MSC: IntegratedProcess:Teaching/Learning NOT: Client Needs Category: Physiological Integrity: Reduction of Risk Potential

A client had an upper gastrointestinal hemorrhage and now has a nasogastric (NG) tube. What comfort measure may the nurse delegate to the unlicensed assistive personnel (UAP)? a. Lavaging the tube with ice water b. Performing frequent oral care c. Re-positioning the tube every 4 hours d. Taking and recording vital signs

ANS: B Clients with NG tubes need frequent oral care both for comfort and to prevent infection. Lavaging the tube is done by the nurse. Re-positioning the tube, if needed, is also done by the nurse. The UAP can take vital signs, but this is not a comfort measure.

3. After teaching a client with early polycystic kidney disease (PKD) about nutritional therapy, the nurse assesses the client's understanding. Which statement made by the client indicates a correct understanding of the teaching? a. "I will take a laxative every night before going to bed." b. "I must increase my intake of dietary fiber and fluids." c. "I shall only use salt when I am cooking my own food." d. "I'll eat white bread to minimize gastrointestinal gas."

ANS: B Clients with PKD often have constipation, which can be managed with increased fiber, exercise, and drinking plenty of water. Laxatives should be used cautiously. Clients with PKD should be on a restricted salt diet, which includes not cooking with salt. White bread has a low fiber count and would not be included in a high-fiber diet.

20. A nurse assesses a client who has a history of heart failure. Which question should the nurse ask to assess the extent of the client's heart failure? a. "Do you have trouble breathing or chest pain?" b. "Are you able to walk upstairs without fatigue?" c. "Do you awake with breathlessness during the night?" d. "Do you have new-onset heaviness in your legs?"

ANS: B Clients with a history of heart failure generally have negative findings, such as shortness of breath. The nurse needs to determine whether the client's activity is the same or worse, or whether the client identifies a decrease in activity level. Trouble breathing, chest pain, breathlessness at night, and peripheral edema are symptoms of heart failure, but do not provide data that can determine the extent of the client's heart failure. DIF: Applying/Application REF: 682 KEY: Heart failure| functional ability| respiratory distress/failure MSC: IntegratedProcess:NursingProcess:Assessment NOT: Client Needs Category: Physiological Integrity: Physiological Adaptation

8. An emergency department nurse assesses a client with kidney trauma and notes that the client's abdomen is tender and distended and blood is visible at the urinary meatus. Which prescription should the nurse consult the provider about before implementation? a. Assessing vital signs every 15 minutes b. Inserting an indwelling urinary catheter c. Administering intravenous fluids at 125 mL/hr d. Typing and crossmatching for blood products

ANS: B Clients with blood at the urinary meatus should not have a urinary catheter inserted via the urethra before additional diagnostic studies are done. The urethra could be torn. The nurse should question the provider about the need for a catheter; if one is needed, the provider can insert a suprapubic catheter. The nurse should monitor the client's vital signs closely, send blood for type and crossmatch in case the client needs blood products, and administer intravenous fluids.

16. A client with a traumatic brain injury is agitated and fighting the ventilator. What drug should the nurse prepare to administer? a. Carbamazepine (Tegretol) b. Dexmedetomidine (Precedex) c. Diazepam (Valium) d. Mannitol (Osmitrol)

ANS: B Dexmedetomidine is often used to manage agitation in the client with traumatic brain injury. Carbamazepine is an antiseizure drug. Diazepam is a benzodiazepine. Mannitol is an osmotic diuretic. DIF: Remembering/Knowledge REF: 955 KEY: Neurologic disorders| sedatives| mechanical ventilation MSC: Integrated Process: Nursing Process: Implementation NOT: Client Needs Category: Physiological Integrity: Pharmacological and Parenteral Therapies

12. A nurse assesses a client with mitral valve stenosis. What clinical manifestation should alert the nurse to the possibility that the client's stenosis has progressed? a. Oxygen saturation of 92% b. Dyspnea on exertion c. Muted systolic murmur d. Upper extremity weakness

ANS: B Dyspnea on exertion develops as the mitral valvular orifice narrows and pressure in the lungs increases. The other manifestations do not relate to the progression of mitral valve stenosis. DIF: Applying/Application REF: 688 KEY: Valve disorder| respiratory distress/failure MSC: IntegratedProcess:NursingProcess:Assessment NOT: Client Needs Category: Physiological Integrity: Reduction of Risk Potential

18. A student nurse is preparing to administer enoxaparin (Lovenox) to a client. What action by the student requires immediate intervention by the supervising nurse? a. Assessing the client's platelet count b. Choosing an 18-gauge, 2-inch needle c. Not aspirating prior to injection d. Swabbing the injection site with alcohol

ANS: B Enoxaparin is given subcutaneously, so the 18-gauge, 2-inch needle is too big. The other actions are appropriate.

A postmenopausal client says that she is experiencing difficulty with vaginal dryness during intercourse and wonders what might be causing this. Which is the nurse's best response? a. "The less frequently you have intercourse, the drier the vaginal tissues become." b. "Estrogen deficiency causes the vaginal tissues to become drier and thinner." c. "Drinking at least 3 liters of water each day will make all your tissues less dry." d. "Try using a water-soluble lubricant during intercourse."

ANS: B Estrogen deprivation, which occurs as a result of menopause, decreases the moisture-secreting capacity of vaginal cells, thereby making the area drier. The vaginal tissues also become thinner and the rugae become smoother. Reduced frequency of intercourse will not dry out the vaginal tissues. Drinking excess water will not make the tissues less dry. A water-soluble lubricant may make intercourse less difficult. However, the client is asking what causes the problem.

4. A client is on intravenous heparin to treat a pulmonary embolism. The client's most recent partial thromboplastin time (PTT) was 25 seconds. What order should the nurse anticipate? a. Decrease the heparin rate. b. Increase the heparin rate. c. No change to the heparin rate. d. Stop heparin; start warfarin (Coumadin).

ANS: B For clients on heparin, a PTT of 1.5 to 2.5 times the normal value is needed to demonstrate the heparin is working. A normal PTT is 25 to 35 seconds, so this client's PTT value is too low. The heparin rate needs to be increased. Warfarin is not indicated in this situation.

13. A nurse is preparing to admit a client on mechanical ventilation from the emergency department. What action by the nurse takes priority? a. Assessing that the ventilator settings are correct b. Ensuring there is a bag-valve-mask in the room c. Obtaining personal protective equipment d. Planning to suction the client upon arrival to the room

ANS: B Having a bag-valve-mask device is critical in case the client needs manual breathing. The respiratory therapist is usually primarily responsible for setting up the ventilator, although the nurse should know and check the settings. Personal protective equipment is important, but ensuring client safety takes priority. The client may or may not need suctioning on arrival.

13. A client has been taking isoniazid (INH) for tuberculosis for 3 weeks. What laboratory results need to be reported to the health care provider immediately? a. Albumin: 5.1 g/dL b. Alanine aminotransferase (ALT): 180 U/L c. Red blood cell (RBC) count: 5.2/mm3 d. White blood cell (WBC) count: 12,500/mm3

ANS: B INH can cause liver damage, especially if the client drinks alcohol. The ALT (one of the liver enzymes) is extremely high and needs to be reported immediately. The albumin and RBCs are normal. The WBCs are slightly high, but that would be an expected finding in a client with an infection.

A client with rheumatoid arthritis (RA) has an acutely swollen, red, and painful joint. What nonpharmacologic treatment does the nurse apply? a. Heating pad b. Ice packs c. Splints d. Wax dip

ANS: B Ice is best for acute inflammation. Heat often helps with joint stiffness. Splinting helps preserve joint function. A wax dip is used to provide warmth to the joint which is more appropriate for chronic pain and stiffness.

10. While assessing a client who is 12 hours postoperative after a thoracotomy for lung cancer, a nurse notices that the lower chest tube is dislodged. Which action should the nurse take first? a. Assess for drainage from the site. b. Cover the insertion site with sterile gauze. \ c. Contact the provider and obtain a suture kit. d. Reinsert the tube using sterile technique.

ANS: B Immediately covering the insertion site helps prevent air from entering the pleural space and causing a pneumothorax. The area will not reseal quickly enough to prevent air from entering the chest. The nurse should not leave the client to obtain a suture kit. An occlusive dressing may cause a tension pneumothorax. The site should only be assessed after the insertion site is covered. The provider should be called to reinsert the chest tube or prescribe other treatment options. DIF: Applying/Application REF: 578 KEY: Drains| surgical care MSC: Integrated Process: Nursing Process: Implementation NOT: Client Needs Category: Physiological Integrity: Physiological Adaptation

6. After teaching a client with nephrotic syndrome and a normal glomerular filtration, the nurse assesses the client's understanding. Which statement made by the client indicates a correct understanding of the nutritional therapy for this condition? a. "I must decrease my intake of fat." b. "I will increase my intake of protein." c. "A decreased intake of carbohydrates will be required." d. "An increased intake of vitamin C is necessary."

ANS: B In nephrotic syndrome, the renal loss of protein is significant, leading to hypoalbuminemia and edema formation. If glomerular filtration is normal or near normal, increased protein loss should be matched by increased intake of protein. The client would not need to adjust fat, carbohydrates, or vitamins based on this disorder.

15. A nurse is caring for four clients who might be brain dead. Which client would best meet the criteria to allow assessment of brain death? a. Client with a core temperature of 95° F (35° C) for 2 days b. Client in a coma for 2 weeks from a motor vehicle crash c. Client who is found unresponsive in a remote area of a field by a hunter d. Client with a systolic blood pressure of 92 mm Hg since admission

ANS: B In order to determine brain death, clients must meet four criteria: 1) coma from a known cause, 2) normal or near-normal core temperature, 3) normal systolic blood pressure, and 4) at least one neurologic examination. The client who was in the car crash meets two of these criteria. The clients with the lower temperature and lower blood pressure have only one of these criteria. There is no data to support assessment of brain death in the client found by the hunter. DIF: Remembering/Knowledge REF: 954 KEY: Neurologic disorders| brain death| neurologic assessment MSC: IntegratedProcess:NursingProcess:Assessment NOT: Client Needs Category: Physiological Integrity: Physiological Adaptation

7. An older adult is brought to the emergency department by a family member, who reports a moderate change in mental status and mild cough. The client is afebrile. The health care provider orders a chest x-ray. The family member questions why this is needed since the manifestations seem so vague. What response by the nurse is best? a. "Chest x-rays are always ordered when we suspect pneumonia." b. "Older people often have vague symptoms, so an x-ray is essential." c. "The x-ray can be done and read before laboratory work is reported." d. "We are testing for any possible source of infection in the client."

ANS: B It is essential to obtain an early chest x-ray in older adults suspected of having pneumonia because symptoms are often vague. Waiting until definitive manifestations are present to obtain the x-ray leads to a costly delay in treatment. Stating that chest x-rays are always ordered does not give the family definitive information. The x-ray can be done while laboratory values are still pending, but this also does not provide specific information about the importance of a chest x-ray in this client. The client has manifestations of pneumonia, so the staff is not testing for any possible source of infection but rather is testing for a suspected disorder.

While assessing a client's lower extremities, a nurse notices that one leg is pale and cooler to the touch. Which assessment should the nurse perform next? a. Ask about a family history of skin disorders. b. Palpate the client's pedal pulses bilaterally. c. Check for the presence of Homans' sign. d. Assess the client's skin for adequate skin turgor

ANS: B Localized, decreased skin temperature and pallor indicate interference with vascular flow to the region. The nurse should assess bilateral pedal pulses to screen for vascular sufficiency. Without adequate blood flow, the client's limb could be threatened. Asking about a family history of skin problems would not take priority over assessing blood flow. Homans' sign is a screening tool for deep vein thrombosis and is often inaccurate. Skin turgor gives information about hydration status. This assessment may be needed but certainly does not take priority over assessing for blood flow.

17. A client is in the family practice clinic reporting a severe cough that has lasted for 5 weeks. The client is so exhausted after coughing that work has become impossible. What action by the nurse is most appropriate? a. Arrange for immediate hospitalization. b. Facilitate polymerase chain reaction testing. c. Have the client produce a sputum sample. d. Obtain two sets of blood cultures.

ANS: B Polymerase chain reaction testing is used to diagnose pertussis, which this client is showing manifestations of. Hospitalization may or may not be needed but is not the most important action. The client may or may not be able to produce sputum, but sputum cultures for this disease must be obtained via deep suctioning. Blood cultures will be negative.

13. A morbidly obese client is admitted to a community hospital that does not typically care for bariatric-sized clients. What action by the nurse is most appropriate? a. Assess the clients readiness to make lifestyle changes. b. Ensure adequate staff when moving the client. c. Leave siderails down to prevent pressure ulcers. d. Reinforce the need to be sensitive to the client.

ANS: B Many hospitals that see bariatric-sized clients have appropriate equipment for this population. A hospital that does not typically see these clients is less likely to have appropriate equipment, putting staff and client safety at risk. The nurse ensures enough staffing is available to help with all aspects of mobility. It may or may not be appropriate to assess the clients willingness to make lifestyle changes. Leaving the siderails down may present a safety hazard. The staff should be sensitive to this clients situation, but safety takes priority.

Which describes marasmus? a. Deficiency of protein with an adequate supply of calories b. Not confined to geographic areas where food supplies are inadequate c. Syndrome that results solely from vitamin deficiencies d. Characterized by thin, wasted extremities and a prominent abdomen resulting from edema (ascites)

ANS: B Marasmus is a syndrome of emotional and physical deprivation. It is not confined to geographic areas were food supplies are inadequate. Marasmus is a deficiency of both protein and calories. It is characterized by gradual wasting and atrophy of body tissues, especially of subcutaneous fat. The child appears very old, with flabby and wrinkled skin.

19. A client is awaiting bariatric surgery in the morning. What action by the nurse is most important? a. Answering questions the client has about surgery b. Beginning venous thromboembolism prophylaxis c. Informing the client that he or she will be out of bed tomorrow d. Teaching the client about needed dietary changes

ANS: B Morbidly obese clients are at high risk of venous thromboembolism and should be started on a regimen to prevent this from occurring as a priority. Answering questions about the surgery is done by the surgeon. Teaching is important, but safety comes first.

19. A client is in the family medicine clinic reporting a dry, sore throat. The provider asks the nurse to assess for odynophagia. What assessment technique is most appropriate? a. Ask the client what foods cause trouble swallowing. b. Assess the client for pain when swallowing. c. Determine if the client can swallow saliva. d. Palpate the client's jaw while swallowing.

ANS: B Odynophagia is painful swallowing. The nurse should assess the client for this either by asking or by having the client attempt to drink water. It is not related to specific foods and is not assessed by palpating the jaw. Being unable to swallow saliva is not odynophagia, but it would be a serious situation.

5. The charge nurse on a medical unit is preparing to admit several "clients" who have possible pandemic flu during a preparedness drill. What action by the nurse is best? a. Admit the "clients" on Contact Precautions. b. Cohort the "clients" in the same area of the unit. c. Do not allow pregnant caregivers to care for these "clients." d. Place the "clients" on enhanced Droplet Precautions.

ANS: B Preventing the spread of pandemic flu is equally important as caring for the clients who have it. Clients can be cohorted together in the same set of rooms on one part of the unit to use distancing to help prevent the spread of the disease. The other actions are not appropriate.

5. A client is hospitalized with a second episode of pulmonary embolism (PE). Recent genetic testing reveals the client has an alteration in the gene CYP2C19. What action by the nurse is best? a. Instruct the client to eliminate all vitamin K from the diet. b. Prepare preoperative teaching for an inferior vena cava (IVC) filter. c. Refer the client to a chronic illness support group. d. Teach the client to use a soft-bristled toothbrush.

ANS: B Often clients are discharged from the hospital on warfarin (Coumadin) after a PE. However, clients with a variation in the CYP2C19 gene do not metabolize warfarin well and have higher blood levels and more side effects. This client is a poor candidate for warfarin therapy, and the prescriber will most likely order an IVC filter device to be implanted. The nurse should prepare to do preoperative teaching on this procedure. It would be impossible to eliminate all vitamin K from the diet. A chronic illness support group may be needed, but this is not the best intervention as it is not as specific to the client as the IVC filter. A soft-bristled toothbrush is a safety measure for clients on anticoagulation therapy.

An emergency room nurse assesses a client after a motor vehicle crash and notes ecchymotic areas across the client's lower abdomen. Which action should the nurse take first? a. Measure the client's abdominal girth. b. Assess for abdominal guarding or rigidity. c. Check the client's hemoglobin and hematocrit. d. Obtain the client's complete health history.

ANS: B On noticing the ecchymotic areas, the nurse should check to see if abdominal guarding or rigidity is present, because this could indicate major organ injury. The nurse should then notify the provider. Measuring abdominal girth or obtaining a complete health history is not appropriate at this time. Laboratory test results can be checked after assessment for abdominal guarding or rigidity.

An appropriate intervention for a patient diagnosed with bulimia nervosa who binges and purges is to teach the patient: a. to eat a small meal after purging. b. not to skip meals or restrict food. c. to increase oral intake after 4 PM daily. d. the value of reading journal entries aloud to others.

ANS: B One goal of health teaching is normalization of eating habits. Food restriction and skipping meals lead to rebound bingeing. Teaching the patient to eat a small meal after purging will probably perpetuate the need to induce vomiting. Teaching the patient to eat a large breakfast but no lunch and increase intake after 4 PM will lead to late-day bingeing. Journal entries are private.

12. A client is wearing a Venturi mask to deliver oxygen and the dinner tray has arrived. What action by the nurse is best? a. Assess the client's oxygen saturation and, if normal, turn off the oxygen. b. Determine if the client can switch to a nasal cannula during the meal. c. Have the client lift the mask off the face when taking bites of food. d. Turn the oxygen off while the client eats the meal and then restart it.

ANS: B Oxygen is a drug that needs to be delivered constantly. The nurse should determine if the provider has approved switching to a nasal cannula during meals. If not, the nurse should consult with the provider about this issue. The oxygen should not be turned off. Lifting the mask to eat will alter the FiO2 delivered. DIF: Applying/Application REF: 517 KEY: Oxygen therapy| oxygen MSC: Integrated Process: Nursing Process: Implementation NOT: Client Needs Category: Physiological Integrity: Pharmacological and Parenteral Therapies

8. A nurse is caring for a client using oxygen while in the hospital. What assessment finding indicates that goals for a priority diagnosis are being met? a. 100% of meals being eaten by the client b. Intact skin behind the ears c. The client understanding the need for oxygen d. Unchanged weight for the past 3 days

ANS: B Oxygen tubing can cause pressure ulcers, so clients using oxygen have the nursing diagnosis of Risk for Impaired Skin Integrity. Intact skin behind the ears indicates that goals for this diagnosis are being met. Nutrition and weight are not related to using oxygen. Understanding the need for oxygen is important but would not take priority over a physical problem. DIF: Evaluating/Synthesis REF: 515 KEY: Oxygen| skin integrity| nursing diagnosis| oxygen therapy MSC: Integrated Process: Nursing Process: Evaluation NOT: Client Needs Category: Physiological Integrity: Reduction of Risk Potential

A nurse is conducting parenting classes for parents of adolescents. Which parenting style should the nurse recommend? a. Laissez-faire b. Authoritative c. Disciplinarian d. Confrontational

ANS: B Parents should be guided toward an authoritative style of parenting in which authority is used to guide the adolescent while allowing developmentally appropriate levels of freedom and providing clear, consistent messages regarding expectations. The authoritative style of parenting has been shown to have both immediate and long-term protective effects toward adolescent risk reduction. The laissez-faire method would not give adolescents enough structure. The disciplinarian and confrontational styles would not allow any autonomy or independence. DIF: Cognitive Level: Apply REF: p. 456 TOP: Integrated Process: Teaching/Learning MSC: Area of Client Needs: Health Promotion and Maintenance

1. A nurse assesses a client with polycystic kidney disease (PKD). Which assessment finding should alert the nurse to immediately contact the health care provider? a. Flank pain b. Periorbital edema c. Bloody and cloudy urine d. Enlarged abdomen

ANS: B Periorbital edema would not be a finding related to PKD and should be investigated further. Flank pain and a distended or enlarged abdomen occur in PKD because the kidneys enlarge and displace other organs. Urine can be bloody or cloudy as a result of cyst rupture or infection.

A client with systemic lupus erythematosus (SLE) was recently discharged from the hospital after an acute exacerbation. The client is in the clinic for a follow-up visit and is distraught about the possibility of another hospitalization disrupting the family. What action by the nurse is best? a. Explain to the client that SLE is an unpredictable disease. b. Help the client create backup plans to minimize disruption. c. Offer to talk to the family and educate them about SLE. d. Tell the client to remain compliant with treatment plans.

ANS: B SLE is an unpredictable disease and acute exacerbations can occur without warning, creating chaos in the family. Helping the client make backup plans for this event not only will decrease the disruption but will give the client a sense of having more control. Explaining facts about the disease is helpful as well but does not engage the client in problem solving. The family may need education, but again this does not help the client to problem-solve. Remaining compliant may help decrease exacerbations, but is not as powerful an intervention as helping the client plan for such events.

What is the earliest age at which a satisfactory radial pulse can be taken in children? a. 1 year b. 2 years c. 3 years d. 6 years

ANS: B Satisfactory radial pulses can be used in children older than 2 years. In infants and young children, the apical pulse is more reliable. The apical pulse can be used for assessment at these ages. DIF: Cognitive Level: Remember REF: p. 103 TOP: Integrated Process: Nursing Process: Assessment MSC: Area of Client Needs: Health Promotion and Maintenance

3. A nurse assesses a client admitted to the cardiac unit. Which statement by the client alerts the nurse to the possibility of right-sided heart failure? a. "I sleep with four pillows at night." b. "My shoes fit really tight lately." c. "I wake up coughing every night." d. "I have trouble catching my breath."

ANS: B Signs of systemic congestion occur with right-sided heart failure. Fluid is retained, pressure builds in the venous system, and peripheral edema develops. Left-sided heart failure symptoms include respiratory symptoms. Orthopnea, coughing, and difficulty breathing all could be results of left-sided heart failure. DIF: Understanding/Comprehension REF: 683 KEY: Heart failure| assessment/diagnostic examination MSC: IntegratedProcess:NursingProcess:Assessment NOT: Client Needs Category: Health Promotion and Maintenance

2. A client is scheduled to have a tracheostomy placed in an hour. What action by the nurse is the priority? a. Administer prescribed anxiolytic medication. b. Ensure informed consent is on the chart. c. Reinforce any teaching done previously. d. Start the preoperative antibiotic infusion.

ANS: B Since this is an operative procedure, the client must sign an informed consent, which must be on the chart. Giving anxiolytics and antibiotics and reinforcing teaching may also be required but do not take priority. DIF: Applying/Application REF: 522 KEY: Informed consent| autonomy MSC: Integrated Process: Communication and Documentation NOT: Client Needs Category: Safe and Effective Care Environment: Management of Care

7. A student nurse is preparing morning medications for a client who had a stroke. The student plans to hold the docusate sodium (Colace) because the client had a large stool earlier. What action by the supervising nurse is best? a. Have the student ask the client if it is desired or not. b. Inform the student that the docusate should be given. c. Tell the student to document the rationale. d. Tell the student to give it unless the client refuses.

ANS: B Stool softeners should be given to clients with neurologic disorders in order to prevent an elevation in intracranial pressure that accompanies the Valsalva maneuver when constipated. The supervising nurse should instruct the student to administer the docusate. The other options are not appropriate. The medication could be held for diarrhea. DIF: Applying/Application REF: 942 KEY: Neurologic disorders| stool softeners| constipation| intracranial pressure MSC: IntegratedProcess:Teaching/Learning NOT: Client Needs Category: Physiological Integrity: Reduction of Risk Potential

13. A nurse cares for a client recovering from prosthetic valve replacement surgery. The client asks, "Why will I need to take anticoagulants for the rest of my life?" How should the nurse respond? a. "The prosthetic valve places you at greater risk for a heart attack." b. "Blood clots form more easily in artificial replacement valves." c. "The vein taken from your leg reduces circulation in the leg." d. "The surgery left a lot of small clots in your heart and lungs."

ANS: B Synthetic valve prostheses and scar tissue provide surfaces on which platelets can aggregate easily and initiate the formation of blood clots. The other responses are inaccurate. DIF: Applying/Application REF: 696 KEY: Valve disorder| patient education| anticoagulants MSC: IntegratedProcess:Teaching/Learning NOT: Client Needs Category: Physiological Integrity: Reduction of Risk Potential

13. After teaching a client with renal cancer who is prescribed temsirolimus (Torisel), the nurse assesses the client's understanding. Which statement made by the client indicates a correct understanding of the teaching? a. "I will take this medication with food and plenty of water." b. "I shall keep my appointment at the infusion center each week." c. "I'll limit my intake of green leafy vegetables while on this medication." d. "I must not take this medication if I have an infection or am feeling ill."

ANS: B Temsirolimus is administered as a weekly intravenous infusion. This medication blocks protein that is needed for cell division and therefore inhibits cell cycle progression. This medication is not taken orally, and clients do not need to follow a specific diet.

A client has a recurrence of gastric cancer and is in the gastrointestinal clinic crying. What response by the nurse is most appropriate? a. "Do you have family or friends for support?" b. "I'd like to know what you are feeling now." c. "Well, we knew this would probably happen." d. "Would you like me to refer you to hospice?"

ANS: B The nurse assesses the client's emotional state with open-ended questions and statements and shows a willingness to listen to the client's concerns. Asking about support people is very limited in nature, and "yes-or-no" questions are not therapeutic. Stating that this was expected dismisses the client's concerns. The client may or may not be ready to hear about hospice, and this is another limited, yes-or-no question.

13. A nurse cares for a client who has an 80% blockage of the right coronary artery (RCA) and is scheduled for bypass surgery. Which intervention should the nurse be prepared to implement while this client waits for surgery? a. Administration of IV furosemide (Lasix) b. Initiation of an external pacemaker c. Assistance with endotracheal intubation d. Placement of central venous access

ANS: B The RCA supplies the right atrium, the right ventricle, the inferior portion of the left ventricle, and the atrioventricular (AV) node. It also supplies the sinoatrial node in 50% of people. If the client totally occludes the RCA, the AV node would not function and the client would go into heart block, so emergency pacing should be available for the client. Furosemide, intubation, and central venous access will not address the primary complication of RCA occlusion, which is AV node malfunction. DIF: Applying/Application REF: 628 KEY: Coronary perfusion| assessment/diagnostic examination| cardiac electrical conduction MSC: Integrated Process: Nursing Process: Planning NOT: Client Needs Category: Physiological Integrity: Reduction of Risk Potential

A school nurse is teaching a group of preadolescent girls about puberty. Which is the mean age of menarche for girls in the United States? a. 13 years b. 12 years c. 11 years d. 14 years

ANS: B The average age of menarche is 12 years 9.5 months in North American girls, with a normal range of to 15 years. Ages , , and 14 are within the normal range for menarche, but these are not the average ages. DIF: Cognitive Level: Remember REF: p. 448 TOP: Integrated Process: Teaching/Learning MSC: Area of Client Needs: Health Promotion and Maintenance

10. A client's mean arterial pressure is 60 mm Hg and intracranial pressure is 20 mm Hg. Based on the client's cerebral perfusion pressure, what should the nurse anticipate for this client? a. Impending brain herniation b. Poor prognosis and cognitive function c. Probable complete recovery d. Unable to tell from this information

ANS: B The cerebral perfusion pressure (CPP) is the intracranial pressure subtracted from the mean arterial pressure: in this case, 60 - 20 = 40. For optimal outcomes, CPP should be at least 70 mm Hg. This client has very low CPP, which will probably lead to a poorer prognosis with significant cognitive dysfunction should the client survive. This data does not indicate impending brain herniation or complete recovery. DIF: Analyzing/Analysis REF: 949 KEY: Neurologic disorders| nursing assessment| neurologic assessment MSC: Integrated Process: Nursing Process: Analysis NOT: Client Needs Category: Physiological Integrity: Physiological Adaptation

The nurse is taking a health history on an adolescent. Which best describes how the chief complaint should be determined? a. Ask for detailed listing of symptoms. b. Ask adolescent, "Why did you come here today?" c. Use what adolescent says to determine, in correct medical terminology, what the problem is. d. Interview parent away from adolescent to determine chief complaint.

ANS: B The chief complaint is the specific reason for the child's visit to the clinic, office, or hospital. Because the adolescent is the focus of the history, this is an appropriate way to determine the chief complaint. A detailed listing of symptoms will make it difficult to determine the chief complaint. The adolescent should be prompted to tell which symptom caused him to seek help at this time. The chief complaint is usually written in the words that the parent or adolescent uses to describe the reason for seeking help. The parent and adolescent may be interviewed separately, but the nurse should determine the reason the adolescent is seeking attention at this time. DIF: Cognitive Level: Apply REF: p. 62 TOP: Integrated Process: Nursing Process: Assessment MSC: Area of Client Needs: Health Promotion and Maintenance

10. A client with a new tracheostomy is being seen in the oncology clinic. What finding by the nurse best indicates that goals for the nursing diagnosis Impaired Self-Esteem are being met? a. The client demonstrates good understanding of stoma care. b. The client has joined a book club that meets at the library. c. Family members take turns assisting with stoma care. d. Skin around the stoma is intact without signs of infection.

ANS: B The client joining a book club that meets outside the home and requires him or her to go out in public is the best sign that goals for Impaired Self-Esteem are being met. The other findings are all positive signs but do not relate to this nursing diagnosis. DIF: Evaluating/Synthesis REF: 528 KEY: Tracheostomy| nursing evaluation| psychosocial response MSC: Integrated Process: Nursing Process: Evaluation NOT: Client Needs Category: Psychosocial Integrity

9. After teaching a client with hypertension secondary to renal disease, the nurse assesses the client's understanding. Which statement made by the client indicates a need for additional teaching? a. "I can prevent more damage to my kidneys by managing my blood pressure." b. "If I have increased urination at night, I need to drink less fluid during the day." c. "I need to see the registered dietitian to discuss limiting my protein intake." d. "It is important that I take my antihypertensive medications as directed."

ANS: B The client should not restrict fluids during the day due to increased urination at night. Clients with renal disease may be prescribed fluid restrictions. These clients should be assessed thoroughly for potential dehydration. Increased nocturnal voiding can be decreased by consuming fluids earlier in the day. Blood pressure control is needed to slow the progression of renal dysfunction. When dietary protein is restricted, refer the client to the registered dietitian as needed.

18. A pulmonary nurse cares for clients who have chronic obstructive pulmonary disease (COPD). Which client should the nurse assess first? a. A 46-year-old with a 30-pack-year history of smoking b. A 52-year-old in a tripod position using accessory muscles to breathe c. A 68-year-old who has dependent edema and clubbed fingers d. A 74-year-old with a chronic cough and thick, tenacious secretions

ANS: B The client who is in a tripod position and using accessory muscles is working to breathe. This client must be assessed first to establish how well the client is breathing and provide interventions to minimize respiratory failure. The other clients are not in acute distress. DIF: Applying/Application REF: 559 KEY: Health screening MSC: IntegratedProcess:NursingProcess:Assessment NOT: Client Needs Category: Safe and Effective Care Environment: Management of Care

A nurse cares for a client who is recovering from a hemorrhoidectomy. The client states, "I need to have a bowel movement." Which action should the nurse take? a. Obtain a bedside commode for the client to use. b. Stay with the client while providing privacy. c. Make sure the call light is in reach to signal completion. d. Gather supplies to collect a stool sample for the laboratory.

ANS: B The first bowel movement after hemorrhoidectomy can be painful enough to induce syncope. The nurse should stay with the client. The nurse should instruct clients who are discharged the same day to have someone nearby when they have their first postoperative bowel movement. Making sure the call light is within reach is an important nursing action too, but it does not take priority over client safety. Obtaining a bedside commode and taking a stool sample are not needed in this situation.

A client is recovering from an esophagogastroduodenoscopy (EGD) and requests something to drink. What action by the nurse is best? a. Allow the client cool liquids only. b. Assess the client's gag reflex. c. Remind the client to remain NPO. d. Tell the client to wait 4 hours.

ANS: B The local anesthetic used during this procedure will depress the client's gag reflex. After the procedure, the nurse should ensure that the gag reflex is intact before offering food or fluids. The client does not need to be restricted to cool beverages only and is not required to wait 4 hours before oral intake is allowed. Telling the client to remain NPO does not inform the client of when he or she can have fluids, nor does it reflect the client's readiness for them

11. A nurse cares for a client who is prescribed magnetic resonance imaging (MRI) of the heart. The client's health history includes a previous myocardial infarction and pacemaker implantation. Which action should the nurse take? a. Schedule an electrocardiogram just before the MRI. b. Notify the health care provider before scheduling the MRI. c. Call the physician and request a laboratory draw for cardiac enzymes. d. Instruct the client to increase fluid intake the day before the MRI.

ANS: B The magnetic fields of the MRI can deactivate the pacemaker. The nurse should call the health care provider and report that the client has a pacemaker so the provider can order other diagnostic tests. The client does not need an electrocardiogram, cardiac enzymes, or increased fluids. DIF: Applying/Application REF: 647 KEY: Assessment/diagnostic examination| patient safety MSC: Integrated Process: Nursing Process: Planning NOT: Client Needs Category: Safe and Effective Care Environment: Safety and Infection Control

A nurse cares for a client who is prescribed patient-controlled analgesia (PCA) after a cholecystectomy. The client states, "When I wake up I am in pain." Which action should the nurse take? a. Administer intravenous morphine while the client sleeps. b. Encourage the client to use the PCA pump upon awakening. c. Contact the provider and request a different analgesic. d. Ask a family member to initiate the PCA pump for the client

ANS: B The nurse should encourage the client to use the PCA pump prior to napping and upon awakening. Administering additional intravenous morphine while the client sleeps places the client at risk for respiratory depression. The nurse should also evaluate dosages received compared with dosages requested and contact the provider if the dose or frequency is not adequate. Only the client should push the pain button on a PCA pump.

Which nursing intervention has the highest priority as a patient diagnosed with anorexia nervosa begins to gain weight? a. Assess for depression and anxiety. b. Observe for adverse effects of refeeding. c. Communicate empathy for the patient's feelings. d. Help the patient balance energy expenditures with caloric intake.

ANS: B The nursing intervention of observing for adverse effects of refeeding most directly relates to weight gain and is a priority. Assessing for depression and anxiety, as well as communicating empathy, relate to coping. Helping the patient achieve balance between energy expenditure and caloric intake is an inappropriate intervention.

Which is most important to document about immunizations in the child's health history? a. Dosage of immunizations received b. Occurrence of any reaction after an immunization c. The exact date the immunizations were received d. Practitioner who administered the immunizations

ANS: B The occurrence of any reaction after an immunization was given is the most important to document in a history because of possible future reactions, especially allergic reactions. Exact dosage of the immunization received may not be recorded on the immunization record. Exact dates are important to obtain but not as important as a history of reaction to an immunization. The practitioner who administered the immunization does not need to be recorded in the health history. A potentially severe physiologic response is the most threatening and most important information to document for safety reasons. DIF: Cognitive Level: Analyze REF: p. 65 TOP: Integrated Process: Communication and Documentation MSC: Area of Client Needs: Health Promotion and Maintenance

A client is scheduled to have a hip replacement. Preoperatively, the client is found to be mildly anemic and the surgeon states the client may need a blood transfusion during or after the surgery. What action by the preoperative nurse is most important? a. Administer preoperative medications as prescribed. b. Ensure that a consent for transfusion is on the chart. c. Explain to the client how anemia affects healing. d. Teach the client about foods high in protein and iron.

ANS: B The preoperative nurse should ensure that all valid consents are on the chart, including one for blood transfusions if this may be needed. Administering preoperative medications is important for all preoperative clients and is not specific to this client. Teaching in the preoperative area should focus on immediate concerns.

19. A nurse auscultated heart tones on an older adult client. Which action should the nurse take based on heart tones heard? (Click the media button to hear the audio clip.) a. Administer a diuretic. b. Document the finding. c. Decrease the IV flow rate. d. Evaluate the client's medications.

ANS: B The sound heard is an atrial gallop S4. An atrial gallop may be heard in older clients because of a stiffened ventricle. The nurse should document the finding, but no other intervention is needed at this time. DIF: Applying/Application REF: 639 KEY: Assessment/diagnostic examination MSC: Integrated Process: Nursing Process: Implementation NOT: Client Needs Category: Health Promotion and Maintenance

18. A nurse teaches a client recovering from a heart transplant who is prescribed cyclosporine (Sandimmune). Which statement should the nurse include in this client's discharge teaching? a. "Use a soft-bristled toothbrush and avoid flossing." b. "Avoid large crowds and people who are sick." c. "Change positions slowly to avoid hypotension." d. "Check your heart rate before taking the medication."

ANS: B These agents cause immune suppression, leaving the client more vulnerable to infection. The medication does not place the client at risk for bleeding, orthostatic hypotension, or a change in heart rate. DIF: Applying/Application REF: 703 KEY: Transplant| immune suppressant MSC: Integrated Process: Nursing Process: Implementation NOT: Client Needs Category: Physiological Integrity: Pharmacological and Parenteral Therapies

14. A client seen in the emergency department reports fever, fatigue, and dry cough but no other upper respiratory symptoms. A chest x-ray reveals mediastinal widening. What action by the nurse is best? a. Collect a sputum sample for culture by deep suctioning. b. Inform the client that antibiotics will be needed for 60 days. c. Place the client on Airborne Precautions immediately. d. Tell the client that directly observed therapy is needed.

ANS: B This client has manifestations of early inhalation anthrax. For treatment, after IV antibiotics are finished, oral antibiotics are continued for at least 60 days. Sputum cultures are not needed. Anthrax is not transmissible from person to person, so Standard Precautions are adequate. Directly observed therapy is often used for tuberculosis.

19. After a craniotomy, the nurse assesses the client and finds dry, sticky mucous membranes and restlessness. The client has IV fluids running at 75 mL/hr. What action by the nurse is best? a. Assess the client's magnesium level. b. Assess the client's sodium level. c. Increase the rate of the IV infusion. d. Provide oral care every hour.

ANS: B This client has manifestations of hypernatremia, which is a possible complication after craniotomy. The nurse should assess the client's serum sodium level. Magnesium level is not related. The nurse does not independently increase the rate of the IV infusion. Providing oral care is also a good option but does not take priority over assessing laboratory results. DIF: Applying/Application REF: 961 KEY: Neurologic disorders| fluid and electrolyte imbalances| nursing assessment MSC: IntegratedProcess:NursingProcess:Assessment NOT: Client Needs Category: Physiological Integrity: Reduction of Risk Potential

23. A nurse is caring for a client on the medical stepdown unit. The following data are related to this client: What action by the nurse is most appropriate? a. Call respiratory therapy for a breathing treatment. b. Facilitate a STAT pulmonary angiography. c. Prepare for immediate endotracheal intubation. d. Prepare to administer intravenous anticoagulants.

ANS: B This client has manifestations of pulmonary embolism (PE); however, many conditions can cause the client's presentation. The gold standard for diagnosing a PE is pulmonary angiography. The nurse should facilitate this test as soon as possible. The client does not have wheezing, so a respiratory treatment is not needed. The client is not unstable enough to need intubation and mechanical ventilation. IV anticoagulants are not given without a diagnosis of PE.

An older client is scheduled to have hip replacement in 2 months and has the following laboratory values: white blood cell count: 8900/mm3, red blood cell count: 3.2/mm3, hemoglobin: 9 g/dL, hematocrit: 32%. What intervention by the nurse is most appropriate? a. Instruct the client to avoid large crowds. b. Prepare to administer epoetin alfa (Epogen). c. Teach the client about foods high in iron. d. Tell the client that all laboratory results are normal.

ANS: B This client is anemic, which needs correction prior to surgery. While eating iron-rich foods is helpful, to increase the client's red blood cells, hemoglobin, and hematocrit within 2 months, epoetin alfa is needed. This colony-stimulating factor will encourage the production of red cells. The client's white blood cell count is normal, so avoiding infection is not the priority.

1. A client is in the emergency department reporting a brief episode during which he was dizzy, unable to speak, and felt like his legs were very heavy. Currently the client's neurologic examination is normal. About what drug should the nurse plan to teach the client? a. Alteplase (Activase) b. Clopidogrel (Plavix) c. Heparin sodium d. Mannitol (Osmitrol)

ANS: B This client's manifestations are consistent with a transient ischemic attack, and the client would be prescribed aspirin or clopidogrel on discharge. Alteplase is used for ischemic stroke. Heparin and mannitol are not used for this condition. DIF: Remembering/Knowledge REF: 930 KEY: Neurologic disorders| antiplatelet medications| patient education MSC: IntegratedProcess:Teaching/Learning NOT: Client Needs Category: Physiological Integrity: Pharmacological and Parenteral Therapies

6. A nurse is caring for four clients on intravenous heparin therapy. Which laboratory value possibly indicates that a serious side effect has occurred? a. Hemoglobin: 14.2 g/dL b. Platelet count: 82,000/L 3 c. Red blood cell count: 4.8/mm 3 d. White blood cell count: 8.7/mm

ANS: B This platelet count is low and could indicate heparin-induced thrombocytopenia. The other values are normal for either gender.

4. After teaching a client how to perform diaphragmatic breathing, the nurse assesses the client's understanding. Which action demonstrates that the client correctly understands the teaching? a. The client lays on his or her side with his or her knees bent. b. The client places his or her hands on his or her abdomen. c. The client lays in a prone position with his or her legs straight. d. The client places his or her hands above his or her head.

ANS: B To perform diaphragmatic breathing correctly, the client should place his or her hands on his or her abdomen to create resistance. This type of breathing cannot be performed effectively while lying on the side or with hands over the head. This type of breathing would not be as effective lying prone. DIF: Applying/Application REF: 562 KEY: Respiratory distress/failure| patient education MSC: IntegratedProcess:Teaching/Learning NOT: Client Needs Category: Physiological Integrity: Physiological Adaptation

A nurse is teaching parents about prevention and treatment of colic. Which should the nurse include in the teaching plan? a. Avoid use of pacifiers. b. Eliminate all second-hand smoke contact. c. Lay infant flat after feeding. d. Avoid swaddling the infant.

ANS: B To prevent and treat colic, teach parents that if household members smoke, avoid smoking near infant; preferably confine smoking activity to outside of home. A pacifier can be introduced for added sucking. The infant should be swaddled tightly with a soft, stretchy blanket and placed in an upright seat after feedings.

According to Erikson, which psychosocial task is developing in adolescence? a. Intimacy b. Identity c. Initiative d. Independence

ANS: B Traditional psychosocial theory holds that the developmental crises of adolescence lead to the formation of a sense of identity. Intimacy is the developmental stage for early adulthood. Independence is not one of Erikson's developmental stages. DIF: Cognitive Level: Understand REF: p. 453 TOP: Integrated Process: Nursing Process: Assessment MSC: Area of Client Needs: Health Promotion and Maintenance

What is an important consideration for the nurse who is communicating with a very young child? a. Speak loudly, clearly, and directly. b. Use transition objects, such as a doll. c. Disguise own feelings, attitudes, and anxiety. d. Initiate contact with child when parent is not present.

ANS: B Using a transition object allows the young child an opportunity to evaluate an unfamiliar person (the nurse). This will facilitate communication with a child this age. Speaking in this manner will tend to increase anxiety in very young children. The nurse must be honest with the child. Attempts at deception will lead to a lack of trust. Whenever possible, the parent should be present for interactions with young children. DIF: Cognitive Level: Understand REF: p. 61 TOP: Integrated Process: Nursing Process: Planning MSC: Area of Client Needs: Psychosocial Integrity

Which communication technique should the nurse avoid when interviewing children and their families? a. Using silence b. Using clichés c. Directing the focus d. Defining the problem

ANS: B Using stereotyped comments or clichés can block effective communication, and this technique should be avoided. After use of such trite phrases, parents will often not respond. Silence can be an effective interviewing tool. Silence permits the interviewee to sort out thoughts and feelings and search for responses to questions. To be effective, the nurse must be able to direct the focus of the interview while allowing maximal freedom of expression. By using open-ended questions, along with guiding questions, the nurse can obtain the necessary information and maintain the relationship with the family. The nurse and parent must collaborate and define the problem that will be the focus of the nursing intervention. DIF: Cognitive Level: Understand REF: p. 59 TOP: Integrated Process: Communication and Documentation MSC: Area of Client Needs: Psychosocial Integrity

16. A client is being discharged soon on warfarin (Coumadin). What menu selection for dinner indicates the client needs more education regarding this medication? a. Hamburger and French fries b. Large chef's salad and muffin c. No selection; spouse brings pizza d. Tuna salad sandwich and chips

ANS: B Warfarin works by inhibiting the synthesis of vitamin K-dependent clotting factors. Foods high in vitamin K thus interfere with its action and need to be eaten in moderate, consistent amounts. The chef's salad most likely has too many leafy green vegetables, which contain high amounts of vitamin K. The other selections, while not particularly healthy, will not interfere with the medication's mechanism of action.

6. A nurse obtains the health history of a client who is newly admitted to the medical unit. Which statement by the client should alert the nurse to the presence of edema? a. "I wake up to go to the bathroom at night." b. "My shoes fit tighter by the end of the day." c. "I seem to be feeling more anxious lately." d. "I drink at least eight glasses of water a day."

ANS: B Weight gain can result from fluid accumulation in the interstitial spaces. This is known as edema. The nurse should note whether the client feels that his or her shoes or rings are tight, and should observe, when present, an indentation around the leg where the socks end. The other answers do not describe edema. DIF: Applying/Application REF: 635 KEY: Heart failure| vascular perfusion MSC: IntegratedProcess:NursingProcess:Assessment NOT: Client Needs Category: Physiological Integrity: Physiological Adaptation

A nurse cares for a client who has elevated levels of antidiuretic hormone (ADH). Which disorder should the nurse identify as a trigger for the release of this hormone? a. Pneumonia b. Dehydration c. Renal failure d. Edema

ANS: B ADH increases tubular permeability to water, leading to absorption of more water into the capillaries. ADH is triggered by a rising extracellular fluid osmolarity, as occurs in dehydration. Pneumonia, renal failure, and edema would not trigger the release of ADH. DIF: Understanding/Comprehension REF: 1355

1. A nurse assesses a client who has a family history of polycystic kidney disease (PKD). For which clinical manifestations should the nurse assess? (Select all that apply.) a. Nocturia b. Flank pain c. Increased abdominal girth d. Dysuria e. Hematuria f. Diarrhea

ANS: B, C, E Clients with PKD experience abdominal distention that manifests as flank pain and increased abdominal girth. Bloody urine is also present with tissue damage secondary to PKD. Clients with PKD often experience constipation, but would not report nocturia or dysuria.

A client has hypertension and high risk factors for cardiovascular disease. The client is overwhelmed with the recommended lifestyle changes. What action by the nurse is best? a. Assess the client's support system. b. Assist in finding one change the client can control. c. Determine what stressors the client faces in daily life. d. Inquire about delegating some of the client's obligations

ANS: B All options are appropriate when assessing stress and responses to stress. However, this client feels overwhelmed by the suggested lifestyle changes. Instead of looking at all the needed changes, the nurse should assist the client in choosing one the client feels optimistic about controlling. Once the client has mastered that change, he or she can move forward with another change. Determining support systems, daily stressors, and delegation opportunities does not directly impact the client's feelings of control. DIF: Applying/Application REF: 712

A client is receiving an infusion of alteplase (Activase) for an intra-arterial clot. The client begins to mumble and is disoriented. What action by the nurse takes priority? a. Assess the client's neurologic status. b. Notify the Rapid Response Team. c. Prepare to administer vitamin K. d. Turn down the infusion rate.

ANS: B Clients on fibrinolytic therapy are at high risk of bleeding. The sudden onset of neurologic signs may indicate the client is having a hemorrhagic stroke. The nurse does need to complete a thorough neurological examination, but should first call the Rapid Response Team based on the client's manifestations. The nurse notifies the Rapid Response Team first. Vitamin K is not the antidote for this drug. Turning down the infusion rate will not be helpful if the client is still receiving any of the drug. DIF: Applying/Application REF: 726

A nurse reviews a female client's laboratory results. Which results from the client's urinalysis should the nurse recognize as abnormal? a. pH 5.6 b. Ketone bodies present c. Specific gravity of 1.020 d. Clear and yellow color

ANS: B Ketone bodies are by-products of incomplete metabolism of fatty acids. Normally no ketones are present in urine. Ketone bodies are produced when fat sources are used instead of glucose to provide cellular energy. A pH between 4.6 and 8, specific gravity between 1.005 and 1.030, and clear yellow urine are normal findings for a female client's urinalysis. DIF: Remembering/Knowledge REF: 1356 KEY

A nurse reviews the health history of a client with an oversecretion of renin. Which disorder should the nurse correlate with this assessment finding? a. Alzheimer's disease b. Hypertension c. Diabetes mellitus d. Viral hepatitis

ANS: B Renin is secreted when special cells in the distal convoluted tubule, called the macula densa, sense changes in blood volume and pressure. When the macula densa cells sense that blood volume, blood pressure, or blood sodium levels are low, renin is secreted. Renin then converts angiotensinogen into angiotensin I. This leads to a series of reactions that cause secretion of the hormone aldosterone. This hormone increases kidney reabsorption of sodium and water, increasing blood pressure, blood volume, and blood sodium levels. Inappropriate or excessive renin secretion is a major cause of persistent hypertension. Renin has no impact on Alzheimer's disease, diabetes mellitus, or viral hepatitis.

The nurse is evaluating a 3-day diet history with a client who has an elevated lipid panel. What meal selection indicates the client is managing this condition well with diet? a. A 4-ounce steak, French fries, iceberg lettuce b. Baked chicken breast, broccoli, tomatoes c. Fried catfish, cornbread, peas d. Spaghetti with meat sauce, garlic bread

ANS: B The diet recommended for this client would be low in saturated fats and red meat, high in vegetables and whole grains (fiber), low in salt, and low in trans fat. The best choice is the chicken with broccoli and tomatoes. The French fries have too much fat and the iceberg lettuce has little fiber. The catfish is fried. The spaghetti dinner has too much red meat and no vegetables. DIF: Evaluating/Synthesis REF: 708

A nurse cares for a client with a urine specific gravity of 1.018. Which action should the nurse take? a. Evaluate the client's intake and output for the past 24 hours. b. Document the finding in the chart and continue to monitor. c. Obtain a specimen for a urine culture and sensitivity. d. Encourage the client to drink more fluids, especially water.

ANS: B This specific gravity is within the normal range for urine. There is no need to evaluate the client's intake and output, obtain a urine specimen, or increase fluid intake. DIF: Applying/Application REF: 1356

A nurse is caring for a client with a deep vein thrombosis (DVT). What nursing assessment indicates a priority outcome has been met? a. Ambulates with assistance b. Oxygen saturation of 98% c. Pain of 2/10 after medication d. Verbalizing risk factors

ANS: B A critical complication of DVT is pulmonary embolism. A normal oxygen saturation indicates that this has not occurred. The other assessments are also positive, but not the priority. DIF: Analyzing/Analysis REF: 729

A client presents to the emergency department with a severely lacerated artery. What is the priority action for the nurse? a. Administer oxygen via non-rebreather mask. b. Ensure the client has a patent airway. c. Prepare to assist with suturing the artery. d. Start two large-bore IVs with normal saline.

ANS: B Airway always takes priority, followed by breathing and circulation. The nurse ensures the client has a patent airway prior to providing any other care measures. DIF: Applying/Application REF: 736

A nurse wants to provide community service that helps meet the goals of Healthy People 2020 (HP2020) related to cardiovascular disease and stroke. What activity would best meet this goal? a. Teach high school students heart-healthy living. b. Participate in blood pressure screenings at the mall. c. Provide pamphlets on heart disease at the grocery store. d. Set up an "Ask the nurse" booth at the pet store.

ANS: B An important goal of HP2020 is to increase the proportion of adults who have had their blood pressure measured within the preceding 2 years and can state whether their blood pressure was normal or high. Participating in blood pressure screening in a public spot will best help meet that goal. The other options are all appropriate but do not specifically help meet a goal. DIF: Applying/Application REF: 711

A nurse is conducting education classes for parents of infants. The nurse plans to discuss sudden infant death syndrome (SIDS). Which risk factors should the nurse include as increasing an infant's risk of a sudden infant death syndrome incident? (Select all that apply.) a. Breastfeeding b. Low Apgar scores c. Male sex d. Birth weight in the 50th or higher percentile e. Recent viral illness

ANS: B, C, E Certain groups of infants are at increased risk for SIDS: low birth weight, low Apgar scores, recent viral illness, and male sex. Breastfed infants and infants of average or above average weight are not at higher risk for SIDS.

A nurse is caring for four clients. Which one should the nurse see first? a. Client who needs a beta blocker, and has a blood pressure of 92/58 mm Hg b. Client who had a first dose of captopril (Capoten) and needs to use the bathroom c. Hypertensive client with a blood pressure of 188/92 mm Hg d. Client who needs pain medication prior to a dressing change of a surgical wound

ANS: B Angiotensin-converting enzyme inhibitors such as captopril can cause hypotension, especially after the first dose. The nurse should see this client first to prevent falling if the client decides to get up without assistance. The two blood pressure readings are abnormal but not critical. The nurse should check on the client with higher blood pressure next to assess for problems related to the reading. The nurse can administer the beta blocker as standards state to hold it if the systolic blood pressure is below 90 mm Hg. The client who needs pain medication prior to the dressing change is not a priority over client safety and assisting the other client to the bathroom. DIF: Analyzing/Analysis REF: 716

A client had a percutaneous transluminal coronary angioplasty for peripheral arterial disease. What assessment finding by the nurse indicates a priority outcome for this client has been met? a. Pain rated as 2/10 after medication b. Distal pulse on affected extremity 2+/4+ c. Remains on bedrest as directed d. Verbalizes understanding of procedure

ANS: B Assessing circulation distal to the puncture site is a critical nursing action. A pulse of 2+/4+ indicates good perfusion. Pain control, remaining on bedrest as directed after the procedure, and understanding are all important, but do not take priority over perfusion. DIF: Evaluating/Synthesis REF: 722

A client has been diagnosed with hypertension but does not take the antihypertensive medications because of a lack of symptoms. What response by the nurse is best? a. "Do you have trouble affording your medications?" b. "Most people with hypertension do not have symptoms." c. "You are lucky; most people get severe morning headaches." d. "You need to take your medicine or you will get kidney failure."

ANS: B Most people with hypertension are asymptomatic, although a small percentage do have symptoms such as headache. The nurse should explain this to the client. Asking about paying for medications is not related because the client has already admitted nonadherence. Threatening the client with possible complications will not increase compliance.

A client is 4 hours postoperative after a femoropopliteal bypass. The client reports throbbing leg pain on the affected side, rated as 7/10. What action by the nurse takes priority? a. Administer pain medication as ordered. b. Assess distal pulses and skin color. c. Document the findings in the client's chart. d. Notify the surgeon immediately.

ANS: B Once perfusion has been restored or improved to an extremity, clients can often feel a throbbing pain due to the increased blood flow. However, it is important to differentiate this pain from ischemia. The nurse should assess for other signs of perfusion, such as distal pulses and skin color/temperature. Administering pain medication is done once the nurse determines the client's perfusion status is normal. Documentation needs to be thorough. Notifying the surgeon is not necessary. DIF: Applying/Application REF: 724

A client has a deep vein thrombosis (DVT). What comfort measure does the nurse delegate to the unlicensed assistive personnel (UAP)? a. Ambulate the client. b. Apply a warm moist pack. c. Massage the client's leg. d. Provide an ice pack.

ANS: B Warm moist packs will help with the pain of a DVT. Ambulation is not a comfort measure. Massaging the client's legs is contraindicated to prevent complications such as pulmonary embolism. Ice packs are not recommended for DVT. DIF: Understanding/Comprehension REF: 731

The nursing student studying rheumatoid arthritis (RA) learns which facts about the disease? (Select all that apply.) a. It affects single joints only. b. Antibodies lead to inflammation. c. It consists of an autoimmune process. d. Morning stiffness is rare. e. Permanent damage is inevitable.

ANS: B, C RA is a chronic autoimmune systemic inflammatory disorder leading to arthritis-type symptoms in the joints and other symptoms that can be seen outside the joints. Antibodies are created that lead to inflammation. Clients often report morning stiffness. Permanent damage can be avoided with aggressive, early treatment.

The nurse must check vital signs on a 2-year-old boy who is brought to the clinic for his 24-month checkup. What criteria should the nurse use in determining the appropriate-size blood pressure cuff? (Select all that apply.) a. The cuff is labeled "toddler." b. The cuff bladder width is approximately 40% of the circumference of the upper arm. c. The cuff bladder length covers 80% to 100% of the circumference of the upper arm. d. The cuff bladder covers 50% to 66% of the length of the upper arm.

ANS: B, C Research has demonstrated that cuff selection with a bladder width that is 40% of the arm circumference will usually have a bladder length that is 80% to 100% of the upper arm circumference. This size cuff will most accurately reflect measured radial artery pressure. The name of the cuff is a representative size that may not be suitable for any individual child. Choosing a cuff by limb circumference more accurately reflects arterial pressure than choosing a cuff by length. DIF: Cognitive Level: Understand REF: p. 86 TOP: Integrated Process: Nursing Process: Assessment MSC: Area of Client Needs: Health Promotion and Maintenance

3. A client in the emergency department is taking rifampin (Rifadin) for tuberculosis. The client reports yellowing of the sclera and skin and bleeding after minor trauma. What laboratory results correlate to this condition? (Select all that apply.) a. Blood urea nitrogen (BUN): 19 mg/dL b. International normalized ratio (INR): 6.3 c. Prothrombin time: 35 seconds d. Serum sodium: 130 mEq/L 3 e. White blood cell (WBC) count: 72,000/mm

ANS: B, C Rifampin can cause liver damage, evidenced by the client's high INR and prothrombin time. The BUN and WBC count are normal. The sodium level is low, but that is not related to this client's problem.

5. A nurse assesses a client who has a chest tube. For which manifestations should the nurse immediately intervene? (Select all that apply.) a. Production of pink sputum b. Tracheal deviation c. Sudden onset of shortness of breath d. Pain at insertion site e. Drainage of 75 mL/hr

ANS: B, C Tracheal deviation and sudden onset of shortness of breath are manifestations of a tension pneumothorax. The nurse must intervene immediately for this emergency situation. Pink sputum is associated with pulmonary edema and is not a complication of a chest tube. Pain at the insertion site and drainage of 75 mL/hr are normal findings with a chest tube. DIF: Applying/Application REF: 579 KEY: Drain| respiratory distress/failure MSC: IntegratedProcess:NursingProcess:Assessment NOT: Client Needs Category: Physiological Integrity: Reduction of Risk Potential

After teaching a client with a parasitic gastrointestinal infection, a nurse assesses the client's understanding. Which statements made by the client indicate that the client correctly understands the teaching? (Select all that apply.) a. "I'll have my housekeeper keep my toilet clean." b. "I must take a shower or bathe every day." c. "I should have my well water tested." d. "I will ask my sexual partner to have a stool test." e. "I must only eat raw vegetables from my own garden."

ANS: B, C, D Parasitic infections can be transmitted to other people. The client himself or herself should keep the toilet area clean instead of possibly exposing another person to the disease. Parasites are transmitted via unclean water sources and sexual practices with rectal contact. The client should test his or her well water and ask sexual partners to have their stool examined for parasites. Raw vegetables are not associated with parasitic gastrointestinal infections. The client can eat vegetables from the store or a home garden as long as the water source is clean. DIF: Applying/Application REF: 1190 KEY: Parasitic infection| infection control MSC: Integrated Process: Nursing Process: Evaluation NOT: Client Needs Category: Safe and Effective Care Environment: Safety and Infection Control

An infant has been diagnosed with cow's milk allergy. What are the clinical manifestations the nurse expects to assess? (Select all that apply.) a. Pink mucous membranes b. Vomiting c. Rhinitis d. Abdominal pain e. Moist skin

ANS: B, C, D An infant with cow's milk allergy will possibly have vomiting, rhinitis, and abdominal pain. The mucous membranes may be pale due to anemia from blood lost in the GI tract, and the skin will be itchy with the possibility of atopic dermatitis.

A nurse collaborates with an unlicensed assistive personnel (UAP) to provide care for a client who is in the healing phase of acute pancreatitis. Which statements focused on nutritional requirements should the nurse include when delegating care for this client? (Select all that apply.) a. "Do not allow the client to eat between meals." b. "Make sure the client receives a protein shake." c. "Do not allow caffeine-containing beverages." d. "Make sure the foods are bland with little spice." e. "Do not allow high-carbohydrate food items."

ANS: B, C, D During the healing phase of pancreatitis, the client should be provided small, frequent, moderate- to high-carbohydrate, high-protein, low-fat meals. Protein shakes can be provided to supplement the diet. Foods and beverages should not contain caffeine and should be bland.

Surgery has informed a nurse that the patient returning to the floor after spinal surgery has an opioid epidural catheter for pain management. The nurse should prepare to monitor the patient for which side effects of an opioid epidural catheter? (Select all that apply.) a. Urinary frequency b. Nausea c. Itching d. Respiratory depression

ANS: B, C, D Respiratory depression, nausea, itching, and urinary retention are dose-related side effects from an epidural opioid. Urinary retention, not urinary frequency, would be seen. DIF: Cognitive Level: Apply REF: p. 132 TOP: Integrated Process: Nursing Process: Planning MSC: Area of Client Needs: Physiologic Integrity

A nurse cares for a client with pancreatic cancer who is prescribed implanted radioactive iodine seeds. Which actions should the nurse take when caring for this client? (Select all that apply.) a. Dispose of dirty linen in a red "biohazard" bag. b. Place the client in a private room. c. Wear a lead apron when providing client care. d. Bundle care to minimize exposure to the client. e. Initiate Transmission-Based Precautions.

ANS: B, C, D The client should be placed in a private room and dirty linens kept in the client's room until the radiation source is removed. The nurse should wear a lead apron while providing care, ensuring that the apron always faces the client. The nurse should also bundle care to minimize exposure to the client. Transmission-Based Precautions will not protect the nurse from the implanted radioactive iodine seeds.

A nurse works with several clients who have gout. Which types of gout and their drug treatments are correctly matched? (Select all that apply.) a. Allopurinol (Zyloprim) - Acute gout b. Colchicine (Colcrys) - Acute gout c. Febuxostat (Uloric) - Chronic gout d. Indomethacin (Indocin) - Acute gout e. Probenecid (Benemid) - Chronic gout

ANS: B, C, D, E Acute gout can be treated with colchicine and indomethacin. Chronic gout can be treated with febuxostat and probenecid. Allopurinol is used for chronic gout.

A nurse assesses a client who is recovering from a Whipple procedure. Which clinical manifestations alert the nurse to a complication from this procedure? (Select all that apply.) a. Clay-colored stools b. Substernal chest pain c. Shortness of breath d. Lack of bowel sounds or flatus e. Urine output of 20 mL/6 hr

ANS: B, C, D, E Myocardial infarction (chest pain), pulmonary embolism (shortness of breath), adynamic ileus (lack of bowel sounds or flatus), and renal failure (urine output of 20 mL/6 hr) are just some of the complications for which the nurse must assess the client after the Whipple procedure. Clay-colored stools are associated with cholecystitis and are not a complication of a Whipple procedure.

A nurse prepares to admit a client who has herpes zoster. Which actions should the nurse take? (Select all that apply.) a.Prepare a room for reverse isolation. b.Assess staff for a history of or vaccination for chickenpox. c.Check the admission orders for analgesia. d.Choose a roommate who also is immune suppressed. e.Ensure that gloves are available in the room.

ANS: B, C, E Herpes zoster (shingles) is caused by reactivation of the same virus, varicella zoster, in clients who have previously had chickenpox. Anyone who has not had the disease or has not been vaccinated for it is at high risk for getting chickenpox. Herpes zoster is very painful and requires analgesia. Use of gloves and good handwashing are sufficient to prevent spread. It is best to put this client in a private room. Herpes zoster is a disease of immune suppression, so no one who is immune-suppressed should be in the same room.

A client's small-bore feeding tube has become occluded after the nurse administered medications. What actions by the nurse are best? (Select all that apply.) a. Attempt to dissolve the clog by instilling a cola product. b. Determine if any of the medications come in liquid form. c. Flush the tube before and after administering medications. d. Mix all medications in the formula and use a feeding pump. e. Try to flush the tube with 30 mL of water and gentle pressure.

ANS: B, C, E LIQUID FLUSH FLUSH

9. A nurse assesses a client who is diagnosed with infective endocarditis. Which assessment findings should the nurse expect? (Select all that apply.) a. Weight gain b. Night sweats c. Cardiac murmur d. Abdominal bloating e. Osler's nodes

ANS: B, C, E Clinical manifestations of infective endocarditis include fever with chills, night sweats, malaise and fatigue, anorexia and weight loss, cardiac murmur, and Osler's nodes on palms of the hands and soles of the feet. Abdominal bloating is a manifestation of heart transplantation rejection. DIF: Remembering/Knowledge REF: 697 KEY: Endocarditis MSC: IntegratedProcess:NursingProcess:Assessment NOT: Client Needs Category: Physiological Integrity: Physiological Adaptation

4. A nurse assesses a client with chronic obstructive pulmonary disease. Which questions should the nurse ask to determine the client's activity tolerance? (Select all that apply.) a. "What color is your sputum?" b. "Do you have any difficulty sleeping?" c. "How long does it take to perform your morning routine?" d. "Do you walk upstairs every day?" e. "Have you lost any weight lately?"

ANS: B, C, E Difficulty sleeping could indicate worsening breathlessness, as could taking longer to perform activities of daily living. Weight loss could mean increased dyspnea as the client becomes too fatigued to eat. The color of the client's sputum would not assist in determining activity tolerance. Asking whether the client walks upstairs every day is not as pertinent as determining if the client becomes short of breath on walking upstairs, or if the client goes upstairs less often than previously. DIF: Applying/Application REF: 559 KEY: Functional ability MSC: IntegratedProcess:NursingProcess:Assessment NOT: Client Needs Category: Physiological Integrity: Physiological Adaptation

4. A clients small-bore feeding tube has become occluded after the nurse administered medications. What actions by the nurse are best? (Select all that apply.) a. Attempt to dissolve the clog by instilling a cola product. b. Determine if any of the medications come in liquid form. c. Flush the tube before and after administering medications. d. Mix all medications in the formula and use a feeding pump. e. Try to flush the tube with 30 mL of water and gentle pressure.

ANS: B, C, E If the tube is obstructed, use a 50-mL syringe and gentle pressure to attempt to open the tube. Cola products should not be used unless water is not effective. To prevent future problems, determine if any of the medications can be dispensed in liquid form and flush the tube with water before and after medication administration. Do not mix medications with the formula.

The nurse working in the rheumatology clinic assesses clients with rheumatoid arthritis (RA) for late manifestations. Which signs/symptoms are considered late manifestations of RA? (Select all that apply.) a. Anorexia b. Felty's syndrome c. Joint deformity d. Low-grade fever e. Weight loss

ANS: B, C, E Late manifestations of RA include Felty's syndrome, joint deformity, weight loss, organ involvement, osteoporosis, extreme fatigue, and anemia, among others. Anorexia and low-grade fever are both seen early in the course of the disease.

2. An emergency room nurse assesses a female client. Which assessment findings should alert the nurse to request a prescription for an electrocardiogram? (Select all that apply.) a. Hypertension b. Fatigue despite adequate rest c. Indigestion d. Abdominal pain e. Shortness of breath

ANS: B, C, E Women may not have chest pain with myocardial infarction, but may feel discomfort or indigestion. They often present with a triad of symptoms—indigestion or feeling of abdominal fullness, feeling of chronic fatigue despite adequate rest, and feeling unable to catch their breath. Frequently, women are not diagnosed and therefore are not treated adequately. Hypertension and abdominal pain are not associated with acute coronary syndrome. DIF: Applying/Application REF: 635 KEY: Cardiac electrical conduction MSC: IntegratedProcess:NursingProcess:Assessment NOT: Client Needs Category: Physiological Integrity: Physiological Adaptation

The nurse is administering activated charcoal to a preschool child with acetaminophen (Tylenol) poisoning. What potential complications from the use of activated charcoal should the nurse plan to assess for? (Select all that apply.) a. Diarrhea b. Vomiting c. Fluid retention d. Intestinal obstruction

ANS: B, D Potential complications from the use of activated charcoal include vomiting and possible aspiration, constipation, and intestinal obstruction. Diarrhea and fluid retention are not potential complications of activated charcoal administration.

The nurse working with clients who have gastrointestinal problems knows that which laboratory values are related to what organ dysfunctions? (Select all that apply.) a. Alanine aminotransferase: biliary system b. Ammonia: liver c. Amylase: liver d. Lipase: pancreas e. Urine urobilinogen: stomach

ANS: B, D Alanine aminotransferase and ammonia are related to the liver. Amylase and lipase are related to the pancreas. Urobilinogen evaluates both hepatic and biliary function.

What identified characteristics occur more frequently in parents who abuse their children? (Select all that apply.) a. Older parents b. Socially isolated c. Middle class parents d. Single-parent families e. Few supportive relationships

ANS: B, D, E Abusive families are often socially isolated and have few supportive relationships. Single-parent families are at higher risk for abuse. Younger parents more often are abusers of their children. Abusive parents have stressors such as low-income circumstances, with little education, and are not middle class parents.

4. A nurse assesses a client who is recovering from a nephrostomy. Which assessment findings should alert the nurse to urgently contact the health care provider? (Select all that apply.) a. Clear drainage b. Bloody drainage at site c. Client reports headache d. Foul-smelling drainage e. Urine draining from site

ANS: B, D, E After a nephrostomy, the nurse should assess the client for complications and urgently notify the provider if drainage decreases or stops, drainage is cloudy or foul-smelling, the nephrostomy sites leaks blood or urine, or the client has back pain. Clear drainage is normal. A headache would be an unrelated finding.

5. A nurse prepares a client for a pharmacologic stress echocardiogram. Which actions should the nurse take when preparing this client for the procedure? (Select all that apply.) a. Assist the provider to place a central venous access device. b. Prepare for continuous blood pressure and pulse monitoring. c. Administer the client's prescribed beta blocker. d. Give the client nothing by mouth 3 to 6 hours before the procedure. e. Explain to the client that dobutamine will simulate exercise for this examination.

ANS: B, D, E Clients receiving a pharmacologic stress echocardiogram will need peripheral venous access and continuous blood pressure and pulse monitoring. The client must be NPO 3 to 6 hours prior to the procedure. Education about dobutamine, which will be administered during the procedure, should be performed. Beta blockers are often held prior to the procedure. DIF: Applying/Application REF: 646 KEY: Assessment/diagnostic examination| medication MSC: Integrated Process: Nursing Process: Planning NOT: Client Needs Category: Physiological Integrity: Pharmacological and Parenteral Therapies

11. A nurse is dismissing a client from the emergency department who has a mild traumatic brain injury. What information obtained from the client represents a possible barrier to self-management? (Select all that apply.) a. Does not want to purchase a thermometer b. Is allergic to acetaminophen (Tylenol) c. Laughing, says "Strenuous? What's that?" d. Lives alone and is new in town with no friends e. Plans to have a beer and go to bed once home

ANS: B, D, E Clients should take acetaminophen for headache. An allergy to this drug may mean the client takes aspirin or ibuprofen (Motrin), which should be avoided. The client needs neurologic checks every 1 to 2 hours, and this client does not seem to have anyone available who can do that. Alcohol needs to be avoided for at least 24 hours. A thermometer is not needed. The client laughing at strenuous activity probably does not engage in any kind of strenuous activity, but the nurse should confirm this. DIF: Evaluating/Synthesis REF: 957 KEY: Neurologic disorders| patient education MSC: Integrated Process: Nursing Process: Evaluation NOT: Client Needs Category: Health Promotion and Maintenance

1. A nurse is caring for five clients. For which clients would the nurse assess a high risk for developing a pulmonary embolism (PE)? (Select all that apply.) a. Client who had a reaction to contrast dye yesterday b. Client with a new spinal cord injury on a rotating bed c. Middle-aged man with an exacerbation of asthma d. Older client who is 1-day post hip replacement surgery e. Young obese client with a fractured femur

ANS: B, D, E Conditions that place clients at higher risk of developing PE include prolonged immobility, central venous catheters, surgery, obesity, advancing age, conditions that increase blood clotting, history of thromboembolism, smoking, pregnancy, estrogen therapy, heart failure, stroke, cancer (particularly lung or prostate), and trauma. A contrast dye reaction and asthma pose no risk for PE.

A nurse is monitoring a patient for side effects associated with opioid analgesics. Which side effects should the nurse expect to monitor for? (Select all that apply.) a. Diarrhea b. Respiratory depression c. Hypertension d. Pruritus e. Sweating

ANS: B, D, E Side effects of opioids include respiratory depression, pruritus, and sweating. Constipation may occur, not diarrhea, and orthostatic hypotension may occur but not hypertension. DIF: Cognitive Level: Understand REF: p. 131 TOP: Integrated Process: Nursing Process: Implementation MSC: Area of Client Needs: Physiologic Integrity

3. A nurse is designing a community education program to meet the Healthy People 2020 objectives for nutrition and weight status. What information about these goals does the nurse use to plan this event? (Select all that apply.) a. Decrease the amount of fruit to 1.1 cups/1000 calories. b. Increase the amount of vegetables to 1.1 cups/1000 calories. c. Increase the number of adults at a healthy weight by 25%. d. Reduce the number of adults who are obese by 10%. e. Reduce the consumption of saturated fat by nearly 10%.

ANS: B, D, E Some of the goals in this initiative include increasing fruit consumption to 0.9 cups/1000 calories, increasing vegetable intake to 1.1 cups/1000 calories, increasing the number of people at a healthy weight by 10%, decreasing the number of adults who are obese by 10%, and reducing the consumption of saturated fats by 9.5%.

2. A nurse assesses a client who has a mediastinal chest tube. Which symptoms require the nurse's immediate intervention? (Select all that apply.) a. Production of pink sputum b. Tracheal deviation c. Pain at insertion site d. Sudden onset of shortness of breath e. Drainage greater than 70 mL/hr f. Disconnection at Y site

ANS: B, D, E, F Immediate intervention is warranted if the client has tracheal deviation because this could indicate a tension pneumothorax. Sudden shortness of breath could indicate dislodgment of the tube, occlusion of the tube, or pneumothorax. Drainage greater than 70 mL/hr could indicate hemorrhage. Disconnection at the Y site could result in air entering the tubing. Production of pink sputum, oxygen saturation less than 95%, and pain at the insertion site are not signs/symptoms that would require immediate intervention. DIF: Applying/Application REF: 579 KEY: Drain| respiratory distress/failure MSC: IntegratedProcess:NursingProcess:Assessment NOT: Client Needs Category: Physiological Integrity: Physiological Adaptation

A nurse teaches a client who is recovering from acute pancreatitis. Which statements should the nurse include in this client's teaching? (Select all that apply.) a. "Take a 20-minute walk at least 5 days each week." b. "Attend local Alcoholics Anonymous (AA) meetings weekly." c. "Choose whole grains rather than foods with simple sugars." d. "Use cooking spray when you cook rather than margarine or butter." e. "Stay away from milk and dairy products that contain lactose." f. "We can talk to your doctor about a prescription for nicotine patches."

ANS: B, D, F The client should be advised to stay sober, and AA is a great resource. The client requires a low-fat diet, and cooking spray is low in fat compared with butter or margarine. If the client smokes, he or she must stop because nicotine can precipitate an exacerbation. A nicotine patch may help the client quit smoking. The client must rest until his or her strength returns. The client requires high carbohydrates and calories for healing; complex carbohydrates are not preferred over simple ones. Dairy products do not cause a problem.

The nurse is talking to a parent of an infant with severe atopic dermatitis (eczema). Which response(s) should the nurse reinforce with the parent? (Select all that apply.) a. "You can use warm wet compresses to relieve discomfort." b. "You will need to keep your infant's skin well hydrated by using a mild soap in the bath." c. "You should bathe your baby in a bubble bath two times a day." d. "You will need to prevent your baby from scratching the area by using a mild antihistamine." e. "You can try a fabric softener in the laundry to avoid rough cloth." f. "You should apply an emollient to the skin immediately after a bath."

ANS: B, D, F The eczematous lesions of atopic dermatitis are intensely pruritic. Scratching can lead to new lesions and secondary infection; an antihistamine can be used. Keeping the skin hydrated is a goal of treating atopic dermatitis. Applying an emollient immediately after a bath helps to trap moisture and prevent moisture loss. Cool wet compresses should be used for relief. Bubble baths and harsh soaps should be avoided, as is bathing excessively, since this leads to drying. Fabric softener should be avoided because of the irritant effects of some of its components.

After the introduction of the Back to Sleep campaign in 1992, an increased incidence has been noted of which of the following pediatric disorders? (Select all that apply.) a. SIDS b. Torticollis c. Failure to thrive d. Apnea of infancy e. Plagiocephaly

ANS: B, E Plagiocephaly is a misshapen head caused by the prolonged pressure on one side of the skull. If that side becomes misshapen, facial asymmetry may result. The sternocleidomastoid muscle may tighten on the affected side, causing torticollis. SIDS has decreased by more than 40% with the introduction of the Back to Sleep campaign. Apnea of infancy and failure to thrive are unrelated to the Back to Sleep campaign.

3. A nurse is caring for a client after a stroke. What actions may the nurse delegate to the unlicensed assistive personnel (UAP)? (Select all that apply.) a. Assess neurologic status with the Glasgow Coma Scale. b. Check and document oxygen saturation every 1 to 2 hours. c. Cluster client care to allow periods of uninterrupted rest. d. Elevate the head of the bed to 45 degrees to prevent aspiration. e. Position the client supine with the head in a neutral midline position.

ANS: B, E The UAP can take and document vital signs, including oxygen saturation, and keep the client's head in a neutral, midline position with correct direction from the nurse. The nurse assesses the Glasgow Coma Scale score. The nursing staff should not cluster care because this can cause an increase in the intracranial pressure. The head of the bed should be minimally elevated, up to 30 degrees. DIF: Applying/Application REF: 938 KEY: Neurologic disorders| stroke| delegation| unlicensed assistive personnel (UAP) MSC: Integrated Process: Communication and Documentation NOT: Client Needs Category: Safe and Effective Care Environment: Management of Care

A nurse assesses a client who presents with an increase in psoriatic lesions. Which questions should the nurse ask to identify a possible trigger for worsening of this client's psoriatic lesions? (Select all that apply.) a."Have you eaten a large amount of chocolate lately?" b."Have you been under a lot of stress lately?" c."Have you recently used a public shower?" d."Have you been out of the country recently?" e."Have you recently had any other health problems?" f."Have you changed any medications recently?"

ANS: B, E, F Systemic factors, hormonal changes, psychological stress, medications, and general health factors can aggravate psoriasis. Psoriatic lesions are not triggered by chocolate, public showers, or international travel.

A nurse assesses clients on a medical-surgical unit. Which client should the nurse evaluate for a wound infection? a.Client with blood cultures pending b.Client who has thin, serous wound drainage c.Client with a white blood cell count of 23,000/mm3 d.Client whose wound has decreased in size

ANS: C A client with an elevated white blood cell count should be evaluated for sources of infection. Pending cultures, thin drainage, and a decrease in wound size are not indications that the client may have an infection.

During a well-child visit, the father of a 4-year-old boy tells the nurse that he is not sure if his son is ready for kindergarten. The boys birthday is close to the cut-off date, and he has not attended preschool. What is the nurses best recommendation? a. Start kindergarten. b. Talk to other parents about readiness. c. Perform a developmental screening. d. Postpone kindergarten and go to preschool.

ANS: C A developmental assessment with a screening tool that addresses cognitive, social, and physical milestones can help identify children who may need further assessment. A readiness assessment involves an evaluation of skill acquisition. Stating the child should start kindergarten or go to preschool and postpone kindergarten does not address the fathers concerns about readiness for school. Talking to other parents about readiness does not ascertain if the child is ready and does not address the fathers concerns.

Which is most likely to encourage parents to talk about their feelings related to their child's illness? a. Be sympathetic. b. Use direct questions. c. Use open-ended questions. d. Avoid periods of silence.

ANS: C Closed-ended questions should be avoided when attempting to elicit parents' feelings. Open- ended questions require the parent to respond with more than a brief answer. Sympathy is having feelings or emotions in common with another person rather than understanding those feelings (empathy). Sympathy is not therapeutic in helping the relationship. Direct questions may obtain limited information. In addition, the parent may consider them threatening. Silence can be an effective interviewing tool. It allows sharing of feelings in which two or more people absorb the emotion in depth. Silence permits the interviewee to sort out thoughts and feelings and search for responses to questions. DIF: Cognitive Level: Apply REF: p. 58 TOP: Integrated Process: Communication and Documentation MSC: Area of Client Needs: Psychosocial Integrity

A nurse assesses a young female client who is prescribed isotretinoin (Accutane). Which question should the nurse ask prior to starting this therapy? a."Do you spend a great deal of time in the sun?" b."Have you or any family members ever had skin cancer?" c."Which method of contraception are you using?" d."Do you drink alcoholic beverages?"

ANS: C Isotretinoin has many side effects. It is a known teratogen and can cause severe birth defects. A pregnancy test is required before therapy is initiated, and strict birth control measures must be used during therapy. Sun exposure, alcohol ingestion, and family history of cancer are contraindications for isotretinoin.

Which is the most frequent source of acute childhood lead poisoning? a. Folk remedies b. Unglazed pottery c. Lead-based paint d. Cigarette butts and ashes

ANS: C Lead-based paint in houses built before 1978 is the most frequent source of lead poisoning. Some folk remedies and unglazed pottery may contain lead, but they are not the most frequent source. Cigarette butts and ashes do not contain lead.

8. A nurse assesses a client 2 hours after a cardiac angiography via the left femoral artery. The nurse notes that the left pedal pulse is weak. Which action should the nurse take? a. Elevate the leg and apply a sandbag to the entrance site. b. Increase the flow rate of intravenous fluids. c. Assess the color and temperature of the left leg. d. Document the finding as "left pedal pulse of +1/4."

ANS: C Loss of a pulse distal to an angiography entry site is serious, indicating a possible arterial obstruction. The pulse may be faint because of edema. The left pulse should be compared with the right, and pulses should be compared with previous assessments, especially before the procedure. Assessing color (pale, cyanosis) and temperature (cool, cold) will identify a decrease in circulation. Once all peripheral and vascular assessment data are acquired, the primary health care provider should be notified. Simply documenting the findings is inappropriate. The leg should be positioned below the level of the heart or dangling to increase blood flow to the distal portion of the leg. Increasing intravenous fluids will not address the client's problem. DIF: Applying/Application REF: 644 KEY: Assessment/diagnostic examination| vascular perfusion MSC: IntegratedProcess:NursingProcess:Assessment NOT: Client Needs Category: Safe and Effective Care Environment: Management of Care

A 4-year-old child tells the nurse that she doesnt want another blood sample drawn because I need all of my insides and I dont want anyone taking them out. What is the nurses best interpretation of this? a. The child is being overly dramatic. b. The child has a disturbed body image. c. Preschoolers have poorly defined body boundaries. d. Preschoolers normally have a good understanding of their bodies.

ANS: C Preschoolers have little understanding of body boundaries, which leads to fears of mutilation. The child is not capable of being dramatic at this age. She truly has fear. Body image is just developing in school-age children. Preschoolers do not have good understanding of their bodies.

A child with corrosive poisoning is being admitted to the emergency department. What clinical manifestation does the nurse expect to assess on this child? a. Nausea and vomiting b. Alterations in sensorium, such as lethargy c. Severe burning pain in the mouth, throat, and stomach d. Respiratory symptoms of acute pulmonary involvement

ANS: C Severe burning pain in the mouth, throat, and stomach is a clinical manifestation of corrosive poisoning. Nausea and vomiting; alterations in sensorium, such as lethargy; and respiratory symptoms of acute pulmonary involvement are clinical manifestations of hydrocarbon poisoning.

5. A nurse cares for a client who has developed esophagitis after undergoing radiation therapy for lung cancer. Which diet selection should the nurse provide for this client? a. Spaghetti with meat sauce, ice cream b. Chicken soup, grilled cheese sandwich c. Omelet, soft whole wheat bread d. Pasta salad, custard, orange juice

ANS: C Side effects of radiation therapy may include inflammation of the esophagus. Clients should be taught that bland, soft, high-calorie foods are best, along with liquid nutritional supplements. Tomato sauce may prove too spicy for a client with esophagitis. A grilled cheese sandwich is too difficult to swallow with this condition, and orange juice and other foods with citric acid are too caustic. DIF: Applying/Application REF: 576 KEY: Cancer| nutrition MSC: IntegratedProcess:NursingProcess:Assessment NOT: Client Needs Category: Physiological Integrity: Basic Care and Comfort

A child with diazepam (Valium) poisoning has been admitted to the emergency department. What antidote does the nurse anticipate being prescribed? a. Succimer (Chemet) b. EDTA (Versenate) c. Flumazenil (Romazicon) d. Octreotide acetate (Sandostatin)

ANS: C The antidote for diazepam (Valium) poisoning is flumazenil (Romazicon). Succimer (Chemet) and EDTA (Versenate) are antidotes for heavy metal poisoning. Octreotide acetate (Sandostatin) is an antidote for sulfonylurea poisoning.

A nurse cares for a client who is prescribed vancomycin (Vancocin) 500 mg IV every 6 hours for a methicillin-resistant Staphylococcus aureus (MRSA) infection. Which action should the nurse take? a.Administer it over 30 minutes using an IV pump. b.Give the client diphenhydramine (Benadryl) before the drug. c.Assess the IV site at least every 2 hours for thrombophlebitis. d.Ensure that the client has increased oral intake during therapy.

ANS: C Vancomycin is very irritating to the veins and can easily cause thrombophlebitis. This drug is given over at least 60 minutes; although it can cause histamine release (leading to "red man syndrome"), it is not customary to administer diphenhydramine before starting the infusion. Increasing oral intake is not specific to vancomycin therapy.

A nurse assesses a client who has ulcerative colitis and severe diarrhea. Which assessment should the nurse complete first? a. Inspection of oral mucosa b. Recent dietary intake c. Heart rate and rhythm d. Percussion of abdomen

ANS: C Although the client with severe diarrhea may experience skin irritation and hypovolemia, the client is most at risk for cardiac dysrhythmias secondary to potassium and magnesium loss from severe diarrhea. The client should have her or his electrolyte levels monitored, and electrolyte replacement may be necessary. Oral mucosa inspection, recent dietary intake, and abdominal percussion are important parts of physical assessment but are lower priority for this client than heart rate and rhythm. DIF: Applying/Application REF: 1172 KEY: Ulcerative colitis| hydration MSC: Integrated Process: Nursing Process: Assessment NOT: Client Needs Category: Safe and Effective Care Environment: Management of Care

A nurse cares for a client who is prescribed mesalamine (Asacol) for ulcerative colitis. The client states, "I am having trouble swallowing this pill." Which action should the nurse take? a. Contact the clinical pharmacist and request the medication in suspension form. b. Empty the contents of the capsule into applesauce or pudding for administration. c. Ask the health care provider to prescribe the medication as an enema instead. d. Crush the pill carefully and administer it in applesauce or pudding.

ANS: C Asacol is the oral formula for mesalamine and is produced as an enteric-coated pill that should not be crushed, chewed, or broken. Asacol is not available as a suspension or elixir. If the client is unable to swallow the Asacol pill, a mesalamine enema (Rowasa) may be administered instead, with a provider's order. DIF: Applying/Application REF: 1176 KEY: Ulcerative colitis| medication safety MSC: Integrated Process: Nursing Process: Implementation NOT: Client Needs Category: Physiological Integrity: Pharmacological and Parenteral Therapies

After teaching a client who has diverticulitis, a nurse assesses the client's understanding. Which statement made by the client indicates a need for additional teaching? a. "I'll ride my bike or take a long walk at least three times a week." b. "I must try to include at least 25 grams of fiber in my diet every day." c. "I will take a laxative nightly at bedtime to avoid becoming constipated." d. "I should use my legs rather than my back muscles when I lift heavy objects."

ANS: C Laxatives are not recommended for clients with diverticulitis because they can increase pressure in the bowel, causing additional outpouching of the lumen. Exercise and a high-fiber diet are recommended for clients with diverticulitis because they promote regular bowel function. Using the leg muscles rather than the back for lifting prevents abdominal straining. DIF: Applying/Application REF: 1187 KEY: Diverticulitis| medication MSC: Integrated Process: Nursing Process: Evaluation NOT: Client Needs Category: Health Promotion and Maintenance

A nurse cares for a teenage girl with a new ileostomy. The client states, "I cannot go to prom with an ostomy." How should the nurse respond? a. "Sure you can. Purchase a prom dress one size larger to hide the ostomy appliance." b. "The pouch won't be as noticeable if you avoid broccoli and carbonated drinks prior to the prom." c. "Let's talk to the enterostomal therapist about options for ostomy supplies and dress styles." d. "You can remove the pouch from your ostomy appliance when you are at the prom so that it is less noticeable."

ANS: C The ostomy nurse is a valuable resource for clients, providing suggestions for supplies and methods to manage the ostomy. A larger dress size will not necessarily help hide the ostomy appliance. Avoiding broccoli and carbonated drinks does not offer reassurance for the client. Ileostomies have an almost constant liquid effluent, so pouch removal during the prom is not feasible. DIF: Applying/Application REF: 1180 KEY: Ostomy care| coping MSC: Integrated Process: Caring NOT: Client Needs Category: Safe and Effective Care Environment: Management of Care

During skin inspection of a client, a nurse observes lesions with wavy borders that are widespread across the client's chest. Which descriptors should the nurse use to document these observations? a. Clustered and annular b. Linear and circinate c. Diffuse and serpiginous d. Coalesced and circumscribed

ANS: C "Diffuse" is used to describe lesions that are widespread. "Serpiginous" describes lesions with wavy borders. "Clustered" describes lesions grouped together. "Linear" describes lesions occurring in a straight line. Annular lesions are ringlike with raised borders, circinate lesions are circular, and circumscribed lesions have well-defined sharp borders. "Coalesced" describes lesions that merge with one another and appear confluent.

3. A client has a pulmonary embolism and is started on oxygen. The student nurse asks why the client's oxygen saturation has not significantly improved. What response by the nurse is best? a. "Breathing so rapidly interferes with oxygenation." b. "Maybe the client has respiratory distress syndrome." c. "The blood clot interferes with perfusion in the lungs." d. "The client needs immediate intubation and mechanical ventilation."

ANS: C A large blood clot in the lungs will significantly impair gas exchange and oxygenation. Unless the clot is dissolved, this process will continue unabated. Hyperventilation can interfere with oxygenation by shallow breathing, but there is no evidence that the client is hyperventilating, and this is also not the most precise physiologic answer. Respiratory distress syndrome can occur, but this is not as likely. The client may need to be mechanically ventilated, but without concrete data on FiO2 and SaO2, the nurse cannot make that judgment.

9. A nurse assesses a client who is recovering after a left-sided cardiac catheterization. Which assessment finding requires immediate intervention? a. Urinary output less than intake b. Bruising at the insertion site c. Slurred speech and confusion d. Discomfort in the left leg

ANS: C A left-sided cardiac catheterization specifically increases the risk for a cerebral vascular accident. A change in neurologic status needs to be acted on immediately. Discomfort and bruising are expected at the site. If intake decreases, a client can become dehydrated because of dye excretion. The second intervention would be to increase the client's fluid status. Neurologic changes would take priority. DIF: Applying/Application REF: 644 KEY: Assessment/diagnostic examination| vascular perfusion MSC: IntegratedProcess:NursingProcess:Assessment NOT: Client Needs Category: Safe and Effective Care Environment: Management of Care

A nurse is caring for an adolescent hospitalized for cellulitis. The nurse notes that the adolescent experiences many "mood swings" throughout the day. How should the nurse interpret this behavior? a. Requires a referral to a mental health counselor b. Requires some further lab testing c. It is normal behavior d. Related to feelings of depression

ANS: C Adolescents vacillate in their emotional states between considerable maturity and childlike behavior. One minute they are exuberant and enthusiastic; the next minute they are depressed and withdrawn. Because of these mood swings, adolescents are frequently labeled as unstable, inconsistent, and unpredictable, but the behavior is normal. The behavior would not require a referral to a mental health counselor or further lab testing. The mood swings do not indicate depression. DIF: Cognitive Level: Understand REF: p. 449 TOP: Integrated Process: Nursing Process: Assessment MSC: Area of Client Needs: Health Promotion and Maintenance

A patient referred to the eating disorders clinic has lost 35 pounds during the past 3 months. To assess eating patterns, the nurse should ask the patient: a. "Do you often feel fat?" b. "Who plans the family meals?" c. "What do you eat in a typical day?" d. "What do you think about your present weight?"

ANS: C Although all the questions might be appropriate to ask, only "What do you eat in a typical day?" focuses on the eating patterns. Asking if the patient often feels fat focuses on distortions in body image. Questions about family meal planning are unrelated to eating patterns. Asking for the patient's thoughts on present weight explores the patient's feelings about weight.

When performing an assessment of the external genitalia of an older man, the nurse observes the scrotum to have smooth skin and to be very pendulous. Which action by the nurse is most appropriate? a. Suggest to the client that he should wear an athletic supporter while awake. b. Ask the client if he has been treated for a sexually transmitted disease. c. Document the observation and continue the assessment. d. Notify the health care provider and facilitate a scrotal ultrasound.

ANS: C As the male client ages, the scrotum loses rugae and becomes increasingly pendulous. This is a normal assessment finding. No further action is needed.

A nurse is assessing a patient admitted for an asthma exacerbation. Which breath sounds does the nurse expect to assess? a. Rubs b. Rattles c. Wheezes d. Crackles

ANS: C Asthma causes bronchoconstriction and narrowed passageways. Wheezes are produced as air passes through narrowed passageways. Rubs are the sound created by the friction of one surface rubbing over another. Pleural friction rub is caused by inflammation of the pleural space. Rattles is the term formerly used for crackles. Crackles are the sounds made when air passes through fluid or moisture. DIF: Cognitive Level: Analyze REF: p. 102 TOP: Integrated Process: Nursing Process: Assessment MSC: Area of Client Needs: Health Promotion and Maintenance

A nurse is reviewing hormone changes that occur during adolescence. What is the hormone responsible for the growth of beard, mustache, and body hair in the male? a. Estrogen b. Pituitary c. Androgen d. Progesterone

ANS: C Beard, mustache, and body hair on the chest, upward along the linea alba, and sometimes on other areas (e.g., back and shoulders) appears in males and is androgen dependent. Estrogen and progesterone are produced by the ovaries in the female and do not contribute to body hair appearance in the male. The pituitary hormone does not have any relationship to body hair appearance in the male. DIF: Cognitive Level: Analyze REF: p. 448 TOP: Integrated Process: Nursing Process: Evaluation MSC: Area of Client Needs: Health Promotion and Maintenance

A client has a continuous passive motion (CPM) device after a total knee replacement. What action does the nurse delegate to the unlicensed assistive personnel (UAP) after the affected leg is placed in the machine while the client is in bed? a. Assess the distal circulation in 30 minutes. b. Change the settings based on range of motion. c. Raise the lower siderail on the affected side. d. Remind the client to do quad-setting exercises.

ANS: C Because the client's leg is strapped into the CPM, if it falls off the bed due to movement, the client's leg (and new joint) can be injured. The nurse should instruct the UAP to raise the siderail to prevent this from occurring. Assessment is a nursing responsibility. Only the surgeon, physical therapist, or specially trained technician adjusts the CPM settings. Quad-setting exercises are not related to the CPM machine.

17. After teaching a client who is recovering from a heart transplant to change positions slowly, the client asks, "Why is this important?" How should the nurse respond? a. "Rapid position changes can create shear and friction forces, which can tear out your internal vascular sutures." b. "Your new vascular connections are more sensitive to position changes, leading to increased intravascular pressure and dizziness." c. "Your new heart is not connected to the nervous system and is unable to respond to decreases in blood pressure caused by position changes." d. "While your heart is recovering, blood flow is diverted away from the brain, increasing the risk for stroke when you stand up."

ANS: C Because the new heart is denervated, the baroreceptor and other mechanisms that compensate for blood pressure drops caused by position changes do not function. This allows orthostatic hypotension to persist in the postoperative period. The other options are false statements and do not correctly address the client's question. DIF: Understanding/Comprehension REF: 703 KEY: Transplant| patient education MSC: IntegratedProcess:NursingProcess:Implementation NOT: Client Needs Category: Physiological Integrity: Physiological Adaptation

A nurse prepares to assess the emotional state of a client with end-stage pancreatic cancer. Which action should the nurse take first? a. Bring the client to a quiet room for privacy. b. Pull up a chair and sit next to the client's bed. c. Determine whether the client feels like talking about his or her feelings. d. Review the health care provider's notes about the prognosis for the client.

ANS: C Before conducting an assessment about the client's feelings, the nurse should determine whether he or she is willing and able to talk about them. If the client is open to the conversation and his or her room is not appropriate, an alternative meeting space may be located. The nurse should be present for the client during this time, and pulling up a chair and sitting with the client indicates that presence. Because the nurse is assessing the client's response to a terminal diagnosis, it is not necessary to have detailed information about the projected prognosis; the nurse knows that the client is facing an end-of-life illness.

Parent guidelines for relieving colic in an infant include: a. avoiding touching abdomen. b. avoiding using a pacifier. c. changing infant's position frequently. d. placing infant where family cannot hear the crying.

ANS: C Changing the infant's position frequently may be beneficial. The parent can walk holding the child face down and with the child's chest across the parent's arm. The parent's hand can support the child's abdomen, applying gentle pressure. Gently massaging the abdomen is effective in some children. Pacifiers can be used for meeting additional sucking needs. The child should not be placed where monitoring cannot be done. The child can be placed in the crib and allowed to cry. Periodically, the child should be picked up and comforted.

While caring for a critically ill child, the nurse observes that respirations are gradually increasing in rate and depth, with periods of apnea. What pattern of respiration will the nurse document? a. Dyspnea b. Tachypnea c. Cheyne-Stokes respirations d. Seesaw (paradoxic) respirations

ANS: C Cheyne-Stokes respirations are a pattern of respirations that gradually increase in rate and depth, with periods of apnea. Dyspnea is defined as distress during breathing. Tachypnea is an increased respiratory rate. In seesaw respirations, the chest falls on inspiration and rises on expiration. DIF: Cognitive Level: Understand REF: p. 102 TOP: Integrated Process: Communication and Documentation MSC: Area of Client Needs: Health Promotion and Maintenance

A nurse is counseling parents of a child beginning to show signs of being overweight. The nurse accurately relates which body mass index (BMI)-for-age percentile indicates a risk for being overweight? a. 10th percentile b. 9th percentile c. 85th percentile d. 95th percentile

ANS: C Children who have BMI-for-age greater than or equal to the 85th percentile and less than the 95th percentile are at risk for being overweight. Children in the 9th and 10th percentiles are within normal limits. Children who are greater than or equal to the 95th percentile are considered overweight. DIF: Cognitive Level: Apply REF: p. 79 TOP: Integrated Process: Nursing Process: Implementation MSC: Area of Client Needs: Health Promotion and Maintenance

14. A nurse cares for a client who has pyelonephritis. The client states, "I am embarrassed to talk about my symptoms." How should the nurse respond? a. "I am a professional. Your symptoms will be kept in confidence." b. "I understand. Elimination is a private topic and shouldn't be discussed." c. "Take your time. It is okay to use words that are familiar to you." d. "You seem anxious. Would you like a nurse of the same gender to care for you?"

ANS: C Clients may be uncomfortable discussing issues related to elimination and the genitourinary area. The nurse should encourage the client to use language that is familiar to the client. The nurse should not make promises that cannot be kept, like keeping the client's symptoms confidential. The nurse must assess the client and cannot take the time to stop the discussion or find another nurse to complete the assessment.

A nurse teaches a client who is recovering from a colon resection. Which statement should the nurse include in this client's plan of care? a. "You may experience nausea and vomiting for the first few weeks." b. "Carbonated beverages can help decrease acid reflux from anastomosis sites." c. "Take a stool softener to promote softer stools for ease of defecation." d. "You may return to your normal workout schedule, including weight lifting."

ANS: C Clients recovering from a colon resection should take a stool softener as prescribed to keep stools a soft consistency for ease of passage. Nausea and vomiting are symptoms of intestinal obstruction and perforation and should be reported to the provider immediately. The client should be advised to avoid gas-producing foods and carbonated beverages, and avoid lifting heavy objects or straining on defecation.

6. The nurse is caring for a client with lung cancer who states, "I don't want any pain medication because I am afraid I'll become addicted." How should the nurse respond? a. "I will ask the provider to change your medication to a drug that is less potent." b. "Would you like me to use music therapy to distract you from your pain?" c. "It is unlikely you will become addicted when taking medicine for pain." d. "Would you like me to give you acetaminophen (Tylenol) instead?"

ANS: C Clients should be encouraged to take their pain medications; addiction usually is not an issue with a client in pain. The nurse would not request that the pain medication be changed unless it was not effective. Other methods to decrease pain can be used, in addition to pain medication. DIF: Applying/Application REF: 576 KEY: Cancer| pain| pharmacologic pain management MSC: Integrated Process: Nursing Process: Implementation NOT: Client Needs Category: Physiological Integrity: Pharmacological and Parenteral Therapies 7. After teaching a client who is prescribed salmeterol (Serevent), the nurse assesses the client's understanding. Which statement by the client indicates a need for additional teaching? a. "I will be certain to shake the inhaler well before I use it." b. "It may take a while before I notice a change in my asthma." c. "I will use the drug when I have an asthma attack." d. "I will be careful not to let the drug escape out of my nose and mouth." ANS: C Salmeterol is designed to prevent an asthma attack; it does not relieve or reverse symptoms. Salmeterol has a slow onset of action; therefore, it should not be used as a rescue drug. The drug must be shaken well because it has a tendency to separate easily. Poor technique on the client's part allows the drug to escape through the nose and mouth. DIF: Applying/Application REF: 554 KEY: Medication| patient education MSC: Integrated Process: Teaching/Learning NOT: Client Needs Category: Health Promotion and Maintenance

16. A nurse cares for a client who is recovering from a myocardial infarction. The client states, "I will need to stop eating so much chili to keep that indigestion pain from returning." How should the nurse respond? a. "Chili is high in fat and calories; it would be a good idea to stop eating it." b. "The provider has prescribed an antacid for you to take every morning." c. "What do you understand about what happened to you?" d. "When did you start experiencing this indigestion?"

ANS: C Clients who experience myocardial infarction often respond with denial, which is a defense mechanism. The nurse should ask the client what he or she thinks happened, or what the illness means to him or her. The other responses do not address the client's misconception about recent pain and the cause of that pain. DIF: Applying/Application REF: 640 KEY: Coronary perfusion| coping MSC: Integrated Process: Teaching/Learning NOT: Client Needs Category: Psychosocial Integrity

A nurse cares for an older adult client who has a chronic skin disorder. The client states, "I have not been to church in several weeks because of the discoloration of my skin." How should the nurse respond? a. "I will consult the chaplain to provide you with spiritual support." b. "You do not need to go to church; God is everywhere." c. "Tell me more about your concerns related to your skin." d. "Religious people are nonjudgmental and will accept you."

ANS: C Clients with chronic skin disorders often become socially isolated related to the fear of rejection by others. Nurses should assess how the client's skin changes are affecting the client's body image and encourage the client to express his or her feelings about a change in appearance. The other responses are not appropriate.

2. A nurse assesses a client in an outpatient clinic. Which statement alerts the nurse to the possibility of left-sided heart failure? a. "I have been drinking more water than usual." b. "I am awakened by the need to urinate at night." c. "I must stop halfway up the stairs to catch my breath." d. "I have experienced blurred vision on several occasions."

ANS: C Clients with left-sided heart failure report weakness or fatigue while performing normal activities of daily living, as well as difficulty breathing, or "catching their breath." This occurs as fluid moves into the alveoli. Nocturia is often seen with right-sided heart failure. Thirst and blurred vision are not related to heart failure. DIF: Understanding/Comprehension REF: 682 KEY: Heart failure| assessment/diagnostic examination MSC: IntegratedProcess:NursingProcess:Assessment NOT: Client Needs Category: Health Promotion and Maintenance

4. A nurse cares for a middle-aged female client with diabetes mellitus who is being treated for the third episode of acute pyelonephritis in the past year. The client asks, "What can I do to help prevent these infections?" How should the nurse respond? a. "Test your urine daily for the presence of ketone bodies and proteins." b. "Use tampons rather than sanitary napkins during your menstrual period." c. "Drink more water and empty your bladder more frequently during the day." d. "Keep your hemoglobin A1c under 9% by keeping your blood sugar controlled."

ANS: C Clients with long-standing diabetes mellitus are at risk for pyelonephritis for many reasons. Chronically elevated blood glucose levels spill glucose into the urine, changing the pH and providing a favorable climate for bacterial growth. The neuropathy associated with diabetes reduces bladder tone and reduces the client's sensation of bladder fullness. Thus, even with large amounts of urine, the client voids less frequently, allowing stasis and overgrowth of microorganisms. Increasing fluid intake (specifically water) and voiding frequently prevent stasis and bacterial overgrowth. Testing urine and using tampons will not help prevent pyelonephritis. A hemoglobin A1c of 9% is too high.

16. A nurse cares for a client with chronic obstructive pulmonary disease (COPD) who appears thin and disheveled. Which question should the nurse ask first? a. "Do you have a strong support system?" b. "What do you understand about your disease?" c. "Do you experience shortness of breath with basic activities?" d. "What medications are you prescribed to take each day?"

ANS: C Clients with severe COPD may not be able to perform daily activities, including bathing and eating, because of excessive shortness of breath. The nurse should ask the client if shortness of breath is interfering with basic activities. Although the nurse should know about the client's support systems, current knowledge, and medications, these questions do not address the client's appearance. DIF: Applying/Application REF: 561 KEY: Functional ability MSC: IntegratedProcess:NursingProcess:Assessment NOT: Client Needs Category: Physiological Integrity

A school nurse is teaching a group of preadolescent boys about puberty. By which age should concerns about pubertal delay be considered? a. 12 years b. 13 years c. 14 years d. 15 years

ANS: C Concerns about pubertal delay should be considered for boys who exhibit no enlargement of the testes or scrotal changes from to 14 years. Ages 12 to 13 years is too young for initial concern. DIF: Cognitive Level: Remember REF: p. 449 TOP: Integrated Process: Teaching/Learning MSC: Area of Client Needs: Health Promotion and Maintenance

19. The nurse is teaching a client with chronic obstructive pulmonary disease who has been prescribed continuous oxygen therapy at home. Which statement indicates the client correctly understands the teaching? a. "I plan to wear my oxygen when I exercise and feel short of breath." b. "I will use my portable oxygen when grilling burgers in the backyard." c. "I plan to use cotton balls to cushion the oxygen tubing on my ears." d. "I will only smoke while I am wearing my oxygen via nasal cannula."

ANS: C Cotton balls can decrease pressure ulcers from the oxygen tubing. Continuous oxygen orders mean the client should wear the oxygen at all times. Oxygen fuels a fire. Wearing oxygen while grilling and smoking increases the risk for fire. DIF: Applying/Application REF: 563 KEY: Safety| patient education| oxygen therapy MSC: IntegratedProcess:NursingProcess:Assessment NOT: Client Needs Category: Safe and Effective Care Environment: Safety and Infection Control

22. A nurse cares for a female client who has a family history of cystic fibrosis. The client asks, "Will my children have cystic fibrosis?" How should the nurse respond? a. "Since many of your family members are carriers, your children will also be carriers of the gene." b. "Cystic fibrosis is an autosomal recessive disorder. If you are a carrier, your children will have the disorder." c. "Since you have a family history of cystic fibrosis, I would encourage you and your partner to be tested." d. "Cystic fibrosis is caused by a protein that controls the movement of chloride. Adjusting your diet will decrease the spread of this disorder."

ANS: C Cystic fibrosis is an autosomal recessive disorder in which both gene alleles must be mutated for the disorder to be expressed. The nurse should encourage both the client and partner to be tested for the abnormal gene. The other statements are not true. DIF: Applying/Application REF: 567 KEY: Gene| allele| health screening MSC: Integrated Process: Teaching/Learning NOT: Client Needs Category: Safe and Effective Care Environment: Management of Care

Vitamin A supplementation may be recommended for the young child who has: a. Mumps. b. Measles (rubeola). c. Rubella. d. Erythema infectiosum.

B Evidence shows that vitamin A decreases morbidity and mortality associated with measles. Vitamin A will not lessen the effects of mumps, rubella, or fifth disease.

A client has fibromyalgia and is prescribed duloxetine hydrochloride (Cymbalta). The client calls the clinic and asks the nurse why an antidepressant drug has been prescribed. What response by the nurse is best? a. "A little sedation will help you get some rest." b. "Depression often accompanies fibromyalgia." c. "This drug works in the brain to decrease pain." d. "You will have more energy after taking this drug."

ANS: C Duloxetine works to increase the release of the neurotransmitters serotonin and norepinephrine, which reduces the pain from fibromyalgia. The other answers are inaccurate.

An infant has been pronounced dead from sudden infant death syndrome (SIDS) in the emergency department. Which is an appropriate question to ask the parents? a. "Did you hear the infant cry out?" b. "Why didn't you check on the infant earlier?" c. "What time did you find the infant?" d. "Was the head buried in a blanket?"

ANS: C During a SIDS incident, if the infant is not pronounced dead at the scene, he or she may be transported to the emergency department to be pronounced dead by a physician. While they are in the emergency department, the parents are asked only factual questions, such as when they found the infant, how he or she looked, and whom they called for help. The nurse avoids any remarks that may suggest responsibility, such as "Why didn't you go in earlier?" "Didn't you hear the infant cry out?" "Was the head buried in a blanket?"

7. A lumbar puncture is needed on a school-age child. What should the nurse apply to provide the most appropriate analgesia during this procedure? a. TAC (tetracaine-adrenaline-cocaine) 15 minutes b. Transdermal fentanyl (Duragesic) patch immediately c. EMLA (eutectic mixture of local anesthetics) 1 hour d. EMLA (eutectic mixture of local anesthetics) 30 minutes

ANS: C EMLA is an effective analgesic agent when applied to the skin 60 minutes before a procedure. It eliminates or reduces the pain from most procedures involving skin puncture. TAC provides skin anesthesia about 15 minutes after application to nonintact skin. The gel can be placed on the wound for suturing. Transdermal fentanyl patches are useful for continuous pain control, not rapid pain control. For maximal effectiveness, EMLA must be applied approximately 60 minutes in advance. DIF: Cognitive Level: Apply REF: p. 143 TOP: Integrated Process: Nursing Process: Implementation MSC: Area of Client Needs: Physiologic Integrity

8. A nurse is assisting the health care provider who is intubating a client. The provider has been attempting to intubate for 40 seconds. What action by the nurse takes priority? a. Ensure the client has adequate sedation. b. Find another provider to intubate. c. Interrupt the procedure to give oxygen. d. Monitor the client's oxygen saturation.

ANS: C Each intubation attempt should not exceed 30 seconds (15 is preferable) as it causes hypoxia. The nurse should interrupt the intubation attempt and give the client oxygen. The nurse should also have adequate sedation during the procedure and monitor the client's oxygen saturation, but these do not take priority. Finding another provider is not appropriate at this time.

*A nurse conducting group therapy on the eating disorders unit schedules the sessions immediately after meals for the primary purpose of:* a. maintaining patients' concentration and attention. b. shifting the patients' focus from food to psychotherapy. c. promoting processing of anxiety associated with eating. d. focusing on weight control mechanisms and food preparation.

ANS: C Eating produces high anxiety for patients with eating disorders. Anxiety levels must be lowered if the patient is to be successful in attaining therapeutic goals. Shifting the patients' focus from food to psychotherapy and focusing on weight control mechanisms and food preparation are not desirable. Maintaining patients' concentration and attention is important, but not the primary purpose of the schedule.

A 5-year-old child was diagnosed with encopresis. Which assessment finding would the nurse expect associated with this diagnosis? The child: a. frequently smears feces on clothing and toys. b. experiences frequent nocturnal episodes of bedwetting. c. has accidents of defecation at kindergarten three times a week. d. has occasional episodes of voiding accidents at the day care center.

ANS: C Encopresis refers to unsuccessful bowel control. Bowel control is expected by age 5, so frequent involuntary defecation is associated with this diagnosis. Smearing feces is behavioral. Enuresis refers to the voiding of urine during the day (diurnal) or at night (nocturnal).

The nurse is assessing a client with a history of irregular periods. Which condition does the nurse possibly correlate with this problem? a. Childhood mumps b. Past valve replacement surgery c. Diabetes mellitus d. Mild intermittent asthma

ANS: C Endocrine disorders can affect the hypothalamic-pituitary-gonadal function of both men and women. Mumps would be important to know if the client were male. Past valve replacement surgery would not be contributory. Mild intermittent asthma also would not contribute to this problem. However, a client with more severe asthma who takes steroids on a long-term basis may develop secondary diabetes.

The nurse working in the orthopedic clinic knows that a client with which factor has an absolute contraindication for having a total joint replacement? a. Needs multiple dental fillings b. Over age 85 c. Severe osteoporosis d. Urinary tract infection

ANS: C Osteoporosis is a contraindication to joint replacement because the bones have a high risk of shattering as the new prosthesis is implanted. The client who needs fillings should have them done prior to the surgery. Age greater than 85 is not an absolute contraindication. A urinary tract infection can be treated prior to surgery.

2. A client is admitted with a pulmonary embolism (PE). The client is young, healthy, and active and has no known risk factors for PE. What action by the nurse is most appropriate? a. Encourage the client to walk 5 minutes each hour. b. Refer the client to smoking cessation classes. c. Teach the client about factor V Leiden testing. d. Tell the client that sometimes no cause for disease is found.

ANS: C Factor V Leiden is an inherited thrombophilia that can lead to abnormal clotting events, including PE. A client with no known risk factors for this disorder should be referred for testing. Encouraging the client to walk is healthy, but is not related to the development of a PE in this case, nor is smoking. Although there are cases of disease where no cause is ever found, this assumption is premature.

10. Fentanyl and midazolam (Versed) are given before débridement of a child's burn wounds. Which is the rationale for administration of these medications? a. Promote healing b. Prevent infection c. Provide pain relief d. Limit amount of débridement that will be necessary

ANS: C Fentanyl and midazolam provide excellent intravenous sedation and analgesia to control procedural pain in children with burns. These drugs are for sedation and pain control, not healing, preventing infection, or limiting the amount of débridement. DIF: Cognitive Level: Understand REF: p. 127 TOP: Integrated Process: Nursing Process: Implementation MSC: Area of Client Needs: Physiologic Integrity: Pharmacologic and Parenteral Therapies

A client has newly diagnosed systemic lupus erythematosus (SLE). What instruction by the nurse is most important? a. "Be sure you get enough sleep at night." b. "Eat plenty of high-protein, high-iron foods." c. "Notify your provider at once if you get a fever." d. "Weigh yourself every day on the same scale."

ANS: C Fever is the classic sign of a lupus flare and should be reported immediately. Rest and nutrition are important but do not take priority over teaching the client what to do if he or she develops an elevated temperature. Daily weights may or may not be important depending on renal involvement.

1. A nurse working in a geriatric clinic sees clients with "cold" symptoms and rhinitis. Which drug would be appropriate to teach these clients to take for their symptoms? a. Chlorpheniramine (Chlor-Trimeton) b. Diphenhydramine (Benadryl) c. Fexofenadine (Allegra) d. Hydroxyzine (Vistaril)

ANS: C First-generation antihistamines are not appropriate for use in the older population. These drugs include chlorpheniramine, diphenhydramine, and hydroxyzine. Fexofenadine is a second-generation antihistamine.

13. The nurse assesses the client using the device pictured below to deliver 50% O2: The nurse finds the mask fits snugly, the skin under the mask and straps is intact, and the flow rate of the oxygen is 3 L/min. What action by the nurse is best? a. Assess the client's oxygen saturation. b. Document these findings in the chart. c. Immediately increase the flow rate. d. Turn the flow rate down to 2 L/min.

ANS: C For the Venturi mask to deliver high flow of oxygen, the flow rate must be set correctly, usually between 4 and 10 L/min. The client's flow rate is too low and the nurse should increase it. After increasing the flow rate, the nurse assesses the oxygen saturation and documents the findings. DIF: Analyzing/Analysis REF: 519 KEY: Oxygen| patient safety| oxygen therapy MSC: Integrated Process: Nursing Process: Implementation NOT: Client Needs Category: Safe and Effective Care Environment: Safety and Infection Control MULTIPLE RESPONSE

A nurse notes that a 10-month-old infant has a larger head circumference than chest. The nurse interprets this as a normal finding because the head and chest circumference become equal at which age? a. 1 month b. 6 to 9 months c. 1 to 2 years d. to 3 years

ANS: C Head circumference begins larger than chest circumference. Between ages 1 and 2 years, they become approximately equal. Head circumference is larger than chest circumference before age 1. Chest circumference is larger than head circumference at to 3 years. DIF: Cognitive Level: Remember REF: p. 80 TOP: Integrated Process: Nursing Process: Assessment MSC: Area of Client Needs: Health Promotion and Maintenance

The school nurse tells adolescents in the clinic that confidentiality and privacy will be maintained unless a life-threatening situation arises. How should this practice be interpreted? a. Not appropriate in a school setting b. Never appropriate because adolescents are minors c. Important in establishing trusting relationships d. Suggestive that the nurse is meeting his or her own needs

ANS: C Health professionals who work with adolescents should consider adolescents' increasing independence and responsibility while maintaining privacy and ensuring confidentiality. However, in some circumstances, such as self-destructive behavior or maltreatment by others, they are not able to maintain confidentiality. Confidentiality and privacy are necessary to build trust with this age group. The nurse must be aware of the limits placed on confidentiality by local jurisdiction. DIF: Cognitive Level: Understand REF: p. 456 TOP: Integrated Process: Communication and Documentation MSC: Area of Client Needs: Management of Care

Parents report that they have been giving a multivitamin to their 1-year-old infant. The nurse counsels the parents that which vitamin can cause a toxic reaction at a low dose? a. Niacin b. B6 c. D d. C

ANS: C Hypervitaminosis of vitamin D presents the greatest problem because this fat-soluble vitamin is stored in the body. Vitamin D is the most likely of all vitamins to cause toxic reactions in relatively small overdoses. The water-soluble vitamins, primarily niacin, B6, and C, can also cause toxicity but not at the low dose that occurs with vitamin D.

A nurse provides health teaching for a patient diagnosed with binge-purge bulimia. Priority information the nurse should provide relates to: a. self-monitoring of daily food and fluid intake. b. establishing the desired daily weight gain. c. how to recognize hypokalemia. d. self-esteem maintenance.

ANS: C Hypokalemia results from potassium loss associated with vomiting. Physiological integrity can be maintained if the patient can self-diagnose potassium deficiency and adjust the diet or seek medical assistance. Self-monitoring of daily food and fluid intake is not useful if the patient purges. Daily weight gain may not be desirable for a patient with bulimia nervosa. Self-esteem is an identifiable problem but is of lesser priority than the dangers associated with hypokalemia.

12. A nurse has educated a client on isoniazid (INH). What statement by the client indicates teaching has been effective? a. "I need to take extra vitamin C while on INH." b. "I should take this medicine with milk or juice." c. "I will take this medication on an empty stomach." d. "My contact lenses will be permanently stained."

ANS: C INH needs to be taken on an empty stomach, either 1 hour before or 2 hours after meals. Extra vitamin B needs to be taken while on the drug. Staining of contact lenses commonly occurs while taking rifampin (Rifadin).

8. A client experiences impaired swallowing after a stroke and has worked with speech-language pathology on eating. What nursing assessment best indicates that a priority goal for this problem has been met? a. Chooses preferred items from the menu b. Eats 75% to 100% of all meals and snacks c. Has clear lung sounds on auscultation d. Gains 2 pounds after 1 week

ANS: C Impaired swallowing can lead to aspiration, so the priority goal for this problem is no aspiration. Clear lung sounds is the best indicator that aspiration has not occurred. Choosing menu items is not related to this problem. Eating meals does not indicate the client is not still aspirating. A weight gain indicates improved nutrition but still does not show a lack of aspiration. DIF: Evaluating/Synthesis REF: 942 KEY: Neurologic disorders| stroke| aspiration MSC: IntegratedProcess:NursingProcess:Evaluation NOT: Client Needs Category: Physiological Integrity: Reduction of Risk Potential

7. A nurse assesses an older adult client who is experiencing a myocardial infarction. Which clinical manifestation should the nurse expect? a. Excruciating pain on inspiration b. Left lateral chest wall pain c. Disorientation and confusion d. Numbness and tingling of the arm

ANS: C In older adults, disorientation or confusion may be the major manifestation of myocardial infarction caused by poor cardiac output. Pain manifestations and numbness and tingling of the arm could also be related to the myocardial infarction. However, the nurse should be more concerned about the new onset of disorientation or confusion caused by decreased perfusion. DIF: Applying/Application REF: 637 KEY: Coronary perfusion| older adult MSC: IntegratedProcess:NursingProcess:Assessment NOT: Client Needs Category: Physiological Integrity: Physiological Adaptation

When interviewing the mother of a 3-year-old child, the nurse asks about developmental milestones such as the age of walking without assistance. How should this question be considered? a. Unnecessary information because child is age 3 years b. An important part of the family history c. An important part of the child's past history d. An important part of the child's review of systems

ANS: C Information about the attainment of developmental milestones is important to obtain. It provides data about the child's growth and development that should be included in the past history. Developmental milestones provide important information about the child's physical, social, and neurologic health and should be included in the history for a 3-year-old child. If pertinent, attainment of milestones by siblings would be included in the family history. The review of systems does not include the developmental milestones. DIF: Cognitive Level: Understand REF: p. 65 TOP: Integrated Process: Communication and Documentation MSC: Area of Client Needs: Health Promotion and Maintenance

A nurse assesses a client who has cholecystitis. Which clinical manifestation indicates that the condition is chronic rather than acute? a. Temperature of 100.1° F (37.8° C) b. Positive Murphy's sign c. Light-colored stools d. Upper abdominal pain after eating

ANS: C Jaundice, clay-colored stools, and dark urine are more commonly seen with chronic cholecystitis. The other symptoms are seen equally with both chronic and acute cholecystitis.

2. A nursing student is studying nutritional problems and learns that kwashiorkor is distinguished from marasmus with which finding? a. Deficit of calories b. Lack of all nutrients c. Specific lack of protein d. Unknown cause of malnutrition

ANS: C Kwashiorkor is a lack of protein when total calories are adequate. Marasmus is a caloric malnutrition.

3. After teaching a client who is prescribed a long-acting beta2 agonist medication, a nurse assesses the client's understanding. Which statement indicates the client comprehends the teaching? a. "I will carry this medication with me at all times in case I need it." b. "I will take this medication when I start to experience an asthma attack." c. "I will take this medication every morning to help prevent an acute attack." d. "I will be weaned off this medication when I no longer need it."

ANS: C Long-acting beta2 agonist medications will help prevent an acute asthma attack because they are long acting. The client will take this medication every day for best effect. The client does not have to always keep this medication with him or her because it is not used as a rescue medication. This is not the medication the client will use during an acute asthma attack because it does not have an immediate onset of action. The client will not be weaned off this medication because this is likely to be one of his or her daily medications. DIF: Applying/Application REF: 554 KEY: Medication| patient education MSC: Integrated Process: Teaching/Learning NOT: Client Needs Category: Physiological Integrity: Pharmacological and Parenteral Therapies

8. A nurse cares for a client with chronic obstructive pulmonary disease (COPD). The client states that he no longer enjoys going out with his friends. How should the nurse respond? a. "There are a variety of support groups for people who have COPD." b. "I will ask your provider to prescribe you with an antianxiety agent." c. "Share any thoughts and feelings that cause you to limit social activities." d. "Friends can be a good support system for clients with chronic disorders."

ANS: C Many clients with moderate to severe COPD become socially isolated because they are embarrassed by frequent coughing and mucus production. They also can experience fatigue, which limits their activities. The nurse needs to encourage the client to verbalize thoughts and feelings so that appropriate interventions can be selected. Joining a support group would not decrease feelings of social isolation if the client does not verbalize feelings. Antianxiety agents will not help the client with social isolation. Encouraging a client to participate in activities without verbalizing concerns also would not be an effective strategy for decreasing social isolation. DIF: Applying/Application REF: 561 KEY: Coping| support MSC: IntegratedProcess:Caring NOT: Client Needs Category: Psychosocial Integrity 9. A nurse is teaching a client who has cystic fibrosis (CF). Which statement should the nurse include in this client's teaching? a. "Take an antibiotic each day." b. "Contact your provider to obtain genetic screening." c. "Eat a well-balanced, nutritious diet." d. "Plan to exercise for 30 minutes every day." ANS: C Clients with CF often are malnourished due to vitamin deficiency and pancreatic malfunction. Maintaining nutrition is essential. Daily antibiotics and daily exercise are not essential actions. Genetic screening would not help the client manage CF better. DIF: Applying/Application REF: 567 KEY: Nutrition| patient education MSC: Integrated Process: Teaching/Learning NOT: Client Needs Category: Health Promotion and Maintenance

A nurse is assessing a child with an unrepaired ventricular septal defect. Which heart sound does the nurse expect to assess? a. S3 b. S4 c. Murmur d. Physiologic splitting

ANS: C Murmurs are the sounds that are produced in the heart chambers or major arteries from the back-and-forth flow of blood. These are the sounds expected to be heard in a child with a ventricular septal defect because of the abnormal opening between the ventricles. S3 is a normal heart sound sometimes heard in children. S4 is rarely heard as a normal heart sound. If heard, medical evaluation is required. Physiologic splitting is the distinction of the two sounds in S2 , which widens on inspiration. It is a significant normal finding. DIF: Cognitive Level: Analyze REF: p. 103 TOP: Integrated Process: Nursing Process: Assessment MSC: Area of Client Needs: Health Promotion and Maintenance

A nurse assesses clients at a community health center. Which client is at highest risk for pancreatic cancer? a. A 32-year-old with hypothyroidism b. A 44-year-old with cholelithiasis c. A 50-year-old who has the BRCA2 gene mutation d. A 68-year-old who is of African-American ethnicity

ANS: C Mutations in both the BRCA2 and p16 genes increase the risk for developing pancreatic cancer in a small number of cases. The other factors do not appear to be linked to increased risk.

A nurse is working with a community group promoting healthy aging. What recommendation is best to help prevent osteoarthritis (OA)? a. Avoid contact sports. b. Get plenty of calcium. c. Lose weight if needed. d. Engage in weight-bearing exercise.

ANS: C Obesity can lead to OA, and if the client is overweight, losing weight can help prevent OA or reduce symptoms once it occurs. Arthritis can be caused by contact sports, but this is less common than obesity. Calcium and weight-bearing exercise are both important for osteoporosis.

10. A nurse is caring for several older clients in the hospital that the nurse identifies as being at high risk for healthcare-associated pneumonia. To reduce this risk, what activity should the nurse delegate to the unlicensed assistive personnel (UAP)? a. Encourage between-meal snacks. b. Monitor temperature every 4 hours. c. Provide oral care every 4 hours. d. Report any new onset of cough.

ANS: C Oral colonization by gram-negative bacteria is a risk factor for healthcare-associated pneumonia. Good, frequent oral care can help prevent this from developing and is a task that can be delegated to the UAP. Encouraging good nutrition is important, but this will not prevent pneumonia. Monitoring temperature and reporting new cough in clients is important to detect the onset of possible pneumonia but do not prevent it.

Over the past year, a woman has cooked gourmet meals for her family but eats only tiny servings. This person wears layered loose clothing. Her current weight is 95 pounds, a loss of 35 pounds. Which medical diagnosis is most likely?* a. Binge eating b. Bulimia nervosa c. Anorexia nervosa d. Eating disorder not otherwise specified

ANS: C Overly controlled eating behaviors, extreme weight loss, preoccupation with food, and wearing several layers of loose clothing to appear larger are part of the clinical picture of an individual with anorexia nervosa. The individual with bulimia usually is near normal weight. The binge eater is often overweight. The patient with eating disorder not otherwise specified may be obese. See relationship to audience response question.

4. A nurse assesses an older adult client who has multiple chronic diseases. The client's heart rate is 48 beats/min. Which action should the nurse take first? a. Document the finding in the chart. b. Initiate external pacing. c. Assess the client's medications. d. Administer 1 mg of atropine.

ANS: C Pacemaker cells in the conduction system decrease in number as a person ages, resulting in bradycardia. The nurse should check the medication reconciliation for medications that might cause such a drop in heart rate, then should inform the health care provider. Documentation is important, but it is not the priority action. The heart rate is not low enough for atropine or an external pacemaker to be needed. DIF: Applying/Application REF: 633 KEY: Medication| health screening MSC: IntegratedProcess:NursingProcess:Assessment NOT: Client Needs Category: Safe and Effective Care Environment: Management of Care

3. A nurse is gathering a history on a school-age child admitted for a migraine headache. The child states, "I have been getting a migraine every 2 or 3 months for the last year." The nurse documents this as which type of pain? a. Acute b. Chronic c. Recurrent d. Subacute

ANS: C Pain that is episodic and reoccurs is defined as recurrent pain. The time frame within which episodes of pain recur is at least 3 months. Recurrent pain in children includes migraine headache, episodic sickle cell pain, recurrent abdominal pain (RAP), and recurrent limb pain. Acute pain is pain that lasts for less than 3 months. Chronic pain is pain that lasts, on a daily basis, for more than 3 months. Subacute is not a term for documenting type of pain. DIF: Cognitive Level: Understand REF: p. 118 TOP: Integrated Process: Communication and Documentation MSC: Area of Client Needs: Physiologic Integrity

A nurse cares for a client with acute pancreatitis. The client states, "I am hungry." How should the nurse reply? a. "Is your stomach rumbling or do you have bowel sounds?" b. "I need to check your gag reflex before you can eat." c. "Have you passed any flatus or moved your bowels?" d. "You will not be able to eat until the pain subsides."

ANS: C Paralytic ileus is a common complication of acute pancreatitis. The client should not eat until this has resolved. Bowel sounds and decreased pain are not reliable indicators of peristalsis. Instead, the nurse should assess for passage of flatus or bowel movement.

22. A nurse is preparing to perform a physical assessment on a toddler. Which approach should the nurse use for this child? a. Always proceed in a head-to-toe direction. b. Perform traumatic procedures first. c. Use minimal physical contact initially. d. Demonstrate use of equipment.

ANS: C Parents can remove clothing, and the child can remain on the parent's lap. The nurse should use minimal physical contact initially to gain the child's cooperation. The head-to-toe assessment can be done in older children but usually must be adapted in younger children. Traumatic procedures should always be performed last. These will most likely upset the child and inhibit cooperation. The nurse should introduce the equipment slowly. The child can inspect the equipment, but demonstrations are usually too complex for toddlers. DIF: Cognitive Level: Apply REF: p. 77 TOP: Integrated Process: Nursing Process: Planning MSC: Area of Client Needs: Health Promotion and Maintenance

A nurse is assigned to four children of different ages. In which age group should the nurse understand that body integrity is a concern? a. Toddler b. Preschooler c. School-age child d. Adolescent

ANS: C School-age children have a heightened concern about body integrity. They place importance and value on their bodies and are oversensitive to anything that constitutes a threat or suggestion of injury. Body integrity is not as important a concern to toddlers, preschoolers, or adolescents. DIF: Cognitive Level: Understand REF: p. 61 TOP: Integrated Process: Nursing Process: Planning MSC: Area of Client Needs: Health Promotion and Maintenance

An 8-year-old girl asks the nurse how the blood pressure apparatus works. What is the most appropriate nursing action? a. Ask her why she wants to know. b. Determine why she is so anxious. c. Explain in simple terms how it works. d. Tell her she will see how it works as it is used.

ANS: C School-age children require explanations and reasons for everything. They are interested in the functional aspect of all procedures, objects, and activities. It is appropriate for the nurse to explain how equipment works and what will happen to the child. A nurse should respond positively for requests for information about procedures and health information. By not responding, the nurse may be limiting communication with the child. The child is not exhibiting anxiety, just requesting clarification of what will be occurring. The nurse must explain how the blood pressure cuff works so that the child can then observe during the procedure. DIF: Cognitive Level: Apply REF: p. 61 TOP: Integrated Process: Teaching/Learning MSC: Area of Client Needs: Health Promotion and Maintenance

During a routine health assessment, the nurse notes that an 8-month-old infant has significant head lag. Which is the nurse's most appropriate action? a. Teach parents appropriate exercises. b. Recheck head control at next visit. c. Refer child for further evaluation. d. Refer child for further evaluation if anterior fontanel is still open.

ANS: C Significant head lag after age 6 months strongly indicates cerebral injury and is referred for further evaluation. Reduction of head lag is part of normal development. Exercises will not be effective. The lack of achievement of this developmental milestone must be evaluated. DIF: Cognitive Level: Apply REF: p. 89 TOP: Integrated Process: Nursing Process: Assessment MSC: Area of Client Needs: Health Promotion and Maintenance

10. A client is on a ventilator and is sedated. What care may the nurse delegate to the unlicensed assistive personnel (UAP)? a. Assess the client for sedation needs. b. Get family permission for restraints. c. Provide frequent oral care per protocol. d. Use nonverbal pain assessment tools.

ANS: C The client on mechanical ventilation needs frequent oral care, which can be delegated to the UAP. The other actions fall within the scope of practice of the nurse.

26. A nurse is caring for four clients in the neurologic/neurosurgical intensive care unit. Which client should the nurse assess first? a. Client who has been diagnosed with meningitis with a fever of 101° F (38.3° C) b. Client who had a transient ischemic attack and is waiting for teaching on clopidogrel (Plavix) c. Client receiving tissue plasminogen activator (t-PA) who has a change in respiratory pattern and rate d. Client who is waiting for subarachnoid bolt insertion with the consent form already signed

ANS: C The client receiving t-PA has a change in neurologic status while receiving this fibrinolytic therapy. The nurse assesses this client first as he or she may have an intracerebral bleed. The client with meningitis has expected manifestations. The client waiting for discharge teaching is a lower priority. The client waiting for surgery can be assessed quickly after the nurse sees the client who is receiving t-PA, or the nurse could delegate checking on this client to another nurse. DIF: Analyzing/Analysis REF: 938 KEY: Neurologic disorders| critical rescue| nursing assessment MSC: IntegratedProcess:NursingProcess:Assessment NOT: Client Needs Category: Safe and Effective Care Environment: Management of Care

A nurse teaches a client who is at risk for colon cancer. Which dietary recommendation should the nurse teach this client? a. "Eat low-fiber and low-residual foods." b. "White rice and bread are easier to digest." c. "Add vegetables such as broccoli and cauliflower to your new diet." d. "Foods high in animal fat help to protect the intestinal mucosa."

ANS: C The client should be taught to modify his or her diet to decrease animal fat and refined carbohydrates. The client should also increase high-fiber foods and Brassica vegetables, including broccoli and cauliflower, which help to protect the intestinal mucosa from colon cancer.

When doing a nutritional assessment on a Hispanic family, the nurse learns that their diet consists mainly of vegetables, legumes, and starches. How should the nurse assess this diet? a. Indicates they live in poverty b. Is lacking in protein c. May provide sufficient amino acids d. Should be enriched with meat and milk

ANS: C The diet that contains vegetable, legumes, and starches may provide sufficient essential amino acids, even though the actual amount of meat or dairy protein is low. Many cultures use diets that contain this combination of foods. It is not indicative of poverty. Combinations of foods contain the essential amino acids necessary for growth. A dietary assessment should be done, but many vegetarian diets are sufficient for growth. DIF: Cognitive Level: Understand REF: p. 66 TOP: Integrated Process: Nursing Process: Assessment MSC: Area of Client Needs: Health Promotion and Maintenance

After teaching a client who is prescribed pancreatic enzyme replacement therapy, the nurse assesses the client's understanding. Which statement made by the client indicates a need for additional teaching? a. "The capsules can be opened and the powder sprinkled on applesauce if needed." b. "I will wipe my lips carefully after I drink the enzyme preparation." c. "The best time to take the enzymes is immediately after I have a meal or a snack." d. "I will not mix the enzyme powder with food or liquids that contain protein."

ANS: C The enzymes should be taken immediately before eating meals or snacks. If the client cannot swallow the capsules whole, they can be opened up and the powder sprinkled on applesauce, mashed fruit, or rice cereal. The client should wipe his or her lips carefully after drinking the enzyme preparation because the liquid could damage the skin. Protein items will be dissolved by the enzymes if they are mixed together.

*As a patient admitted to the eating disorders unit undresses, a nurse observes that the patient's body is covered by fine, downy hair. The patient weighs 70 pounds and is 5 feet 4 inches tall. Which term should be documented?* a. Amenorrhea b. Alopecia c. Lanugo d. Stupor

ANS: C The fine, downy hair noted by the nurse is called lanugo. It is frequently seen in patients with anorexia nervosa. None of the other conditions can be supported by the data the nurse has gathered.

21. A nurse cares for a client who tests positive for alpha1-antitrypsin (AAT) deficiency. The client asks, "What does this mean?" How should the nurse respond? a. "Your children will be at high risk for the development of chronic obstructive pulmonary disease." b. "I will contact a genetic counselor to discuss your condition." c. "Your risk for chronic obstructive pulmonary disease is higher, especially if you smoke." d. "This is a recessive gene and should have no impact on your health."

ANS: C The gene for AAT is a recessive gene. Clients with only one allele produce enough AAT to prevent chronic obstructive pulmonary disease (COPD) unless the client smokes. A client with two alleles is at high risk for COPD even if not exposed to smoke or other irritants. The client is a carrier, and children may or may not be at high risk depending on the partner's AAT levels. Contacting a genetic counselor may be helpful but does not address the client's current question. DIF: Applying/Application REF: 558 KEY: Gene| allele| health screening| a1AT (alpha1-antitrypsin) gene MSC: IntegratedProcess:Teaching/Learning NOT: Client Needs Category: Safe and Effective Care Environment: Management of Care

Where in the health history should the nurse describe all details related to the chief complaint? a. Past history b. Chief complaint c. Present illness d. Review of systems

ANS: C The history of the present illness is a narrative of the chief complaint from its earliest onset through its progression to the present. The focus of the present illness is on all factors relevant to the main problem, even if they have disappeared or changed during the onset, interval, and present. Past history refers to information that relates to previous aspects of the child's health, not to the current problem. The chief complaint is the specific reason for the child's visit to the clinic, office, or hospital. It does not contain the narrative portion describing the onset and progression. The review of systems is a specific review of each body system. DIF: Cognitive Level: Understand REF: p. 64 TOP: Integrated Process: Communication and Documentation MSC: Area of Client Needs: Health Promotion and Maintenance

3. A nurse assesses clients on a medical-surgical unit. Which client should the nurse identify as having the greatest risk for cardiovascular disease? a. An 86-year-old man with a history of asthma b. A 32-year-old Asian-American man with colorectal cancer c. A 45-year-old American Indian woman with diabetes mellitus d. A 53-year-old postmenopausal woman who is on hormone therapy

ANS: C The incidence of coronary artery disease and hypertension is higher in American Indians than in whites or Asian Americans. Diabetes mellitus increases the risk for hypertension and coronary artery disease in people of any race or ethnicity. Asthma, colorectal cancer, and hormone therapy do not increase risk for cardiovascular disease. DIF: Understanding/Comprehension REF: 632 KEY: Health screening MSC: IntegratedProcess:NursingProcess:Assessment NOT: Client Needs Category: Safe and Effective Care Environment: Management of Care

What is the single most important factor to consider when communicating with children? a. The child's physical condition b. Presence or absence of the child's parent c. The child's developmental level d. The child's nonverbal behaviors

ANS: C The nurse must be aware of the child's developmental stage to engage in effective communication. The use of both verbal and nonverbal communication should be appropriate to the developmental level. Although the child's physical condition is a consideration, developmental level is much more important. The parents' presence is important when communicating with young children but may be detrimental when speaking with adolescents. Nonverbal behaviors will vary in importance, based on the child's developmental level. DIF: Cognitive Level: Understand REF: p. 60 TOP: Integrated Process: Communication and Documentation MSC: Area of Client Needs: Psychosocial Integrity

17. A nurse attempted to assist a morbidly obese client back to bed and had immediate pain in the lower back. What action by the nurse is most appropriate? a. Ask another nurse to help next time. b. Demand better equipment to use. c. Fill out and file a variance report. d. Refuse to assist the client again.

ANS: C The nurse should complete a variance report per agency policy. Asking another nurse to help and requesting better equipment are both good ideas, but the nurse may have an injury that needs care. It would be unethical to refuse to care for this client again.

17. A nurse prepares a client for coronary artery bypass graft surgery. The client states, "I am afraid I might die." How should the nurse respond? a. "This is a routine surgery and the risk of death is very low." b. "Would you like to speak with a chaplain prior to surgery?" c. "Tell me more about your concerns about the surgery." d. "What support systems do you have to assist you?"

ANS: C The nurse should discuss the client's feelings and concerns related to the surgery. The nurse should not provide false hope or push the client's concerns off on the chaplain. The nurse should address support systems after addressing the client's current issue. DIF: Applying/Application REF: 647 KEY: Assessment/diagnostic examination| coping| anxiety MSC: IntegratedProcess:Teaching/Learning NOT: Client Needs Category: Psychosocial Integrity

5. A client having a tube feeding begins vomiting. What action by the nurse is most appropriate? a. Administer an antiemetic. b. Check the clients gastric residual. c. Hold the feeding until the nausea subsides. d. Reduce the rate of the tube feeding by half.

ANS: C The nurse should hold the feeding until the nausea and vomiting have subsided and consult with the provider on the rate at which to restart the feeding. Giving an antiemetic is not appropriate. After vomiting, a gastric residual will not be accurate. The nurse should not continue to feed the client while he or she is vomiting.

An infant experienced an apparent life-threatening event (ALTE) and is being placed on home apnea monitoring. Parents have understood the instructions for use of a home apnea monitor when they state? a. "We can adjust the monitor to eliminate false alarms." b. "We should sleep in the same bed as our monitored infant." c. "We will check the monitor several times a day to be sure the alarm is working." d. "We will place the monitor in the crib with our infant."

ANS: C The parents should check the monitor several times a day to be sure the alarm is working and that it can be heard from room to room. The parents should not adjust the monitor to eliminate false alarms. Adjustments could compromise the monitor's effectiveness. The monitor should be placed on a firm surface away from the crib and drapes. The parents should not sleep in the same bed as the monitored infant.

Parents are concerned about the number of hours their teenage daughter spends with peers. The nurse explains that peer relationships are important during adolescence for which reason? a. Adolescents dislike their parents. b. Adolescents no longer need parental control. c. They provide adolescents with a feeling of belonging. d. They promote a sense of individuality in adolescents.

ANS: C The peer group serves as a strong support to teenagers, providing them with a sense of belonging and a sense of strength and power. During adolescence, the parent-child relationship changes from one of protection-dependency to one of mutual affection and quality. Parents continue to play an important role in the personal and health-related decisions. The peer group forms the transitional world between dependence and autonomy. DIF: Cognitive Level: Apply REF: p. 449 TOP: Integrated Process: Teaching/Learning MSC: Area of Client Needs: Health Promotion and Maintenance

30. A client has a traumatic brain injury and a positive halo sign. The client is in the intensive care unit, sedated and on a ventilator, and is in critical but stable condition. What collaborative problem takes priority at this time? a. Inability to communicate b. Nutritional deficit c. Risk for acquiring an infection d. Risk for skin breakdown

ANS: C The positive halo sign indicates a leak of cerebrospinal fluid. This places the client at high risk of acquiring an infection. Communication and nutrition are not priorities compared with preventing a brain infection. The client has a definite risk for a skin breakdown, but it is not the immediate danger a brain infection would be. DIF: Applying/Application REF: 952 KEY: Neurologic disorders| infection control| asepsis MSC: Integrated Process: Nursing Process: Analysis NOT: Client Needs Category: Physiological Integrity: Reduction of Risk Potential

7. A nurse is caring for four clients receiving enteral tube feedings. Which client should the nurse see first? a. Client with a blood glucose level of 138 mg/dL b. Client with foul-smelling diarrhea c. Client with a potassium level of 2.6 mEq/L d. Client with a sodium level of 138 mEq/L

ANS: C The potassium is critically low, perhaps due to hyperglycemia-induced hyperosmolarity. The nurse should see this client first. The blood glucose reading is high, but not extreme. The sodium is normal. The client with the diarrhea should be seen last to avoid cross-contamination.

At about what age does the Babinski sign disappear? a. 4 months b. 6 months c. 1 year d. 2 years

ANS: C The presence of the Babinski reflex after about age 1 year, when walking begins, is abnormal. Four to 6 months is too young for the disappearance of the Babinski reflex. Persistence of the Babinski reflex requires further evaluation. DIF: Cognitive Level: Understand REF: p. 109 TOP: Integrated Process: Nursing Process: Assessment MSC: Area of Client Needs: Health Promotion and Maintenance

15. A client has been brought to the emergency department with a life-threatening chest injury. What action by the nurse takes priority? a. Apply oxygen at 100%. b. Assess the respiratory rate. c. Ensure a patent airway. d. Start two large-bore IV lines.

ANS: C The priority for any chest trauma client is airway, breathing, circulation. The nurse first ensures the client has a patent airway. Assessing respiratory rate and applying oxygen are next, followed by inserting IVs.

6. "Remove the mouthpiece from your mouth, keep your lips closed, and hold your breath for at least 10 seconds." a. 2, 3, 4, 5, 6, 1 b. 3, 4, 5, 1, 6, 2 c. 4, 3, 5, 1, 2, 6 d. 5, 3, 6, 1, 2, 4

ANS: C The proper order for correctly using an inhaler with a spacer is as follows. Insert the mouthpiece of the inhaler into the nonmouthpiece end of the spacer. Shake the whole unit vigorously three or four times. Place the mouthpiece into the mouth, over the tongue, and seal the lips tightly around it. Press down firmly on the canister of the inhaler to release one dose of medication into the spacer. Breathe in slowly and deeply. Remove the mouthpiece from the mouth, and, keeping the lips closed, hold the breath for at least 10 seconds. Then breathe out slowly. Wait at least 1 minute between puffs. DIF: Applying/Application REF: MSC: IntegratedProcess:Teaching/Learning NOT: Client Needs Category: Physiological Integrity: Pharmacological and Parenteral Therapies

A client is scheduled for an ultrasound to evaluate for possible uterine fibroids. Which instruction by the nurse is most appropriate? a. "Do not eat or drink anything after midnight." b. "Take these laxatives the morning of the test." c. "Do not urinate an hour before the test; a full bladder will give best results." d. "Have a designated driver because you will be sleepy from the anesthesia."

ANS: C The scan is noninvasive and painless. The abdominal and pelvic organs are better visualized with the bladder full during the scan. The other statements are inaccurate

11. Nitrous oxide is being administered to a child with extensive burn injuries. Which is the purpose of this medication? a. Promote healing b. Prevent infection c. Provide anesthesia d. Improve urinary output

ANS: C The use of short-acting anesthetic agents, such as propofol and nitrous oxide, has proven beneficial in eliminating procedural pain. Nitrous oxide is an anesthetic agent. DIF: Cognitive Level: Understand REF: p. 144 TOP: Integrated Process: Nursing Process: Implementation MSC: Area of Client Needs: Physiologic Integrity: Pharmacologic and Parenteral Therapies

A nurse is discharging a client after a total hip replacement. What statement by the client indicates good potential for self-management? a. "I can bend down to pick something up." b. "I no longer need to do my exercises." c. "I will not sit with my legs crossed." d. "I won't wash my incision to keep it dry."

ANS: C There are many precautions clients need to take after hip replacement surgery, including not bending more than 90 degrees at the hips, continuing prescribed exercises, not crossing the legs, and washing the incision daily and patting it dry.

20. A client is admitted with suspected pneumonia from the emergency department. The client went to the primary care provider a "few days ago" and shows the nurse the results of what the client calls "an allergy test," as shown below: What action by the nurse takes priority? a. Assess the client for possible items to which he or she is allergic. b. Call the primary care provider's office to request records. c. Immediately place the client on Airborne Precautions. d. Prepare to begin administration of intravenous antibiotics.

ANS: C This "allergy test" is actually a positive tuberculosis test. The client should be placed on Airborne Precautions immediately. The other options do not take priority over preventing the spread of the disease.

2. A nurse in a family practice clinic is preparing discharge instructions for a client reporting facial pain that is worse when bending over, tenderness across the cheeks, and postnasal discharge. What instruction will be most helpful? a. "Ice packs may help with the facial pain." b. "Limit fluids to dry out your sinuses." c. "Try warm, moist heat packs on your face." d. "We will schedule you for a computed tomography scan this week."

ANS: C This client has rhinosinusitis. Comfort measures for this condition include breathing in warm steam, hot packs, nasal saline irrigations, sleeping with the head elevated, increased fluids, and avoiding cigarette smoke. The client does not need a CT scan.

6. A client had an embolectomy for an arteriovenous malformation (AVM). The client is now reporting a severe headache and has vomited. What action by the nurse takes priority? a. Administer pain medication. b. Assess the client's vital signs. c. Notify the Rapid Response Team. d. Raise the head of the bed.

ANS: C This client may be experiencing a rebleed from the AVM. The most important action is to call the Rapid Response Team as this is an emergency. The nurse can assess vital signs while someone else notifies the Team, but getting immediate medical attention is the priority. Administering pain medication may not be warranted if the client must return to surgery. The optimal position for the client with an AVM has not been determined, but calling the Rapid Response Team takes priority over positioning. DIF: Applying/Application REF: 941 KEY: Neurologic disorders| critical rescue| Rapid Response Team| communication MSC: Integrated Process: Communication and Documentation NOT: Client Needs Category: Safe and Effective Care Environment: Management of Care

27. The nurse assesses a client's Glasgow Coma Scale (GCS) score and determines it to be 12 (a 4 in each category). What care should the nurse anticipate for this client? a. Can ambulate independently b. May have trouble swallowing c. Needs frequent re-orientation d. Will need near-total care

ANS: C This client will most likely be confused and need frequent re-orientation. The client may not be able to ambulate at all but should do so independently, not because of mental status. Swallowing is not assessed with the GCS. The client will not need near-total care. DIF: Analyzing/Analysis REF: 934 KEY: Neurologic disorders| neurologic assessment MSC: IntegratedProcess:NursingProcess:Assessment NOT: Client Needs Category: Physiological Integrity: Reduction of Risk Potential

What should the nurse suggest to the parents of an infant who has a prolonged need for middle-of-the-night feedings? a. Decrease daytime feedings. b. Allow child to go to sleep with a bottle. c. Offer last feeding as late as possible at night. d. Put infant to bed after asleep from rocking.

ANS: C To manage an infant who has a prolonged need for middle-of-the-night feedings parents should be taught to offer last feeding as late as possible at night. Parent should increase daytime feeding intervals to 4 hours or more (may need to be done gradually), offer no bottles in bed, put to bed awake and when child is crying, check at progressively longer intervals each night; reassure child but do not hold, rock, take to parent's bed, or give bottle or pacifier.

6. A student nurse is providing tracheostomy care. What action by the student requires intervention by the instructor? a. Holding the device securely when changing ties b. Suctioning the client first if secretions are present c. Tying a square knot at the back of the neck d. Using half-strength peroxide for cleansing

ANS: C To prevent pressure ulcers and for client safety, when ties are used that must be knotted, the knot should be placed at the side of the client's neck, not in back. The other actions are appropriate. DIF: Applying/Application REF: 527 KEY: Tracheostomy| tracheostomy care| patient safety| supervision MSC: Integrated Process: Communication and Documentation NOT: Client Needs Category: Safe and Effective Care Environment: Safety and Infection Control

During examination of the male client's external genitalia, the nurse observes a discharge from the urethra when compressing the glans. Which is the nurse's next action? a. Document the observation. b. Ask the client to turn his head and cough. c. Obtain a specimen for culture. d. Test the cremasteric reflex.

ANS: C Urethral discharge is not considered normal in a continent client and should be cultured. The other options would not help provide information about the nature of the discharge.

The nurse is testing an infant's visual acuity. By what age should the infant be able to fix on and follow a target? a. 1 month b. 1 to 2 months c. 3 to 4 months d. 6 months

ANS: C Visual fixation and following a target should be present by ages 3 to 4 months. One to 2 months is too young for this developmental milestone. If the infant is not able to fix and follow by 6 months, further ophthalmologic evaluation is needed. DIF: Cognitive Level: Understand REF: p. 93 TOP: Integrated Process: Nursing Process: Problem Identification MSC: Area of Client Needs: Health Promotion and Maintenance

How does the nurse assess a child's capillary refill time? a. Inspecting the chest b. Auscultating the heart c. Palpating the apical pulse d. Palpating the skin to produce a slight blanching

ANS: D Capillary refill time is assessed by pressing lightly on the skin to produce blanching, and then noting the amount of time it takes for the blanched area to refill. Inspecting the chest, auscultating the heart, and palpating the apical pulse will not provide an assessment of capillary refill time. DIF: Cognitive Level: Understand REF: p. 102 TOP: Integrated Process: Nursing Process: Assessment MSC: Area of Client Needs: Health Promotion and Maintenance

A nurse is teaching a parent of an infant about treatment of seborrhea dermatitis (cradle cap). Which should the nurse include in the instructions? a. Shampoo every three days with a mild soap. b. The hair should be shampooed with a medicated shampoo. c. Shampoo every day with an antiseborrheic shampoo. d. The loosened crusts should not be removed with a fine-toothed comb.

ANS: C When seborrheic lesions are present, the treatment is directed at removing the crusts. Parents are taught the appropriate procedure to clean the scalp. Shampooing should be done daily with a mild soap or commercial baby shampoo; medicated shampoos are not necessary, but an antiseborrheic shampoo containing sulfur and salicylic acid may be used. Shampoo is applied to the scalp and allowed to remain on the scalp until the crusts soften. Then the scalp is thoroughly rinsed. A fine-tooth comb or a soft facial brush helps remove the loosened crusts from the strands of hair after shampooing.

10. A client tells the nurse about losing weight and regaining it multiple times. Besides eating and exercising habits, for what additional data should the nurse assess as the priority? a. Economic ability to join a gym b. Food allergies and intolerances c. Psychosocial influences on weight d. Reasons for wanting to lose weight

ANS: C While all topics might be important to assess, people who lose and gain weight in cycles often are depressed or have poor self-esteem, which has a negative effect on weight-loss efforts. The nurse assesses the clients psychosocial status as the priority.

After a total knee replacement, a client is on the postoperative nursing unit with a continuous femoral nerve blockade. On assessment, the nurse notes the client's pulses are 2+/4+ bilaterally; the skin is pale pink, warm, and dry; and the client is unable to dorsiflex or plantarflex the affected foot. What action does the nurse perform next? a. Document the findings and monitor as prescribed. b. Increase the frequency of monitoring the client. c. Notify the surgeon or anesthesia provider immediately. d. Palpate the client's bladder or perform a bladder scan.

ANS: C With the femoral nerve block, the client should still be able to dorsiflex and plantarflex the affected foot. Since this client has an abnormal finding, the nurse should notify either the surgeon or the anesthesia provider immediately. Documentation is the last priority. Increasing the frequency of assessment may be a good idea, but first the nurse must notify the appropriate person. Palpating the bladder is not related.

A nurse cares for a client who is recovering from a closed percutaneous kidney biopsy. The client states, "My pain has suddenly increased from a 3 to a 10 on a scale of 0 to 10." Which action should the nurse take first? a. Reposition the client on the operative side. b. Administer the prescribed opioid analgesic. c. Assess the pulse rate and blood pressure. d. Examine the color of the client's urine.

ANS: C An increase in the intensity of pain after a percutaneous kidney biopsy is a symptom of internal hemorrhage. A change in vital signs can indicate that hemorrhage is occurring. Before other actions, the nurse must assess the client's hemodynamic status. DIF: Applying/Application REF: 1363

A nurse teaches a client who is recovering from a urography. Which instruction should the nurse include in this client's discharge teaching? a. "Avoid direct contact with your urine for 24 hours until the radioisotope clears." b. "You may have some dribbling of urine for several weeks after this procedure." c. "Be sure to drink at least 3 liters of fluids today to help eliminate the dye faster." d. "Your skin may become slightly yellow from the dye used in this procedure."

ANS: C Dyes used in urography are potentially nephrotoxic. A large fluid intake will help the client eliminate the dye rapidly. Dyes used in urography are not radioactive, the client should not experience any dribbling of urine, and the dye should not change the color of the client's skin. DIF: Applying/Application REF: 1361

A nurse obtains a sterile urine specimen from a client's Foley catheter. After applying a clamp to the drainage tubing distal to the injection port, which action should the nurse take next? a. Clamp another section of the tube to create a fixed sample section for retrieval. b. Insert a syringe into the injection port and aspirate the quantity of urine required. c. Clean the injection port cap of the drainage tubing with povidone-iodine solution. d. Withdraw 10 mL of urine and discard it; then withdraw a fresh sample of urine.

ANS: C It is important to clean the injection port cap of the catheter drainage tubing with an appropriate antiseptic, such as povidone-iodine solution or alcohol. This will help prevent surface contamination before injection of the syringe. The urine sample should be collected directly from the catheter; therefore, a second clamp to create a sample section would not be appropriate. Every sample from the catheter is usable; there is the need to discard the first sample. DIF: Understanding/Comprehension REF: 1357

A nurse is assessing an obese client in the clinic for follow-up after an episode of deep vein thrombosis. The client has lost 20 pounds since the last visit. What action by the nurse is best? a. Ask if the weight loss was intended. b. Encourage a high-protein, high-fiber diet. c. Measure for new compression stockings. d. Review a 3-day food recall diary.

ANS: C Compression stockings must fit correctly in order to work. After losing a significant amount of weight, the client should be re-measured and new stockings ordered if needed. The other options are appropriate, but not the most important. DIF: Applying/Application REF: 731

Disturbed body image is a nursing diagnosis established for a patient diagnosed with an eating disorder. Which outcome indicator is most appropriate to monitor?* a. Weight, muscle, and fat congruence with height, frame, age, and sex b. Calorie intake is within required parameters of treatment plan c. Weight reaches established normal range for the patient d. Patient expresses satisfaction with body appearance

ANS: D Body image disturbances are considered improved or resolved when the patient is consistently satisfied with his or her own appearance and body function. This is a subjective consideration. The other indicators are more objective but less related to the nursing diagnosis.

A nurse assesses a client with renal insufficiency and a low red blood cell count. The client asks, "Is my anemia related to the renal insufficiency?" How should the nurse respond? a. "Red blood cells produce erythropoietin, which increases blood flow to the kidneys." b. "Your anemia and renal insufficiency are related to inadequate vitamin D and a loss of bone density." c. "Erythropoietin is usually released from the kidneys and stimulates red blood cell production in the bone marrow." d. "Kidney insufficiency inhibits active transportation of red blood cells throughout the blood."

ANS: C Erythropoietin is produced in the kidney and is released in response to decreased oxygen tension in the renal blood supply. Erythropoietin stimulates red blood cell production in the bone marrow. Anemia and renal insufficiency are not manifestations of vitamin D deficiency. The kidneys do not play a role in the transportation of red blood cells or any other cells in the blood. DIF: Remembering/Knowledge REF: 1349

A student nurse asks what "essential hypertension" is. What response by the registered nurse is best? a. "It means it is caused by another disease." b. "It means it is 'essential' that it be treated." c. "It is hypertension with no specific cause." d. "It refers to severe and life-threatening hypertension."

ANS: C Essential hypertension is the most common type of hypertension and has no specific cause such as an underlying disease process. Hypertension that is due to another disease process is called secondary hypertension. A severe, life-threatening form of hypertension is malignant hypertension. DIF: Understanding/Comprehension REF: 710

A nurse working with a client who has possible gastritis assesses the client's gastrointestinal system. Which findings indicate a chronic condition as opposed to acute gastritis? (Select all that apply.) a. Anorexia b. Dyspepsia c. Intolerance of fatty foods d. Pernicious anemia e. Nausea and vomiting

ANS: C, D Intolerance of fatty or spicy foods and pernicious anemia are signs of chronic gastritis. Anorexia and nausea/vomiting can be seen in both conditions. Dyspepsia is seen in acute gastritis.

3. A nurse reviews laboratory results for a client with glomerulonephritis. The client's glomerular filtration rate (GFR) is 40 mL/min as measured by a 24-hour creatinine clearance. How should the nurse interpret this finding? (Select all that apply.) a. Excessive GFR b. Normal GFR c. Reduced GFR d. Potential for fluid overload e. Potential for dehydration

ANS: C, D The GFR refers to the initial amount of urine that the kidneys filter from the blood. In the healthy adult, the normal GFR ranges between 100 and 120 mL/min, most of which is reabsorbed in the kidney tubules. A GFR of 40 mL/min is drastically reduced, with the client experiencing fluid retention and risks for hypertension and pulmonary edema as a result of excess vascular fluid.

A nurse has delegated feeding a client to an unlicensed assistive personnel (UAP). What actions does the nurse include in the directions to the UAP? (Select all that apply.) a. Allow 30 minutes for eating so food doesn't get spoiled. b. Assess the client's mouth while providing premeal oral care. c. Ensure warm and cold items stay at appropriate temperatures. d. Remove bedpans, soiled linens, and other unpleasant items. e. Sit with the client, making the atmosphere more relaxed.

ANS: C, D, E UAP RES!

2. When working with women who are taking hormonal birth control, what health promotion measures should the nurse teach to prevent possible pulmonary embolism (PE)? (Select all that apply.) a. Avoid drinking alcohol. b. Eat more omega-3 fatty acids. c. Exercise on a regular basis. d. Maintain a healthy weight. e. Stop smoking cigarettes.

ANS: C, D, E Health promotion measures for clients to prevent thromboembolic events such as PE include maintaining a healthy weight, exercising on a regular basis, and not smoking. Avoiding alcohol and eating more foods containing omega-3 fatty acids are heart-healthy actions but do not relate to the prevention of PE.

A patient diagnosed with anorexia nervosa is hospitalized for treatment. What features should the milieu provide? Select all that apply. a. Flexible mealtimes b. Unscheduled weight checks c. Adherence to a selected menu d. Observation during and after meals e. Monitoring during bathroom trips f. Privileges correlated with emotional expression

ANS: C, D, E Priority milieu interventions support restoration of weight and normalization of eating patterns. This requires close supervision of the patient's eating and prevention of exercise, purging, and other activities. There is strict adherence to menus. Observe patients during and after meals to prevent throwing away food or purging. Monitor all trips to the bathroom. Mealtimes are structured, not flexible. Weighing is performed on a regular schedule. Privileges are correlated with weight gain and treatment plan compliance.

2. A nurse has delegated feeding a client to an unlicensed assistive personnel (UAP). What actions does the nurse include in the directions to the UAP? (Select all that apply.) a. Allow 30 minutes for eating so food doesnt get spoiled. b. Assess the clients mouth while providing premeal oral care. c. Ensure warm and cold items stay at appropriate temperatures. d. Remove bedpans, soiled linens, and other unpleasant items. e. Sit with the client, making the atmosphere more relaxed.

ANS: C, D, E The UAP should make sure food items remain at the appropriate temperatures for maximum palatability. Removing items such as bedpans, urinals, or soiled linens helps make the atmosphere more conducive to eating. The UAP should sit, not stand, next to the client to promote a relaxing experience. The client, especially older clients who tend to eat more slowly, should not be rushed. Assessment is done by the nurse.

The nurse is caring for a client with systemic sclerosis (SSc). What comfort measures can the nurse delegate to the unlicensed assistive personnel (UAP)? (Select all that apply.) a. Collaborate with a registered dietitian for appropriate foods. b. Inspect the skin and note any areas of ulceration. c. Keep the room at a comfortably warm temperature. d. Place a foot cradle at the end of the bed to lift sheets. e. Remind the client to elevate the head of the bed after eating.

ANS: C, D, E The client with SSc should avoid cold temperatures, which may lead to vasospasms and Raynaud's phenomenon. The UAP can adjust the room temperature for the client's comfort. Keeping the sheets off the feet will help prevent injury; the UAP can apply a foot cradle to the bed to hold the sheets up. Because of esophageal problems, the client should remain in an upright position for 1 to 2 hours after meals. The UAP can remind the client of this once he or she has been taught. The other actions are performed by the registered nurse.

1. A nurse assesses a client with asthma and notes bilateral wheezing, decreased pulse oxygen saturation, and suprasternal retraction on inhalation. Which actions should the nurse take? (Select all that apply.) 555 KEY: Medication safety Arterial Blood Gas Results Vital Signs pH = 7.32 PaCO2 = 62 mm Hg PaO2 = 46 mm Hg HCO3- = 28 mEq/L Heart rate = 110 beats/min Respiratory rate = 12 breaths/min Blood pressure = 145/65 mm Hg Oxygen saturation = 76% a. Administer prescribed salmeterol (Serevent) inhaler. b. Assess the client for a tracheal deviation. c. Administer oxygen to keep saturations greater than 94%. d. Perform peak expiratory flow readings. e. Administer prescribed albuterol (Proventil) inhaler.

ANS: C, E Suprasternal retraction caused by inhalation usually indicates that the client is using accessory muscles and is having difficulty moving air into the respiratory passages because of airway narrowing. Wheezing indicates a narrowed airway; a decreased pulse oxygen saturation also supports this finding. The asthma is not responding to the medication, and intervention is needed. Administration of a rescue inhaler is indicated, probably along with administration of oxygen. The nurse would not do a peak flow reading at this time, nor would a code be called. Midline trachea is a normal and expected finding. DIF: Applying/Application REF: 575 KEY: Respiratory distress/failure MSC: Integrated Process: Nursing Process: Implementation NOT: Client Needs Category: Physiological Integrity: Pharmacological and Parenteral Therapies

What is the result of acute salicylate (ASA, aspirin) poisoning? a. Chemical pneumonitis b. Hepatic damage c. Retractions and grunting d. Disorientation and loss of consciousness

ANS: D ASA poisoning causes disorientation and loss of consciousness. Chemical pneumonitis is caused by hydrocarbon ingestion. Hepatic damage is caused by acetaminophen overdose. ASA does not cause airway obstruction.

A young child has just arrived at the emergency department after ingestion of aspirin at home. The practitioner has ordered activated charcoal. The nurse administers charcoal in which way? a. Administer through a nasogastric tube because the child will not drink it because of the taste. b. Serve in a clear plastic cup so the child can see how much has been drunk. c. Give half of the solution, and then give the other half in 1 hour. d. Serve in an opaque container with a straw.

ANS: D Although the activated charcoal can be mixed with a flavorful beverage, it will be black and resemble mud. When it is served in an opaque container, the child does not have any preconceived ideas about its being distasteful. The nasogastric tube should be used only in children without a gag reflex. The ability to see the charcoal solution may affect the child's desire to drink it. The child should be encouraged to drink the solution all at once.

. A child with acetylsalicylic acid (aspirin) poisoning is being admitted to the emergency department. What early clinical manifestation does the nurse expect to assess on this child? a. Hematemesis b. Hematochezia c. Hyperglycemia d. Hyperventilation

ANS: D An early clinical manifestation of acetylsalicylic acid (aspirin) poisoning is hyperventilation. Hematemesis, hematochezia, and hyperglycemia are clinical manifestations of iron poisoning.

After teaching a client how to care for a furuncle in the axilla, a nurse assesses the client's understanding. Which statement indicates the client correctly understands the teaching? a."I'll apply cortisone cream to reduce the inflammation." b."I'll apply a clean dressing after squeezing out the pus." c."I'll keep my arm down at my side to prevent spread." d."I'll cleanse the area prior to applying antibiotic cream."

ANS: D Cleansing and topical antibiotics can eliminate the infection. Warm compresses enhance comfort and open the lesion, allowing better penetration of the topical antibiotic. Cortisone cream reduces the inflammatory response but increases the infectious process. Squeezing the lesion may introduce infection to deeper tissues and cause cellulitis. Keeping the arm down increases moisture in the area and promotes bacterial growth.

What do inflicted immersion burns often appear as? a. Partial-thickness, asymmetrical burns b. Splash pattern burns on hands or feet c. Any splash burn with dry linear marks d. Sharply demarcated, symmetrical burns

ANS: D Immersion burns are sharply demarcated symmetrical burns. Asymmetrical burns and splash burns are often accidental.

A nurse cares for clients who have various skin infections. Which infection is paired with the correct pharmacologic treatment? a.Viral infection - Clindamycin (Cleocin) b.Bacterial infection - Acyclovir (Zovirax) c.Yeast infection - Linezolid (Zyvox) d.Fungal infection - Ketoconazole (Nizoral)

ANS: D Ketoconazole is an antifungal. Clindamycin and linezolid are antibiotics. Acyclovir is an antiviral drug.

A young boy is found squirting lighter fluid into his mouth. His father calls the emergency department. The nurse taking the call should know that the primary danger is which result? a. Hepatic dysfunction b. Dehydration secondary to vomiting c. Esophageal stricture and shock d. Bronchitis and chemical pneumonia

ANS: D Lighter fluid is a hydrocarbon. The immediate danger is aspiration. Acetaminophen overdose, not hydrocarbons, causes hepatic dysfunction. Dehydration is not the primary danger. Esophageal stricture is a late or chronic issue of hydrocarbon ingestion.

20. A nurse cares for a client who has a pleural chest tube. Which action should the nurse take to ensure safe use of this equipment? a. Strip the tubing to minimize clot formation and ensure patency. b. Secure tubing junctions with clamps to prevent accidental disconnections. c. Connect the chest tube to wall suction at the level prescribed by the provider. d. Keep padded clamps at the bedside for use if the drainage system is interrupted.

ANS: D Padded clamps should be kept at the bedside for use if the drainage system becomes dislodged or is interrupted. The nurse should never strip the tubing. Tubing junctions should be taped, not clamped. Wall suction should be set at the level indicated by the device's manufacturer, not the provider. DIF: Remembering/Knowledge REF: 578 KEY: Drains| postsurgical care MSC: Integrated Process: Nursing Process: Implementation NOT: Client Needs Category: Safe and Effective Care Environment: Safety and Infection Control

3. A nurse receives a report on a client who had a left-sided stroke and has homonymous hemianopsia. What action by the nurse is most appropriate for this client? a. Assess for bladder retention and/or incontinence. b. Listen to the client's lungs after eating or drinking. c. Prop the client's right side up when sitting in a chair. d. Rotate the client's meal tray when the client stops eating.

ANS: D This condition is blindness on the same side of both eyes. The client must turn his or her head to see the entire visual field. The client may not see all the food on the tray, so the nurse rotates it so uneaten food is now within the visual field. This condition is not related to bladder function, difficulty swallowing, or lack of trunk control. DIF: Applying/Application REF: 936 KEY: Neurologic disorders| stroke| visual disorders MSC: Integrated Process: Nursing Process: Implementation NOT: Client Needs Category: Physiological Integrity: Basic Care and Comfort

A nurse cares for a client who has food poisoning resulting from a Clostridium botulinum infection. Which assessment should the nurse complete first? a. Heart rate and rhythm b. Bowel sounds c. Urinary output d. Respiratory rate

ANS: D Severe infection with C. botulinum can lead to respiratory failure, so assessments of oxygen saturation and respiratory rate are of high priority for clients with suspected C. botulinum infection. The other assessments may be completed after the respiratory system has been assessed. DIF: Applying/Application REF: 1191 KEY: Hydration| inflammatory bowel disorder MSC: Integrated Process: Nursing Process: Assessment NOT: Client Needs Category: Safe and Effective Care Environment: Management of Care

18. A client has the diagnosis of "valley fever" accompanied by myalgias and arthralgias. What treatment should the nurse educate the client on? a. Intravenous amphotericin B b. Long-term anti-inflammatories c. No specific treatment d. Oral fluconazole (Diflucan)

ANS: D "Valley fever," or coccidioidomycosis, is a fungal infection. Many people do not need treatment and the disease resolves on its own. However, the presence of joint and muscle pain indicates a moderate infection that needs treatment with antifungal medications. IV amphotericin is reserved for pregnant women and those with severe infection. Anti-inflammatory medications may be used to treat muscle aches and pain but are not used long term.

1. A 2-year-old child has been returned to the nursing unit after an inguinal hernia repair. Which pain assessment tool should the nurse use to assess this child for the presence of pain? a. FACES pain rating tool b. Numeric scale c. Oucher scale d. FLACC tool

ANS: D A behavioral pain tool should be used when the child is preverbal or doesn't have the language skills to express pain. The FLACC (face, legs, activity, cry, consolability) tool should be used with a 2-year-old child. The FACES, numeric, and Oucher scales are all self-report pain rating tools. Self-report measures are not sufficiently valid for children younger than 3 years of age because many are not able to accurately self-report their pain. DIF: Cognitive Level: Apply REF: p. 115 TOP: Integrated Process: Nursing Process: Assessment MSC: Area of Client Needs: Physiologic Integrity

A nurse assesses a client with a mechanical bowel obstruction who reports intermittent abdominal pain. An hour later the client reports constant abdominal pain. Which action should the nurse take next? a. Administer intravenous opioid medications. b. Position the client with knees to chest. c. Insert a nasogastric tube for decompression. d. Assess the client's bowel sounds.

ANS: D A change in the nature and timing of abdominal pain in a client with a bowel obstruction can signal peritonitis or perforation. The nurse should immediately check for rebound tenderness and the absence of bowel sounds. The nurse should not medicate the client until the provider has been notified of the change in his or her condition. The nurse may help the client to the knee-chest position for comfort, but this is not the priority action. The nurse need not insert a nasogastric tube for decompression.

13. A nurse cares for a client who had a chest tube placed 6 hours ago and refuses to take deep breaths because of the pain. Which action should the nurse take? a. Ambulate the client in the hallway to promote deep breathing. b. Auscultate the client's anterior and posterior lung fields. c. Encourage the client to take shallow breaths to help with the pain. d. Administer pain medication and encourage the client to take deep breaths.

ANS: D A chest tube is placed in the pleural space and may be uncomfortable for a client. The nurse should provide pain medication to minimize discomfort and encourage the client to take deep breaths. The other responses do not address the client's discomfort and need to take deep breaths to prevent complications. DIF: Applying/Application REF: 580 KEY: Pain| pharmacologic pain management| drain| surgical care MSC: Integrated Process: Nursing Process: Implementation NOT: Client Needs Category: Physiological Integrity: Reduction of Risk Potential

What is an important nursing responsibility when dealing with a family experiencing the loss of an infant from sudden infant death syndrome (SIDS)? a. Explain how SIDS could have been predicted and prevented. b. Interview parents in depth concerning the circumstances surrounding the child's death. c. Discourage parents from making a last visit with the infant. d. Make a follow-up home visit to parents as soon as possible after the child's death.

ANS: D A competent, qualified professional should visit the family at home as soon as possible after the death and provide the family with printed information about SIDS. An explanation of how SIDS could have been predicted and prevented is inappropriate. SIDS cannot be prevented or predicted. Discussions about the cause will only increase parental guilt. The parents should be asked only factual questions to determine the cause of death. Parents should be allowed and encouraged to make a last visit with their child.

14. The nurse is caring for four clients with traumatic brain injuries. Which client should the nurse assess first? a. Client with cerebral perfusion pressure of 72 mm Hg b. Client who has a Glasgow Coma Scale score of 12 c. Client with a PaCO2 of 36 mm Hg who is on a ventilator d. Client who has a temperature of 102° F (38.9° C)

ANS: D A fever is a poor prognostic indicator in clients with brain injuries. The nurse should see this client first. A Glasgow Coma Scale score of 12, a PaCO2 of 36, and cerebral perfusion pressure of 72 mm Hg are all desired outcomes. DIF: Applying/Application REF: 953 KEY: Neurologic disorders| neurologic assessment MSC: IntegratedProcess:NursingProcess:Assessment NOT: Client Needs Category: Safe and Effective Care Environment: Management of Care

*A nurse finds a patient diagnosed with anorexia nervosa vigorously exercising before gaining the agreed-upon weekly weight. Which response by the nurse is appropriate?* a. "You and I will have to sit down and discuss this problem." b. "It bothers me to see you exercising. I am afraid you will lose more weight." c. "Let's discuss the relationship between exercise, weight loss, and the effects on your body." d. "According to our agreement, no exercising is permitted until you have gained a specific amount of weight."

ANS: D A matter-of-fact statement that the nurse's perceptions are different will help to avoid a power struggle. Treatment plans have specific goals for weight restoration. Exercise is limited to promote weight gain. Patients must be held accountable for required behaviors.

Pulses can be graded according to certain criteria. Which is a description of a normal pulse? a. 0 b. +1 c. +2 d. +3

ANS: D A normal pulse is described as +3. A pulse that is easy to palpate and not easily obliterated with pressure is considered normal. A pulse graded 0 is not palpable. A pulse graded +1 is difficult to palpate, thready, weak, and easily obliterated with pressure. A pulse graded +2 is difficult to palpate and may be easily obliterated with pressure. DIF: Cognitive Level: Remember REF: p. 85 TOP: Integrated Process: Nursing Process: Assessment MSC: Area of Client Needs: Health Promotion and Maintenance: Techniques of Physical Assessment

21. A nurse is weighing and measuring a client with severe kyphosis. What is the best method to obtain this clients height? a. Add the trunk and leg measurements. b. Ask the client how tall he or she is. c. Estimate by measuring clothing. d. Use knee-height calipers.

ANS: D A sliding blade knee-height caliper is used to obtain the height of a client who cannot stand upright, such as those with kyphosis or lower extremity contractures. The other methods will not yield accurate data.

18. An emergency department nurse triages clients who present with chest discomfort. Which client should the nurse plan to assess first? a. A 42-year-old female who describes her pain as a dull ache with numbness in her fingers b. A 49-year-old male who reports moderate pain that is worse on inspiration c. A 53-year-old female who reports substernal pain that radiates to her abdomen d. A 58-year-old male who describes his pain as intense stabbing that spreads across his chest

ANS: D All clients who have chest pain should be assessed more thoroughly. To determine which client should be seen first, the nurse must understand common differences in pain descriptions. Intense stabbing, vise-like substernal pain that spreads through the client's chest, arms, jaw, back, or neck is indicative of a myocardial infarction. The nurse should plan to see this client first to prevent cardiac cell death. A dull ache with numbness in the fingers is consistent with anxiety. Pain that gets worse with inspiration is usually related to a pleuropulmonary problem. Pain that spreads to the abdomen is often associated with an esophageal-gastric problem, especially when this pain is experienced by a male client. Female clients may experience abdominal discomfort with a myocardial event. Although clients with anxiety, pleuropulmonary, and esophageal-gastric problems should be seen, they are not a higher priority than myocardial infarction. DIF: Applying/Application REF: 644 KEY: Assessment/diagnostic examination MSC: Integrated Process: Nursing Process: Planning NOT: Client Needs Category: Safe and Effective Care Environment: Management of Care

A client is scheduled for a total gastrectomy for gastric cancer. What preoperative laboratory result should the nurse report to the surgeon immediately? a. Albumin: 2.1 g/dL b. Hematocrit: 28% c. Hemoglobin: 8.1 mg/dL d. International normalized ratio (INR): 4.2

ANS: D An INR as high as 4.2 poses a serious risk of bleeding during the operation and should be reported. The albumin is low and is an expected finding. The hematocrit and hemoglobin are also low, but this is expected in gastric cancer.

A nurse is discharging a client to a short-term rehabilitation center after a joint replacement. Which action by the nurse is most important? a. Administering pain medication before transport b. Answering any last-minute questions by the client c. Ensuring the family has directions to the facility d. Providing a verbal hand-off report to the facility

ANS: D As required by The Joint Commission and other accrediting agencies, a hand-off report must be given to the new provider to prevent error. The other options are valid responses but do not take priority.

10. A nurse assesses a client who is scheduled for a cardiac catheterization. Which assessment should the nurse complete prior to this procedure? a. Client's level of anxiety b. Ability to turn self in bed c. Cardiac rhythm and heart rate d. Allergies to iodine-based agents

ANS: D Before the procedure, the nurse should ascertain whether the client has an allergy to iodine-containing preparations, such as seafood or local anesthetics. The contrast medium used during the procedure is iodine based. This allergy can cause a life-threatening reaction, so it is a high priority. Second, it is important for the nurse to assess anxiety, mobility, and baseline cardiac status. DIF: Remembering/Knowledge REF: 643 KEY: Assessment/diagnostic examination| allergies| patient safety MSC: IntegratedProcess:NursingProcess:Assessment NOT: Client Needs Category: Safe and Effective Care Environment: Safety and Infection Control

2. A nurse assesses a client after administering a prescribed beta blocker. Which assessment should the nurse expect to find? a. Blood pressure increased from 98/42 mm Hg to 132/60 mm Hg b. Respiratory rate decreased from 25 breaths/min to 14 breaths/min c. Oxygen saturation increased from 88% to 96% d. Pulse decreased from 100 beats/min to 80 beats/min

ANS: D Beta blockers block the stimulation of beta1-adrenergic receptors. They block the sympathetic (fight-or-flight) response and decrease the heart rate (HR). The beta blocker will decrease HR and blood pressure, increasing ventricular filling time. It usually does not have effects on beta2-adrenergic receptor sites. Cardiac output will drop because of decreased HR. DIF: Applying/Application REF: 630 KEY: Beta blocker| medication MSC: IntegratedProcess:NursingProcess:Assessment NOT: Client Needs Category: Physiological Integrity: Pharmacological and Parenteral Therapies

At what blood level is chelation therapy for lead poisoning initiated in a child? a. 10 to 14 g/dl b. 15 to 19 g/dl c. 20 to 44 g/dl d. ≥45 g/dl

ANS: D Chelation therapy is initiated if the child's blood level is greater than or equal to 45 g/dl. At 10 to 14 g/dl, the family should have lead-poisoning education and follow-up level. At 15 to 19 g/dl, the family should have lead-poisoning education and follow-up level but if it persists, initiate environmental investigation. At 20 to 44 g/dl environmental investigation and lead hazard control are necessary.

2. The nurse is caring for a 6-year-old girl who had surgery 12 hours ago. The child tells the nurse that she does not have pain, but a few minutes later she tells her parents that she does. Which should the nurse consider when interpreting this? a. Truthful reporting of pain should occur by this age. b. Inconsistency in pain reporting suggests that pain is not present. c. Children use pain experiences to manipulate their parents. d. Children may be experiencing pain even though they deny it to the nurse.

ANS: D Children may deny pain to the nurse because they fear receiving an injectable analgesic or because they believe they deserve to suffer as a punishment for a misdeed. They may refuse to admit pain to a stranger but readily tell a parent. Truthfully reporting pain and inconsistency in pain reporting suggesting that pain is not present are common fallacies about children and pain. Pain is whatever the experiencing person says it is, whenever the person says it exists. Pain would not be questioned in an adult 12 hours after surgery. DIF: Cognitive Level: Analyze REF: p. 116 TOP: Integrated Process: Nursing Process: Planning MSC: Area of Client Needs: Physiologic Integrity

2. A nurse cares for a client with autosomal dominant polycystic kidney disease (ADPKD). The client asks, "Will my children develop this disease?" How should the nurse respond? a. "No genetic link is known, so your children are not at increased risk." b. "Your sons will develop this disease because it has a sex-linked gene." c. "Only if both you and your spouse are carriers of this disease." d. "Each of your children has a 50% risk of having ADPKD."

ANS: D Children whose parent has the autosomal dominant form of PKD have a 50% chance of inheriting the gene that causes the disease. ADPKD is transmitted as an autosomal dominant trait and therefore is not gender specific. Both parents do not need to have this disorder.

A client with rheumatoid arthritis (RA) is on the postoperative nursing unit after having elective surgery. The client reports that one arm feels like "pins and needles" and that the neck is very painful since returning from surgery. What action by the nurse is best? a. Assist the client to change positions. b. Document the findings in the client's chart. c. Encourage range of motion of the neck. d. Notify the provider immediately.

ANS: D Clients with RA can have cervical joint involvement. This can lead to an emergent situation in which the phrenic nerve is compressed, causing respiratory insufficiency. The client can also suffer a permanent spinal cord injury. The nurse needs to notify the provider immediately. Changing positions and doing range of motion may actually worsen the situation. The nurse should document findings after notifying the provider.

The nurse is caring for a client using a continuous passive motion (CPM) machine and has delegated some tasks to the unlicensed assistive personnel (UAP). What action by the UAP warrants intervention by the nurse? a. Checking to see if the machine is working b. Keeping controls in a secure place on the bed c. Placing padding in the machine per request d. Storing the CPM machine under the bed after removal

ANS: D For infection control (and to avoid tripping on it), the CPM machine is never placed on the floor. The other actions are appropriate.

10. A nurse teaches a client who has a history of heart failure. Which statement should the nurse include in this client's discharge teaching? a. "Avoid drinking more than 3 quarts of liquids each day." b. "Eat six small meals daily instead of three larger meals." c. "When you feel short of breath, take an additional diuretic." d. "Weigh yourself daily while wearing the same amount of clothing."

ANS: D Clients with heart failure are instructed to weigh themselves daily to detect worsening heart failure early, and thus avoid complications. Other signs of worsening heart failure include increasing dyspnea, exercise intolerance, cold symptoms, and nocturia. Fluid overload increases symptoms of heart failure. The client should be taught to eat a heart-healthy diet, balance intake and output to prevent dehydration and overload, and take medications as prescribed. The most important discharge teaching is daily weights as this provides the best data related to fluid retention. DIF: Applying/Application REF: 687 KEY: Heart failure| patient education MSC: Integrated Process: Teaching/Learning NOT: Client Needs Category: Health Promotion and Maintenance

11. A nurse teaches a client who is recovering from a nephrectomy secondary to kidney trauma. Which statement should the nurse include in this client's teaching? a. "Since you only have one kidney, a salt and fluid restriction is required." b. "Your therapy will include hemodialysis while you recover." c. "Medication will be prescribed to control your high blood pressure." d. "You need to avoid participating in contact sports like football."

ANS: D Clients with one kidney need to avoid contact sports because the kidneys are easily injured. The client will not be required to restrict salt and fluids, end up on dialysis, or have new hypertension because of the nephrectomy.

A client who has had numerous children is having her annual examination. The nurse wishes to discuss contraception, but the client is not interested. Which action by the nurse is most appropriate? a. Provide education on the value of spacing children. b. Explain the many alternatives from which to choose. c. Ask the client how her husband feels about so many children. d. Assess the client's religious and cultural background.

ANS: D Cultural and religious backgrounds can have a great deal of influence on clients' attitudes toward sexuality and reproduction. Because the client does not seem interested in the topic, the nurse should gently assess for these background influences and respect them. Providing education that the client does not want is not helpful and is disrespectful. Asking about the husband's preferences diminishes the nurse-client relationship, which should be focused on the client.

16. A client is being discharged on long-term therapy for tuberculosis (TB). What referral by the nurse is most appropriate? a. Community social worker for Meals on Wheels b. Occupational therapy for job retraining c. Physical therapy for homebound therapy services d. Visiting Nurses for directly observed therapy

ANS: D Directly observed therapy is often utilized for managing clients with TB in the community. Meals on Wheels, job retraining, and home therapy may or may not be appropriate.

11. The nurse is having difficulty communicating with a hospitalized 6-year-old child. What technique might be most helpful? a. Suggest that the child keep a diary. b. Suggest that the parent read fairy tales to the child. c. Ask the parent if the child is always uncommunicative. d. Ask the child to draw a picture.

ANS: D Drawing is one of the most valuable forms of communication. Children's drawings tell a great deal about them because they are projections of the child's inner self. It would be difficult for a 6-year-old child who is most likely learning to read to keep a diary. Parents reading fairy tales to the child is a passive activity involving the parent and child. It would not facilitate communication with the nurse. The child is in a stressful situation and is probably uncomfortable with strangers. DIF: Cognitive Level: Apply REF: p. 64 TOP: Integrated Process: Communication and Documentation MSC: Area of Client Needs: Psychosocial Integrity

The nurse suspects that a child has ingested some type of poison. Which clinical manifestation would be most suggestive that the poison was a corrosive product? a. Tinnitus b. Disorientation c. Stupor, lethargy, coma d. Edema of lips, tongue, pharynx

ANS: D Edema of lips, tongue, and pharynx indicates a corrosive ingestion. Tinnitus is indicative of aspirin ingestion. Corrosives do not act on the central nervous system (CNS).

What is the most common cause of death in the adolescent age group? a. Drownings b. Firearms c. Drug overdoses d. Motor vehicles

ANS: D Forty percent of all adolescent deaths in the United States are the result of motor vehicle accidents. Drownings, firearms, and drug overdoses are major concerns in adolescence but are not the most common cause of death. DIF: Cognitive Level: Understand REF: p. 458 TOP: Integrated Process: Nursing Process: Assessment MSC: Area of Client Needs: Safe and Effective Care Environment: Safety and Infection Control

9. A nurse teaches a client who is prescribed digoxin (Lanoxin) therapy. Which statement should the nurse include in this client's teaching? a. "Avoid taking aspirin or aspirin-containing products." b. "Increase your intake of foods that are high in potassium." c. "Hold this medication if your pulse rate is below 80 beats/min." d. "Do not take this medication within 1 hour of taking an antacid."

ANS: D Gastrointestinal absorption of digoxin is erratic. Many medications, especially antacids, interfere with its absorption. Clients are taught to hold their digoxin for bradycardia; a heart rate of 80 beats/min is too high for this cutoff. Potassium and aspirin have no impact on digoxin absorption, nor do these statements decrease complications of digoxin therapy. DIF: Applying/Application REF: 686 KEY: Heart failure| digoxin| medication| patient education MSC: Integrated Process: Nursing Process: Implementation NOT: Client Needs Category: Physiological Integrity: Pharmacological and Parenteral Therapies

A nurse is examining a client reporting right upper quadrant (RUQ) abdominal pain. What technique should the nurse use to assess this client's abdomen? a. Auscultate after palpating. b. Avoid any palpation. c. Palpate the RUQ first. d. Palpate the RUQ last.

ANS: D If pain is present in a certain area of the abdomen, that area should be palpated last to keep the client from tensing up, which could possibly affect the rest of the examination. Auscultation of the abdomen occurs prior to palpation.

Which is an appropriate action when an infant becomes apneic? a. Shake vigorously b. Roll head side to side c. Hold by feet upside down with head supported d. Gently stimulate trunk by patting or rubbing

ANS: D If the infant is apneic, the infant's trunk should be gently stimulated by patting or rubbing. If the infant is prone, turn onto the back. The infant should not be shaken vigorously, the head rolled side to side, or held by the feet upside down with the head supported. These can cause injury.

What is the initial indication of puberty in girls? a. Menarche b. Growth spurt c. Growth of pubic hair d. Breast development

ANS: D In most girls, the initial indication of puberty is the appearance of breast buds, an event known as thelarche. The usual sequence of secondary sex characteristic development in girls is breast changes, rapid increase in height and weight, growth of pubic hair, appearance of axillary hair, menstruation, and abrupt deceleration of linear growth. DIF: Cognitive Level: Understand REF: p. 448 TOP: Integrated Process: Nursing Process: Assessment MSC: Area of Client Needs: Health Promotion and Maintenance

9. A client has been admitted for suspected inhalation anthrax infection. What question by the nurse is most important? a. "Are any family members also ill?" b. "Have you traveled recently?" c. "How long have you been ill?" d. "What is your occupation?"

ANS: D Inhalation anthrax is rare and is an occupational hazard among people who work with animal wool, bone meal, hides, and skin, such as taxidermists and veterinarians. Inhalation anthrax seen in someone without an occupational risk is considered a bioterrorism event and must be reported to authorities immediately. The other questions are appropriate for anyone with an infection.

14. A nurse cares for a client who has a chest tube. When would this client be at highest risk for developing a pneumothorax? a. When the insertion site becomes red and warm to the touch b. When the tube drainage decreases and becomes sanguineous c. When the client experiences pain at the insertion site d. When the tube becomes disconnected from the drainage system

ANS: D Intrathoracic pressures are less than atmospheric pressures; therefore, if the chest tube becomes disconnected from the drainage system, air can be sucked into the pleural space and cause a pneumothorax. A red, warm, and painful insertion site does not increase the client's risk for a pneumothorax. Tube drainage should decrease and become serous as the client heals. Sanguineous drainage is a sign of bleeding but does not increase the client's risk for a pneumothorax. DIF: Applying/Application REF: 578 KEY: Drain| respiratory distress/failure MSC: Integrated Process: Nursing Process: Implementation NOT: Client Needs Category: Physiological Integrity: Reduction of Risk Potential

During examination of a toddler's extremities, the nurse notes that the child is bowlegged. What should the nurse recognize regarding this finding? a. Abnormal and requires further investigation b. Abnormal unless it occurs in conjunction with knock-knee c. Normal if the condition is unilateral or asymmetric d. Normal because the lower back and leg muscles are not yet well developed

ANS: D Lateral bowing of the tibia (bowlegged) is common in toddlers when they begin to walk. It usually persists until all their lower back and leg muscles are well developed. Further evaluation is needed if it persists beyond ages 2 to 3 years, especially in African-American children. DIF: Cognitive Level: Understand REF: p. 108 TOP: Integrated Process: Nursing Process: Problem Identification MSC: Area of Client Needs: Health Promotion and Maintenance

17. The nurse is caring for a client who is prescribed a long-acting beta2 agonist. The client states, "The medication is too expensive to use every day. I only use my inhaler when I have an attack." How should the nurse respond? a. "You are using the inhaler incorrectly. This medication should be taken daily." b. "If you decrease environmental stimuli, it will be okay for you to use the inhaler only for asthma attacks." c. "Tell me more about your fears related to feelings of breathlessness." d. "It is important to use this type of inhaler every day. Let's identify potential community services to help you."

ANS: D Long-acting beta2 agonists should be used every day to prevent asthma attacks. This medication should not be taken when an attack starts. Asthma medications can be expensive. Telling the client that he or she is using the inhaler incorrectly does not address the client's financial situation, which is the main issue here. Clients with limited incomes should be provided with community resources. Asking the client about fears related to breathlessness does not address the client's immediate concerns. DIF: Applying/Application REF: 554 KEY: Case management| medication MSC: Integrated Process: Communication and Documentation NOT: Client Needs Category: Psychosocial Integrity

Although infants may be allergic to a variety of foods, the most common allergens are: a. fruit and eggs. b. fruit, vegetables, and wheat. c. cow's milk and green vegetables. d. eggs, cow's milk, and wheat.

ANS: D Milk products, eggs, and wheat are three of the most common food allergens. Ingestion of these products can cause sensitization and, with subsequent exposure, an allergic reaction. Eggs are a common allergen, but fruit is not. Wheat is a common allergen, but fruit and vegetables are not. Cow's milk is a common allergen, but green vegetables are not.

An older client has returned to the surgical unit after a total hip replacement. The client is confused and restless. What intervention by the nurse is most important to prevent injury? a. Administer mild sedation. b. Keep all four siderails up. c. Restrain the client's hands. d. Use an abduction pillow.

ANS: D Older clients often have trouble metabolizing anesthetics and pain medication, leading to confusion or restlessness postoperatively. To prevent the hip from dislocating, the nurse should use an abduction pillow since the client cannot follow directions at this time. Sedation may worsen the client's mental status and should be avoided. Using all four siderails may be considered a restraint. Hand restraints are not necessary in this situation.

28. A nurse evaluates the following arterial blood gas and vital sign results for a client with chronic obstructive pulmonary disease (COPD): Which action should the nurse take first? a. Administer a short-acting beta2 agonist inhaler. b. Document the findings as normal for a client with COPD. c. Teach the client diaphragmatic breathing techniques. d. Initiate oxygenation therapy to increase saturation to 92%.

ANS: D Oxygen should be administered to a client who is hypoxic even if the client has COPD and is a carbon dioxide retainer. The other interventions do not address the client's hypoxia, which is the priority. DIF: Applying/Application REF: 563 KEY: Oxygen therapy| respiratory distress/failure MSC: Integrated Process: Nursing Process: Implementation NOT: Client Needs Category: Safe and Effective Care Environment: Management of Care MULTIPLE RESPONSE

23. A nurse is caring for a client with acute pericarditis who reports substernal precordial pain that radiates to the left side of the neck. Which nonpharmacologic comfort measure should the nurse implement? a. Apply an ice pack to the client's chest. b. Provide a neck rub, especially on the left side. c. Allow the client to lie in bed with the lights down. d. Sit the client up with a pillow to lean forward on.

ANS: D Pain from acute pericarditis may worsen when the client lays supine. The nurse should position the client in a comfortable position, which usually is upright and leaning slightly forward. Pain is decreased by using gravity to take pressure off the heart muscle. An ice pack and neck rub will not relieve this pain. DIF: Applying/Application REF: 699 KEY: Nonpharmacologic pain management MSC: Integrated Process: Nursing Process: Implementation NOT: Client Needs Category: Physiological Integrity: Basic Care and Comfort

4. Physiologic measurements in children's pain assessment are: a. the best indicator of pain in children of all ages. b. essential to determine whether a child is telling the truth about pain. c. of most value when children also report having pain. d. of limited value as sole indicator of pain.

ANS: D Physiologic manifestations of pain may vary considerably, not providing a consistent measure of pain. Heart rate may increase or decrease. The same signs that may suggest fear, anxiety, or anger also indicate pain. In chronic pain, the body adapts, and these signs decrease or stabilize. Physiologic measurements are of limited value and must be viewed in the context of a pain-rating scale, behavioral assessment, and parental report. When the child states that pain exists, it does. That is the truth. DIF: Cognitive Level: Understand REF: p. 119 TOP: Integrated Process: Nursing Process: Assessment MSC: Area of Client Needs: Physiologic Integrity

A nurse is caring for a client after joint replacement surgery. What action by the nurse is most important to prevent wound infection? a. Assess the client's white blood cell count. b. Culture any drainage from the wound. c. Monitor the client's temperature every 4 hours. d. Use aseptic technique for dressing changes.

ANS: D Preventing surgical wound infection is a primary responsibility of the nurse, who must use aseptic technique to change dressings or empty drains. The other actions do not prevent infection but can lead to early detection of an infection that is already present.

*The nursing care plan for a patient diagnosed with anorexia nervosa includes the intervention "monitor for complications of refeeding." Which system should a nurse closely monitor for dysfunction?* a. Renal b. Endocrine c. Integumentary d. Cardiovascular

ANS: D Refeeding resulting in too-rapid weight gain can overwhelm the heart, resulting in cardiovascular collapse. Focused assessment is a necessity to ensure the patient's physiological integrity. The other body systems are not initially involved in the refeeding syndrome.

22. A student nurse asks for an explanation of "refractory hypoxemia." What answer by the nurse instructor is best? a. "It is chronic hypoxemia that accompanies restrictive airway disease." b. "It is hypoxemia from lung damage due to mechanical ventilation." c. "It is hypoxemia that continues even after the client is weaned from oxygen." d. "It is hypoxemia that persists even with 100% oxygen administration."

ANS: D Refractory hypoxemia is hypoxemia that persists even with the administration of 100% oxygen. It is a cardinal sign of acute respiratory distress syndrome. It does not accompany restrictive airway disease and is not caused by the use of mechanical ventilation or by being weaned from oxygen.

A nurse assesses clients on the medical-surgical unit. Which client should the nurse identify as at high risk for pancreatic cancer? a. A 26-year-old with a body mass index of 21 b. A 33-year-old who frequently eats sushi c. A 48-year-old who often drinks wine d. A 66-year-old who smokes cigarettes

ANS: D Risk factors for pancreatic cancer include obesity, older age, high intake of red meat, and cigarette smoking. Sushi and wine intake are not risk factors for pancreatic cancer.

When palpating the child's cervical lymph nodes, the nurse notes that they are tender, enlarged, and warm. What is the best explanation for this? a. Some form of cancer b. Local scalp infection common in children c. Infection or inflammation distal to the site d. Infection or inflammation close to the site

ANS: D Small nontender nodes are normal. Tender, enlarged, and warm lymph nodes may indicate infection or inflammation close to their location. Tender lymph nodes are not usually indicative of cancer. A scalp infection would usually not cause inflamed lymph nodes. The lymph nodes close to the site of inflammation or infection would be inflamed. DIF: Cognitive Level: Analyze REF: p. 89 TOP: Integrated Process: Nursing Process: Assessment MSC: Area of Client Needs: Health Promotion and Maintenance

4. A client is in the family practice clinic reporting a severe "cold" that started 4 days ago. On examination, the nurse notes the client also has a severe headache and muscle aches. What action by the nurse is best? a. Educate the client on oseltamivir (Tamiflu). b. Facilitate admission to the hospital. c. Instruct the client to have a flu vaccine. d. Teach the client to sneeze in the upper sleeve.

ANS: D Sneezing and coughing into one's sleeve helps prevent the spread of upper respiratory infections. The client does have manifestations of the flu (influenza), but it is too late to start antiviral medications; to be effective, they must be started within 24 to 48 hours of symptom onset. The client does not need hospital admission. The client should be instructed to have a flu vaccination, but now that he or she has the flu, vaccination will have to wait until next year.

14. A client is on mechanical ventilation and the client's spouse wonders why ranitidine (Zantac) is needed since the client "only has lung problems." What response by the nurse is best? a. "It will increase the motility of the gastrointestinal tract." b. "It will keep the gastrointestinal tract functioning normally." c. "It will prepare the gastrointestinal tract for enteral feedings." d. "It will prevent ulcers from the stress of mechanical ventilation."

ANS: D Stress ulcers occur in many clients who are receiving mechanical ventilation, and often prophylactic medications are used to prevent them. Frequently used medications include antacids, histamine blockers, and proton pump inhibitors. Zantac is a histamine blocking agent.

Which assessment finding for a patient diagnosed with an eating disorder meets criteria for hospitalization? a. Urine output 40 mL/hr b. Pulse rate 58 beats/min c. Serum potassium 3.4 mEq/L d. Systolic blood pressure 62 mm Hg

ANS: D Systolic blood pressure less than 70 mm Hg is an indicator for inpatient care. Many people without eating disorders have bradycardia (pulse less than 60 beats/min). Urine output should be more than 30 mL/hr. A potassium level of 3.4 mEq/L is within the normal range.

1. A nurse assesses several clients who have a history of asthma. Which client should the nurse assess first? a. A 66-year-old client with a barrel chest and clubbed fingernails b. A 48-year-old client with an oxygen saturation level of 92% at rest c. A 35-year-old client who has a longer expiratory phase than inspiratory phase d. A 27-year-old client with a heart rate of 120 beats/min

ANS: D Tachycardia can indicate hypoxemia as the body tries to circulate the oxygen that is available. A barrel chest is not an emergency finding. Likewise, a pulse oximetry level of 92% is not considered an acute finding. The expiratory phase is expected to be longer than the inspiratory phase in someone with airflow limitation. DIF: Applying/Application REF: 552 KEY: Respiratory distress/failure| assessment/diagnostic examination MSC: IntegratedProcess:NursingProcess:Assessment NOT: Client Needs Category: Safe and Effective Care Environment: Management of Care

Which vitamin is recommended for all women of childbearing age to reduce the risk of neural tube defects such as spina bifida? a.A b.C c.Niacin d.Folic acid

ANS: D The vitamin supplement that is recommended for all women of childbearing age is a daily dose of 0.4 mg of folic acid. Folic acid taken before conception and during pregnancy can reduce the risk of neural tube defects by 70%. No correlation exists between vitamins A, C, or folic acid and neural tube defects.

The nurse is using the Centers for Disease Control and Prevention (CDC) growth chart for an African-American child. Which statement should the nurse consider? a. This growth chart should not be used. b. Growth patterns of African-American children are the same as for all other ethnic groups. c. A correction factor is necessary when the CDC growth chart is used for non- Caucasian ethnic groups. d. The CDC charts are accurate for US ferer African-American children.

ANS: D The CDC growth charts can serve as reference guides for all racial or ethnic groups. US African-American children were included in the sample population. The growth chart can be used with the perspective that different groups of children have varying normal distributions on the growth curves. No correction factor exists. DIF: Cognitive Level: Understand REF: p. 77 TOP: Integrated Process: Nursing Process: Assessment MSC: Area of Client Needs: Health Promotion and Maintenance

A nurse is preparing to test a school-age child's vision. Which eye chart should the nurse use? a. Denver Eye Screening Test b. Allen picture card test c. Ishihara vision test d. Snellen letter chart

ANS: D The Snellen letter chart, which consists of lines of letters of decreasing size, is the most frequently used test for visual acuity for school-age children. Single cards (Denver—letter E; Allen—pictures) are used for children ages 2 years and older who are unable to use the Snellen letter chart. The Ishihara vision test is used for color vision. DIF: Cognitive Level: Apply REF: p. 92 TOP: Integrated Process: Nursing Process: Assessment MSC: Area of Client Needs: Health Promotion and Maintenance

At what age should the nurse expect the anterior fontanel to close? a. 2 months b. 2 to 4 months c. 6 to 8 months d. 12 to 18 months

ANS: D The anterior fontanel normally closes between ages 12 and 18 months. Two to 8 months is too early. The expected closure of the anterior fontanel occurs between ages 12 and 18 months; if it closes between ages 2 and 8 months, the child should be referred for further evaluation. DIF: Cognitive Level: Remember REF: p. 90 TOP: Integrated Process: Nursing Process: Assessment MSC: Area of Client Needs: Health Promotion and Maintenance

A 14-year-old male mentions that he now has to use deodorant but never had to before. The nurse's response should be based on knowledge that which occurs during puberty? a. Eccrine sweat glands in the axillae become fully functional during puberty. b. Sebaceous glands become extremely active during puberty. c. New deposits of fatty tissue insulate the body and cause increased sweat production. d. Apocrine sweat glands reach secretory capacity during puberty.

ANS: D The apocrine sweat glands, nonfunctional in children, reach secretory capacity during puberty. They secrete a thick substance as a result of emotional stimulation that, when acted on by surface bacteria, becomes highly odoriferous. They are limited in distribution and grow in conjunction with hair follicles, in the axilla, genital, anal, and other areas. Eccrine sweat glands are present almost everywhere on the skin and become fully functional and respond to emotional and thermal stimulation. Sebaceous glands become extremely active at this time, especially those on the genitalia and the "flush" areas of the body such as face, neck, shoulders, upper back, and chest. This increased activity is important in the development of acne. New deposits of fatty tissue is not the etiology of apocrine sweat gland activity. DIF: Cognitive Level: Understand REF: p. 449 TOP: Integrated Process: Teaching/Learning MSC: Area of Client Needs: Health Promotion and Maintenance

15. A nurse cares for a client who has advanced cardiac disease and states, "I am having trouble sleeping at night." How should the nurse respond? a. "I will consult the provider to prescribe a sleep study to determine the problem." b. "You become hypoxic while sleeping; oxygen therapy via nasal cannula will help." c. "A continuous positive airway pressure, or CPAP, breathing mask will help you breathe at night." d. "Use pillows to elevate your head and chest while you are sleeping."

ANS: D The client is experiencing orthopnea (shortness of breath while lying flat). The nurse should teach the client to elevate the head and chest with pillows or sleep in a recliner. A sleep study is not necessary to diagnose this client. Oxygen and CPAP will not help a client with orthopnea. DIF: Understanding/Comprehension REF: 635 KEY: Heart failure| respiratory distress/failure| patient education MSC: IntegratedProcess:Teaching/Learning NOT: Client Needs Category: Physiological Integrity: Basic Care and Comfort

19. A nurse cares for a client with end-stage heart failure who is awaiting a transplant. The client appears depressed and states, "I know a transplant is my last chance, but I don't want to become a vegetable." How should the nurse respond? a. "Would you like to speak with a priest or chaplain?" b. "I will arrange for a psychiatrist to speak with you." c. "Do you want to come off the transplant list?" d. "Would you like information about advance directives?"

ANS: D The client is verbalizing a real concern or fear about negative outcomes of the surgery. This anxiety itself can have a negative effect on the outcome of the surgery because of sympathetic stimulation. The best action is to allow the client to verbalize the concern and work toward a positive outcome without making the client feel as though he or she is crazy. The client needs to feel that he or she has some control over the future. The nurse personally provides care to address the client's concerns instead of pushing the client's issues off on a chaplain or psychiatrist. The nurse should not jump to conclusions and suggest taking the client off the transplant list, which is the best treatment option. DIF: Applying/Application REF: 691 KEY: Transplant| psychosocial response| anxiety MSC: Integrated Process: Nursing Process: Implementation NOT: Client Needs Category: Psychosocial Integrity

A psychiatric clinical nurse specialist uses cognitive-behavioral therapy for a patient diagnosed with anorexia nervosa. Which statement by the staff nurse supports this type of therapy? a. "What are your feelings about not eating foods that you prepare?" b. "You seem to feel much better about yourself when you eat something." c. "It must be difficult to talk about private matters to someone you just met." d. "Being thin doesn't seem to solve your problems. You are thin now but still unhappy."

ANS: D The correct response is the only strategy that attempts to question the patient's distorted thinking.

10. A nurse cares for a client who is recovering after a nephrostomy tube was placed 6 hours ago. The nurse notes drainage in the tube has decreased from 40 mL/hr to 12 mL over the last hour. Which action should the nurse take? a. Document the finding in the client's record. b. Evaluate the tube as working in the hand-off report. c. Clamp the tube in preparation for removing it. d. Assess the client's abdomen and vital signs.

ANS: D The nephrostomy tube should continue to have a consistent amount of drainage. If the drainage slows or stops, it may be obstructed. The nurse must notify the provider, but first should carefully assess the client's abdomen for pain and distention and check vital signs so that this information can be reported as well. The other interventions are not appropriate.

A nurse is preparing to feed a 12-month-old infant with failure to thrive. Which intervention should the nurse implement? a. Provide stimulation during feeding. b. Avoid being persistent during feeding time. c. Limit feeding time to 10 minutes. d. Maintain a face-to-face posture with the infant during feeding.

ANS: D The nurse preparing to feed an infant with failure to thrive should maintain a face-to-face posture with the infant when possible. Encourage eye contact and remain with the infant throughout the meal. Stimulation is not recommended; a quiet, unstimulating atmosphere should be maintained. Persistence during feeding may need to be implemented. Calm perseverance through 10 to 15 minutes of food refusal will eventually diminish negative behavior. Although forced feeding is avoided, "strictly encouraged" feeding is essential. The length of the feeding should be established (usually 30 minutes); limiting the feeding to 10 minutes would make the infant feel rushed.

7. A nurse assesses a client who is recovering from a radical nephrectomy for renal cell carcinoma. The nurse notes that the client's blood pressure has decreased from 134/90 to 100/56 mm Hg and urine output is 20 mL for this past hour. Which action should the nurse take? a. Position the client to lay on the surgical incision. b. Measure the specific gravity of the client's urine. c. Administer intravenous pain medications. d. Assess the rate and quality of the client's pulse.

ANS: D The nurse should first fully assess the client for signs of volume depletion and shock, and then notify the provider. The radical nature of the surgery and the proximity of the surgery to the adrenal gland put the client at risk for hemorrhage and adrenal insufficiency. Hypotension is a clinical manifestation associated with both hemorrhage and adrenal insufficiency. Hypotension is particularly dangerous for the remaining kidney, which must receive adequate perfusion to function effectively. Re-positioning the client, measuring specific gravity, and administering pain medication would not provide data necessary to make an appropriate clinical decision, nor are they appropriate interventions at this time.

A nursing diagnosis for a patient diagnosed with bulimia nervosa is Ineffective coping related to feelings of loneliness as evidenced by overeating to comfort self, followed by self-induced vomiting. The best outcome related to this diagnosis is that within 2 weeks the patient will: a. appropriately express angry feelings. b. verbalize two positive things about self. c. verbalize the importance of eating a balanced diet. d. identify two alternative methods of coping with loneliness.

ANS: D The outcome of identifying alternative coping strategies is most directly related to the diagnosis of Ineffective coping. Verbalizing positive characteristics of self and verbalizing the importance of eating a balanced diet are outcomes that might be used for other nursing diagnoses. Appropriately expressing angry feelings is not measurable.

*A patient being admitted to the eating disorders unit has a yellow cast to the skin and fine, downy hair over the trunk. The patient weighs 70 pounds; height is 5 feet 4 inches. The patient says, "I won't eat until I look thin." Select the priority initial nursing diagnosis.* a. Anxiety related to fear of weight gain b. Disturbed body image related to weight loss c. Ineffective coping related to lack of conflict resolution skills d. Imbalanced nutrition: less than body requirements related to self-starvation

ANS: D The physical assessment shows cachexia, which indicates imbalanced nutrition. Addressing the patient's self-starvation is the priority.

4. A client with a stroke is being evaluated for fibrinolytic therapy. What information from the client or family is most important for the nurse to obtain? a. Loss of bladder control b. Other medical conditions c. Progression of symptoms d. Time of symptom onset

ANS: D The time limit for initiating fibrinolytic therapy for a stroke is 3 to 4.5 hours, so the exact time of symptom onset is the most important information for this client. The other information is not as critical. DIF: Applying/Application REF: 938 KEY: Stroke| neurologic disorders| nursing assessment| fibrinolytic therapy MSC: IntegratedProcess:NursingProcess:Assessment NOT: Client Needs Category: Physiological Integrity: Pharmacological and Parenteral Therapies

11. A nurse is caring for a client on mechanical ventilation. When double-checking the ventilator settings with the respiratory therapist, what should the nurse ensure as a priority? a. The client is able to initiate spontaneous breaths. b. The inspired oxygen has adequate humidification. c. The upper peak airway pressure limit alarm is off. d. The upper peak airway pressure limit alarm is on.

ANS: D The upper peak airway pressure limit alarm will sound when the airway pressure reaches a preset maximum. This is critical to prevent damage to the lungs. Alarms should never be turned off. Initiating spontaneous breathing is important for some modes of ventilation but not others. Adequate humidification is important but does not take priority over preventing injury.

The nurse is helping parents achieve a more nutritionally adequate vegetarian diet for their child. Which is most likely lacking in their particular diet? a. Fat b. Protein c. Vitamins C and A d. Complete protein

ANS: D The vegetarian diet can be extremely healthy, meeting the overall nutrition objectives for Healthy People. Parents should be taught about food preparation to ensure that complete proteins are available for growth. When parents use a strict vegetarian diet, likelihood exists of inadequate protein for growth and calories for energy. Fat and vitamins C and A are readily available from vegetable sources. Plant proteins are available. Foods must be combined to provide complete proteins for growth.

11. A client asks the nurse about drugs for weight loss. What response by the nurse is best? a. All weight-loss drugs can cause suicidal ideation. b. No drugs are currently available for weight loss. c. Only over-the-counter medications are available. d. There are three drugs currently approved for this.

ANS: D There are three drugs available by prescription for weight loss, including orlistat (Xenical), lorcaserin (Belviq), and phentermine-topiramate (Qsymia). Suicidal thoughts are possible with lorcaserin and phentermine-topiramate. Orlistat is also available in a reduced-dose over-the-counter formulation.

An adolescent boy tells the nurse that he has recently had homosexual feelings. What knowledge should the nurse's response be based on? a. This indicates the adolescent is homosexual. b. This indicates the adolescent will become homosexual as an adult. c. The adolescent should be referred for psychotherapy. d. The adolescent should be encouraged to share his feelings and experiences.

ANS: D These adolescents are at increased risk for health-damaging behaviors, not because of the sexual behavior itself, but because of society's reaction to the behavior. The nurse's first priority is to give the young man permission to discuss his feelings about this topic, knowing that the nurse will maintain confidentiality, appreciate his feelings, and remain sensitive to his need to talk about the topic. In recent studies among self-identified gay, lesbian, and bisexual adolescents, many of the adolescents report changing self-labels one or more times during their adolescence. An assessment must be made about any risks to himself or others. If these do not exist, the adolescent needs a supportive person to talk with. DIF: Cognitive Level: Apply REF: p. 449 TOP: Integrated Process: Communication and Documentation MSC: Area of Client Needs: Health Promotion and Maintenance

21. A client is brought to the emergency department after sustaining injuries in a severe car crash. The client's chest wall does not appear to be moving normally with respirations, oxygen saturation is 82%, and the client is cyanotic. What action by the nurse is the priority? a. Administer oxygen and reassess. b. Auscultate the client's lung sounds. c. Facilitate a portable chest x-ray. d. Prepare to assist with intubation.

ANS: D This client has manifestations of flail chest and, with the other signs, needs to be intubated and mechanically ventilated immediately. The nurse does not have time to administer oxygen and wait to reassess, or to listen to lung sounds. A chest x-ray will be taken after the client is intubated.

18. A client in the intensive care unit is scheduled for a lumbar puncture (LP) today. On assessment, the nurse finds the client breathing irregularly with one pupil fixed and dilated. What action by the nurse is best? a. Ensure that informed consent is on the chart. b. Document these findings in the client's record. c. Give the prescribed preprocedure sedation. d. Notify the provider of the findings immediately.

ANS: D This client is exhibiting signs of increased intracranial pressure. The nurse should notify the provider immediately because performing the LP now could lead to herniation. Informed consent is needed for an LP, but this is not the priority. Documentation should be thorough, but again this is not the priority. The preprocedure sedation (or other preprocedure medications) should not be given as the LP will most likely be canceled. DIF: Applying/Application REF: 952 KEY: Neurologic disorders| neurologic assessment| communication MSC: Integrated Process: Communication and Documentation NOT: Client Needs Category: Physiological Integrity: Reduction of Risk Potential

9. A nurse is assessing a client who has a tracheostomy. The nurse notes that the tracheostomy tube is pulsing with the heartbeat as the client's pulse is being taken. No other abnormal findings are noted. What action by the nurse is most appropriate? a. Call the operating room to inform them of a pending emergency case. b. No action is needed at this time; this is a normal finding in some clients. c. Remove the tracheostomy tube; ventilate the client with a bag-valve-mask. d. Stay with the client and have someone else call the provider immediately.

ANS: D This client may have a trachea-innominate artery fistula, which can be a life-threatening emergency if the artery is breached and the client begins to hemorrhage. Since no bleeding is yet present, the nurse stays with the client and asks someone else to notify the provider. If the client begins hemorrhaging, the nurse removes the tracheostomy and applies pressure at the bleeding site. The client will need to be prepared for surgery. DIF: Applying/Application REF: 523 KEY: Tracheostomy| medical emergencies| communication MSC: Integrated Process: Nursing Process: Implementation NOT: Client Needs Category: Physiological Integrity: Physiological Adaptation

A client presents to the family practice clinic reporting a week of watery, somewhat bloody diarrhea. The nurse assists the client to obtain a stool sample. What action by the nurse is most important? a. Ask the client about recent exposure to illness. b. Assess the client's stool for obvious food particles. c. Include the date and time on the specimen container. d. Put on gloves prior to collecting the sample.

ANS: D To avoid possible exposure to infectious agents, the nurse dons gloves prior to handling any bodily secretions. Recent exposure to illness is not related to collecting a stool sample. The nurse can visually inspect the stool for food particles, but it still needs analysis in the laboratory. The container should be dated and timed, but safety for the staff and other clients comes first.

The nurse in the rheumatology clinic is assessing clients with rheumatoid arthritis (RA). Which client should the nurse see first? a. Client taking celecoxib (Celebrex) and ranitidine (Zantac) b. Client taking etanercept (Enbrel) with a red injection site c. Client with a blood glucose of 190 mg/dL who is taking steroids d. Client with a fever and cough who is taking tofacitinib (Xeljanz)

ANS: D Tofacitinib carries a Food and Drug Administration black box warning about opportunistic infections, tuberculosis, and cancer. Fever and cough may indicate tuberculosis. Ranitidine is often taken with celecoxib, which can cause gastrointestinal distress. Redness and itchy rashes are frequently seen with etanercept injections. Steroids are known to raise blood glucose levels.

Which following parameters correlates best with measurements of the body's total protein stores? a. Height b. Weight c. Skinfold thickness d. Upper arm circumference

ANS: D Upper arm circumference is correlated with measurements of total muscle mass. Muscle serves as the body's major protein reserve and is considered an index of the body's protein stores. Height is reflective of past nutritional status. Weight is indicative of current nutritional status. Skinfold thickness is a measurement of the body's fat content. DIF: Cognitive Level: Understand REF: p. 72 TOP: Integrated Process: Nursing Process: Assessment MSC: Area of Client Needs: Health Promotion and Maintenance

A nurse reviews the urinalysis of a client and notes the presence of glucose. Which action should the nurse take? a. Document findings and continue to monitor the client. b. Contact the provider and recommend a 24-hour urine test. c. Review the client's recent dietary selections. d. Perform a capillary artery glucose assessment.

ANS: D Glucose normally is not found in the urine. The normal renal threshold for glucose is about 220 mg/dL, which means that a person whose blood glucose is less than 220 mg/dL will not have glucose in the urine. A positive finding for glucose on urinalysis indicates high blood sugar. The most appropriate action would be to perform a capillary artery glucose assessment. The client needs further evaluation for this abnormal result; therefore, documenting and continuing to monitor is not appropriate. Requesting a 24-hour urine test or reviewing the client's dietary selections will not assist the nurse to make a clinical decision related to this abnormality. DIF: Applying/Application REF: 1348

A nurse cares for a client with a urine specific gravity of 1.040. Which action should the nurse take? a. Obtain a urine culture and sensitivity. b. Place the client on restricted fluids. c. Assess the client's creatinine level. d. Increase the client's fluid intake.

ANS: D Normal specific gravity for urine is 1.005 to 1.030. A high specific gravity can occur with dehydration, decreased kidney blood flow (often because of dehydration), and the presence of antidiuretic hormone. Increasing the client's fluid intake would be a beneficial intervention. Assessing the creatinine or obtaining a urine culture would not provide data necessary for the nurse to make a clinical decision. DIF: Applying/Application REF: 1356

A nurse reviews the urinalysis results of a client and notes a urine osmolality of 1200 mOsm/L. Which action should the nurse take? a. Contact the provider and recommend a low-sodium diet. b. Prepare to administer an intravenous diuretic. c. Obtain a suction device and implement seizure precautions. d. Encourage the client to drink more fluids.

ANS: D Normal urine osmolality ranges from 300 to 900 mOsm/L. This client's urine is more concentrated, indicating dehydration. The nurse should encourage the client to drink more water. Dehydration can be associated with elevated serum sodium levels. Although a low-sodium diet may be appropriate for this client, this diet change will not have a significant impact on urine osmolality. A diuretic would increase urine output and decrease urine osmolality further. Low serum sodium levels, not elevated serum levels, place the client at risk for seizure activity. These options would further contribute to the client's dehydration or elevate the osmolality. DIF: Applying/Application REF: 1359

A nurse cares for a client who is having trouble voiding. The client states, "I cannot urinate in public places." How should the nurse respond? a. "I will turn on the faucet in the bathroom to help stimulate your urination." b. "I can recommend a prescription for a diuretic to improve your urine output." c. "I'll move you to a room with a private bathroom to increase your comfort." d. "I will close the curtain to provide you with as much privacy as possible."

ANS: D The nurse should provide privacy to clients who may be uncomfortable or have issues related to elimination or the urogenital area. Turning on the faucet and administering a diuretic will not address the client's concern. Although moving the client to a private room with a private bathroom would be nice, this is not realistic. The nurse needs to provide as much privacy as possible within the client's current room. DIF: Applying/Application REF: 1363

A nursing student is caring for a client with an abdominal aneurysm. What action by the student requires the registered nurse to intervene? a. Assesses the client for back pain b. Auscultates over abdominal bruit c. Measures the abdominal girth d. Palpates the abdomen in four quadrants

ANS: D Abdominal aneurysms should never be palpated as this increases the risk of rupture. The registered nurse should intervene when the student attempts to do this. The other actions are appropriate. DIF: Applying/Application REF: 726

An older client with peripheral vascular disease (PVD) is explaining the daily foot care regimen to the family practice clinic nurse. What statement by the client may indicate a barrier to proper foot care? a. "I nearly always wear comfy sweatpants and house shoes." b. "I'm glad I get energy assistance so my house isn't so cold." c. "My daughter makes sure I have plenty of lotion for my feet." d. "My hands shake when I try to do things requiring coordination."

ANS: D Clients with PVD need to pay special attention to their feet. Toenails need to be kept short and cut straight across. The client whose hands shake may cause injury when trimming toenails. The nurse should refer this client to a podiatrist. Comfy sweatpants and house shoes are generally loose and not restrictive, which is important for clients with PVD. Keeping the house at a comfortable temperature makes it less likely the client will use alternative heat sources, such as heating pads, to stay warm. The client should keep the feet moist and soft with lotion. DIF: Analyzing/Analysis REF: 725

A student nurse is assessing the peripheral vascular system of an older adult. What action by the student would cause the faculty member to intervene? a. Assessing blood pressure in both upper extremities b. Auscultating the carotid arteries for any bruits c. Classifying capillary refill of 4 seconds as normal d. Palpating both carotid arteries at the same time

ANS: D The student should not compress both carotid arteries at the same time to avoid brain ischemia. Blood pressure should be taken and compared in both arms. Prolonged capillary refill is considered to be greater than 5 seconds in an older adult, so classifying refill of 4 seconds as normal would not require intervention. Bruits should be auscultated. DIF: Remembering/Knowledge REF: 707

A client has a possible connective tissue disease and the nurse is reviewing the client's laboratory values. Which laboratory values and their related connective tissue diseases (CTDs) are correctly matched? (Select all that apply.) a. Elevated antinuclear antibody (ANA) - Normal value; no connective tissue disease b. Elevated sedimentation rate - Rheumatoid arthritis c. Lowered albumin - Indicative only of nutritional deficit d. Positive human leukocyte antigen B27 (HLA-B27) - Reiter's syndrome or ankylosing spondylitis e. Positive rheumatoid factor - Possible kidney disease

ANS: D, E The HLA-B27 is diagnostic for Reiter's syndrome or ankylosing spondylitis. A positive rheumatoid factor can be seen in autoimmune CTDs, kidney and liver disease, or leukemia. An elevated ANA is indicative of inflammatory CTDs, although a small minority of healthy adults also have this finding. An elevated sedimentation rate indicates inflammation, whether from an infection, an injury, or an autoimmune CTD. Lowered albumin is seen in nutritional deficiencies but also in chronic infection or inflammation.

A nurse assesses an older adult's skin. Which findings require immediate referral? (Select all that apply.) a. Excessive moisture under axilla b. Increased hair thinning c. Increased presence of fungal toenails d. Lesion with various colors e. Spider veins on legs f. Asymmetric 6-mm dark lesion on forehead

ANS: D, F The lesion with various colors, as well as the asymmetric 6-mm dark lesion, fits two of the American Cancer Society's hallmark signs for cancer according to the ABCD method. Other manifestations are variants of normal seen in various age groups.

b. "I will avoid sources of strong electromagnetic fields."

After teaching a client who has an implantable cardioverter-defibrillator (ICD), a nurse assesses the client's understanding. Which statement by the client indicates a correct understanding of the teaching? a. "I should wear a snug-fitting shirt over the ICD." b. "I will avoid sources of strong electromagnetic fields." c. "I should participate in a strenuous exercise program." d. "Now I can discontinue my antidysrhythmic medication."

The unit secretary tells the nurse that the practitioner has just ordered a low-calorie diet for a patient who is overweight. Place these nursing interventions in the order in which they should be implemented. 1. Verify the dietary order 2. Determine food preferences 3. Teach specifics about a low-calorie diet 4. Review a meal plan designed by the patient 5. Assess the patient's motivation to follow the diet

Answer: 1, 5, 2, 3, 4 1. This should be done first because a diet requires a practitioner's order; following a specific diet is a dependent function of the nurse. 5. Assessing motivation is one of the most important factors influencing learning. The learner must recognize that the need exists and that the need will be addressed through the learning. 2. Determining food preferences is part of nursing assessment. Food preferences can then be included in the teaching plan about the low-calorie diet. 3. Details of the diet can be taught after the order is verified, motivation is determined, and preferences identified. 4. Evaluation is the final step of teaching. A meal plan designed by the patient requires not just an understanding of the information but an ability to apply the information.

Which is probably the most important criterion on which to base the decision to report suspected child abuse? a. Inappropriate parental concern for the degree of injury b. Absence of parents for questioning about child's injuries c. Inappropriate response of child d. Incompatibility between the history and injury observed

Answer: D Conflicting stories about the "accident" are the most indicative red flags of abuse. Inappropriate response of caregiver or child may be present, but is subjective. Parents should be questioned at some point during the investigation.

The client receives multiple antibiotics to treat a serious infection. What will the priority assessment of the client by the nurse include?

Assessing changes in stool, white patches in the mouth, and urogenital itching or rash

The nurse is caring for a hospitalized 4-year-old boy, Ryan. His parents tell the nurse that they will be back to visit at 6 PM. When Ryan asks the nurse when his parents are coming, the nurse's best response is: a. "They will be here soon." b. "They will come after dinner." c. "Let me show you on the clock when 6 PM is." d. "I will tell you every time I see you how much longer it will be."

B A 4-year-old understands time in relation to events such as meals. Children perceive "soon" as a very short time. The nurse may lose the child's trust if his parents do not return in the time he perceives as "soon." Children cannot read or use a clock for practical purposes until age 7 years. This answer assumes that the child understands the concept of hours and minutes, which is not developed until age 5 or 6 years.

The parent of a 4-year-old son tells the nurse that the child believes "monsters and the boogeyman" are in his bedroom at night. The nurse's best suggestion for coping with this problem is to: a. Insist that the child sleep with his parents until the fearful phase passes. b. Suggest involving the child to find a practical solution such as a night-light. c. Help the child understand that these fears are illogical. d. Tell the child frequently that monsters and the boogeyman do not exist.

B A night-light shows a child that imaginary creatures do not lurk in the darkness. Letting the child sleep with parents or telling the child that these creatures do not exist will not get rid of the fears. A 4-year-old is in the preconceptual age and cannot understand logical thought.

In terms of language and cognitive development, a 4-year-old child would be expected to: a. Think in abstract terms. b. Follow simple commands. c. Understand conservation of matter. d. Comprehend another person's perspective.

B Children ages 3 to 4 years can give and follow simple commands. Children cannot think abstractly at age 4 years. Conservation of matter is a developmental task of the school-age child. A 4-year-old child cannot comprehend another's perspective.

What is descriptive of the preschooler's understanding of time? a. Has no understanding of time b. Associates time with events c. Can tell time on a clock d. Uses terms like "yesterday" appropriately

B In a preschooler's understanding, time has a relation with events such as, "We'll go outside after lunch." Preschoolers develop an abstract sense of time at age 3 years. Children can tell time on a clock at age 7 years. Children do not fully understand use of time-oriented words until age 6 years.

A 4-year-old boy is hospitalized with a serious bacterial infection. He tells the nurse that he is sick because he was "bad." The nurse's best interpretation of this comment is that it is: a. A sign of stress. b. Common at this age. c. Suggestive of maladaptation. d. Suggestive of excessive discipline at home.

B Preschoolers cannot understand the cause and effect of illness. Their egocentrism makes them think that they are directly responsible for events, making them feel guilt for things outside of their control. Children of this age show stress by regressing developmentally or acting out. Maladaptation is unlikely. This comment does not imply excessive discipline at home.

Which statement, made by a 4-year-old child's father, is true about the care of the preschooler's teeth? a. "Because the 'baby teeth' are not permanent, they are not important to the child." b. "My son can be encouraged to brush his teeth after I have thoroughly cleaned his teeth." c. "My son's 'permanent teeth' will begin to come in at 4 to 5 years of age." d. "Fluoride supplements can be discontinued when my son's 'permanent teeth' erupt."

B Toddlers and preschoolers lack the manual dexterity to remove plaque adequately, so parents must assume this responsibility. Deciduous teeth are important because they maintain spacing and play an important role in the growth and development of the jaws and face and in speech development. Secondary teeth erupt at about 6 years of age. If the family does not live in an area where fluoride is included in the water supply, fluoride supplements should be continued.

Parents tell the nurse that they found their 3-year-old daughter and a male cousin of the same age inspecting each other closely as they used the bathroom. Which is the most appropriate recommendation the nurse should make? a. Punish children so this behavior stops. b. Neither condone nor condemn the curiosity. c. Allow children unrestricted permission to satisfy this curiosity. d. Get counseling for this unusual and dangerous behavior.

B Three-year-olds become aware of anatomic differences and are concerned about how the other "works." Such exploration should not be condoned or condemned. Children should not be punished for this normal exploration. Encouraging the children to ask questions of the parents and redirecting their activity are more appropriate than giving permission. Exploration is age-appropriate and not dangerous behavior.

A 4-year-old female child sometimes wakes her parents up at night screaming, thrashing, sweating, and apparently frightened. Yet she is not aware of her parents' presence when they check on her. She lies down and sleeps without any parental intervention. This is MOST likely described as: A. a nightmare. B. sleep terror. C. seizure activity. D. sleep apnea.

B. sleep terror Nightmares are associated with difficulty returning to sleep as opposed to sleep terrors where the individual easily goes back to sleep. With the advent of a nightmare, the child has a memory of the dream like state, is comforted by traditional methods of contact and thrashing type behaviors cease upon awakening. In sleep terrors, the child has no memory of the event, continues thrashing behaviors when awaken, and is not comforted by traditional methods of contact.

A mother tells the nurse that her daughter's favorite toy is a large, empty box that contained a stove. She plays "house" in it with her toddler brother. Based on the nurse's knowledge of growth and development, the nurse recognizes that this is: A. unsafe play that should be discouraged. B. creative play that should be encouraged. C. suggestive of limited family resources. D. suggestive of limited adult supervision.

B. creative play that should be encouraged This type of play should be encouraged. After children create something new, they can then transfer it to other situations. There should be some supervision to prevent injury or accidents. As long as the play is supervised, it should be encouraged. This is not considered unsafe play. There is no indication of limited resources. There is no indication of limited adult supervision.

When preparing parents to teach their preschool child about human sexuality, the nurse should emphasize that: A. a parent's words may have a greater influence on the child's understanding than the parent's actions. B. parents should determine exactly what the child wants to know before answering a question about sex. C. parents should avoid using correct anatomic terms because they are confusing to the preschooler. D. parents should allow children to satisfy their sexual curiosity by playing "doctor."

B. parents should determine exactly what the child wants to know before answering a question about sex. It is important that the parent answer the question that the child is asking. Actions may have a greater influence because language is not fully developed in the preschool years. Using correct terminology lays the foundation for later discussion of human sexuality. Parents should encourage children to ask questions to provide accurate information at their cognitive level.

During a well-child visit, the father of a 4-year-old boy tells the nurse that he is not sure if his son is ready for kindergarten. His birthday is close to the cutoff date, and he has not attended preschool. The nurse's BEST recommendation is to: A. start kindergarten. B. perform developmental screening. C. observe a kindergarten class. D. postpone kindergarten and go to preschool.

B. perform developmental screening A developmental screening will provide the necessary information to help the family determine readiness. Encouraging the father to have the child start kindergarten does not address the father's concern about readiness and suggests that his concerns are not warranted. Recommending to the father that he postpone kindergarten and send the child to preschool assumes that the child is not ready for kindergarten, but the recommendation is not based on any data or facts. Recommending to the father that he simply place his child in preschool may lead to the child's boredom with school. Having the father observe a kindergarten class and then decide if the child would enjoy the experience will provide information about kindergarten but not about whether his child is ready to begin and thrive there.

The nurse's BEST approach for effective communication with a preschool age child is through: A. speech. B. play. C. drawing. D. actions.

B. play Preschoolers' most effective means of communication is through play. Play allows preschoolers to understand, adjust to, and work out life's experiences through their imagination and ability to invent and imitate. Speech is not effective, because preschoolers assume that everyone thinks as they do and that a brief explanation of their thinking makes them understood by others, which is often not true. Also, preschoolers often do not understand the meaning of words and often take statements literally. Drawing is still being developed as a fine motor skill; therefore, it is not the most effective means of communication. Actions are not an appropriate means of communication for a preschooler.

A 4-year-old boy has been having increasingly more frequent angry outbursts in preschool. He is very aggressive toward the other children and the teachers. This behavior has been a problem for approximately 8 to 10 weeks. His parent asks the nurse for advice. The MOST appropriate intervention is to: A. explain that this is normal in preschoolers, especially boys. B. refer the child for professional help. C. talk to the preschool teacher to obtain validation for the behavior the parent reports. D. encourage the parent to try more consistent and firm discipline.

B. refer the child for professional help. This is not expected behavior. The child should be referred to a competent professional to deal with his aggression so that an accurate assessment can be made and a care plan determined. Outward aggression to others is not normal behavior and should be evaluated. The validation will be helpful for the referral, but the referral is the priority action. This may be recommended by the professional once an accurate assessment is made.

In providing anticipatory guidance to parents whose child will soon be entering kindergarten, which is a critical factor in preparing a child for kindergarten entry? a. The child's ability to sit still b. The child's sense of learned helplessness c. The parent's interactions and responsiveness to the child d. Attending a preschool program

C Interactions between the parent and child are an important factor in the development of academic competence. Parental encouragement and support maximize a child's potential. The child's ability to sit still is important to learning; however, parental responsiveness and involvement are more important factors. Learned helplessness is the result of a child feeling that he or she has no effect on the environment and that his or her actions do not matter. Parents who are actively involved in a supportive learning environment will demonstrate a more positive approach to learning. Preschool and day care programs can supplement the developmental opportunities provided by parents at home, but they are not critical in preparing a child for entering kindergarten.

The nurse is performing an assessment on a child and notes the presence of Koplik's spots. In which communicable disease are Koplik's spots present? a. Rubella b. Chickenpox (varicella) c. Measles (rubeola) d. Exanthema subitum (roseola)

C Koplik's spots are small, irregular red spots with a minute, bluish white center found on the buccal mucosa 2 days before systemic rash. Koplik's spots are not present with rubella, varicella, or roseola.

Imaginary playmates are beneficial to the preschool child because they: a. Take the place of social interactions. b. Take the place of pets and other toys. c. Become friends in times of loneliness. d. Accomplish what the child has already successfully accomplished.

C One purpose of an imaginary friend is to be a friend in time of loneliness. Imaginary friends do not take the place of social interactions but may encourage conversation. Imaginary friends do not take the place of pets or toys. They accomplish what the child is still attempting, not what has already been accomplished.

A 4-year-old child tells the nurse that she does not want another blood sample drawn because "I need all my insides, and I don't want anyone taking them out." Which is the nurse's best interpretation of this? a. Child is being overly dramatic. b. Child has a disturbed body image. c. Preschoolers have poorly defined body boundaries. d. Preschoolers normally have a good understanding of their bodies.

C Preschoolers have little understanding of body boundaries, which leads to fears of mutilation. The child is not capable of being dramatic at 4 years of age. She truly has fear. Body image is just developing in the school-age child. Preschoolers do not have good understanding of their bodies.

When is a child with chickenpox considered to be no longer contagious? a. When fever is absent b. 24 hours after lesions erupt c. When lesions are crusted d. 8 days after onset of illness

C When the lesions are crusted, the chickenpox is no longer contagious. This may be a week after onset of disease. The child is still contagious once the fever has subsided and after the lesions erupt, and may or may not be contagious any time after 6 days as long as all lesions are crusted over.

Acyclovir (Zovirax) is given to children with chickenpox to: a. Minimize scarring. b. Prevent aplastic anemia. c. Decrease the number of lesions. d. Prevent spread of the disease.

C Acyclovir decreases the number of lesions, shortens duration of fever, and decreases itching, lethargy, and anorexia; however, it does not prevent scarring. Preventing aplastic anemia is not a function of acyclovir. Only quarantine of the infected child can prevent the spread of disease.

A normal characteristic of the language development of a preschool-age child is: a. Lisp. b. Echolalia. c. Stammering. d. Repetition without meaning.

C Stammering and stuttering are normal dysfluencies in preschool-age children. Lisps are not a normal characteristic of language development. Echolalia and repetition are traits of toddlers' language.

The nurse is giving anticipatory guidance to the parent of a 5-year-old. In this guidance, it is MOST important to: A. prepare the parent for increased aggression. B. encourage the parent to offer the child choices. C. inform the parent to expect a more tranquil period at this age. D. advise parents that this is the age when stuttering may develop.

C. inform the parent to expect a more tranquil period at this age. The end of preschool and the beginning of school age is a more tranquil period. Preparing the parent for increased aggression is anticipatory guidance for 4-year-old children. Encouraging the parent to offer the child choices is anticipatory guidance for 3-year-old children. Advising the parent that this is the age when stuttering may develop is anticipatory guidance for 3-year-old children.

The parents of a 4-year-old girl are worried because she has an imaginary playmate. The nurse's BEST response is to tell the parents: A. a psychosocial evaluation is indicated. B. an evaluation of possible parent-child conflict is indicated. C. having imaginary playmates is normal and useful at this age. D. having imaginary playmates is abnormal after about age 2 years.

C. having imaginary playmates is normal and useful at this age. Imaginary playmates are a part of normal development at this age and serve many purposes, including being a friend in times of loneliness, accomplishing what the preschooler is still attempting, and experiencing what the preschooler wants to forget or remember. Because an imaginary playmate is part of normal development, a psychosocial evaluation is not warranted. Because an imaginary playmate is part of normal development, an evaluation of the parent-child relationship is not warranted. Imaginary playmates are commonly present during the preschool years; therefore, they are not abnormal after the age of 2 years.

According to Erikson, the primary psychosocial task of the preschool period is developing a sense of: A. identity. B. intimacy. C. initiative. D. industry.

C. initiative Preschoolers focus on developing initiative. The stage is known as initiative versus guilt. Identity versus role confusion is associated with adolescence. Intimacy versus isolation is associated with young adulthood. Industry versus inferiority is associated with the school-aged child.

When caring for a preschool age child, the nurse should incorporate knowledge that body image has developed to include: A. a well-defined body boundary. B. knowledge about his or her internal anatomy. C. fear of intrusive procedures. D. anxiety and fear of separation.

C. fear of intrusive procedures. Preschoolers fear that their insides will come out with intrusive procedures. Preschoolers have poorly defined body images. Preschoolers have little or no knowledge of their internal anatomy. The fear of looking different is a concept that occurs in later school-aged children and adolescents.

Which tool measures body fat most accurately? a. Stadiometer b. Calipers c. Cloth tape measure d. Paper or metal tape measure

Calipers are used to measure skinfold thickness, which is an indicator of body fat content. Stadiometers are used to measure height. Cloth tape measures should not be used because they can stretch. Paper or metal tape measures can be used for recumbent lengths and other body measurements that must be made. DIF: Cognitive Level: Understand REF: p. 80 TOP: Integrated Process: Nursing Process: Assessment MSC: Area of Client Needs: Health Promotion and Maintenance

Which characteristic best describes the language of a 3-year-old child? a. Asks meanings of words b. Follows directional commands c. Can describe an object according to its composition d. Talks incessantly, regardless of whether anyone is listening

D Because of the dramatic vocabulary increase at this age, 3-year-olds are known to talk incessantly, regardless of whether anyone is listening. A 4- to 5-year-old asks lots of questions and can follow simple directional commands. A 6-year-old can describe an object according to its composition.

The nurse is guiding parents in selecting a day care facility for their child. When making the selection, it is especially important to consider: a. Structured learning environment. b. Socioeconomic status of children. c. Cultural similarities of children. d. Teachers knowledgeable about development.

D A teacher knowledgeable about development will structure activities for learning. A structured learning environment is not necessary at this age. Socioeconomic status is not the most important factor in selecting a preschool. Preschool is about expanding experiences with others; cultural similarities are not necessary.

By what age would the nurse expect that most children could understand prepositional phrases such as "under," "on top of," "beside," and "in back of"? a. 18 months b. 3 years c. 24 months d. 4 years

D At 4 years, children can understand directional phrases. Children 18 to 24 months and 3 years of age are too young.

Which is probably the most important criterion on which to base the decision to report suspected child abuse? a. Inappropriate parental concern for the degree of injury b. Absence of parents for questioning about child's injuries c. Inappropriate response of child d. Incompatibility between the history and injury observed

D Conflicting stories about the "accident" are the most indicative red flags of abuse. Inappropriate response of caregiver or child may be present, but is subjective. Parents should be questioned at some point during the investigation.

Which is the causative agent of scarlet fever? a. Enteroviruses b. Corynebacterium organisms c. Scarlet fever virus d. Group A -hemolytic streptococci (GABHS)

D GABHS infection causes scarlet fever. Enteroviruses do not cause the same complications. Corynebacterium organisms cause diphtheria. Scarlet fever is not caused by a virus.

In terms of fine motor development, what could the 3-year-old child be expected to do? a. Tie shoelaces. b. Use scissors or a pencil very well. c. Draw a person with seven to nine parts. d. Copy (draw) a circle.

D Three-year-olds are able to accomplish the fine motor skill of drawing a circle. Tying shoelaces, using scissors or a pencil very well, and drawing a person with multiple parts are fine motor skills of 5-year-old children.

Which accomplishment would the nurse expect of a healthy 3-year-old child? a. Jump rope b. Ride a two-wheel bicycle c. Skip on alternate feet d. Balance on one foot for a few seconds

D Three-year-olds are able to accomplish the gross motor skill of balancing on one foot. Jumping rope, riding a two-wheel bike, and skipping on alternate feet are gross motor skills of 5-year-old children.

Which medication may be given to high risk children after exposure to chickenpox to prevent varicella? a. Acyclovir b. Vitamin A c. Diphenhydramine hydrochloride d. Varicella zoster immune globulin (VZIG)

D VZIG is given to high risk children to help prevent the development of chickenpox. Immune globulin intravenous may also be recommended. Acyclovir is given to immunocompromised children to reduce the severity of symptoms. Vitamin A reduces morbidity and mortality associated with the measles. The antihistamine diphenhydramine is administered to reduce the itching associated with chickenpox.

What do nurses teach?

Disease information Information about medications Procedures/psychomotor skills Disease prevention and health promotion Clinical processes

A 4-year-old female child is afraid of dogs. What should the nurse recommend to her parents to help her with this fear? A. Keep her away from dogs B. Buy her a stuffed dog toy C. Force her to touch a dog briefly D. Let her watch other children play with a dog

D. Let her watch other children play with a dog The parents should actively seek ways to deal with fear. By observing other children at play with dogs, the child can adapt. Keeping their child away from dogs avoids the object of fear rather than addressing the fear and finding solutions. Buying a child a stuffed dog toy avoids the object of fear rather than addressing the fear and finding solutions. Forcing the child to touch a dog without working up to it may increase the level of fear.

The nurse in a long-term care facility is teaching a group of residents about increasing dietary fiber. Which foods should she explain are high in fiber? 1) White bread, pasta, and white rice 2) Oranges, raisins, and strawberries 3) Whole milk, eggs, and bacon 4) Peaches, orange juice and bananas

Oranges, raisins, and strawberries Oranges, raisins, and strawberries are high in fiber. White bread, pasta, and white rice are carbohydrates. Whole milk, eggs, and bacon are high in cholesterol. Peaches, orange juice, and bananas are sources of potassium.

A nurse cares for a client who is recovering from an open Whipple procedure. Which action should the nurse take? a. Clamp the nasogastric tube. b. Place the client in semi-Fowler's position. c. Assess vital signs once every shift. d. Provide oral rehydration.

Postoperative care for a client recovering from an open Whipple procedure should include placing the client in a semi-Fowler's position to reduce tension on the suture line and anastomosis sites, setting the nasogastric tube to low suction to remove free air buildup and pressure, assessing vital signs frequently to assess fluid and electrolyte complications, and providing intravenous fluids.

For a patient with a newly fractured pelvis, not yet in a cast, which of the following actions is appropriate when placing the patient on a bedpan? 1) Place the patient in semi-Fowler's position to defecate. 2) Ask the patient to push up with his feet to lift his hips while you place the bedpan. 3) Place a fracture pan under the buttocks, small end toward the feet. 4) Raise the siderail on the opposite side from where you are working.

Raise the siderail on the opposite side from where you are working. The nurse should always raise the siderail on the opposite side from where he is working to protect the patient from falls. Placing the patient in semi-Fowler's position or asking the patient to push up with his feet would cause pain and possible dislocation of the fracture. A fracture pan should be used, but the small large end is pointed toward the feet.

What should be closely monitored by the healthcare provider for a patient taking cyclosporine

Renal Function

What are my Teaching Responsibilities?

Teaching + MAJOR part of clinical practice skills Independent Nursing Function American Nurses Association (ANA) states nurse are RESPONSIBLE for promoting and protecting health. The Joint Commission: sets accreditation standards on considerations when conducting education American Hospital Association (AHA) Patient Care Partnership (Former Patients Bill of Rights) - simple language to describe the patients right to receive high-quality care.

A client's medical record reveals the diagnosis of tinea unguium. The nurse would assess which body part for this disorder?

The nails

c. "Clients who use cocaine are at risk for fatal dysrhythmias."

The nurse asks a client who has experienced ventricular dysrhythmias about substance abuse. The client asks, "Why do you want to know if I use cocaine?" How should the nurse respond? a. "Substance abuse puts clients at risk for many health issues." b. "The hospital requires that I ask you about cocaine use." c. "Clients who use cocaine are at risk for fatal dysrhythmias." d. "We can provide services for cessation of substance abuse."

b. Initiate cardiopulmonary resuscitation (CPR). v fib, immediate defibrillation

The nurse is caring for a client on the medical-surgical unit who suddenly becomes unresponsive and has no pulse. The cardiac monitor shows the rhythm below: After calling for assistance and a defibrillator, which action should the nurse take next? a. Perform a pericardial thump. b. Initiate cardiopulmonary resuscitation (CPR). c. Start an 18-gauge intravenous line. d. Ask the client's family about code status.

Which of the following goals is appropriate for a patient with a nursing diagnosis of Constipation? The patient increases the intake of: 1) milk and cheese. 2) bread and pasta. 3) fruits and vegetables. 4) lean meats.

The nurse should encourage the patient to increase his intake of foods rich in fiber because they promote peristalsis and defecation, thereby relieving constipation. Low-fiber foods, such as bread, pasta, and other simple carbohydrates, as well as milk, cheese, and lean meat, slow peristalsis.

a patient asks why it is essential that HAART meds be taken everyday at the same time. what is the nurses best response? a. missing or delaying doses of these drugs decreases blood conenctrations needed to inhibit viral replication b. missing or delayed doses of these drugs decreases the risk of developing infections c. missing or delaying doses of these drugs decreases the effectiveness missing or delaying doses can decrease the risk of developing HIV resistant mutations

a

an IV drug user who regularly shares needles is in the ER. what information does the nurse provide to decrease he patients risk of HIV through shared needles after each use? A. fill and flush syringe with clear water, fill with bleach and shake for 30-60 seconds and rinse with clear water B. fill and flush with water then soap and hot water, shake for 2 minutes and flush with cold water C. rinse needles with bleach and water solution and allow to air dry D. rinse needles after each use with rubbing alcohol and water, then rinse with water

a

which point are you sure to include when teaching a new RN to prevent HIV transmission from patients? A. wear gloves when in contact with patients mucous membrane or non-intact skin B. be sure to wear protective gear when providing any care to HIV positive patients C. always war a mask D. use PEP whether a patient is positive or not

a

13. A 65-year-old woman calls the clinic for an appointment stating that she has developed weakness, fatigue, and nausea over the past 2 weeks. The nurse should A have her come in immediately or go to the local emergency department. B make an appointment for later in the day. C make an appointment within 1 week. D discuss with her the need for referral to a hematologist.

a Recognition of coronary artery disease in women is important because they are more likely to die from a myocardial infarction (MI) than men. MIs tend to present with atypical vague symptoms in women that can delay recognition and treatment. Women may report having some vague symptoms that signal an imminent acute MI, such as fatigue, for several weeks before seeking care. Other symptoms may include nausea, vomiting, sweating, and dizziness.

18. A woman with premenstrual syndrome (PMS) may benefit from which of the following management techniques? A Relaxation therapy B Increasing caffeine C High-protein diets D One alcoholic drink at night during the luteal phase

a Relaxation therapy has shown benefits for women with more severe PMS symptoms. Avoiding alcohol and caffeine can help, and carbohydrate-rich foods can also help relieve PMS symptoms.

A client just returned to the surgical unit after a gastric bypass. What action by the nurse is the priority? a. Assess the client's pain. b. Check the surgical incision. c. Ensure an adequate airway. d. Program the morphine pump.

c. Ensure an adequate airway.

23. The simple procedure a nurse in a gynecologic clinic can do to assist patients with early detection of osteoporosis is to A measure the height of the patient at each annual appointment. B assess the spinal column for changes at each annual appointment. C recommend that the patient have serum calcium levels checked twice a year. D recommend that the patient have a bone mineral analysis done once a year.

a The first noticeable evidence of bone mass loss is the loss of height. Later signs include the dowager's hump on the spinal column. Serum calcium levels will not assist in determining osteoporosis. A bone mineral analysis is done to diagnose osteoporosis.

17. An 18-year-old has been diagnosed with primary dysmenorrhea. Prostaglandin inhibitors have been prescribed. The nurse recognizes that teaching concerning the use of prostaglandin inhibitors has been effective when the woman states: A "I will take ibuprofen as soon as my period starts and will continue around the clock for about 2 or 3 days." B "I will take ibuprofen once a day starting 2 days before my period should begin." C "I will take naproxen once a day every day." D "I will take naproxen around the clock every day that I am bleeding."

a The most effective prostaglandin inhibitors are nonsteroidal antiinflammatory drugs (NSAIDs) such as ibuprofen and naproxen. To be effective, the NSAID should be taken around the clock for at least 48 to 72 hours, beginning when menstrual flow starts.

What is personal space?

a zone that individuals maintain around themselves in most casual social situations

A client has undergone transurethral resection of the prostate (TURP). Which interventions does the nurse incorporate in this client's postoperative care? (Select all that apply.) a) Administer antispasmodic medications. b) Encourage the client to urinate around the catheter if pressure is felt. c) Perform intermittent urinary catheterization every 4 to 6 hours. d) Place the client in a supine position with his knees flexed. e) Assist the client to mobilize as soon as permitted. (Chp 72, elsevier resources)

a) Administer antispasmodic medications. e) Assist the client to mobilize as soon as permitted. Antispasmodic drugs can be administered to decrease the bladder spasms that may occur due to catheter use. Assisting the client to a chair as soon as permitted postoperatively will help to decrease the risk of complications from immobility. An indwelling catheter and continuous bladder irrigation are in place for about 24 hours after TURP. The client should not try to void around the catheter, which causes the bladder muscles to contract and may result in painful spasms. Intermittent urinary catheterization is not necessary. Typically, the catheter is taped to the client's thigh, so he should keep his leg straight. (Chp 72, elsevier resources)

After returning from transurethral resection of the prostate, the client's urine in the continuous bladder irrigation system is a burgundy color. Which client needs does the nurse anticipate after the surgeon sees the client? (Select all that apply.) a) Antispasmodic drugs b) Emergency surgery c) Forced fluids d) Increased intermittent irrigation e) Monitoring for anemia (Chp 72, elsevier resources)

a) Antispasmodic drugs e) Monitoring for anemia Although not a common occurrence, bleeding may occur in the postoperative period. Venous bleeding is more common than arterial bleeding. The surgeon may apply traction on the catheter for a few hours to control the venous bleeding. Traction on the catheter is uncomfortable and increases the risk for bladder spasms, so analgesics or antispasmodics are usually prescribed. Hemoglobin and hematocrit should be monitored and trended for indications of anemia. Emergency surgery and increased intermittent irrigation would be indicated for an arterial bleed, which would be a brighter red color. Forced fluids are indicated after the catheter is removed. (Chp 72, elsevier resources)

A client with benign prostatic hyperplasia is being discharged with alpha-adrenergic blockers. Which information is important for the nurse to include when teaching the client about this type of pharmacologic management? (Select all that apply.) a) Avoid drugs used to treat erection problems. b) Be careful when changing positions. c) Keep all appointments for follow-up laboratory testing. d) Hearing tests will need to be conducted periodically. e) Take the medication in the afternoon. (Chp 72, elsevier resources)

a) Avoid drugs used to treat erection problems. b) Be careful when changing positions. c) Keep all appointments for follow-up laboratory testing. Drugs used to treat erectile dysfunction can worsen side effects, such as hypotension. Alpha-adrenergic blockers may cause orthostatic hypotension and can cause liver damage, so it is important to keep appointments for follow-up laboratory testing. These drugs do not affect hearing. Alpha-adrenergic blockers should be taken in the evening to decrease the risk of problems related to hypotension. (Chp 72, elsevier resources)

The nurse is caring for a client with erectile dysfunction who has not had success with other treatment modalities. The nurse anticipates that the health care provider will recommend which treatment for this client? a) Penile implants b) Penile injections c) Transurethral suppository d) Vacuum constriction device (Chp 72, elsevier resources)

a) Penile implants Penile implants (prostheses), which require surgery, are used when other modalities fail. Devices include semi-rigid, flexible, or hydraulic inflatable and multi-component or one-piece instruments. Penile injections are tried before using the option of last resort. Transurethral suppository is tried before using the option of last resort. A vacuum constriction device is easy to use, and is often the first option that is tried. (Chp 72, elsevier resources)

which conditions cause severe pain in HIV and AIDS (Select all that apply) a. enlarged organs b. peripheral neuropathy c. tumors d. high fever e. dry skin

a, b, c

an HIV positive women who is pregnant asks if her baby is at risk for HIV. which points must the nurse be sure to include when teaching? (Select all that apply) A. HIV can cross the placenta B. infant can contract HIV with exposure to blood and vaginal secretions during birth C. once your baby is born, you should be able to breastfeed D. there is a risk for perinatal transmission of HIV from you to your child. because you are on drug therapy, that risk is about 8% E. consider oral contraceptives o protect yourself from other STDs

a, b, d

which actions are useful in helping orient a patient (Select all that apply) a. repeating person, place, time b. using clocks and calendars c. using MMSE screening test d. having familiar items present e. providing uninterrupted time

a, b, d

which descriptions are characteristic of a non progressor? (Select all that apply) A. has been infected for 10 years B. is asymptomatic C. has no CD4+ or t-lymphocytes D. is immunocompetent E. are functional antibodies

a, b, d

the nurse assesses a patient diagnosed with advanced AIDS for malnutirition. which findings does the nurse most likely assess (Select all that apply) a. pain b. anorexia c. urinary incontinence d. diarrhea e. vomiting

a, b, d, e

Which statement concerning a crisis experience is true and should be used as a guideline for crisis management care? Select all that apply a. A crisis is self-limiting and usually resolves within 4-6 weeks b. The earlier interventions are implemented, the better the expected prognosis c. The nurse should maintain a nondirective role d. The patient in crisis is assumed to be mentally unhealthy and in an extreme state of disequilibrium e. The goal of crisis management is to return the patient to at least the pre-crisis level of functioning

a, b, e a. A crisis is self-limiting and usually resolves within 4-6 weeks b. The earlier interventions are implemented, the better the expected prognosis e. The goal of crisis management is to return the patient to at least the pre-crisis level of functioning

corticosteroids perform which actions (Select all that apply) a. block movement of neutrophils and monoctyes through cell membrane b. increase cell production in the bone marrow c. reduce number of circulating t cells, resulting in suppressed cell mediated immunity d. decrease ICP e. contrict blood vessels

a, c, d

a patient presenting with toxicoplasmosis may have with s/s? (Select all that apply) A. speech difficulty B. Shortness of breath C. visual changes D. impaired gait E. mental status changes

a, c, d, e

which practices are recommended to prevent transmission of HIV? (Select all that apply) A. latex condoms for genital and anal intercourse B. natural membrane condoms for genital and anal intercourse C. topical contraceptives D. antiviral meds E. latex barrier for genital and anal intercourse

a, e

38. Which measures provide comfort and prevent secondary infections when a sexually transmitted disease has been diagnosed? (Select all that apply.) A Keep the vulva clean but avoid strong soaps, creams, and ointments unless prescribed by the health care provider. B Keep the vulva dry. C Take analgesics (aspirin or acetaminophen) as directed by the health care provider. D Hot sitz baths may provide relief from itching. E Wipe the vulva from back to front after urination or defecation

a,b,c To provide comfort and prevent secondary infections, keep the vulva clean but avoid strong soaps, creams, and ointments unless prescribed by the health care provider, keep the vulva dry; using a hair dryer on low heat is helpful, wear absorbent cotton underwear and avoid pantyhose and tight pants as much as possible, take analgesics (aspirin or acetaminophen) as directed by the health care provider, cool or tepid sitz baths may provide relief from itching, wipe vulva from front to back after urination or defecation, and then carefully wash hands.

36. Which risk factors would necessitate performing a bone density scan on a woman younger than 65 years? (Select all that apply.) A Family history of osteoporosis B Fall history C Active life style D Chronic steroid use E Normal levels of estrogen F Overweight

a,b,d A bone density scan should be performed on a woman younger than 65 years when there is a family history of osteoporosis, history of falls, underweight, estrogen deficiency, or chronic steroid use.

Which patient statement indicates the helpfulness of the nurse-patient relationship? a. "I appreciate the time you spent with me I have a better understanding of what I can do to manage my problem" b. "I really need to talk with you. You always give me good advice about how to address my anger issues" c. "If it wasn't for you and the hours we've spent talking, I don't think I would be on my way to getting my anxiety under control" d. "You always showed me sympathy when I was at my lowest point after the sexual assault. Knowing you had been there too was such a help"

a. "I appreciate the time you spent with me I have a better understanding of what I can do to manage my problem"

A client has been prescribed lorcaserin (Belviq). What teaching is most appropriate? a. "Increase the fiber and water in your diet." b. "Reduce fat to less than 30% each day." c. "Report dry mouth and decreased sweating." d. "Lorcaserin may cause loose stools for a few days."

a. "Increase the fiber and water in your diet."

A client in sickle cell crisis is dehydrated and in the emergency department. The nurse plans to start an IV. Which fluid choice is best? a. 0.45% normal saline b. 0.9% normal saline c. Dextrose 50% (D50) d. Lactated Ringer's solution

a. 0.45% normal saline

Which statement about crisis theory will provide a basis for nursing intervention? a. A crisis is an active time-limited phenomenon experienced as an overwhelming emotional reaction to a problem perceived as unsolvable b. A person in crisis has always has adjustment problems and has coped inadequately in the usual life situations c. Crisis is precipitated by an event that enhances a person's self-concept and self-esteem d. Nursing intervention in crisis situations rarely has the effect of stopping the crisis

a. A crisis is an active time-limited phenomenon experienced as an overwhelming emotional reaction to a problem perceived as unsolvable

Which belief would be least helpful for a nurse working in crisis intervention? a. A person in crisis is incapable of responding to instruction b. The crisis counseling relationship is one between partners c. Crisis counseling helps the patient refocus to gain new perspectives on the situations d. Anxiety-reduction techniques are used so the patient's inner resources can be accessed

a. A person in crisis is incapable of responding to instruction

A nurse is caring for a client receiving enteral feedings through a Dobhoff tube. What action by the nurse is best to prevent hyperosmolarity? a. Administer free-water boluses. b. Change the client's formula. c. Dilute the client's formula. d. Slow the rate of infusion.

a. Administer free-water boluses.

A client presents to the emergency department in sickle cell crisis. What intervention by the nurse takes priority? a. Administer oxygen. b. Apply an oximetry probe. c. Give pain medication. d. Start an IV line.

a. Administer oxygen.

A client is in the family practice clinic. Today the client weighs 186.4 pounds (84.7 kg). Six months ago the client weighed 211.8 pounds (96.2 kg). What action by the nurse is best? a. Ask the client if the weight loss was intentional. b. Determine if there are food allergies or intolerances. c. Perform a comprehensive nutritional assessment. d. Perform a rapid bedside blood glucose test.

a. Ask the client if the weight loss was intentional.

A client is receiving total parenteral nutrition (TPN). On assessment, the nurse notes the client's pulse is 128 beats/min, blood pressure is 98/56 mm Hg, and skin turgor is dry. What action should the nurse perform next? a. Assess the 24-hour fluid balance. b. Assess the client's oral cavity. c. Prepare to hang a normal saline bolus. d. Turn up the infusion rate of the TPN.

a. Assess the 24-hour fluid balance.

A nurse is preparing to administer a blood transfusion to an older adult. Understanding age-related changes, what alterations in the usual protocol are necessary for the nurse to implement? (Select all that apply.) a. Assess vital signs more often. b. Hold other IV fluids running. c. Premedicate to prevent reactions. d. Transfuse smaller bags of blood. e. Transfuse each unit over 8 hours.

a. Assess vital signs more often. b. Hold other IV fluids running.

A client has frequent hospitalizations for leukemia and is worried about functioning as a parent to four small children. What action by the nurse would be most helpful? a. Assist the client to make "sick day" plans for household responsibilities. b. Determine if there are family members or friends who can help the client. c. Help the client inform friends and family that they will have to help out. d. Refer the client to a social worker in order to investigate respite child care.

a. Assist the client to make "sick day" plans for household responsibilities.

7. What is usually considered to be an unfair labor practice? a. Being fired because a physician does not like a nurse's attitude b. Being passed over for promotion with an explanation of the rationale c. Being assigned to work five weekends in a row when the policy states that nurses will be required to work as needed d. Only being allowed 2 weeks of vacation during the first year of work.

a. Being fired because a physician does not like a nurse's attitude

A client has a platelet count of 9000/mm3. The nurse finds the client confused and mumbling. What action takes priority? a. Calling the Rapid Response Team b. Delegating taking a set of vital signs c. Instituting bleeding precautions d. Placing the client on bedrest

a. Calling the Rapid Response Team

A student studying leukemias learns the risk factors for developing this disorder. Which risk factors does this include? (Select all that apply.) a. Chemical exposure b. Genetically modified foods c. Ionizing radiation exposure d. Vaccinations e. Viral infections

a. Chemical exposure c. Ionizing radiation exposure e. Viral infections

A nurse is caring for four clients with leukemia. After hand-off report, which client should the nurse see first? a. Client who had two bloody diarrhea stools this morning b. Client who has been premedicated for nausea prior to chemotherapy c. Client with a respiratory rate change from 18 to 22 breaths/min d. Client with an unchanged lesion to the lower right lateral malleolus

a. Client who had two bloody diarrhea stools this morning

A client hospitalized with sickle cell crisis frequently asks for opioid pain medications, often shortly after receiving a dose. The nurses on the unit believe the client is drug seeking. When the client requests pain medication, what action by the nurse is best? a. Give the client pain medication if it is time for another dose. b. Instruct the client not to request pain medication too early. c. Request the provider leave a prescription for a placebo. d. Tell the client it is too early to have more pain medication.

a. Give the client pain medication if it is time for another dose.

A patient asks the nurse why the healthcare provider had advised against use of calcium carbonate as an antacid. What is the nurse's best response? a. Its use may result in kidney stones. b. It causes decreased gastric acid production. c. It often causes severe diarrhea. d. It may result in fluid retention and edema.

a. Its use may result in kidney stones.

When responding to the patient in question 7, the intervention that takes priority is to: a. Reduce anxiety b. Arrange shelter c. Contact out-of-area family d. Hospitalize and place the patient on suicide precautions

a. Reduce anxiety

7. What is the primary reason that a patient will attempt to bargain with the nurse during the course of their grief process? a. The patient is attempting to avoid anticipated bad things that may happen. b. The nurse can provide security when accepting the bargaining process. c. The patient is trying to pay the nurse for future favors and special treatment. d. Patients will only bargain with the nurse when they are angry.

a. The patient is attempting to avoid anticipated bad things that may happen.

A patient being treated for peptic ulcer disease (PUD) due to Helicobacter pylori asks the nurse why two or more antibiotics need to be taken. What is the nurse's best response? a. They lower the potential for bacterial resistance. b. They decrease the chances of development of duodenal ulcers. c. They completely eliminate redevelopment of gastric ulcers. d. They decrease the cost of future drug therapies.

a. They lower the potential for bacterial resistance.

10. What is the primary role of the Federal Mediation and Conciliation Service? a. To bring both sides together to work out a settlement b. To prevent nurses and other health-care groups from going on strike c. To develop a solution to the conflict that is binding on both sides d. To force management into accepting the employee demands

a. To bring both sides together to work out a settlement

8. Select all the actions that are correct for a co-worker who is using the wet-blanket behavior approach on the nursing unit. a. Uses complaints to dampen other peoples' attitudes. b. Likes to tear down the ideas of other in the group. c. Seek help with problems they are having from the group. d. Presents strong and useful ideas for solving problems. e. Likes to sow seeds of disappointment and failure. f. Places responsibility for problems on others in the group.

a. Uses complaints to dampen other peoples' attitudes. b. Likes to tear down the ideas of other in the group. e. Likes to sow seeds of disappointment and failure.

the nurse is caring for a patient with gastroesophageal reflux disease and would question an order for which of the following? a. amoxicillin (Amoxil) b. Rantidine (Zantac) c. Pantoprazole (Protonix) d. calcium carbonate (Tums)

a. amoxicillin (Amoxil)

Simethicone (Gas-X, Mylicon) may be added to some medications or given plain for what therapeutic effect? a. decrease the amount of gas associated with GI disorders b. increase the acid-fighting ability of some medications c. prevent constipation associated with gastrointestinal drugs d. prevent diarrhea associated with gastrointestinal drugs

a. decrease the amount of gas associated with GI disorders

20. Women who are past menopause are at higher risk for cardiovascular disease. One of the physiologic changes that occurs with menopause that might lead to cardiovascular disease is A atrophy of the heart. B a rise in the low-density lipoproteins. C a rise in the high-density lipoproteins. D spasms of the vascular system.

b Absence of estrogen is associated with an adverse change in serum lipid levels. Serum levels of low-density lipoproteins increase. Levels of high-density lipoproteins decrease.

35. When providing care to a young single woman just diagnosed with acute pelvic inflammatory disease (PID), the nurse should A point out that inappropriate sexual behavior caused the infection. B prepare the woman for the need of IV antibiotics for the next 48 hours. C explain to the woman that infertility is a likely outcome of this type of infection. D tell her that antibiotics need to be taken until the pelvic pain is relieved.

b Acute PID is often treated with IV administration of broad-spectrum antibiotics. The IV antibiotics can be changed to oral treatment after 48 hours; total duration of antibiotic therapy should be 14 days. Although sexual behavior may well have contributed to the infection, the nurse must discuss these practices in a nonjudgmental manner and provide information about prevention measures. Until treatment is complete and healing has occurred, the outcome is unknown and should not be suggested.

28. A woman is trying to decrease her urinary incontinence without medication or surgery. The nurse can recommend that the woman A decrease fluid intake at night. B decrease alcohol and caffeine intake. C decrease 10% of her average weight. D increase fluid intake in the morning and decrease the intake in the afternoon.

b Alcohol and caffeine can irritate the bladder and worsen incontinence. Obesity is associated with urinary incontinence, and the woman should attempt to be at her ideal weight range. Decreased fluid intake can lead to concentrated urine, which can irritate the bladder's mucous membranes and increase the urge to void.

21. A woman who has had a hysterectomy has been prescribed hormone replacement therapy. The nurse can anticipate which type of hormones that will be prescribed to this woman? A Combination of estrogen and progesterone B Estrogen therapy alone C Hormone therapy is not recommended for women after hysterectomies. D Progesterone therapy alone

b Estrogen therapy alone can be given to women who have had a hysterectomy because uterine hyperplasia is not a risk. Estrogen and progesterone are given to women with a uterus to prevent hyperplasia.

9. A 22-year-old woman has come to the clinic complaining of a "mass in my breast." The nurse practitioner notes two firm, freely mobile nodules in the upper outer quadrant of the right breast. The nurse is aware that the nurse practitioner will A refer the patient for a needle biopsy. B have the patient return during her menstrual period to reevaluate the masses. C schedule the woman for a mammography. D do nothing. Masses at this age are always benign.

b Fibroadenomas are benign tumors of the breast and are most common during the teenage years and the 20s. Fibroadenomas are firm, freely mobile nodules that may or may not be tender when palpated. Fibroadenomas do not change during the menstrual cycle. They are generally located in the upper outer quadrant of the breast, and more than one is often present. Treatment may involve careful observation for a few months to determine if the mass is stable. If the mass enlarges, a biopsy is done.

3. When scheduling times for women to have a pelvic examination and Papanicolaou (Pap) test, what question is important to ask the woman? A When was her last examination? B On what date will her next menstrual period start? C Does she have insurance coverage of the examination? D Does she use any type of birth control?

b Pelvic examinations should be scheduled between menstrual periods.

15. A 25-year-old woman comes to the clinic for her regular annual gynecologic examination. When taking the history, the woman tells the nurse that she has been dieting for the past year and has lost 150 lb (from 250 to 100 lb). Her menstruation stopped 6 months ago. A chart review indicates that prior to this visit her menses had been regular every 28 days. The nurse can classify this woman with A primary amenorrhea. B secondary amenorrhea. C amenorrhea of unknown origin. D possible pregnancy.

b Secondary amenorrhea is the cessation of menstruation for at least 6 months in a woman who has established a pattern of menstruation. Poor nutrition is one reason for secondary amenorrhea.

22. Which one of the following women is at greatest risk for osteoporosis? A African-American, weight 165 lb, height 5?2'3?3?; does not smoke cigarettes or drink alcohol B Asian, weight 105 lb, height 4?2'14?3?; smokes two packs of cigarettes a day C White, weight 145 lb, height 5?2'8?3?; had a hysterectomy with removal of ovaries at age 45 D Native American, weight 165 lb, height 5?2'7?3?; alcoholic for 15 years, has been without a drink for the past 2 years

b Small-boned, fair-skinned, White, and Asian women are at greatest risk for osteoporosis. Other risk factors include early menopause, smoking, and alcohol intake. The more risk factors, the higher the risk for developing osteoporosis. The Asian woman is small-boned and smokes, which gives her three risk factors. All the other women had zero to two risk factors.

16. A 32-year-old woman complains of excessive bleeding with menses for the past 3 months. With a chart review, the nurse notes that the woman had a urinary tract infection 3 months ago that was treated with an antibiotic, has hypothyroidism, uses condoms with foam for contraception, and uses antidepressants. Which one of these is the most likely cause of the woman's excessive bleeding? A Urinary tract infections treated with an antibiotic B Hypothyroidism C Use of condoms with foam for contraception D Use of antidepressants

b Systemic disorders such as hypothyroidism may be a cause of dysfunctional bleeding.

34. A woman calls the clinic concerned that a neighbor has been diagnosed with herpes genitalis type 2. The woman is upset and tells the nurse that this neighbor "used my toilet last week, so what should I do?" The nurse's response should be based on knowledge that herpes genitalis type 2 is transmitted only through A sexual intercourse. B direct contact. C blood contamination. D blood or body fluid contamination.

b Transmission occurs only through direct contact with an infected person.

how does HSV manifest itself in patients with HIV/AIDS (Select all that apply) a. maculopapular lesions that can spread b. chronic ulceration after vesicles rupture c. vesicles ocated in the perirectal, oral, and genital area d. numbness and tingling before vesicle forms e. itching localized to perianal area

b, c, d

where can candidiasis occur in the body (Select all that apply) a. nose b. esophagus c. vagina d. mouth e. ears

b, c, d

2. A school nurse is teaching a group of high school seniors about gynecologic care. It is important to include instructions on A.scheduling a mammogram within the next 3 years. B vulvar self-examination. C use of contraceptives by the menopausal woman. D breast self-examination.

b Vulvar self-examinations should begin in women 18 years old and in women younger than 18 if they are sexually active. Mammograms are routinely started at the age of 45. Due to lack of evidence of clear benefit, breast self-examinations are no longer recommended. Adolescents will not benefit from information about contraceptive use by menopausal women.

6. A nurse teaching adolescents concerning care during menses should include that A only perineal pads should be used until the woman is at least 24 years old to allow closure of the cervical os. B perineal pads should be worn at night. C tampons can be used around the clock. D tampons should be replaced every 6 to 8 hours.

b When using tampons, they should be changed at least every 4 hours to prevent excessive bacterial growth. Perineal pads should be used at night during sleep, which usually exceeds 4 hours.

A woman is undergoing chemotherapy for breast cancer. During the discharge teaching, it is important that the nurse teach the woman to A not wash off the marks on her skin made by the technician. B avoid crowds and anyone who is sick. C take the medication before her menstrual period. D have her calcium levels checked every 2 months.

b Chemotherapeutics may kill off normal cells along with the cancer cells, especially rapidly dividing cells such as those in blood cells. Therefore the woman may be more susceptible to infection during the treatment.

HIV is most commonly transmitted by which routes? (Select all that apply) A. oral B. sexual C. parenteral D. airborne E. perinatal

b, c, e

A client is having a radical prostatectomy. Which preoperative teaching specific to this surgery does the nurse emphasize? a) Incentive spirometry b) Kegel exercises c) Pain control d) Penile implants (Chp 72, elsevier resources)

b) Kegel exercises Kegel perineal exercises may reduce the severity of urinary incontinence after radical prostatectomy. The client is taught to contract and relax the perineal and gluteal muscles in several ways. Incentive spirometry and pain control are important for everyone who undergoes surgery; neither is specific to radical prostatectomy. Penile implants are not important to discuss during preoperative teaching; however, they may be necessary to discuss later. (Chp 72, elsevier resources)

A client had an orchiectomy and laparoscopic radical retroperitoneal lymph node dissection this morning. What is the nurse's priority for care? a) assess the client's pain level and provide pain management b) ensure that the client's urinary catheter is draining clear yellow urine c) observe the client's incision for redness, swelling, and drainage d) apply oxygen therapy via nasal cannula at 2 L/min (Ignatavicius & Workman, p. 1515)

b) ensure that the client's urinary catheter is draining clear yellow urine (Ignatavicius & Workman, p. 1515)

A client had a transurethral resection of the prostate (TURP) with continuous bladder irrigation yesterday. The staff nurse notes that the urinary drainage is bright red and thick. What is the nurse's best action? a) notify the charge nurse as soon as possible b) increase the rate of bladder irrigation c) document the assessment in the medical record d) prepare the patient for a blood transfusion (Ignatavicius & Workman, p. 1506)

b) increase the rate of bladder irrigation (Ignatavicius & Workman, p. 1506)

which statement about the transmission of HIV is true? (Select all that apply) A. can only be transmitted during end stage B. those with recent HIV infection and high viral load are very infectious C. those with end stage HIV and no drug therapy are very infectious D. HIV is only transmitted with sexual contact E. all people infected with HIV will quickly progress to AIDS

b, c

9. What is the most important feature of the shared governance model? a. Nursing administration retains most of the power over nurses to better regulate practice. b. Power and authority are transferred to the nursing staff rather than being located primarily in nursing administration. c. Clients are billed for nursing care as a separate item similar to the way they are billed for physician services. d. The nursing staff hierarchy structure is similar to the medical staff structure.

b. Power and authority are transferred to the nursing staff rather than being located primarily in nursing administration.

A client received a drug for treatment of nausea and vomiting and is now complaining of dry mouth, constipation, and a rapid heart rate. The nurse concludes that which drug was taken by the client? a. Loperamide (Imodium) b. Prochlorperazine (Compazine) c. Peppermint d. Diphenoxylate (Lomotil)

b. Prochlorperazine (Compazine)

8. Select a practice that is an indication of failure to bargain in good faith. a. Agreeing to meet at reasonable times b. Sending individuals to negotiate who cannot make binding decisions c. Unwillingness to negotiate on all issues d. Exchanging lists of demands by both sides

b. Sending individuals to negotiate who cannot make binding decisions

Several nurses have just helped a morbidly obese client get out of bed. One nurse accesses the client's record because "I just have to know how much she weighs!" What action by the client's nurse is most appropriate? a. Make an anonymous report to the charge nurse. b. State "That is a violation of client confidentiality." c. Tell the nurse "Don't look; I'll tell you her weight." d. Walk away and ignore the other nurse's behavior.

b. State "That is a violation of client confidentiality."

A client who will be traveling on a plane is prescribed dimenhydrinate (Dramamine) for management of motion sickness. Which instruction about administration of the drug should the nurse provide the client? a. Apply the patch behind the ear the day before travel. b. Take the medication by mouth 20-60 minutes prior to the trip. c. Take the medication by mouth at onset of motion sickness. d. Inject the medication on the thigh intramuscularly.

b. Take the medication by mouth 20-60 minutes prior to the trip.

the nurse has administered prochlorperazine (compazine) to a patient for postoperative nausea. Before administering this medication, it is essential that the nurse check which of the following? a. pain level b. blood pressure c. breath sounds d. temperature

b. blood pressure

a 35-year old male patient has been prescribed omeprazole (Prilosec) for the treatment of gastroesophageal reflux disease. which of the following assessment findings would assist the nurse to determine whether drug therapy has been effective? SATA. a. decreased "gnawing" upper abdominal pain on an empty stomach b. decreased belching c. decreased appetitie d. decreased nausea e. decrease dysphagia

b. decreased belching d. decreased nausea e. decrease dysphagia

The HIV positive patient tells the nurse that his HIV negative partner will be using preexposure drugs (Truvada). which statement indicates the need for additional teaching? A. my partner will need to be tested q3m B. this drug will decrease the chances of my partner becoming positive C. once we start using Truvada I will no longer need a condom D. my partner will need to be monitored for any side effects on this drug

c

the patient with HIV/AIDS tells the nurse that food tastes funny and is difficult to swallow. what is the nurses priority action at this time? a. Check the patients gag reflex b. ask about blood cultures c. examine the patient's mouth and throat d. collaborate with the dietitian to provide a soft diet

c

24. A 57-year-old woman eats two servings of calcium-rich food a day, usually in the form of 8 oz of skim milk or yogurt. To meet her calcium needs, she would need to take a calcium supplement that contains how much calcium? A 400 mg B 500 mg C 600 mg D 800 mg

c 8 oz of skim milk or yogurt contain between 300 and 350 mg of calcium. Two servings would give this woman at least 600 mg. A woman older than 50 years needs 1200 mg of calcium daily, so she would need to take 600 mg in a supplement.

30. When assisting a woman into the lithotomy position for a pelvic examination, the nurse notes a frothy, malodorous, yellow-green vaginal discharge. The nurse should anticipate the need for a A culture and sensitivity test of the discharge. B serologic test. C wet mount preparation test. D biopsy.

c A frothy, malodorous, and yellow-green discharge is an indication of trichomoniasis. The diagnosis is made by identifying the organism in a wet mount preparation.

14. Which one of the following women is at highest risk for cardiovascular disease? A 55-year-old who is overweight and participates in no physical activity during the week B 65-year-old who has type 2 diabetes C 45-year-old with type 2 diabetes, hypertension, overweight, and smokes D 70-year-old in good health but with a family history of cardiovascular disease

c Age, being overweight, no physical activity, type 2 diabetes, hypertension, family history, and smoking are all risk factors for cardiovascular disease. The more risk factors a woman has, the higher her risk for developing cardiovascular disease. The 45-year-old is the youngest in the choices shown, but she has the most risk factors.

29. During an annual gynecologic examination, the physician notes an enlarged left ovary in a 28-year-old woman. The woman has no complaints of pain or tenderness. The nurse can anticipate A an appointment for an ultrasound. B scheduling the woman for a laparoscopy. C scheduling the woman for a follow-up examination after her next menses. D nothing. The finding is insignificant.

c Follicle ovarian cysts are usually asymptomatic and generally regress during the subsequent menstrual cycle. If the woman is in her childbearing years, when the risk of ovarian cancer is less, the physician may wait until after the next menstrual cycle and examine the woman again.

5. The nurse is reviewing laboratory reports from several patients who had Pap tests done 3 days ago. One result stated, "high-grade squamous intraepithelial lesion." The nurse is aware that this report indicates A negative results and no follow-up is required. B a negative result but a 3-month repeat Pap test should be done. C this result has a high likelihood of becoming cancerous, and a follow-up is necessary for treatment. D the results are inconclusive and the woman should have a repeat test done in 6 months.

c High-grade squamous intraepithelial lesion was previously categorized as carcinoma in situ. These cell changes are likely to become cancerous without definitive treatment. This woman requires immediate follow-up on the results.

27. After teaching a woman about Kegel exercises, the nurse assesses that the teaching has been effective when the woman states: A "Once I can contract the muscles for 10 seconds at a time I can stop the exercise." B "I will need to do these exercises until I get up to 45 daily repetitions." C "I will need to do these exercises for the rest of my life." D "If I can stop the stream of urine I don't need to do the exercises."

c Kegel exercises involve conscious contracting and relaxing of the pelvic muscles. To maintain pelvic muscle tone, the woman should continue Kegel exercises for the rest of her life.

1. A 25-year-old woman is in for her first gynecologic examination. She is in good health and has no family history of cancer or reproductive diseases. She asks the nurse if a mammogram will be performed. The nurse is aware that this woman will A need a mammogram this year, and it should be repeated every 2 years. B need a mammogram this year, and it should be repeated every 5 years. C not need a mammogram until she is 45 years old. D not need a mammogram until she is 30 years old.

c The American Cancer Society and American College of Obstetricians and Gynecologists recommends that mammograms start at the age of 45 unless the woman is at high risk for breast cancer.

26. A 65-year-old woman, gravida 6, para 6, is complaining of increasing stress incontinence and pelvic pressure and fullness. Pelvic examination reveals a bulging in the anterior vaginal wall. This woman is most likely experiencing A uterine prolapse. B rectocele. C cystocele. D vesicovaginal fistula.

c The classic clinical manifestations of cystocele are described in the question. Prolapse or downward displacement of the uterus could result in protrusion of the uterus through the vagina. Rectocele results in herniation of the rectal wall through the posterior vagina. Clinical manifestations relate to alterations in bowel elimination. A vesicovaginal fistula is an abnormal passage between the bladder and vagina, resulting in urinary incontinence and excoriation of the vaginal mucosa.

33. A woman confides in the nurse about a painless chancre that developed next to her vaginal opening about 8 weeks ago. The woman is not concerned because it has gone away. The nurse should teach this woman A that the use of condoms will decrease chances of this developing again. B to wash the perineum with mild soap and water after intercourse to prevent recurrences. C that a serologic test is indicated. D that a vaginal culture is indicated.

c The first sign of primary syphilis is a painless chancre that heals in about 6 weeks. About 2 months after the initial infection, serologic tests are generally positive for syphilis. The infection does not heal but moves into the secondary stage of syphilis.

25. A 50-year-old woman is trying to decrease her chances of developing osteoporosis. Which exercise can the nurse recommend that will be beneficial? A Swimming B Water aerobics C Walking D Yoga

c Weight-bearing and resistance exercise have been shown to be beneficial in slowing loss of bone mass to maintain bone density. Water-based exercises such as swimming do not help limit bone loss.

12. A woman who was diagnosed with breast cancer 1 month ago is making an appointment for a follow-up appointment with the physician. During the conversation with the nurse, the woman becomes angry and yells, "You never have any appointments available when I can come. I always have to rearrange my day to agree with your schedule." The best response by the nurse is: A "We have so many patients and there are few available times for an appointment. I am sorry it is inconvenient for you." B "Let's look to see if we have another time that is better for you." C "You seem to be upset today." D "I am doing the best job I can. Tell me when you can come and I will try and work you in at that time."

c During the stages of cancer, women think that they have lost control and that their lives have been taken over by cancer. The nurse should provide time and demonstrate genuine interest in the woman's concerns using communication techniques, such as reflecting feelings, and open-ended statements to encourage her to express her concerns.

A nurse attempted to assist a morbidly obese client back to bed and had immediate pain in the lower back. What action by the nurse is most appropriate? a. Ask another nurse to help next time. b. Demand better equipment to use. c. Fill out and file a variance report. d. Refuse to assist the client again.

c. Fill out and file a variance report.

With which male client does the nurse conduct prostate screening and education? a) Young adult with a history of urinary tract infections b) Client who has sustained an injury to the external genitalia c) Adult who is older than 50 years d) Sexually active client (Chp 72, elsevier resources)

c) Adult who is older than 50 years A man who is 50 years or older is at higher risk for prostate cancer. A history of urinary tract infections, injury to the external genitalia, and sexual activity are not risk factors for prostate cancer. (Chp 72, elsevier resources)

The potential problem of grief is most relevant to a client after which procedure? a) Cystoscopy b) Transurethral microwave therapy c) Radical prostatectomy d) Sperm banking (Chp 72, elsevier resources)

c) Radical prostatectomy A radical prostatectomy may lead to erectile dysfunction, which could present a potential problem of grief at loss of function. Cystoscopy, a test to view the interior of the bladder, the bladder neck, and the urethra, does not affect sexuality. Transurethral microwave therapy is a minimally invasive procedure involving high temperatures that heat and destroy excess prostate tissue, and does not affect sexuality. The process of sperm banking would not result in a diagnosis of altered self-image; however, the diagnosis leading to the necessity of sperm banking might cause this. (Chp 72, elsevier resources)

A nurse is caring for four clients receiving enteral tube feedings. Which client should the nurse see first? a. Client with a blood glucose level of 138 mg/dL b. Client with foul-smelling diarrhea c. Client with a potassium level of 2.6 mEq/L d. Client with a sodium level of 138 mEq/L

c. Client with a potassium level of 2.6 mEq/L

Which method is a common complementary and alternative therapy for benign prostatic hyperplasia (BPH)? a) Acupuncture b) Calcium supplements c) Serenoa repens d) Yoga (Chp 72, elsevier resources)

c) Serenoa repens Serenoa repens (saw palmetto), a plant extract, is often used by men with early to moderate BPH. They believe that this agent relieves their symptoms and prefer this treatment over prescription drugs or surgery. (It should be noted, however, that studies on the effectiveness of Serenoa repens have not shown that it is effective.) Acupuncture, calcium, and yoga are not common alternative therapies for BPH. (Chp 72, elsevier resources)

A client has thrombocytopenia. What client statement indicates the client understands self-management of this condition? a. "I brush and use dental floss every day." b. "I chew hard candy for my dry mouth." c. "I usually put ice on bumps or bruises." d. "Nonslip socks are best when I walk."

c. "I usually put ice on bumps or bruises."

The nurse is engaged in crisis intervention with a patient reporting, "I have no reason to keep on living." What is the nurse's initial interventions? a. Advise the patient about the services available to help them b. Ask the patient, "Have you ever been this depressed before?" c. Ask the patient, "Do you have any plan to hurt yourself or anyone else?" d. Assure the patient that he or she is in a safe place and will be well cared for

c. Ask the patient, "Do you have any plan to hurt yourself or anyone else?"

A nurse is caring for a patient preparing to undergo a colonoscopy. The nurse should anticipate administering which drug? a. Laxative b. Diuretic c. Cathartic d. Antihypertensive

c. Cathartic

A client has a platelet count of 25,000/mm3. What actions does the nurse delegate to the unlicensed assistive personnel (UAP)? (Select all that apply.) a. Assist with oral hygiene using a firm toothbrush. b. Give the client an enema if he or she is constipated. c. Help the client choose soft foods from the menu. d. Shave the male client with an electric razor. e. Use a lift sheet when needed to re-position the client.

c. Help the client choose soft foods from the menu. d. Shave the male client with an electric razor. e. Use a lift sheet when needed to re-position the client.

A nurse is caring for a client who is about to receive a bone marrow transplant. To best help the client cope with the long recovery period, what action by the nurse is best? a. Arrange a visitation schedule among friends and family. b. Explain that this process is difficult but must be endured. c. Help the client find things to hope for each day of recovery. d. Provide plenty of diversionary activities for this time.

c. Help the client find things to hope for each day of recovery.

A client having a tube feeding begins vomiting. What action by the nurse is most appropriate? a. Administer an antiemetic. b. Check the client's gastric residual. c. Hold the feeding until the nausea subsides. d. Reduce the rate of the tube feeding by half.

c. Hold the feeding until the nausea subsides.

A client has a sickle cell crisis with extreme lower extremity pain. What comfort measure does the nurse delegate to the unlicensed assistive personnel (UAP)? a. Apply ice packs to the client's legs. b. Elevate the client's legs on pillows. c. Keep the lower extremities warm. d. Place elastic bandage wraps on the client's legs.

c. Keep the lower extremities warm.

A patient tells the nurse that she has been taking sodium bicarbonate antacid 3-4 times a day for the past 3 weeks to relieve symptoms of GERD. For what should the nurse assess in this patient? a. Constipation b. Respiratory acidosis c. Metabolic alkalosis d. Hypokalemia

c. Metabolic alkalosis

A client tells the nurse about losing weight and regaining it multiple times. Besides eating and exercising habits, for what additional data should the nurse assess as the priority? a. Economic ability to join a gym b. Food allergies and intolerances c. Psychosocial influences on weight d. Reasons for wanting to lose weight

c. Psychosocial influences on weight

A patient whose history includes experiences with abusive partners is being treats for major depressive disorder. The patient's care plan includes rape-trauma syndrome among the nursing diagnoses. What goal is directly associated with this diagnosis? a. Remains free from self-harm b. Wears appropriate clothing c. Reports feeling stronger and having a sense of hopefulness d. Demonstrates appropriate affect for both positive and negative

c. Reports feeling stronger and having a sense of hopefulness

A nursing student is studying nutritional problems and learns that kwashiorkor is distinguished from marasmus with which finding? a. Deficit of calories b. Lack of all nutrients c. Specific lack of protein d. Unknown cause of malnutrition

c. Specific lack of protein

A nurse is caring for a young male client with lymphoma who is to begin treatment. What teaching topic is a priority? a. Genetic testing b. Infection prevention c. Sperm banking d. Treatment options

c. Sperm banking

10. Which of the following would be effective approaches to chronic complainers to help them recognize that their behaviors are inappropriate and that expressing their needs by complaining is harmful to themselves and the work environment? a. Reinforce their attitudes towards people and work b. Completely ameliorate the behavior c. Use active listening d. Use an aggressive confrontation to change their behavior

c. Use active listening

a female patient reports using OTC aluminum hydroxide (alternaGEL) for the relief of gastric upset. she is on renal dialysis 3 times a week. what should the nurse teach this patient? a. continue using the antacids but if she needs to continue them beyond a few months, she should consult the health care provider about different therapies b. take the antacid no longer than for two weeks. if it has not worked by then, it will not be effective c. consult with the health care provider about the appropriate amount and type of antacid d. continue to take the antacid; it is OTC and safe

c. consult with the health care provider about the appropriate amount and type of antacid

Tetracycline contraindications

contraindicated in patients with hypersensitivity to drugs in this class - not to be used in the second half of pregnancy - not to be used n children 8 years or younger - not to be used in patients with severe renal or hepatic impairment

a patient with HIV is receiving meds to reduce viral load and improve cd4+ counts. which term accurately describes this HIV drug regimen a. interferon treatment b. antiviremia c. ELISA administration d. HAART

d

cryptosporidiosis is a form of intestinal infection in which diarrhea can amount to a loss of how many liters of fluid per day? A. 1-2 B. 3-5 c. 5-8 d. 15-20

d

which is the most common route for HCP to contract HIV: A. blood B. bodily fluids C. mucous membranes D. needle sticks

d

8. The nurse should refer the patient for further testing if which one of the following is noted on inspection of the breasts of a 55-year-old woman? A Left breast slightly smaller than the right breast B Eversion (elevation) of both nipples C Bilateral symmetry of venous network that is faintly visible D Small dimple located in the upper outer quadrant of the right breasts

d Dimpling or retraction is often associated with an underlying mass or tumor. The other choices are all expected findings.

19. A woman who is 17 weeks pregnant because of incest asks the nurse about having a therapeutic abortion. The nurse's best response should be based on the knowledge that A therapeutic abortions are not available in this country for a woman more than 14 weeks pregnant. B mifepristone (RU486) can be used up to week 20 of pregnancy. C methotrexate (Folex, Mexate) can be used up to week 24 of pregnancy. D a dilation of the cervix with removal of the fetus and placenta can be performed during the 17th week of pregnancy.

d Medications such as mifepristone and methotrexate are used for early abortions. For second-trimester abortions, dilation with removal of the fetus and placenta is performed.

The nurse is educating a group of young men about testicular self-examination (TSE). Which statement by a member of the group indicates teaching has been effective? a) "I will examine my testicles right before taking a shower." b) "I should squeeze each testicle in my hand to feel any lumps." c) "I should only report any large lumps to my health care provider." d) "I will look and feel for any lumps or changes to my testes." (Chp 72, elsevier resources)

d) "I will look and feel for any lumps or changes to my testes." With early detection by monthly TSE and treatment, testicular cancer can be successfully cured. In TSE, the client should look and feel for any lumps or changes to the testes. Any lumps that are detected should be immediately reported. A TSE should be performed immediately following a shower. The client should gently roll each testicle between the thumb and forefinger. All lumps should be reported to the provider, no matter the size. (Chp 72, elsevier resources)

The nurse is teaching a client about taking sildenafil (Viagra) for erectile dysfunction. Which statement by the client indicates a need for further teaching? a) "I should have sex within an hour after taking the drug." b) "I should avoid alcohol when on the drug or it might not work well." c) "I can expect to maybe get a stuffy nose or headache when I take the drug." d) "If I have chest pain during sex, I should take a nitroglycerin tablet." (Ignatavicius & Workman, p. 1513)

d) "If I have chest pain during sex, I should take a nitroglycerin tablet." (Ignatavicius & Workman, p. 1513)

A client receiving external beam radiation therapy calls the nurse to report rectal urgency, cramping, and passing of mucus and blood. What is the nurse's best response? a) "This is an emergency. Go directly to the emergency department." b) "This is normal and will resolve as soon as the treatment stops." c) "Avoid caffeine and continue drinking plenty of water and other fluids." d) "Limit spicy or fatty foods, caffeine, and dairy products." (Chp 72, elsevier resources)

d) "Limit spicy or fatty foods, caffeine, and dairy products." The client's symptoms indicate that he is experiencing radiation proctitis, a common complication of external beam radiation therapy. The nurse's instructions to limit spicy or fatty foods, caffeine, and dairy products describe what the client should do to alleviate these symptoms. The client's symptoms do not indicate an emergency, but they should be reported to the health care provider. The client's symptoms should resolve 4 to 6 weeks after the treatment stops. Avoiding caffeine and drinking water and other fluids describe what the client should do if he is experiencing radiation cystitis, which he is not. (Chp 72, elsevier resources)

which groups are experiencing increased numbers of HIV infection? (Select all that apply) A. men having sex with other men B. IV drug users C. women having sex with men D. african americans E. hispanics

d, e

A nursing student is struggling to understand the process of graft-versus-host disease. What explanation by the nurse instructor is best? a. "Because of immunosuppression, the donor cells take over." b. "It's like a transfusion reaction because no perfect matches exist." c. "The client's cells are fighting donor cells for dominance." d. "The donor's cells are actually attacking the client's cells."

d. "The donor's cells are actually attacking the client's cells."

A client asks the nurse about drugs for weight loss. What response by the nurse is best? a. "All weight-loss drugs can cause suicidal ideation." b. "No drugs are currently available for weight loss." c. "Only over-the-counter medications are available." d. "There are three drugs currently approved for this."

d. "There are three drugs currently approved for this."

A nursing student is caring for a client with leukemia. The student asks why the client is still at risk for infection when the client's white blood cell count (WBC) is high. What response by the registered nurse is best? a. "If the WBCs are high, there already is an infection present." b. "The client is in a blast crisis and has too many WBCs." c. "There must be a mistake; the WBCs should be very low." d. "Those WBCs are abnormal and don't provide protection."

d. "Those WBCs are abnormal and don't provide protection."

The highest-priority goal of crisis intervention is: a. Anxiety reduction b. Identification of situational supports c. Teaching specific coping skills that are lacking d. Patient safety

d. Patient safety

A client is receiving total parenteral nutrition (TPN). What action by the nurse is most important? a. Assessing blood glucose as directed b. Changing the IV dressing each day c. Checking the TPN with another nurse d. Performing appropriate hand hygiene

d. Performing appropriate hand hygiene

3. In caring for a patient with third degree burns over 22 percent of his body, which of the following needs of Maslow have the highest priority? a. Love and belonging b. Psychological and social fulfillment c. Safety d. Physiological

d. Physiological

A nurse is preparing to hang a blood transfusion. Which action is most important? a. Documenting the transfusion b. Placing the client on NPO status c. Placing the client in isolation d. Putting on a pair of gloves

d. Putting on a pair of gloves

The nurse assesses a client's oral cavity and makes the discovery shown in the photo below: What action by the nurse is most appropriate? a. Encourage the client to have genetic testing. b. Instruct the client on high-fiber foods. c. Place the client in protective precautions. d. Teach the client about cobalamin therapy.

d. Teach the client about cobalamin therapy.

A nurse is weighing and measuring a client with severe kyphosis. What is the best method to obtain this client's height? a. Add the trunk and leg measurements. b. Ask the client how tall he or she is. c. Estimate by measuring clothing. d. Use knee-height calipers.

d. Use knee-height calipers.

A client with sickle cell disease (SCD) takes hydroxyurea (Droxia). The client presents to the clinic reporting an increase in fatigue. What laboratory result should the nurse report immediately? a. Hematocrit: 25% b. Hemoglobin: 9.2 mg/dL c. Potassium: 3.2 mEq/L d. White blood cell count: 38,000/mm3

d. White blood cell count: 38,000/mm3

A client with multiple myeloma demonstrates worsening bone density on diagnostic scans. About what drug does the nurse plan to teach this client? a. Bortezomib (Velcade) b. Dexamethasone (Decadron) c. Thalidomide (Thalomid) d. Zoledronic acid (Zometa)

d. Zoledronic acid (Zometa)

In taking a new client's history, the nurse notices that he has been taking omeprazole (Prilosec) consistently over the past 6 months for treatment of epigastric pain. Which recommendation would be the best for the nurse to give this client? a. try switching to a different form of the drug b. try a drug like cimetidine (Tagamet) or famotidine (Pepcid) c. try taking the drug after meals instead of before meals d. check with his health care provider about his continued discomfort

d. check with his health care provider about his continued discomfort

a patient with constipation is prescribed psyllium (metamucil) by his health care provider. what essential teaching will the nurse provide to the patient? a. take the drug with meals and at bedtime b. take the drug with minimal water so that it will not be diluted in the GI tract c. avoid caffeine and chocolate while taking this drug d. mix the product in a full glass of water and drink another full glass of water after taking the drug

d. mix the product in a full glass of water and drink another full glass of water after taking the drug

Fluvicin side effects

granulocytopenia, cholestatic hepatitis, neurotropenia - if symptoms worsen or do not improve call doctor.

Hypersensitivity reaction with Fungizone

immunosuppression, renal impairment, and liver toxicity

Pathogenicity is different than virulence in that pathogenicity can

lead to the ability of organisms to cause infection.

What drug to drug interaction results in increase of cyclosporine levels

macrolide antibiotics

Lab results in Retrovir

mean corpuscular volume may be increased during zidovudine therapy. White blood cell and hemoglobin may decrease due to neutropenia and anemia, respectively

superinfections

occur when miccroorganisms normally present in the body are destroyed

What is the primary adverse effect of cyclosporine

occurs in the kidneys, resulting in reduction of urine output

What is the most productive way to start the communication process?

open-ended language

What is amoxicillin used for?

prescribed for sinus and upper reparatory and genitourinary tract infections

The primary function of the dermis is to

provide foundation for hair and nails.

The client asks the nurse how skin cells are replaced. What is the best response by the nurse?

"The epidermis supplies new cells after older cells have been damaged or lost."

The client tells the nurse that the doctor told him his antibiotic did not kill his infection but just slowed its growth. The client is anxious. What is the best response by the nurse to decrease the client's anxiety?

"This is okay because your body will help kill the infection too."

The nurse has taught a client how to manage constipation. Which action by the client would provide evidence of learning? (Select all that apply.) The patient: 1) increases his intake of high-fiber foods. 2) drinks at least four 8-ounce glasses of water a day. 3) goes to the bathroom to evacuate after meals. 4) takes a daily laxative.

1) increases his intake of high-fiber foods. 3) goes to the bathroom to evacuate after meals. The urge to defecate typically comes after eating; the nurse can help manage the patient's constipation by assisting the patient to the bathroom after meals. The nurse should also encourage the patient to increase his intake of high-fiber food and drink at least eight glasses of water a day (not four). Laxatives should be administered or taken only when absolutely necessary.

The nurse plans to teach the client with acquired immune deficiency syndrome (AIDS) about bacterial infections. Which information should the nurse include in this teaching?

1. "If just a few bacteria make you sick, this is virulence." 4. "Pathogenicity means the bacteria can cause an infection." 5. "Actually, most bacteria will not harm us."

The client receives a topical medication for treatment of an acne-like skin disorder. The nurse completes medication education and evaluates learning has occurred when the client makes which statements?

2. "I will call my doctor if I notice a change in my symptoms." 3. "I will apply the medication only to the affected area."

A nurse assesses a client who has appendicitis. Which clinical manifestation should the nurse expect to find? a. Severe, steady right lower quadrant pain b. Abdominal pain associated with nausea and vomiting c. Marked peristalsis and hyperactive bowel sounds d. Abdominal pain that increases with knee flexion

ANS: A Right lower quadrant pain, specifically at McBurney's point, is characteristic of appendicitis. Usually if nausea and vomiting begin first, the client has gastroenteritis. Marked peristalsis and hyperactive bowel sounds are not indicative of appendicitis. Abdominal pain due to appendicitis decreases with knee flexion. DIF: Remembering/Knowledge REF: 1169 KEY: Inflammatory bowel disorder| assessment/diagnostic examination MSC: Integrated Process: Nursing Process: Assessment NOT: Client Needs Category: Physiological Integrity: Physiological Adaptation

A nurse teaches a client who has viral gastroenteritis. Which dietary instruction should the nurse include in this client's teaching? a. "Drink plenty of fluids to prevent dehydration." b. "You should only drink 1 liter of fluids daily." c. "Increase your protein intake by drinking more milk." d. "Sips of cola or tea may help to relieve your nausea."

ANS: A The client should drink plenty of fluids to prevent dehydration. Milk products may not be tolerated. Caffeinated beverages increase intestinal motility and should be avoided. DIF: Applying/Application REF: 1173 KEY: Inflammatory bowel disorder| nutritional requirements MSC: Integrated Process: Teaching/Learning NOT: Client Needs Category: Physiological Integrity: Basic Care and Comfort

An older client has had an instance of drug toxicity and asks why this happens, since the client has been on this medication for years at the same dose. What response by the nurse is best? a. "Changes in your liver cause drugs to be metabolized differently." b. "Perhaps you don't need as high a dose of the drug as before." c. "Stomach muscles atrophy with age and you digest more slowly." d. "Your body probably can't tolerate as much medication anymore."

ANS: A Decreased liver enzyme activity depresses drug metabolism, which leads to accumulation of drugs—possibly to toxic levels. The other options do not accurately explain this age-related change.

A nurse assesses a client who is admitted with inflamed soft-tissue folds around the nail plates. Which question should the nurse ask to elicit useful information about the possible condition? a. "What do you do for a living?" b. "Are your nails professionally manicured?" c. "Do you have diabetes mellitus?" d. "Have you had a recent fungal infection?"

ANS: A The condition chronic paronychia is common in people with frequent intermittent exposure to water, such as homemakers, bartenders, and laundry workers. The other questions would not provide information specifically related to this assessment finding.

A nurse cares for a client who has obstructive jaundice. The client asks, "Why is my skin so itchy?" How should the nurse respond? a. "Bile salts accumulate in the skin and cause the itching." b. "Toxins released from an inflamed gallbladder lead to itching." c. "Itching is caused by the release of calcium into the skin." d. "Itching is caused by a hypersensitivity reaction."

ANS: A In obstructive jaundice, the normal flow of bile into the duodenum is blocked, allowing excess bile salts to accumulate on the skin. This leads to itching, or pruritus. The other statements are not accurate.

A nurse assesses a client who is prescribed alosetron (Lotronex). Which assessment question should the nurse ask this client? a. "Have you been experiencing any constipation?" b. "Are you eating a diet high in fiber and fluids?" c. "Do you have a history of high blood pressure?" d. "What vitamins and supplements are you taking?"

ANS: A Ischemic colitis is a life-threatening complication of alosetron. The nurse should assess the client for constipation. The other questions do not identify complications related to alosetron.

A client scheduled for a percutaneous transhepatic cholangiography (PTC) denies allergies to medication. What action by the nurse is best? a. Ask the client about shellfish allergies. b. Document this information on the chart. c. Ensure that the client has a ride home. d. Instruct the client on bowel preparation.

ANS: A PTC uses iodinated dye, so the client should be asked about seafood allergies, specifically to shellfish. Documentation should occur, but this is not the priority. The client will need a ride home afterward if the procedure is done on an outpatient basis. There is no bowel preparation for PTC.

A client is scheduled for a colonoscopy and the nurse has provided instructions on the bowel cleansing regimen. What statement by the client indicates a need for further teaching? a. "It's a good thing I love orange and cherry gelatin." b. "My spouse will be here to drive me home." c. "I should refrigerate the GoLYTELY before use." d. "I will buy a case of Gatorade before the prep."

ANS: A The client should be advised to avoid beverages and gelatin that are red, orange, or purple in color as their residue can appear to be blood. The other statements show a good understanding of the preparation for the procedure.

The nurse is working with a client who is recovering after a cervical biopsy. Which statement by the client indicates a need for further instruction? a. "I can resume vaginal intercourse after 6 weeks." b. "I should report heavy bleeding to the health care provider." c. "I must not lift heavy objects for about 2 weeks." d. "I will use the antiseptic rinse on a regular basis."

ANS: A The client should be instructed to keep the perineum clean and dry by using antiseptic solution rinses (as directed by her health care provider) and changing pads frequently. In addition, the client is instructed not to lift heavy objects for 2 weeks and to report excessive bleeding (more than like a normal period). She can resume intercourse in about 2 weeks, when the site has healed; she does not need to wait 6 weeks.

A nurse cares for a client who is recovering from laparoscopic cholecystectomy surgery. The client reports pain in the shoulder blades. How should the nurse respond? a. "Ambulating in the hallway twice a day will help." b. "I will apply a cold compress to the painful area on your back." c. "Drinking a warm beverage can relieve this referred pain." d. "You should cough and deep breathe every hour."

ANS: A The client who has undergone a laparoscopic cholecystectomy may report free air pain due to retention of carbon dioxide in the abdomen. The nurse assists the client with early ambulation to promote absorption of the carbon dioxide. Cold compresses and drinking a warm beverage would not be helpful. Coughing and deep breathing are important postoperative activities, but they are not related to discomfort from carbon dioxide.

After teaching a client who was hospitalized for Salmonella food poisoning, a nurse assesses the client's understanding. Which statement made by the client indicates a need for additional teaching? a. "I will let my husband do all of the cooking for my family." b. "I'll take the ciprofloxacin until the diarrhea has resolved." c. "I should wash my hands with antibacterial soap before each meal." d. "I must place my dishes into the dishwasher after each meal."

ANS: B Ciprofloxacin should be taken for 10 to 14 days to treat Salmonella infection, and should not be stopped once the diarrhea has cleared. Clients should be advised to take the entire course of medication. People with Salmonella should not prepare foods for others because the infection may be spread in this way. Hands should be washed with antibacterial soap before and after eating to prevent spread of the bacteria. Dishes and eating utensils should not be shared and should be cleaned thoroughly. Clients can be carriers for up to 1 year. DIF: Applying/Application REF: 1173 KEY: Inflammatory bowel disorder| medications| antibiotics| medication safety MSC: Integrated Process: Teaching/Learning NOT: Client Needs Category: Health Promotion and Maintenance

A nurse cares for an older adult client who has Salmonella food poisoning. The client's vital signs are heart rate: 102 beats/min, blood pressure: 98/55 mm Hg, respiratory rate: 22 breaths/min, and oxygen saturation: 92%. Which action should the nurse complete first? a. Apply oxygen via nasal cannula. b. Administer intravenous fluids. c. Provide perineal care with a premedicated wipe. d. Teach proper food preparation to prevent contamination.

ANS: B Dehydration caused by diarrhea can occur quickly in older clients with Salmonella food poisoning, so maintenance of fluid balance is a high priority. Monitoring vital signs and providing perineal care are important nursing actions but are of lower priority than fluid replacement. The nurse should teach the client about proper hand hygiene to prevent the spread of infection, and preparation of food and beverages to prevent contamination. DIF: Applying/Application REF: 1173 KEY: Inflammatory bowel disorder| hydration MSC: Integrated Process: Nursing Process: Implementation NOT: Client Needs Category: Safe and Effective Care Environment: Management of Care

After teaching a client who is recovering from laparoscopic cholecystectomy surgery, the nurse assesses the client's understanding. Which statement made by the client indicates a correct understanding of the teaching? a. "Drinking at least 2 liters of water each day is suggested." b. "I will decrease the amount of fatty foods in my diet." c. "Drinking fluids with my meals will increase bloating." d. "I will avoid concentrated sweets and simple carbohydrates."

ANS: B After cholecystectomy, clients need a nutritious diet without a lot of excess fat; otherwise a special diet is not recommended for most clients. Good fluid intake is healthy for all people but is not related to the surgery. Drinking fluids between meals helps with dumping syndrome, which is not seen with this procedure. Restriction of sweets is not required.

The nurse is counseling a postmenopausal woman about her new stress incontinence. Which statement by the nurse is most important? a. "You can try a variety of briefs and undergarments." b. "It will be important to keep that area clean and dry." c. "I can refer you to a good incontinence clinic." d. "Unfortunately, incontinence is common in women your age."

ANS: B After menopause, the vagina becomes dry, thinner, and smoother. This atrophy places the vagina at risk for infection. The combination of this fact with the presence of urine places the woman at higher risk for infection. The nurse should teach the client good hygienic practices to reduce the likelihood of infection. Education about briefs/undergarments may be needed, and a referral to an incontinence clinic would be very helpful, but neither takes priority over preventing infection. Stating that incontinence is common is not a helpful strategy.

An older female client has been prescribed esomeprazole (Nexium) for treatment of chronic gastric ulcers. What teaching is particularly important for this client? a. Check with the pharmacist before taking other medications. b. Increase intake of calcium and vitamin D. c. Report any worsening of symptoms to the provider. d. Take the medication as prescribed by the provider.

ANS: B All of this advice is appropriate for any client taking this medication. However, long-term use is associated with osteoporosis and osteoporosis-related fractures. This client is already at higher risk for this problem and should be instructed to increase calcium and vitamin D intake. The other options are appropriate for any client taking any medication and are not specific to the use of esomeprazole.

A nurse assesses a client who is recovering from a hemorrhoidectomy that was done the day before. The nurse notes that the client has lower abdominal distention accompanied by dullness to percussion over the distended area. Which action should the nurse take? a. Assess the client's heart rate and blood pressure. b. Determine when the client last voided. c. Ask if the client is experiencing flatus. d. Auscultate all quadrants of the client's abdomen

ANS: B Assessment findings indicate that the client may have an over-full bladder. In the immediate postoperative period, the client may experience difficulty voiding due to urinary retention. The nurse should assess when the client last voided. The client's vital signs may be checked after the nurse determines the client's last void. Asking about flatus and auscultating bowel sounds are not related to a hemorrhoidectomy.

The student nurse studying the gastrointestinal system understands that chyme refers to what? a. Hormones that reduce gastric acidity b. Liquefied food ready for digestion c. Nutrients after being absorbed d. Secretions that help digest food

ANS: B Before being digested, food must be broken down into a liquid form. This liquid is called chyme. Secretin is the hormone that inhibits acid production and decreases gastric motility. Absorption is carried out as the nutrients produced by digestion move from the lumen of the GI tract into the body's circulatory system for uptake by individual cells. The secretions that help digest food include hydrochloric acid, bile, and digestive enzymes.

After teaching a client with irritable bowel syndrome (IBS), a nurse assesses the client's understanding. Which menu selection indicates that the client correctly understands the dietary teaching? a. Ham sandwich on white bread, cup of applesauce, glass of diet cola b. Broiled chicken with brown rice, steamed broccoli, glass of apple juice c. Grilled cheese sandwich, small banana, cup of hot tea with lemon d. Baked tilapia, fresh green beans, cup of coffee with low-fat milk

ANS: B Clients with IBS are advised to eat a high-fiber diet (30 to 40 g/day), with 8 to 10 cups of liquid daily. Chicken with brown rice, broccoli, and apple juice has the highest fiber content. They should avoid alcohol, caffeine, and other gastric irritants.

While assessing a client, a nurse detects a bluish tinge to the client's palms, soles, and mucous membranes. Which action should the nurse take next? a. Ask the client about current medications he or she is taking. b. Use pulse oximetry to assess the client's oxygen saturation. c. Auscultate the client's lung fields for adventitious sounds. d. Palpate the client's bilateral radial and pedal pulses.

ANS: B Cyanosis can be present when impaired gas exchange occurs. In a client with dark skin, cyanosis can be seen because the palms, soles, and mucous membranes have a bluish tinge. The nurse should assess for systemic oxygenation before continuing with other assessments.

An older woman is asking the nurse about her husband's sexual functioning. Which statement by the nurse is most accurate? a. "Men his age tend to have a rapid decline in sexual abilities." b. "His testosterone levels will decrease only slightly until he is quite old." c. "Changes in testosterone levels do not affect sexual performance." d. "You are lucky your husband is healthy enough for sexual activity."

ANS: B Men experience a gradual but slight decrease in testosterone until they are in their 80s. Low testosterone levels do affect sexual performance. Stating that the woman is lucky does not give accurate information about sexual functioning.

A client has a pyloric obstruction and reports sudden muscle weakness. What action by the nurse takes priority? a. Document the findings in the chart. b. Request an electrocardiogram (ECG). c. Facilitate a serum potassium test. d. Place the client on bedrest.

ANS: B Pyloric stenosis can lead to hypokalemia, which is manifested by muscle weakness. The nurse first obtains an ECG because potassium imbalances can lead to cardiac dysrhythmias. A potassium level is also warranted, as is placing the client on bedrest for safety. Documentation should be thorough, but none of these actions takes priority over the ECG.

A nurse plans care for a client with acute pancreatitis. Which intervention should the nurse include in this client's plan of care to reduce discomfort? a. Administer morphine sulfate intravenously every 4 hours as needed. b. Maintain nothing by mouth (NPO) and administer intravenous fluids. c. Provide small, frequent feedings with no concentrated sweets. d. Place the client in semi-Fowler's position with the head of bed elevated.

ANS: B The client should be kept NPO to reduce GI activity and reduce pancreatic enzyme production. IV fluids should be used to prevent dehydration. The client may need a nasogastric tube. Pain medications should be given around the clock and more frequently than every 4 to 6 hours. A fetal position with legs drawn up to the chest will promote comfort.

The nurse is preparing a teaching plan for a client who is scheduled to undergo mammography for the first time. What instruction by the nurse is accurate? a. "The test should be carried out even if you are pregnant." b. "Do not use deodorant on breasts or underarms before the test." c. "You will not experience any discomfort because this is just an x-ray." d. "The entire test should not take longer than 1 hour."

ANS: B The client should be reminded not to use creams, powders, or deodorant on breast or underarm areas before mammography because these products can show on the x-ray. The test should be rescheduled if any possibility exists that the client is pregnant. Women can experience discomfort as the breasts are compressed. The test is generally much less than an hour in duration.

The nurse is caring for a client with peptic ulcer disease who reports sudden onset of sharp abdominal pain. On palpation, the client's abdomen is tense and rigid. What action takes priority? a. Administer the prescribed pain medication. b. Notify the health care provider immediately. c. Percuss all four abdominal quadrants. d. Take and document a set of vital signs.

ANS: B This client has manifestations of a perforated ulcer, which is an emergency. The priority is to get the client medical attention. The nurse can take a set of vital signs while someone else calls the provider. The nurse should not percuss the abdomen or give pain medication since the client may need to sign consent for surgery.

When transferring a client into a chair, a nurse notices that the pressure-relieving mattress overlay has deep imprints of the client's buttocks, heels, and scapulae. Which action should the nurse take next? a.Turn the mattress overlay to the opposite side. b.Do nothing because this is an expected occurrence. c.Apply a different pressure-relieving device. d.Reinforce the overlay with extra cushions.

ANS: C "Bottoming out," as evidenced by deep imprints in the mattress overlay, indicates that this device is not appropriate for this client, and a different device or strategy should be implemented to prevent pressure ulcer formation.

A nurse assesses clients on a medical-surgical unit. Which client is at greatest risk for pressure ulcer development? a.A 44-year-old prescribed IV antibiotics for pneumonia b.A 26-year-old who is bedridden with a fractured leg c.A 65-year-old with hemi-paralysis and incontinence d.A 78-year-old requiring assistance to ambulate with a walker

ANS: C Being immobile and being incontinent are two significant risk factors for the development of pressure ulcers. The client with pneumonia does not have specific risk factors. The young client who has a fractured leg and the client who needs assistance with ambulation might be at moderate risk if they do not move about much, but having two risk factors makes the 65-year-old the person at highest risk.

A nurse assesses a client who is hospitalized with an exacerbation of Crohn's disease. Which clinical manifestation should the nurse expect to find? a. Positive Murphy's sign with rebound tenderness to palpitation b. Dull, hypoactive bowel sounds in the lower abdominal quadrants c. High-pitched, rushing bowel sounds in the right lower quadrant d. Reports of abdominal cramping that is worse at night

ANS: C The nurse expects high-pitched, rushing bowel sounds due to narrowing of the bowel lumen in Crohn's disease. A positive Murphy's sign is indicative of gallbladder disease, and rebound tenderness often indicates peritonitis. Dullness in the lower abdominal quadrants and hypoactive bowel sounds are not commonly found with Crohn's disease. Nightly worsening of abdominal cramping is not consistent with Crohn's disease. DIF: Applying/Application REF: 1182 KEY: Crohn's disease| assessment/diagnostic examination MSC: Integrated Process: Nursing Process: Analysis NOT: Client Needs Category: Physiological Integrity: Physiological Adaptation

A nurse prepares a client for a colonoscopy scheduled for tomorrow. The client states, "My doctor told me that the fecal occult blood test was negative for colon cancer. I don't think I need the colonoscopy and would like to cancel it." How should the nurse respond? a. "Your doctor should not have given you that information prior to the colonoscopy." b. "The colonoscopy is required due to the high percentage of false negatives with the blood test." c. "A negative fecal occult blood test does not rule out the possibility of colon cancer." d. "I will contact your doctor so that you can discuss your concerns about the procedure."

ANS: C A negative result from a fecal occult blood test does not completely rule out the possibility of colon cancer. To determine whether the client has colon cancer, a colonoscopy should be performed so the entire colon can be visualized and a tissue sample taken for biopsy. The client may want to speak with the provider, but the nurse should address the client's concerns prior to contacting the provider.

A client with a bleeding gastric ulcer is having a nuclear medicine scan. What action by the nurse is most appropriate? a. Assess the client for iodine or shellfish allergies. b. Educate the client on the side effects of sedation. c. Inform the client a second scan may be needed. d. Teach the client about bowel preparation for the scan.

ANS: C A second scan may be performed in 1 to 2 days to see if interventions have worked. The nuclear medicine scan does not use iodine-containing contrast dye or sedation. There is no required bowel preparation.

A client had a colonoscopy and biopsy yesterday and calls the gastrointestinal clinic to report a spot of bright red blood on the toilet paper today. What response by the nurse is best? a. Ask the client to call back if this happens again today. b. Instruct the client to go to the emergency department. c. Remind the client that a small amount of bleeding is possible. d. Tell the client to come in to the clinic this afternoon.

ANS: C After a colonoscopy with biopsy, a small amount of bleeding is normal. The nurse should remind the client of this and instruct him or her to go to the emergency department for large amounts of bleeding, severe pain, or dizziness.

A client is having an esophagogastroduodenoscopy (EGD) and has been given midazolam hydrochloride (Versed). The client's respiratory rate is 8 breaths/min. What action by the nurse is best? a. Administer naloxone (Narcan). b. Call the Rapid Response Team. c. Provide physical stimulation. d. Ventilate with a bag-valve-mask.

ANS: C For an EGD, clients are given mild sedation but should still be able to follow commands. For shallow or slow respirations after the sedation is given, the nurse's first action is to provide a physical stimulation such as a sternal rub and directions to breathe deeply. Naloxone is not the antidote for Versed. The Rapid Response Team is not needed at this point. The client does not need manual ventilation.

After teaching a client who expressed concern about a rash located beneath her breast, a nurse assesses the client's understanding. Which statement indicates the client has a good understanding of this condition? a. "This rash is probably due to fluid overload." b. "I need to wash this daily with antibacterial soap." c. "I can use powder to keep this area dry." d. "I will schedule a mammogram as soon as I can."

ANS: C Rashes limited to skinfold areas (e.g., on the axillae, beneath the breasts, in the groin) may reflect problems related to excessive moisture. The client needs to keep the area dry; one option is to use powder. Good hygiene is important, but the rash does not need an antibacterial soap. Fluid overload and breast cancer are not related to rashes in skinfolds.

A nurse assessing a client with colorectal cancer auscultates high-pitched bowel sounds and notes the presence of visible peristaltic waves. Which action should the nurse take? a. Ask if the client is experiencing pain in the right shoulder. b. Perform a rectal examination and assess for polyps. c. Contact the provider and recommend computed tomography. d. Administer a laxative to increase bowel movement activity.

ANS: C The presence of visible peristaltic waves, accompanied by high-pitched or tingling bowel sounds, is indicative of partial obstruction caused by the tumor. The nurse should contact the provider with these results and recommend a computed tomography scan for further diagnostic testing. This assessment finding is not associated with right shoulder pain; peritonitis and cholecystitis are associated with referred pain to the right shoulder. The registered nurse is not qualified to complete a rectal examination for polyps, and laxatives would not help this client.

A client with peptic ulcer disease is in the emergency department and reports the pain has gotten much worse over the last several days. The client's blood pressure when lying down was 122/80 mm Hg and when standing was 98/52 mm Hg. What action by the nurse is most appropriate? a. Administer ibuprofen (Motrin). b. Call the Rapid Response Team. c. Start a large-bore IV with normal saline. d. Tell the client to remain lying down.

ANS: C This client has orthostatic changes to the blood pressure, indicating fluid volume loss. The nurse should start a large-bore IV with isotonic solution. Ibuprofen will exacerbate the ulcer. The Rapid Response Team is not needed at this point. The client should be put on safety precautions, which includes staying in bed, but this is not the priority.

A nurse is caring for a client who has a pressure ulcer on the right ankle. Which action should the nurse take first? a.Draw blood for albumin, prealbumin, and total protein. b.Prepare for and assist with obtaining a wound culture. c.Place the client in bed and instruct the client to elevate the foot. d.Assess the right leg for pulses, skin color, and temperature.

ANS: D A client with an ulcer on the foot should be assessed for interruption in arterial flow to the area. This begins with the assessment of pulses and color and temperature of the skin. The nurse can also assess for pulses noninvasively with a Doppler flowmeter if unable to palpate with his or her fingers. Tests to determine nutritional status and risk assessment would be completed after the initial assessment is done. Wound cultures are done after it has been determined that drainage, odor, and other risks for infection are present. Elevation of the foot would impair the ability of arterial blood to flow to the area.

After teaching a client with diverticular disease, a nurse assesses the client's understanding. Which menu selection made by the client indicates the client correctly understood the teaching? a. Roasted chicken with rice pilaf and a cup of coffee with cream b. Spaghetti with meat sauce, a fresh fruit cup, and hot tea c. Garden salad with a cup of bean soup and a glass of low-fat milk d. Baked fish with steamed carrots and a glass of apple juice

ANS: D Clients who have diverticular disease are prescribed a low-residue diet. Whole grains (rice pilaf), uncooked fruits and vegetables (salad, fresh fruit cup), and high-fiber foods (cup of bean soup) should be avoided with a low-residue diet. Canned or cooked vegetables are appropriate. Apple juice does not contain fiber and is acceptable for a low-residue diet. DIF: Applying/Application REF: 1187 KEY: Diverticular disease| nutritional requirements MSC: Integrated Process: Teaching/Learning NOT: Client Needs Category: Physiological Integrity: Basic Care and Comfort

The nurse is conducting a reproductive assessment of a postmenopausal woman. Which assessment finding reported by the client requires immediate intervention by the nurse? a. Urinary incontinence b. Vaginal dryness c. Painful intercourse d. Returning periods

ANS: D All client reports require some action by the nurse, but the priority would be to further investigate and report the "returning periods." In a postmenopausal woman, this can signal cancer.

A nurse assesses a client who has multiple areas of ecchymosis on both arms. Which question should the nurse ask first? a. "Are you using lotion on your skin?" b. "Do you have a family history of this?" c. "Do your arms itch?" d. "What medications are you taking?"

ANS: D Certain drugs such as aspirin, warfarin, and corticosteroids can lead to easy or excessive bruising, which can result in ecchymosis. The other options would not provide information about bruising.

The nurse is working with a client who is recovering after a laparoscopy. Which assessment finding is considered a priority by the nurse? a. Slight drainage from the incision site b. Grogginess after the anesthesia c. Discomfort from the catheter d. Reports of shoulder pain

ANS: D Clients should expect mild drainage or blood from the incision site. Grogginess from the anesthesia and discomfort from a catheter are also expected minor occurrences post-laparoscopy. The nurse would not be concerned about these but should intervene and treat the client with shoulder pain. Shoulder pain is referred pain from phrenic nerve irritation and can be expected.

After teaching a client who has a history of cholelithiasis, the nurse assesses the client's understanding. Which menu selection made by the client indicates the client clearly understands the dietary teaching? a. Lasagna, tossed salad with Italian dressing, and low-fat milk b. Grilled cheese sandwich, tomato soup, and coffee with cream c. Cream of potato soup, Caesar salad with chicken, and a diet cola d. Roasted chicken breast, baked potato with chives, and orange juice

ANS: D Clients with cholelithiasis should avoid foods high in fat and cholesterol, such as whole milk, butter, and fried foods. Lasagna, low-fat milk, grilled cheese, cream, and cream of potato soup all have high levels of fat. The meal with the least amount of fat is the chicken breast dinner.

A nurse assesses clients at a community health center. Which client is at highest risk for the development of colorectal cancer? a. A 37-year-old who drinks eight cups of coffee daily b. A 44-year-old with irritable bowel syndrome (IBS) c. A 60-year-old lawyer who works 65 hours per week d. A 72-year-old who eats fast food frequently

ANS: D Colon cancer is rare before the age of 40, but its incidence increases rapidly with advancing age. Fast food tends to be high in fat and low in fiber, increasing the risk for colon cancer. Coffee intake, IBS, and a heavy workload do not increase the risk for colon cancer.

A client is being taught about drug therapy for Helicobacter pylori infection. What assessment by the nurse is most important? a. Alcohol intake of 1 to 2 drinks per week b. Family history of H. pylori infection c. Former smoker still using nicotine patches d. Willingness to adhere to drug therapy

ANS: D Treatment for this infection involves either triple or quadruple drug therapy, which may make it difficult for clients to remain adherent. The nurse should assess the client's willingness and ability to follow the regimen. The other assessment findings are not as critical.

A patient is admitted to the hospital with severe diarrhea. The patient should be monitored for which complication associated with diarrhea? 1) Hypokalemia 2) Hypocalcemia 3) Hyperglycemia 4) Thrombocytopenia

Hypokalemia Diarrhea causes fluid loss and hypokalemia, not hypocalcemia, hyperglycemia, or thrombocytopenia.

Which type of bowel diversion allows the patient to be free from an appliance? 1) Colostomy in the transverse colon 2) Double-barreled colostomy 3) Ileostomy 4) Kock pouch

Kock pouch A Kock pouch, also known as a continent ileostomy, creates an internal pouch to collect ileal drainage. To drain the pouch, the patient inserts a tube through the external stoma into a pouch several times a day. This allows the patient to be free from an appliance. A colostomy, double-barreled colostomy, and ileostomy all require an appliance.

A nurse is conducting a health history assessment of a client and determines that which factor places the client at risk for development of irritable bowel disease (IBD)? a. Stress b. Peptic ulcers c. GERD (gastroesophageal reflux disease) d. Helicobacter pylori

a. Stress

A patient is taking omeprazole (Prilosec) for the treatment of gastroesophageal reflux disease (GERD). Which information should the nurse emphasize when teaching about this medication? (Select all that apply.) a. The medication should not be taken with alcohol. b. This medication is for long-term therapy. c. The medication can affect kidney function. d. Although rare, blood disorders may occur. e. Common side effects include headache and diarrhea.

a. The medication should not be taken with alcohol d. Although rare, blood disorders may occur. e. Common side effects include headache and diarrhea.

2. Which of the following statements is most accurate concerning the personal happiness of the nurse when dealing with people with difficult behavior? a. The nurse must believe that she or he is 100 percent responsible for their own happiness. b. Difficult people can put the nurse in a persistent state of unhappiness. c. It is up to others to make the nurse happy or unhappy. d. Hanging out with unhappy people will allow the nurse to identify factors in her or his own life that causes unhappiness and counter act them.

a. The nurse must believe that she or he is 100 percent responsible for their own happiness.

which definition of immunodeficiency is accurate? A. disease/deficiency acquired as a result of viral infection, contact with toxin, or medical therapy B. deficient immune response as a result of imapired or missing immune components C. chronic infection wih immunodeficiency virus D. disease/deficiency pesent since birth

b

1. Which of the following is an accurate statement about behavior changes when dealing with people who are displaying difficult behaviors? a. With consistent communication practices, co-workers are likely to change their behaviors. b. One of the important goals when dealing with patients with difficult behavior is to change their behavior. c. The nurse needs to be consistent and not change her behaviors in response to co-workers with difficult behavior. d. All behavior is a matter of perception.

b. One of the important goals when dealing with patients with difficult behavior is to change their behavior.

The nurse should be concerned with which aspect of the health history of a patient who is taking a magnesium-based antacid? a. Peptic ulcer disease b. Renal failure c. Hypertension d. Heart failure

b. Renal failure

A nurse is caring for a client receiving chemotherapy with high emetogenic potential. Which drugs should the nurse prepare to administer prior to chemotherapy administration? (Select all that apply.) a. Anticholinergics b. Serotonin (5-HT3) antagonists c. Corticosteroids d. Neurokinin receptor antagonists e. Phenothiazines

b. Serotonin (5-HT3) antagonists c. Corticosteroids d. Neurokinin receptor antagonists e. Phenothiazines

the HCP prescribes an integrase inhibitor for an HIV patient. the patient asks the nurse how this drug works. what is the nurses best response? A. it reduces how well HIV genetic material can be converted into human genetic material B. it reinforces the immune systems ability to fight off an infection C. it prevents viral DNA from integrating into hosts DNA D. prevent HIV infection from progressing to AIDS

c

4. Which of the following statements made by a client during an individual therapy session would the nurse most identify as reflecting schizoaffective disorder? a. I want to cut my arms with a knife. b. My mind is racing and I can't control my thoughts c. I just got fired from my third job this month because the boss belongs to the CIA. d. Life has no meaning for me anymore and I just bought a gun from a street pusher.

c. I just got fired from my third job this month because the boss belongs to the CIA.

A patient has been using over-the-counter omeprazole (Prilosec) for relief of gastric upset. The nurse should provide which teaching regarding appropriate administration of this medication? a. Drink a full glass of water with administration. b. Do not take the medication with antacids. c. Never crush or chew the medication. d. Take medication 30 minutes after meals.

c. Never crush or chew the medication.

A nurse conducting a health history with a patient identifies that which health condition places the patient at risk for developing gastroesophageal reflux disease (GERD)? a. Cigarette smoking b. Type II diabetes mellitus c. Obesity d. Alcohol use

c. Obesity

5. What is the primary reason that a nurse use silence when communicating with a patient about his diagnosis? a. A period of silence makes the patient feel uncomfortable b. Several periods of silence makes the teaching session longer c. Silence periods allow the patient to gather their thoughts before speaking d. Silence periods allow the nurse to think of her next question for the patient

c. Silence periods allow the patient to gather their thoughts before speaking

4. A nurse is sitting by the bedside close to a patient to better hear him and letting him know that she has his full attention. What factor does the nurse need to keep in mind that may interfere with effective communication with this patient? a. The patient may not be hard of hearing b. The volume on the TV may be at a high setting c. The patient may have person space issues d. The nurse may need to sit closer to be most effective

c. The patient may have person space issues

What is primary diversity?

more obvious, such as nationality, race, color, gender, age, religious beliefs

What is secondary diversity?

socioeconomic status, education, occupation, length of time away from the country or origin, gender issues, residential status, and sexual orientation

Replication of HIV

• HIV targets CD4 receptor on T4 lymphocyte - Using reverse transcriptase, makes viral DNA from RNA • Virions bud from host cell - Enzyme protease enables virion to infect other T4 lymphocytes - Result is gradual destruction of immune system • HIV called "retrovirus" because of reverse synthesis process

Therapy for Viral Infections

• Mature particles called virions • Most viruses are self-limiting; require no pharmacotherapy - Example: rhinovirus that causes common cold • Some viruses cause serious disease and require aggressive therapy • Examples: - HIV fatal if left untreated - Herpes viruses can cause significant pain and disability if left untreated

Structure of Viruses

• Surrounded by capsid (protein coat) • Contain a few dozen genes, either RNA or DNA • DNA contains information needed for replication • May have lipid envelope with protein "spikes" that trigger immune response


Set pelajaran terkait

Chapitre 2: les régimes totalitaires

View Set

Chapter 3- Connections and Interactions

View Set

Chapter 20: Analysis of Convertible bonds

View Set

chapter 4, 38, 32, 34, 45, 43, 6

View Set

Left Side Vocabulario two boxes (Andres Sandoval)

View Set

PSYC 2400 SOCIAL PSYCHOLOGY FINAL EXAM MATERIAL

View Set

Marketing Ch 10 Customer heterogeneity. Segmentation techniques in the reference market. Individualized demand analysis: Customer panels and Relationship Management

View Set